Otolaryngology Alberta Notes

Pataasin ang iyong marka sa homework at exams ngayon gamit ang Quizwiz!

What are the 4 R's of radiotherapy injury mechanisms?

3: Repair - Sublethal injury will be repaired by the cell if it takes no further hits, increased fractionation increases the opportunity for repair 3: Reoxygenation - Presence of oxygen increases the effects of Ionizing Radiation, radiosensitivity stays the same down to 20 mmHg oxygen, below this sensitivity decreases 3: Redistribution - Maximum Radioresistance occurs in late S phase of cell cycle, maximum Radiosensitivity occurs in early M phase, increased fractionation allows increased Redistribution of tumor cells into radiosensitive phases 3: Repopulation - Tumors Accelerate Repopulation after cell reduction from Surgery or Radiation

Two Disadvantages of Postoperative Radiotherapy for SCC

3: Surgery may interrupt Blood Flow to remaining tumor cells making them less radiosensitive 3: Wound Breakdown or Infectious Complications may Delay the onset or Prevent radiotherapy delivery

Name 4 antibiotics which can cross the blood-brain barrier

A. Ampicillin A: Ceftriaxone A: Flagyl A: Chloramphenicol

Seven Patterns of Nystagmus

A: "Jerk", or "Sawtooth" (VOR, OKN) - Constant slow phase velocity interrupted by quick phase saccades; Horizontal or Vertical? A: Torsional A: Gaze-paretic (Cerebellar) - Decreasing slow phase velocity A: Congenital - Increasing slow phase velocity A: Pendular - Equal to and fro velocities A: See-saw - Disconjugate A: Vergence - Disconjugate

Consensus Auditory Perceptual Evaluation for Voice

A: "The blue spot is on the key again" - Contains all of the vowel sounds in English A: "How hard did he hit him?" - Emphasizes easy onset A: "We were away a year ago" - Is all-voiced A: "We eat eggs every Easter" - Elicits hard glottal onsets A: "My mama makes lemon muffins" - Uses nasal sounds A: "Peter will keep at the peak" - Is loaded with voiceless plosive sounds

Management of Bilateral VCP

A: +/- urgent intubation A: +/- ACM management A: Supportive measures (upright positioning, thickening of formula, observation, management) A: Tracheotomy (> 2 years to allow for spontaneous recovery, 50%) A: Lateralization (arytenoidectomy +/- CO2 laser cordotomy, arytenoidopexy, cordotomy) A: Open procedures / Reanimation / Electrical pacers 3: Decannulation rates > 60% A: ? New procedure - cricothyroid myotomy (help to relax and open paralyzed cords enough to get them extubated while waiting for spontaneous recovery)

Type A vs Type C tympanogram peak division (Baileys and KJ Lee)

A: -100 mmH2O (daPa)

Wermer's classification of Grave's ophthalmopathy (NO SPECS)

A: 0 = Normal A: I = Only signs (lid retraction, lid lag, and stare) A: II = Soft tissue Swelling (deep conjunctival injection, mild chemosis, edema of caruncle or periorbita) A: III = Proptosis (>3 mm? use exophthalmometer) A: IV = Extraocular muscle involvement (inferior and medial rectus most commonly, use forced duction test) A: V = Corneal exposure (exposure keratopathy) A: VI = Sight loss

Describe the Skin Endpoint Titration technique and results

A: 0.01 mL injected - Wheel response of 5 mm normal with control A: Induration (not erythema) is measured, serial increase by 2 mm is the endpoint titration. Recall that need a confirmatory wheel of greater than 2 mm to confirm previous wheel (which also grew by at least 2 mm)

Physiologic frequency response of the SCCs

A: 0.1-10 Hz angular accelerations

List the 5 grades of middle ear barotrauma

A: 1 - TM injection A: 2 - Hemorrhage within TM A: 3 - Gross hemorrhage A: 4 - Gross hemotympanum A: 5 - TM perforation

Discuss Malignant Hyperthermia

A: 1 in 15,000 children, 1 in 50-100,000 adults (i.e. more common in children) A: AD, reduced penetrance, variable expressivity A: Sudden increase in Ca in muscle sarcoplasm; either increased release or decreased reuptake from sarcoplasmic reticulum A: Clinical - Fever, tachycardia, tachypnea, cyanosis, hypercarbia A: Associated with halogenated inhalational anesthetics, depolarizing muscle relaxants A: Investigations - Muscle biopsy, CPK, LDH, ALP, urine Myoglobin A: Treatment - ABC, Discontinuation of precipitating agent, Dantrolene 1mg/kg slow IV push up to 10mg/kg, cooling, 100% oxygen, cardiovascular support

Temporal Bone Epidermoids

A: 1% of all Intracranial tumors, 3% of CPA lesions A: Four Anatomic groups - Petrous Apex (most common), CPA, Perigeniculate, Middle ear A: Entrapped Epithelial Rests - Slow growing, Infiltrative & Expanding, Local Inflammatory reaction A: Symptoms - Imbalance & Hearing Loss; Facial Weakness & Spasm; Trigeminal numbness & pain; Very Gradual onset A: Complete excision nearly impossible, 30% recurrence

At what rate/year do these tumours tend to grow?

A: 1-4 mm/year 3: Study by Jansen (Cancer, 2000) showed a mean doubling time of 7 years

Enzyme stimulated by PTH to activate vitamin D

A: 1-alpha-hydroxylase

Typical radiation dose

A: 1.8 - 2.2 Gy given 5x/week for 6-8 weeks

Pediatric dosing of Local anaesthesia

A: 1.8 cc of 2% lidocaine/20 lbs

What is the Pheochromocytoma rule of 10s?

A: 10% bilateral A: 10% malignant A: 10% extraadrenal

Normal parents, with child with SNHL, chance of further children with SNHL (all comers)

A: 14%

Greatest period of flap failure and most common site

A: 15-20 min post anastamosis, then the first 3 days A: Venous anastamosis

Describe the embryologic derivations of the Ethmoturbinals (appear in 8th week gestation)

A: 1st ethmoturbinal, Ascending portion - Agger nasi A: 1st ethmoturbinal, Descending portion - Uncinate process A: 2nd ethmoturbinal - Middle turbinate A: 3rd ethmoturbinal - Superior turbinate A: 4th to 6th ethmoturbinals - Usually fuse and degenerate, but occasionally can form a Supreme turbinate 3: Maxilloturbinal - Arise inferiorly, eventually becomes the Inferior turbinate

Pediatric bronchoscope sizes (inner diameter)

A: 2.5 (3.5) = premie A: 3.0 (4.3) = 0-3 months A: 3.5 (5.0) = 3-18 months A: 3.7 (5.7) = 1-3 years A: 4.0 (6.0) = 2-6 years A: 5.0 (7.1) = 5-10 years A: 6.0 (7.5) = >10 years A: 6.5 (8.5) = adult 3: Outer diameter always equals inner diameter + 0.7 mm

Ingredient in Hurricaine spray

A: 20% benzocaine in polyethylene glycol base

What are the associations between CRS, Polyps (NP), and Asthma?

A: 20% of CRS have NP A: 20% of CRS have Asthma A: 40% of FESS have Asthma A: 50% of NP have Asthma (35% of which have Samter's triad) A: 10% of NP have Samter's triad

Etiology of vasomotor Rhinitis during Pregnancy

A: 20% of pregnancies, usually starts in 2nd trimesters A: Increased Estrogen levels inhibits Acetylcholinesterase activity, leads to increased ACh in parasympathetic ganglia, causes swelling & edema of nasal mucosa A: Treatment - Conservative, saline spray, possible use of Budesonide (Rhinocort) spray, Consult obstetrician; no use of decongestants

Physiology of salivation

A: 24h Salivary flow volume = 1-1.5L A: Submandibular glands responsible for majority of nonstimulated salivary flow, ~70% in 24 hour period A: Parotid supplies ~66% of total stimulated salivary flow, overall ~25% of total 24 hour flow A: Parotid saliva serous; SLG mucinous, SMG mixed serous/mucinous saliva

Percentage of Otosclerosis with Vestibular symptoms (most commonly disequilibrium)

A: 25%

Prognosis of Tinnitus

A: 25% better A: 50% slight improvement A: 25% unchanged

Degree of CHL with Negative Rinne test

A: 256 Hz = >15 dB or more (KJ Lee); 20-30dB (Cummings) A: 512 Hz = >25 dB loss (KJ Lee); 30-45dB (Cummings) A: 1024 Hz = >35 dB loss (KJ Lee); 45-60dB (Cummings)

Number of dB gained by middle ear mechanism

A: 25dB (25-30 dB in Ben's notes)

Discuss Herpes Zoster Oticus/Ramsay Hunt syndrome (only Ramsay hunt if have the facial palsy)

A: 2nd most common cause of facial paralysis (5-9%) A: Clinical - Present with Vesicles on Pinna, Retroauricular area, Face, Mouth with Acute Facial Palsy; 25% of patients will have Hyperacusis, Tinnitus, Hearing loss, Pain, Vertigo A: Complications - Postherpetic neuralgia A: Prognosis - Depends on Severity; Complete loss = full recovery in 10%; Incomplete loss = full recovery in 66% A: Treatment - Corticosteroids (reduces Acute Pain, Postherpetic Neuralgia, & Vertigo), Valacyclovir 1 g PO tid x 7 days

Describe Optic Nerve Glioma

A: 2nd most common pediatric orbital tumor A: Cell origin - Fibrillary astrocyte, associated with neurofibromatosis 1 in 18-50% A: Radiographic - CT fusiform, lobular, isodense homogenous enlargement of optic nerve; MRI T1 hypo, T2 hyper A: Treatment - Observation vs. resection depending on whether indolent or progressive growth

Polymorphous Low Grade (PLGA)

A: 2nd most common tumor of the MSG's A: Seen on palate, buccal mucosa, upper lip A: Gross - firm, painless mucosalized mass A: Histo - variable tumor cell differentiation & organization, mitotic figures & necrosis unusual

dB required to double sound energy

A: 3 dB (IL) or 6 dB (SPL)

Time required post-XRT to call biopsy reliable

A: 3 months 3: Cell lysis occurs at mitosis, 4-5 divisions occur before Lysis 3: Lethally injured cells and Surviving cells are morphologically identical

Outline the timeline of Cleft Lip and Palate repair

A: 3 months - Cleft lip repair, PET insertion A: 1 year - Cleft palate repair A: 5 years - Columellar lengthening (for bilateral cleft lip) A: 8-16 years - Orthodontics A: 10 years - Alveolar cancellous bone graft A: 14 years - Definitive rhinoplasty and orthognatic surgery

Describe Grave's Ophthalmopathy

A: 3 to 1 female predominance, 5th-6th decades, incidence ~10-45% (!), most severe forms with CN II involvement rare, ~2-5% of Graves' patients require surgical intervention 3: Due to Thyroid Stimulating Immunoglobulins, T-lymphocyte Infiltration into orbital tissues, Fibroblast response causes Glycosaminoglycan and Collagen deposition in the Extraocular Muscles leading to Fibrosis and Ophthalmoplegia

New perforation of TM - How long to wait until repair?

A: 3-6 months

Smallest bronchoscope able to accommodate peanut grasper

A: 3.5

Percentage of extralaryngeal spread of adult and juvenile onset RRP

A: 30% Children A: 15% Adult

Risks of Posttreatment Recurrence, and of 2nd Primary development with continued Smoking and EtOH consumption

A: 30-50% risk of Locoregional Recurrence A: 10-40% risk of 2nd Primary development

Why do all noise induced HL have 4000 Hz notch?

A: 3000 Hz is the natural Resonance Frequency of the EAC but routine audio only tests 4000Hz A: 5000 Hz is the natural Resonance Frequency of the Concha A: Hair cells at the Basal Turn are most susceptible to Oxidative Stress A: Greatest Sensitivity of human ear is to frequencies between 1-5 kHz A: Protective Effect of Stapedial Reflex is <2 kHz intermittent loud sounds; more protective for the Lower Frequencies

Average volume of the orbital cavity

A: 30ml 3: 5ml increase in size (16%) will cause 5mm proptosis

How much distraction of the cricoid is required for a cartilage graft to be placed in the anterior split

A: 3mm

Assessment of Cerebral Blood Flow in skull base surgery

A: 4 Vessel Arteriography with Venous Phase A: Trial balloon occlusion A: Single-Photon Emission Computed Tomography scanning (SPECT, using Tc-99m HMPAO) A: Functional Cerebral Blood Flow evaluation - Xenon-CBF study 3: Unselected ICA interruption - 26% rate of cerebral infarction (depends on patient age)

What week embryologically does thyroid development begin and end

A: 4 th week to 10 th week (TGD disappears)

Briefly outline the Embryology of the Larynx

A: 4 weeks - Formation of the Laryngotracheal Groove in the ventral wall of the primitive pharynx A: 5-7 weeks - Elongation of the Laryngotracheal Diverticulum, separated from the dorsal foregut by the Tracheoesophageal Septum A: 8-9 weeks - Epithelial proliferation obliterates the laryngeal lumen A: 10 weeks - Recanalization of the laryngeal lumen is complete

Describe the Embryology of the Thyroid gland

A: 4 weeks GA - Endoderm on the floor of the pharynx between the 1st & 2nd arches invaginates and descends into the mesenchyme of the neck; this diverticulum is situated between the Tuberculum impar (forms the oral tongue along with the ligual swellings) and the Copula (forms the base of tongue) A: 4.5 weeks GA - The connection between the ventral thyroid diverticulum and the floor of the pharynx (Foramen Cecum) disappears; the diverticulum develops into the median thyroid anlage A: 6 weeks GA - The thyroglossal duct has completely degenerated, and cellular proliferation results in the right and left thyroid lobes, separated by an isthmus; in up to 50% of individuals, the distal aspect of the duct persists as a pyramidal lobe A: The Ultimobranchial body of the 4th arch fuses and is incorporated into the supero-lateral aspects of the thyroid lobes, and forms the parafollicular C-cells which secrete calcitonin

Recurrent acute adenoiditis

A: 4+ episodes in a 12 month period (as per 2012 AAO clinical indicators for adenoidectomy)

Four Tests to assess the adequacy of Collateral Circulation preop Carotid Resection

A: 4-vessel angiography A: Trial balloon occlusion A: Single-photon emission computed tomography scanning (SPECT, using Tc-99m HMPAO) A: Xenon-133 flow scan

Normal term subglottis

A: 4.5 - 5 mm A: Size 3.5 ETT

Mean age at presentation and Gender predilection for Congenital Cholesteatoma

A: 4.5 years A: Male:Female ratio 3:1

TEOAE are found when hearing is better than...

A: 40 dB, but not effective for evaluating higher frequency hearing loss (better at 500, 1000 hz)

Describe the pathology for labyrinthine anomalies

A: 40% of radiologically abnormal (osseous) Cochleas will have SCC abnormality A: SCC Dysplasia 4x as common as SCC Aplasia A: Lateral canal affected most often = Last to develop

Botox - how long until onset of paralysis and how long overall effect

A: 48 hours (presynaptic vesicles depleted) A: 90 days (although new axons begin sprouting at 28 days)

Embryology of the External Auditory Canal

A: 4th gestational week - 1st Arch groove invaginates over next 4 weeks, ectoderm forms a core of tissue that comes into apposition with endoderm of 1st pouch A: 28th week - Epithelial core canalizes from medial to lateral to form EAC

Embryology of the Middle ear

A: 4th week - Tubotympanic Recess formed by laterally migrating 1st Arch Pouch A: 5th-6th weeks - Mesoderm between branchial cleft & otic capsule condense to form Ossicles A: Endodermal pouch continues growing by surrounding the ossicles & supporting structures; remains slitlike into the 5th month, expands into an open space by the 8th month A: Mastoid pneumatization starts in late fetal life, Antrum present at birth

Distance of tumor to resection Margin that is considered Close

A: 5 mm 3: Found to have Same Prognostic significance for Recurrence as Positive Margins

DPOAE are found when hearing is better than...

A: 50 dB, and are effective up to 6000 Hz (better at 4000, 6000 hz.....both about equal at 2000hz)

Define Phonetically Balanced word list

A: 50 single syllable words which contains the same proportion of Phonemes as that which occurs in connected American English discourse A: Used to determine the SDS

Percentage of laryngeal anomalies with other airway anomaly

A: 50%

Success in OSA surgery is defined as ...

A: 50% reduction in the RDI, and A: RDI <20 (?<15 ie. less than moderate disease)

Percent of Robin sequence associated with a syndrome

A: 50-80% A: Stickler A: VCFS

Describe the External ear Embryology (Bailey ch. 128)

A: 6 hillocks of His, derived from 1st/2nd branchial arches during 5th gestational week; reach adult form (but not size) by 18th week A: 1st Arch derivatives - 1st) Tragus, 2nd) Helical crus, 3rd - Ascending Helix A: 2nd Arch derivatives - 4th) Horizontal helix, scapha, antihelix crus, 5th) Descending helix, scapha, antihelix, 6th) Antitragus, inferior aspect of helix, and lobule?

By what percentage will 1 mm of subglottic edema reduce the airway in a neonate?

A: 67%

What if the half-life of azithromycin?

A: 68 hours

Embryologic development Timeline of the Malleus & Incus

A: 6th week - Single mass A: 8th week - Separated, with Malleo-Incudal joint formed A: 16th week - Ossification in long process of Incus A: 17th week - Ossification in medial neck of Malleus A: Birth - Adult size & shape 3: Most common abnormality in middle ear atresia = Fusion of malleus to a bony atretic plate, incudomalleolar fusion

Number of Botox exotoxins, and most clinically useful

A: 7 (A to G) exotoxins; A - longest lasting A: Active form has a heavy chain for binding, and a light chain for intracellular toxicity

Six anatomic relationships of the Sphenoid Ostium

A: 7 cm from the Anterior Nasal Spine A: 30o angle from the floor of the nose (Pasha: most reliable) A: 1.5 cm above the upper limit of the choana A: 1/3 up from the choana to the skull base A: Adjacent the posterior border of the nasal Septum A: Inferomedial to the posterior end of the Superior Turbinate

Recurrent Acute Tonsillitis

A: 7 infections in a year A: 5 infections for 2 consecutive years A: 3 infections for 3 consecutive years

Pearls on Tracheal tumors

A: 75% malignant in adults, 90% benign in pediatrics A: Chondroma most common benign lesion in adults A: SCC & Adenoid Cystic most common (75%) malignant tumors; SCC 60% in males; Adenoid cystic 60% in females A: Others adenocarcinoma, chondrosarcoma, rhabdomyosarcoma, small cell, melanoma A: Gentle inhalational induction, rigid bronchoscopy, and endotracheal debridement A: Do not free more than 2 cm of trachea circumferentially if that part remaining (risk of devascularization) A: Tension-free end-to-end anastomosis with coated polyglactin suture A: Strap muscle flap for innominate artery protection A: Postop radiation for SCC & Adenoid Cystic

Age when cricoid is no longer narrowest segment of airway

A: 8 years

Percentage of Otosclerosis which is Bilateral

A: 80%

Likelihood of malignancy for a cystic neck mass in someone >40 years old, and top 3 differential

A: 80% A: Thyroid cancer A: Cancer in Waldeyer's ring (oropharynx or nasopharynx) A: Branchiogenic cancer

Percentage of tumors that occur in the different salivary glands and percentage which are benign

A: 80% Parotid - 80% A: 15% Submandibular - 50% A: 5% Sublingual - 30%

Percentage of pediatric NHL that are high grade

A: 90%

Percentage of thyroid hormone as T4

A: 98%

Describe the Breslow classification and the associated risk of regional metastasis

A: <0.75 mm = 0% A: 0.76-1.5 mm = 25% A: 1.51-4 mm (subdivided into 3a 1.51-3 mm, and 3b 3.01-4 mm) = 60% A: >4 mm = 65%

Techniques for Vessels Mismatch

A: <2:1 - Uneven stitches end to end, dilation A: 2-3:1 - Beveled or spatulation (max angle of beveling is 30 degrees) A: >3:1 - End to side

CSF leak anterior skull base - Size of defect that is suitable for a) mucosa only coverage and b) composite (multilayer) coverage

A: <3 or 5? mm mucosal defect A: >3 or 5? mm mucosal or >2 cm bony defect

SGS at term & premie?

A: <4 mm & <3.5 mm

Pathologies associated with low CD4 counts and prophylaxis

A: <400 cells/uL = TB A: <200 cells/uL = NHL (including sinonasal), PCP - Septra A: <150 cells/uL = Fungal sinusitis A: <100 cells/uL = Kaposi sarcoma, Cryptococcal meningitis, Toxoplasmosis A: <50 cells/uL = Aspergillus, Cryptosporidiosis, CMV, MAC - Azithromycin 3: Hodgkin's is associated with a wide range of CD4 3: Remember in order 400, 200, 100, 50 = TB, NHL/PCP, HHV-8/CCM/TPM, ASP/CSD/CMV/MAC

Stapes reflex thresholds

A: <50 db SNHL - Normal stapes reflex threshold A: 50-80 db SNHL - Elevated stapes reflex threshold A: >80 db SNHL - Likely no reflex A: Represent Cochlear hearing losses; for VIII nerve losses, despite the degree of loss, there is likely an absent reflex, also VIII losses are associated with reflex decay

Criteria for use of adjuvant therapies for RRP

A: >4 surgeries/year A: Rapid regrowth with airway compromise A: Distal multisite spread of disease

Abnormal SP/AP Ratios

A: >50% for EAC probe A: >40% for TM A: >30% for Transtympanic

Define an adequate FNAB for thyroid

A: >6 properly prepared smears with 15-20 groups of well preserved clumps of follicular epithelium (bethesda says at least 6 clusters of at least 10 cells to be considered adequate)

Indication for early surgical intervention for chylous fistula

A: >600 ml in 24 hours A: Persistent >1 week A: Chylothorax A: Cachexia

Eight circumstances in which to screen for primary immunodeficiency

A: >8 new ear infections per year A: >2 serious sinusitis per year A: >2 pneumonias per year, or bonchiectasis A: >2 months on antibiotics without improvement, or patient not responding to treatment as expected A: Recurrent deep skin or organ abscesses A: Persistent thrush after age 1 A: Failure to thrive A: Family history of PID A: Any patient with recurrent, unusual, severe infections

Describe the high grade NHL lesions

A: >90% of children have high grade lesions A: Burkitt lymphoma (small noncleaved) - Diffuse B-cell malignancy, classic starry sky pattern of phagocytic histiocytes and tumor cells A: Lymphoblastic - Immature T-cell origin, small lymphoblasts with round/convoluted nuclei, distinct nuclear membranes, basophilic cytoplasm A: Immunoblastic/Large cell - Heterogeneous group of lymphocytic & histiocytic tumors; 80% of adults are B-cell in origin; in children equal numbers originate from T-cell, B-cell or indeterminate origin

Kadish staging for Esthesioneuroblastoma and 5-year survivals

A: A - Confined to Nasal cavity, 85% A: B - Extension into Paranasal sinuses, 70% A: C - Spread Beyond nasal cavity/sinuses (orbit, BOS, intracranial, neck LNs), 45%

Structures of the superior orbital fissure and tendon of Zinn (Left orbit in this example)

A: A - Optic nerve, ophthalmic artery A: B - CN IV, lacrimal and frontal divisions of V1, superior ophthalmic vein A: C - CN III (possibly in superior & inferior divisions), CN VI, nasociliary division of V1 A: D - Zygomaticofacial and zygomaticotemporal divisions of V2, inferior ophthalmic vein

Define Spondee

A: A 2 syllable word with equal emphasis on both syllables A: Used to determine the SRT

Define Vertigo

A: A Symptom characterized by an illusion of movement of the environment, often rotatory, and often accompanied by dysequilibrium and vegetative symptoms

What is Hyrtl's fissure?

A: A connection between the Subarachnoid space near the Glossopharyngeal ganglion to the Hypotympanum inferior & anterior to the Round Window A: Present in infants, closes off as child grows

Definition of Diplacusis

A: A difference in the perception of pitch between the ears

What is Lupus Pernio?

A: A form of Sarcoidosis characterized by chronic, violaceous cutaneous lesions with a predilection for cold-sensitive areas such as the nose, cheeks, ears, and fingers A: Intranasal involvement is characterized by diffuse nasal crusting, or a vasomotor-like appearance to the nasal mucosa, causing diffuse mucosal swelling

Definition of Rollover

A: A paradoxical Decrease in Discrimination ability with increasing signal intensity, suggestive of Retrocochlear pathology (positive if a 20% decrease in discrim)

Define Allele

A: A particular form of a gene on each chromosome

What is Auditory Neuropathy?

A: A pattern of Hearing Loss suggesting loss of Cochlear Nerve Function A: Audiogram shows SNHL with significantly depressed SDS out of keeping with HL A: Normal OAE & ECoG (cochlear microphonics) testing, Abnormal ABR testing

What is the infection caused by Pseudomonas Pyocyaneus and its sequelae?

A: A perichondritis characterized by diffuse inflammation of the pinna and severe pain A: If not treated aggressively with Antibiotics +/- drainage, loss of cartilage and permanent deformity can result

Seven Criteria for diagnosis of pure Sensioneural Otosclerosis

A: A positive Schwartze's sign A: Family History of surgically confirmed stapedial otosclerosis A: Symmetric Progressive SNHL, with Stapes Fixation in one ear A: Unusually good Speech Discrimination for a pure sensorineural loss A: Onset of hearing loss at usual Age for stapes fixation and progressing without other known etiology A: CT scan in patient with one or more of above criteria that shows Cochlear Capsule Demineralization (a negative scan does not preclude diagnosis because mature disease resemble normal bone on CT) A: On-Off effect of Impedance tympanometry

What is Leser-Trelat sign, and what cancer is most commonly associated with this?

A: A shower of Seborrheic Keratoses as a sign of internal malignancy, most commonly Colonic Adenocarcinoma

Define Onodi cell

A: A sphenoethmoidal cell, a posterior ethmoid air cell pneumatizing into the sphenoid (found in 51% of asians)

Define Oscillopsia

A: A symptom of jumping, blurring, or other movement of the visual scene; if it occurs only with head movements, it points to the possibility of severe bilateral vestibular loss

Describe the Guttman's test

A: A test for SLN paralysis (Cricothyroid paralysis) A: Normally, anterior thyroid cartilage pressure decreases pitch, lateral thyroid cartilage pressure increased pitch; with cricothyroid paralysis this reverses

Initial management strategy for Caustic Ingestions

A: ABCs, ABG, NPO, IVF A: Judicious dilution with water or milk up to 15 ml/kg (do not induce vomiting or give neutralizing agents) A: CXR, Abdominal series A: Esophagoscopy - Timing controversial, between 24-48 hours; insertion of NG tube at time of esophagoscopy; identifies amount of damage, circumferential or not A: Gastrograffin swallow - Initial evaluation if >48 hours postingestion, to identify perforation, repeat at 6 weeks to identify stricture formation A: Steroids most useful for grade 2 injuries (1-2 mg/Kg/day, max 60 mg/day, for 21 days) A: Antibiotics controversial usage empirically (14 days) A: Antireflux medications A: Lathyrogens - Reduce collagen cross binding, to decrease esophageal stricture, unproven benefit (penicillamine, ß-aminoproprionitrile, acetylcysteine) A: Sucralfate - Protects esophageal mucosa against gastric acid, and reduces granulation 3: Presence of oral injury cannot accurately predict presence or absence of more distal involvement

Describe von Hippel Lindau syndrome (HIPPEL)

A: AD A: Hemangioblastomas of CNS & retinas A: renal cysts/carcInoma A: Pheochromocytoma A: Pancreatic cysts A: Epididymal papillary cystademonata A: endoLymphatic sac tumors in 11%

Features of Stickler syndrome

A: AD mutation of COL2A1 gene, chromosome 12, for type II collagen A: Robin sequence A: Eye - Myopia, cataracts, & retinal detachment A: Joint - Hypermobility & enlarged joints, early arthritis, occ. spondyloepiphyseal dysplasia A: SNHL or mixed HL in 80%

Treacher-Collins syndrome (mandibulofacial dysostosis)

A: AD, 60% sporatic A: Mutation in TCOF1 (TREACLE) gene, chromosome 5q A: Malformation of 1st & 2nd branchial arches A: Antimongoloid palpebral fissures, coloboma of the lower eyelids A: Malformed auricules, ossicles, aural atresia, preauricular fistulas, CHL in 30%, occasional SNHL (Mondini) A: Mandibular and malar hypoplasia A: May have cleft lip and palate, choanal atresia A: Normal IQ

Branchiootorenal syndrome (Melnick-Fraser syndrome)

A: AD, EYA1 gene, chromosome 8q A: Branchial cleft anomalies (63%) A: Otologic malformations - Hearing loss (89%), preauricular pits (77%), auricle abnormalities (41%), ossicular & cochlear malformations, lacrimal duct stenosis A: Renal dysplasia (66%) - Agenesis, polycystic kidneys, duplicated ureters 3: Renal abnormalities identifiable on IVP or renal U/S

Apert (Acrocephalosyndactyly), Crouzon (Craniofacial dysostosis) and Pfeiffer syndromes

A: AD, mutations of FGFR-2 gene, chromosome 10q26 A: Craniosynostosis, midface hypoplasia, low nasal bridge, Parrot-beaked nose, choanal stenosis or atresia, mandibular prognathism, high arched palate, bifid uvula, cleft palate, and cervical fusion A: Hypertelorism, exophthalmos, and strabismus A: Cognitively normal to severe mental retardation A: Apert specific - Syndactyly, Stapes fixation (CHL) and patent Cochlear aqueduct A: Pfeiffer specific - Digital broadening

Indication for treatment of OSA

A: AHI ≥5 in presence of symptoms of EDS (excessive daytime sleepiness) A: AHI ≥5 in presence of risk factors - cardiac disease, smoking, HTN, high cholesterol 3: AHI ≥20 - associated with increased mortality (other article says moderate OSA ie. AHI>15)

Describe Cicatricial Pemphigoid

A: AKA Benign Mucus Membrane Pemphigoid (BMMP) A: Subepithelial bullae, Nikolsky sign can be Positive - rubbing away of blister skin/mucosa

Discuss the Causes of Gradenigo syndrome

A: AKA Petrous Apex Syndrome A: Originally due to Extradural Abscess involving Petrous bone A: Today, more commonly caused by Tumor at the Petrous Apex (Cholesteatoma, Meningioma, etc) 3: Abscess results from Petrositis with Inflammation of Dorello's canal between Petrous Tip and Petroclinoid Ligament; Contains CN VI and the Inferior Petrosal sinus

Describe Functional dysphonia

A: AKA Psychogenic dysphonia; related to psychologic issues A: Women > men A: Ddx = Spasmodic dysphonia, secondary MTD, Tremor A: Treatment = THERAPY! - Speech, Psycho, Pharmaco

Three risk profiles scales for well differentiated thyroid cancer

A: AMES (Lahey) - Age, distant Metastasis, Extrathyroidal extension, Size A: AGES (Mayo) - Age, Grade, Extrathyroidal extension, Size A: MACIS - distant Metastases, Age, Completeness of resection, extrathyroidal Invasion, Size

Five Indications for Tympanocentesis/Myringotomy

A: AOM in a seriously-ill, toxic, newborn or immune deficient patient A: Severe pain A: Suppurative complications (facial paralysis, meningitis, etc.) A: Unsatisfactory response to antibiotics A: Suspected unusual pathogen (newborns, immunodeficiency)

Hurler's syndrome (MPS I)

A: AR A: Deficiency of α-1-iduronidase which breaks down heparan-, dermatan- and keratan-sulfates A: Visceromegaly, macroglossia, macrocephaly, progressive neurologic dysfunction, death in 1st decade

What muscles are the main abductor & adductors of the vocal cord?

A: Abductor - Posterior cricoarytenoid, supplied by the abductor branch of the recurrent laryngeal nerve A: Adductors (3) - Lateral cricoarytenoid, Thyroarytenoid/vocalis, Interarytenoid

Ddx of Objective Pulsatile Tinnitus (12)

A: Aberrant Carotid artery A: Persistent Stapedial artery A: High riding/Dehiscent Jugular bulb A: Transverse Sinus Obstruction A: Venous Hum (HTN, Thyrotoxicosis, Pregnancy, Anemia, or Benign intracranial hypertension) A: Congenital ArterioVenous Malformations (Occipital Artery to Transverse Sinus, Internal Carotid to Vertebral Vessel, Middle meningeal to greater superficial petrosal artery) A: Acquired Carotid-Cavernous sinus fistula (Spontaneous,Posttraumatic, or Ehlers-Danlos syndrome) A: Vascular tumors (Paraganglioma, Hemagioma) A: Muscle Contraction tinnitus - Palatal myoclonus (associated with MS, CVA, Intracranial Neoplasms, Psychogenic) A: Paget's Disease A: Atherosclerosis A: Cardiac murmur A: Vascular loops

Pathophysiology & diagnosis of Frey's syndrome (aka. Gustatory sweating)

A: Aberrant reinnervation of postganglionic parasympathetic nerves to the sweat glands of the face A: 10% of patients overtly symptomatic A: Diagnosis: Minor's starch iodine test

Four Advantages of the Modified Radial Mastoidectomy (CWD Mastoidectomy)

A: Ability to deal with the disease in One Setting A: Ability to See Recurrent Disease more easily A: Lower Incidence of Recurrent Disease A: Cost effective

What is the Paracusis of Willis

A: Ability to hear better in the presence of background noise (seen with CHL)

Three advantages of Flexible endoscopes

A: Ability to view the larynx during speech and singing A: Glottic gap can be more accurately described A: Allows the clinician to assess the nasal cavity and velopharyngeal port 3: Preferred when the question is one of movement rather than structure or mucosal health such as in spasmodic dysphonia and vocal-fold motion impairment

High risk groups for Pediatric OSAS

A: Abnormal airway anatomy - Down's, Pierre-Robin, Achondroplasia, Craniosynostoses, Treacher-Collins, Macroglossia, Klippel-Feil (Wildervanck), Pyriform aperture stenosis, Laryngomalacia, Masses A: Neuromuscular disease - CP, Down, Hypothyroidism, Muscular dystrophies, Seizures, Prader-Willi, Chiari-malformations A: Cardiovascular disease - Pulmonary/systemic HTN, Congenital heart disease A: Morbid obesity A: Pectus excavatum/Scoliosis

Two absolute and 1 relative contraindications to surgical correction of Aural Atresia

A: Abnormal inner ear Morphology demonstrated on CT scan A: Abnormal Cochlear function demonstrated by audiologic testing A: Jahrsdoerfer score ≤5/10 (relative)

Name 16 Perceptual voice abnormalities

A: Abnormally High fundamental frequency A: Abnormally Low fundamental frequency A: Abnormally Loud voice A: Abnormally Soft voice A: Decreased Smoothness A: Decreased Intelligibility A: Hypernasality A: Hyponasality A: Hoarseness A: Harshness A: Diplophonia A: Tremorous A: Stridorous A: Breathy A: Arrest of phonation A: Aphonia

Discuss the management of Migraine

A: Abortive medications - Sumatriptan, Ergotamine, Dihydroergotamine, NSAIDs, Lidocaine, Butorphanol, Midring A: Acute therapy - Prochlorperazine, metoclopramide, Dihydroergotamine, Chlorpromazine, Haloperidol A: Prevention - Lifestyle modification to avoid triggers A: Prophylaxis (if >2 migraines/month, ABCDE!) - Antidepressants (mainly TCA's), Beta-blockers, Calcium channel blockers, nsaiDs, Ergotamines

Five types of Stenting

A: Aboulker or Cotton-Lorenz stent (rigid teflon - polytef II, hollow lumen) A: Montgomery T tube (hollow silicone) A: Montgomery laryngeal stent (solid silicone) A: Single stage LTR (ETT used as alternative to stenting) A: Finger cot

Incidence of Cerebral Complications following Common Carotid Occlusion

A: Abrupt ligation = 42% A: Occluded gradually over <7 days = 30.6% A: Occluded gradually over ≥8 days = significantly decreased to 5.3% 3: Unselected ICA interruption - 26% rate of cerebral infarction (depends on patient age)

Good prognostic indicators for SSNHL

A: Absence of Vestibular symptoms A: Low frequency loss (upsloping shape) A: Minimal hearing loss A: Early treatment (within 3 days) A: No changes in N1 latency on ECoG

Three Late findings Audio/Impedance in Otosclerosis

A: Absent Stapedial Reflex A: Maximal CHL - Flat audiogram, ~50 dB, Mass effect A: Type As Tympanogram - from Tympanosclerosis, scarred TM

Six Absolute and 2 Relative Contraindications for Craniofacial Resection

A: Absolute - A: Medical or nutritionally unfit A: Distant metastasis A: Prevertebral fascia invasion A: Invasion of Optic Chiasm or both optic nerves A: Cavernous sinus invasion by high grade lesion A: Carotid invasion in a high risk patient A: Relative - A: Dural invasion A: Intracranial nerve involvement by Adenoid Cystic

Contraindications to airway surgery

A: Absolute - A: Tracheotomy dependent for other reasons (aspiration, severe BPD) A: Severe GER refractory to surgical and medical therapy A: Unfit for GA A: Relative - A: Steroid use A: Diabetes A: Cardiac, renal or pulmonary disease

3 Absolute and 4 relative indications for surgical treatment for compression due to vascular anomalies (AFP RESP)

A: Absolute - Reflex Apnea (from vagal stimulation) A: Absolute - Failure of medical management of Severe respiratory distress after 48 hours A: Absolute - Prolonged intubation A: Relative - Repeated URTI episodes A: Relative - Exercise intolerance A: Relative - Significant FTT & dysphagia A: Relative - Coexisting Pathology (SGS, asthma, CF, or previous TEF repair)

Four tissue effects of lasers

A: Absorption A: Reflection A: Diffraction/Scattering A: Transmission

Classification of Hyposmia/Anosmia (3)

A: Access of odorant changed - Conductive A: Damage to olfactory nerve & receptors - Sensory A: Damage to central olfactory pathways - Neural

Six characteristics of the ideal qualities of the soft tissue component of a vascularized composite free tissue transfer

A: Accessible for a two team approach A: Well vascularized A: Sensate A: Thin and pliable A: Mobile relative to bone A: Minimal morbidity at the donor site A: Lubricated

Manometric findings of achalasia & scleroderma (normal LES pressure is about 20 mm Hg)

A: Achalasia - Increased LES pressure (40-60 mmHg), Aperistalsis, Absent LES relaxation A: Scleroderma - Absent LES contraction (<10 mmHg), Aperistalsis of lower 2/3s of esophagus, Esophageal dilatation A: Dermatomyositis - Aperistalsis of upper 1/3s of esophagus, frequent nasal regurgitation

Two Histologic findings of Sialosis/Sialoadenosis

A: Acinar hypertrophy A: Fatty infiltration A: Combination

Name 4 Low Grade salivary gland malignancies

A: Acinic cell ca A: Low grade mucoepidermoid A: Polymorphous low grade adenocarcinoma A: Low grade Adenocarcinoma A: Clear cell carcinoma A: Adenoid cystic ca

Parotid tumors arising from Acinar cells

A: Acinic cell carcinoma

Thirteen Side effects of corticosteroids

A: Acne A: Behavioral disturbances A: Osteoporosis A: Avascular necrosis of the hip A: Cataracts A: Central obesity/Cushingoid habitus A: Hyperglycemia A: Electrolyte disturbances A: Fluid imbalance A: Glycosuria A: Hirsutism A: Hypertension A: Susceptibility to infections A: Peptic ulcer disease

Five Indications for Facial nerve Monitoring

A: Acoustic neuroma (Standard of Care) A: Skull base surgery (Standard of Care) A: EAC Atresia Repair (Standard of Care) A: Revision mastoid surgery (Standard of Care) A: Mastoidectomy (Recommended) A: Exostosis surgery (Recommended) 3: Not routinely used for Stapedectomy, Tympanoplasty

Ddx of Common CPA lesions (7)

A: Acoustic neuroma - 80%, most common adult lesion A: Meningioma - 5%, 2nd most common adult lesion A: Epidermoid - 3%, 3rd most common adult lesion A: Arachnoid cyst - 1% A: Nonacoustic neuroma - Facial schwannoma most common, 1% A: Paraganglioma A: Hemangioma

How to detect Eosinophilia on nasal smears

A: Acquire sample of nasal mucus (wax paper or swab), smear on slide A: Stain with Hansel stain (Eosin & Methylene Blue) A: Positive criteria = >20% eosinophil content, or if Eosinophils, Mast cells & Goblet cells (EMG) are present

What is the cause of post-extubation stridor occurring after short-term, uneventful intubation?

A: Acquired subglottic cyst A: Usually lateral

Three Premalignant lesions in cutaneous SCC malignancies

A: Actinic keratosis A: Bowen's disease A: Keratoacanthoma (low grade malignant)

A patient has a firm neck mass after dental surgery, FNA shows sulfur granules

A: Actinomycosis 3: Nocardia can also produce sulfur granules; but these are differentiated from actinomyces by positive acid fast staining

Stimulation of the Horizontal SCC causes the activation and inhibition of which muscles?

A: Activation - ipsilateral MR, contralateral LR A: Inhibition - ipsilateral LR, contralateral MR

Stimulation of the Posterior SCC causes the activation and inhibition of which muscles?

A: Activation - ipsilateral SO, contralateral IR A: Inhibition - ipsilateral IO, contralateral SR

Stimulation of the Superior SCC causes the activation and inhibition of which muscles?

A: Activation - ipsilateral SR, contralateral IO A: Inhibition - ipsilateral IR, contralateral SO

Mechanism of action and 4 examples of Antimetabolite chemotherapeutic agents

A: Actively interfere with Cellular Metabolism, act in S-phase 3: Methotrexate - Interferes with Folate Metabolism, inhibits Dihydrofolate Reductase enzyme, interrupts DNA synthesis in S-phase mechanism 3: 5-FluoroUracil - S phase Uracil Analogue, either incorporated into DNA and halts replication or is activated and blocks Thymidilate Synthase 3: Hydroxyurea, Gemcitabine

Timeframe of Rhinosinusitis

A: Acute - <4 weeks (symptom-free interval if Recurrent) A: Subacute - 4-12 weeks (not used much anymore, can be grouped with acute or recurrent acute) A: Chronic - >12 weeks

What are the effects of radiation on tissue healing?

A: Acute - Severity related to the radiation fraction; reduction in proliferation of fibroblasts, myofibroblasts, endothelial cells A: Intermediate - 3-6 months after radiation, diminished endothelial & connective tissue proliferation A: Late - Related to total radiation dosage, hyalinization of collagen, rupture of elastic fibrils, fibrinous exudate deposition, induction of atypical fibroblasts, hyalinization & sclerosis of blood vessels (obliterative endarteritis)

Describe 3 phases of allergic rhinitis

A: Acute - due to Histamine and other preformed/newly synthesized mediators; causes sneezing, itchy eyes, nose and throat, nasal discharge A: Intermediate or transitional - due to recruitment and activation of leukocytes into nasal tissue, asymptomatic A: Late phase - 4-6 hours post exposure, due to leukocytes in nasal tissue; main symptom is nasal congestion

Clinical features and treatment of Letterer-Siwe disease

A: Acute disseminated and rapidly progressive form of Histiocytosis X; affects patients under 3 years of age, uniformly fatal A: Fever, proptosis, adenopathy, hepatosplenomegaly, multiple bone lesions, anemia, thrombocytopenia, exfoliative dermatitis A: Treatment - Chemo-Radiation

Classification of Otitis

A: Acute otitis media (AOM) - 1) Acute Onset of signs & symptoms, 2) Presence of MEE, 3) Signs & symptoms of middle ear Inflammation A: Recurrent Acute otitis media (R-AOM) - ≥3x/6 mo or ≥4x/1 yr; recurrences in <1 mo = Same pathogen, in >1 mo = Different strain A: Otitis media with effusion (OME) - Average 40 days, persistent in <2 yo, caucasians A: Chronic Suppurative otitis media

Two contraindications for sialography

A: Acute sialoadenitis A: Iodine allergy

3 and 4th Branchial abnormalities can manifest as what condition in children

A: Acute suppurative thyroiditis

Six types of Fungal sinusitis classification

A: Acute/Fulminant invasive - Mucormycosis, Rhizopus, Aspergillus A: Chronic/Indolent invasive A: Granulomatous invasive - Aspergillus flavus A: Allergic fungal - Can be divided into AFS/EFS/EMRS/ECRS based on fungal cultures and IgE status; all have polyps and eosinophilic mucin A: Mycetoma - Fungal ball A: Saprophytic fungal infestation 3: Three invasive, and 3 noninvasive

Calculation for Long-Acting Opioids

A: Add up Total Daily Opioid Dose including Regular and Breakthrough, and divide by 2; Prescribe L.A. form q12h

Eleven indications for Botox for laryngopharyngeal pathology

A: Adductor Spasmodic dysphonia (into TA) A: Abductor Spasmodic dysphonia (into PCA) A: Stuttering (into TA) A: Vocal Tic (into TA) A: Mutational dysphonia (Puberophonia) (into Cricothyoid) A: Ventricular dysphonia/Dysphonia plica ventricularis (injection into false cord/aryepiglottic muscle) A: Dysphagia due to cricopharygeal dysfunction (into cricopharygeus) A: TEP speech failure due to cricopharygeal spasm (into cricopharygeus) A: Vocal fold granuloma and prevention of posterior glottic stenosis (into TA- decreases strength of vocal cord closure, tends to take a more abducted position at rest) A: Arytenoid rebalancing (post-dislocation, into adductor muscles on that side) A: Bilateral true vocal fold paralysis (into adductors)

Parotid tumors arising from Intercalated duct cells

A: Adenocarcinoma A: Pleomorphic adenoma

Discuss the components of the pituitary gland

A: Adenohypophysis (Anterior) - Derived from Rathke's pouch (ectoderm); secretes ACTH, FSH/LH, GH, PRL, TSH A: Neurohypophysis (Posterior) - Axons of cell bodies from supraoptic & paraventricular nuclei of hypothalamus; secretes Oxytocin & ADH

Incidence of PTH pathologies in primary hyperparathyroidism

A: Adenoma - 85% A: Hyperplasia - 10% A: Multiple adenoma - 5% A: Carcinoma - <1%

Name 5 viral vectors

A: Adenovirus - DNA, nonintegrating episome; immunogenic A: Adeno-Associated virus - ssDNA, when inserts into 19q13.4 causes B-cell leukemia A: Retrovirus - RNA, Lentivirus, risk of insertional mutagenesis A: Herpes - Large DNA, nonintegrating, contains TK for suicide with Acyclovir A: Vaccinia - Large DNA, pox-virus, nonintegrating, immunogenic

What five conditions must be met for normal phonation to occur?

A: Adequate expiratory force A: Approximation of the vocal folds A: Favorable vibratory properties A: Control of length and tension A: Favorable vocal fold shape

Six characteristics of the ideal qualities of the bony component of a vascularized composite free tissue transfer

A: Adequate length to restore a segmental defect of nearly any length A: Natural shape or easy contourability to match the mandibular defect A: Well vascularized A: Vascular anatomy that is well preserved while contouring the graft A: Sufficient height & width for reliable placement of endosteal dental implants for prosthetic rehabilitation A: No significant functional or esthetic defects at the donor site after harvest

Safe length of time for intubation in adults, children, and neonates

A: Adults - 5-10 days A: Children - up to 50 days A: Neonates - up to 6 months

What is the most common cause of bilateral facial nerve paralysis in Adults and in Children?

A: Adults - Guillain Barre Syndrome (Otorhinolaryngol Nova 2002-03;12:149-152) A: Children - Lyme disease (Am J Otolaryngol 1997;18:320-3)

Name 10 preexisting conditions which increase Susceptibility to Ototoxicity (ACDC R 14 PIGS)

A: Advanced Age (>65 years) A: Coadministration of other known Ototoxic agents (Loop diuretics & Aminoglycosides) A: Elevated peak and trough Drug Levels A: Collagen vascular disorders A: Renal or Liver failure A: Treatment course >14 days A: Prior Ototoxicity A: Immunocompromise A: Genetic - Mitochondrial 12S rRNA gene mutation in Chinese population A: Preexisting SNHL

Summarize the VA study

A: Advanced Laryngeal Cancer, randomized into 2 arms, comparing Surgery + XRT to Chemo + XRT 3: Group 1 = Standard Surgery + Postop XRT 3: Group 2 = Neoadjuvant Cisplatinum + 5-FU, then XRT; but if no (even partial) Response after 2 cycles or Recurrent/Residual disease after XRT => Surgery 3: Overall Survival rate at 2 years Identical = 68% 3: Laryngeal Preservation rate in group 2 = 64%

Summarize RTOG 91-11

A: Advanced Laryngeal cancer, randomized into 3 arms, looking at Laryngeal Preservation, Locoregional Control, and OS rates 3: Group 1 = Concomitant Cisplatinum + XRT => 88%, 78% 3: Group 2 = Neoadjuvant Cisplatinum + 5-FU, then XRT => 75%, 61% 3: Group 3 = XRT alone => 70%, 56% 3: Overall Survival rate was Similar for all groups => 54-56% at 5 years

Three indications for post op XRT for thyroid Ca

A: Advanced locoregional WDTC whether superficial excision is complete or incomplete A: Tumor no longer picks up radioiodine A: Palliation of unresectable bone metastasis

What are the three main types of local flaps

A: Advancement - Monopedicled, Bipedicled, V-Y, or Hinged A: Pivotal - Rotation, or Transposition (Rhomboid, Dufourmental, Bilobed) A: Interpolated - Linear, can be classified as pivotal, but base located some distance away from defect (Paramedian Forehead, Nasolabial, Postauricular)

Discuss the 2 Advantages & 3 Disadvantages of the Middle Fossa Approach for Acoustic Neuromas excision

A: Advantages - CN VIII nerve preservation possible; Ideal for Small Intracanalicular tumors A: Disadvantages - Contraindicated if >1 cm or extension into CPA; Increased Risk to CN VII, significant Temporal Lobe Retraction, More technically Difficult

Discuss the 3 Advantages & 4 Disadvantages of the Retrosigmoid Approach for Acoustic Neuromas excision

A: Advantages - Excellent for Hearing preservation; useful for Larger tumors with Limited lateral IAC involvement, Decreased Risk to CN VII cf. MCF approach A: Disadvantages - Contraindicated if tumor extends to Fundus of IAC; Increased Risk to CN VII cf. TL approach; 10% postoperative Headaches; need for Cerebellar Retraction

Discuss the 4 Advantages & 1 Disadvantage of the Translabyrinthine Approach for Acoustic Neuromas excision

A: Advantages - Safest for CN VII function Preservation; Direct approach to IAC; Wide Exposure not limited by Tumor Size; Minimal cerebellar Retraction A: Disadvantange - Total Hearing Eradication, Increased risk of CSF Leak cf. Retrosigmoid approach

Organisms found in acute sialoadenitis

A: Aerobes - S. aureus, S. viridans/pyogenes/pneumoniae, H. influenzae, E. coli A: Anaerobes - B. melaninogenicus, Strep. micros

Bacteriology of chronic pediatric sinusitis

A: Aerobes: S. pneumonia, M. catarrhalis, H. influenzae, S. aureus, α-hemolytic Strep, P. aeruginosa A: Anaerobes: Peptococcus, Peptostreptococcus, Bacteroides

List the sites of Ototoxic activity for Cisplatin

A: Affects OHC's at Basal turn, Degeneration of Stria and Ganglion cells also noted; possibly due to Oxygen Free Radicals A: Bilateral Symmetric High Frequency SNHL & Tinnitus; usually Permanent

List the sites of Ototoxic activity for Aminoglycosides

A: Affects OHCs from Base to Apex A: Effects usually Irreversible A: Gentamycin & Streptomycin more Vestibulotoxic; Tinnitus common ("Gentlemen wear Striped Vests") A: Kanamycin, Amikacin, Neomycin, Tobramycin more Cochleotoxic ("KANT be more Cochleotoxic")

List the sites of Ototoxic activity for Loop diuretics

A: Affects Stria Vascularis by altering Potassium ion transport, alters Endocochleal Potential A: Reversible (Furosemide) or Permanent (Ethacrynic acid) Cochleotoxicity, with Tinnitus and Vertigo

The 5 "A" of stridor

A: Age A: Acuteness A: Appearance (toxic or non-toxic) A: Acoustics (volume, pitch, phase) A: Associated symptoms (dysphonia, cough, drooling, posturing, dysphagia)

Six factors influencing Skin Endpoint Titration

A: Age A: Antihistamines A: Skin reactivity (eg to trauma, Dermatographia) A: Food allergens A: Increased Allergen exposure - Greater response during allergic season A: Volume injected

Ten host risk factors for AOM

A: Age (<2 yo, first onset <6 mo) A: Gender (Male) A: Race (First Nations) A: Genetic predisposition A: Ciliary dyskinesia A: Adenoids (reservoir of infection & mechanical ET obstruction) A: ET dysfunction (short, horizontal, compliant) A: Cleft palate, Craniofacial abnormality, Down's A: Immune deficiency A: Atopy (disputed)

Rule for choosing the appropriate ETT size

A: Age/4 + 4

Thirteen indications for Moh's surgery in BCC

A: Aggressive Histology or growth pattern - Morpheaform or Keratotic for BCC; infiltrative type, sclerosing and keratinizing (Basosquamous or metatypical), Poorly differentiated SCC A: Incompletely excised tumor A: Tumors with poorly defined Margins A: Immunosuppressed patients A: High risk location - Central face, Ear, Periauricular, Temporal, Periocular, Nasal tip, Ala, Melolabial sulcus, Upper Lip, Chin, Mandible A: Skin cancer in Irradiated skin A: Recurrent tumor A: Tumors with Neurotropism A: Size >0.6 cm in high risk area, or >1 cm on scalp, face, neck A: Young patients, Cosmetically important area A: Tumors potentially involving Vital structures 3: "HIGHER RISC, Cosmetic, Vital, with Positive Margins"

Define Suprabullar recess

A: Air cell space left between the ethmoid bulla and the fovea ethmoidalis when the bulla does not extend up to the fovea

Describe Airflow voice measures

A: Airflow through glottis & subglottic pressure measured, which determines the glottic resistance (Ohms law) A: Vocal efficiency = Acoustic power/subglottic power (pressure)

Six Complications of Peritonsillar Abscess

A: Airway obstruction A: Dehydration A: Spread to other spaces A: Carotid artery erosion A: IJV thrombophlebitis (Lemierre's syndrome) A: Sepsis

Complications of posterior nasal packing

A: Airway obstruction & exacerbation of OSA A: Dyspnea A: Nasopulmonary reflex - Bronchoconstriction, hypoxemia, apnea, and cardiac dysrhythmia A: Sinusitis A: Otitis media A: Toxic shock syndrome A: Septal or alar necrosis

Five indications for Surgery

A: Airway obstruction - BOT macroglossia A: Speech impediment - particularly consonants requiring tip contact w/ alveolar ridge, palate A: Dysphagia - failure to thrive A: Maxillofacial abnormalities - anterior open bite, prognathism, increased ramus-body angle A: Cosmesis

Four primary functions of the larynx

A: Airway protection A: Respiration A: Phonation A: Valsalva (for heavy lifting)

Four Reversible surgical procedures for Chronic Aspiration

A: Airway stenting (eg. Eliachar stent) A: Vertical laryngoplasty A: Supraglottic closure - Epiglottic flap A: Laryngotracheal separation or tracheoesophageal diversion, >90% successful

Define Grisel's syndrome

A: Alanto-axial joint subluxation due to ligament laxity due to inflammation (from RP abscess, adenoidectomy etc.) A: Vertebral body decalcification and laxity of anterior transverse ligament between axis and atlas due to inflammation/infection in the nasopharynx A: Spontaneous subluxation occurs 1 week post op - Pain and Torticollis

Eight investigations other than Ca and intact PTH that should preformed with dx of hyperPTH (AABBCC TUG A PUP!)

A: Albumin A: Alkaline phosphatase A: Bone densitometry A: BUN and CR A: Calcitonin (R/O MEN IIA) A: TFT A: 24 h Urine Ca (R/O familial hypocaluric hypercalcemia) A: Gastrin (R/O MEN I) A: ACE (R/O Sarcoid) A: Phosphorous A: Urine catecholamines (R/O MEN IIA) A: Prolactin (R/O MEN I)

Discuss Staging of Rhabdomyosarcoma of Head and neck in Peds

A: All head and neck (non parameningeal) M0 = Stage 1 A: Parameningeal ≤5 cm = Stage 2 A: Parameningeal >5 cm = Stage 3 A: Any site M1 = Stage 4

ABR findings in Conductive Hearing loss

A: All is shifted to right A: Lower amplitude A: Delayed or absent wave I

Four broad categories of Rhinitis

A: Allergic A: Infectious A: Structural A: Other 3: "All Other Infra-Structures"

Four Conditions that steroid are indicated for the professional voice user

A: Allergic vocal fold edema A: Edema from episodic abuse A: Mild to moderate laryngitis A: Vocal fold hemorrhage 3: Do not give for first time on night of performance

What 10 local factors influence nasal ciliary motility?

A: Allergy A: Rhinitis medicamentosa (chronic abuse) A: Infection (viral, bacterial) A: Primary ciliary abnormalities A: Temperatures <18 C A: Humidity <50% A: Hypertonic or hypotonic solutions A: Dehydration A: Excessive acidic or basic environment (optimal ciliary function @ pH 7.0) A: Mucosal to mucosal surface contact (coapting)

Ddx of Acute Angioedema <6 weeks (AID)

A: Allergy - Food, Drug, Contact, Venom A: Infection A: Drug - ACE Inhibitor 3: Oxygenation, epinephrine, steroids, antihistamines, aminophylline; later avoidance, discontinuing medications, etc.

Reconstruction options for Mandibular defect

A: Alloplast Kirschner wire Steinmann pins Titanium tray, Screws, Plates Acrylic A: Autogenous Free bone Grafts Cortical Cancellous Corticocancellous blocks Particulate cortical bone combined with cancellous marrow A: Free bone Homograph = Bone trays (ie cadavic) A: Pedicled vascularized bone Flaps (5) Rib-Pectoralis Rib-Lat dorsi Scapula-Trapezius Clavicle-SCM Calvarium-Temporalis A: Vascularized bone Free Flaps (8) Fibula Scapula Iliac Rib Radius Humerus Ulna Metatarsus A: Distraction Osteogenesis 3: Anterior - Free Fibular (if minimal soft tissue defect) or Scapula (circumflex scapular artery for large tissue defect) 3: Lateral - No reconstruction of mandible (soft tissue only), Low profile plate, Iliac crest (Descending Circumflex iliac artery), or Scapula

Oxygen free radical scavengers

A: Allopurinol (xanthine oxidase inhibitors) A: Superoxide dismutase A: Vitamins A, C, E A: Deferoxamine A: Glutathione

Oncocytoma Histology

A: Almost exclusive to parotid A: Gross = Slow growing, circumscribed but unencapsulated A: Microscopic = Brown, plump, granular eosinophilic cells with small indented nuclei & high mitochondrial density A: Electon microscopy = Mitochondrial hyperplasia

Pharmacologic action of Phenylephrine

A: Alpha-adrenergic agonist, noncatechol sympathomimetic

What is a Chloroma?

A: Also called Granulocytic Sarcoma, an immature myeloid cell collection seen in AML & CML, green color secondary to myeloperoxidase A: Symptoms - Painless proptosis & violaceous lid swelling over weeks A: Treatment - Systemic chemotherapy, will develop into leukemia without treatment A: Prognosis - Poor, death usually within 18 months

Four Disadvantages of the Modified Radial Mastoidectomy (CWD Mastoidectomy)

A: Altered Anatomy A: Necessity to Clean Mastoid bowl for rest of patient's life A: More frequent episodes of postoperative Discharge A: Greater difficulty achieving Hearing Improvement

Describe voice changes in Essential Tremor

A: Altered Pitch & Loudness A: Phonation Stoppages A: Decreased Intelligibility A: Tremor Absent at rest, attenuated at end of movement, maximal during posturing 3: Treatment - Botox, inject muscles with most tremor

Indications for orbital decompression in Graves orbitopathy

A: Altered vision - Decreasing acuity, field defects, abnormal visual evoked potentials, disk edema; typically after failed course of steroids A: Corneal exposure keratitis not responding to medical therapy A: Cosmetic decompression only if eye findings stable for at least 6 months

Name 3 types of Barotrauma

A: Alternobaric trauma A: Atmospheric inner ear barotrauma A: Inner ear decompression sickness and isobaric gas counterdiffusion sickness

Histologic classification of Rhabdomyosarcoma (APE)

A: Alveolar (20%) - Worst, most common in adolescents, small round cells with fibrous septae A: Pleomorphic (5%) - Most common in adults A: Embryonal/botryoid (75%) - Best, most common in infants and children, spindle shaped cells with eosinophilic cytoplasm A: A fourth class may be Undifferentiated

Layers of bone involved with Otosclerosis

A: Always begins in Endochondral bone A: Later can involve Endosteal and Periosteal layers of bone

Name 3 augmented penicillins

A: Amoxicillin-clavulanate A: Ticarcillin-clavulanate A: Ampicillin-sulbactam

Eight causes of Mikulicz Syndrome

A: Amyloid A: Hypovitaminnosis A: Bulemia A: Chronic fatty infiltration secondsary to EtOH A: Lead and Mercury toxicity A: Lymphadenitis A: Tuberculosis A: Sarcoidosis 3: "And He Bets Even Little Monkeys Love Their Soaps!"

Define Vection

A: An Illusion of self-movement caused by slow continuous movement of the visual surround, with the optokinetic system stimulating the vestibular nuclei and the false interpretation that it is not the visual scene but rather the self which is moving

Definition of Recruitment (Cummings)

A: An abnormal increase in the perceived Loudness produced by relatively small increases in Intensity above threshold, causing a Compression in the Dynamic Range of perceived sound in patients with SNHL A: Observed in the frequencies that are most impaired (usually High frequencies), which also carry critical information for speech understanding

Describe Spasmodic Dysphonia and 3 types

A: An action induced laryngeal motion disorder; a focal dystonia, which may have an associated family history of other dystonias in 12% A: Adductor dysphonia - More common (80%); Hyperactivity of Thyroaryteniod/vocalis muscle complex; choked, strained or strangled voice with abrupt initiation & termination causing short phonation breaks; vocal tremor, slow speech rate & decreased speech smoothness, reduced loudness, reduced speech intelligibility, normal whispering, singing or laughing can all be present; Botox - initial dose 1-1.25U, injection via cricothyroid membrane A: Abductor dysphonia - More difficult to treat; Hyperactivity of posterior cricoarytenoid muscle; breathy, effortful voice quality with abrupt termination, causes aphonic segments of speech; vocal tremor often seen, reduced loudness, reduced speech intelligibility, abnormal whispering; Botox - initial dose 3.75U A: Mixed dysphonia

Define Epipericardial ridge, 4 muscles and 2 nerves which are derived from it

A: An elevation of mesoderm separating the developing pharyngeal region from the embryonic pericardium A: Forms SCM, Trapezius, Lingual and Infrahyoid musculature A: Nerves present include XI and XII

What is the Foramen of Huschke?

A: An embryologic remnant that forms a defect which allows passage of Infection or Neoplasm from the Medial EAC into the Preauricular Parotid tissues and Glenoid fossa A: Management of OE, 4 basic principles A: Thorough Cleaning & Debridement A: Antibiotic treatment - Topical +/- Oral depending on severity A: Instruction of the patient - Avoid instrumentation, Keep ear Dry A: Analgesia - Depending on severity

Define Haller cell

A: An infraorbital ethmoid cell, pneumatizes into the maxilla

Five Circuitry options for Hearing Aids

A: Analog - Acoustic to Mechanical to Electrical signal and back to Sound waves A: Digitally controlled Analog - Computer control, more options in smaller size A: Digital signal processing A: Linear Amplification - Ratio of Input to Output always remains one; uses Peak Clipping, Distortion a problem at higher frequencies A: Compression - Limits output within the Dynamic Range of the user in an Non-Linear fashion

Discuss Acoustic Rhinometry

A: Analyzes 10 msec sound pulses reflected from the airway, can accurately identify the minimal cross-sectional area (MCA), the location of the MCA, and the cross-sectional area at various distances from the nostrils A: I-Notch - Isthmus of the internal nasal valve, within 2 cm of nostril, usually narrowest = 0.73 cm2 A: C-Notch - anterior inferior Concha, ~3.3 cm from nostril A: Does not rely on nasal airflow, but cannot measure the effects of narrow regions on airflow dynamics or resistance 3: Three areas of nasal resistance - Vestibule (1/3), Valve (1/2), Turbinated cavity

Accuracy of FNAB at identifying each thyroid tumor histology

A: Anaplastic & Medullary = 90% A: Papillary = 80% A: Follicular = 40%? 3: FNAB cannot diagnose follicular cancer because Vascular &/or Extracapsular invasion are required on pathology to make this diagnosis

Describe the Nasal valve angle

A: Angle between nasal septum and caudal edge of upper lateral cartilage A: Normally 10-15 degrees

List the 4 types of Perennial Allergens

A: Animal Danders A: Dust mite A: Cockroach allergens A: Mold (notes say only outdoor, not black mold?)

Contents of Jugular Foramen

A: Anterior - Inferior petrosal sinus, CN IX and Jacobson's nerve (passes through inferior tympanic canaliculus between jugular and carotid foramina A: Middle (pars nervosa) - CN X/XI A: Posterior (pars vasculara) - IJV, meningeal branches from occipital and ascending pharyngeal arteries, Nodes of Krause

Boundaries of the Infratemporal Fossa

A: Anterior - posterior surface of the maxilla A: Superior - greater wing of the sphenoid bone, foramen ovale & spinosum A: Medial - lateral pterygoid plate, pterygomaxillary fissure A: Posterior - articular tubercle of temporal bone, spine of sphenoid bone A: Lateral - coronoid process and ramus of the mandible A: Inferior - alveolar border of the maxilla

Boundaries of the pterygopalatine fossa

A: Anterior - posterior wall of the maxilla A: Posterior - sphenoid bone, base of pterygoid process, inferior portion of anterior aspect of greater wing A: Medial - perpendicular plate of Palatine bone A: Superior - undersurface of sphenoid bone and orbital process of palatine bone, opens into inferior orbital fissure A: Lateral - Pterygomaxillary fissure

Two Surgical Approaches to the Anterior Cranial Fossa

A: Anterior Craniofacial Resection - Bifrontal Craniotomy combined with a Transfacial exposure of Nasal cavity, Ethmoid, Maxillary & Orbital areas A: Basal Subfrontal - Similar to Craniofacial approach; Less Extensive Transfacial exposure, target area is Sphenoid & Clivus

Internal carotid artery branches supplying the internal nose

A: Anterior Ethmoid artery A: Posterior Ethmoid artery 3: Branches off the Ophthalmic artery, each divides into a medial branch (Little's area & septum) and a lateral branch (superior & middle turbinates)

Anatomy of Internal Acoustic Canal

A: Anterior Superior - Facial nerve A: Anterior Inferior - Cochlear nerve A: Posterior Superior - Superior Vestibular nerve A: Posterior Inferior - Inferior Vestibular nerve A: Bill's Bar (Crista Verticalis) - Superior ridge A: Falciform Crest (Crista Falciformis) - Horizontally divides Upper and Lower halves 3: Remember, Anteriorly: "Seven Up, Coke Down"

Dividing line between the Pars Flaccida and Tensa

A: Anterior and Posterior Mallear folds A: Also dividing the Mesotympanum and Epitympanum

Mosher's 5 landmarks for surgical approaches to the infected neck

A: Anterior border of SCM A: Lateral tip of greater cornu of hyoid A: Cricoid cartilage A: Posterior belly of digastric A: Styloid process

Define the Holman-Miller sign

A: Anterior bowing of posterior maxillary sinus wall A: Indicates JNA invasion of pterygopalatine fossa

Define the Iter Chordae Anterior

A: Anterior canal that the Chorda Tympani enters the Petrotympanic fissure through A: AKA Canal of Huguier

Most common techniques for laryngotracheal reconstruction (LTR)

A: Anterior cartilage graft + tracheotomy + no stent A: Short term stenting (4-6 weeks) + cartilage grafts (anterior &/or posterior) A: Long-term stenting (several months) +/- cartilage graft A: Single stage LTR (SSLTR) - cartilage grafts + brief period of nasotracheal intubation (7 - 10 days for ant graft, 10 - 14 days for post graft, older = shorter)

Four Indications for VPL/laryngoplasty in early glottic carcinoma

A: Anterior commissure involvement A: Extension to the vocal process of the arytenoid A: Select superficial transglottic lesions A: Carcinoma recurring after radiotherapy

Blood supply to the Anterior crus of the Stapes

A: Anterior crural artery - From anastomosis of Superior & Inferior tympanic arteries

Name the three classes of internal derangements of the TMJ

A: Anterior displacement with reduction on opening the mouth - Clicking & popping, can be painless or painful; treat pain with NSAID's, bite-block appliance, muscle relaxant prn for pterygoid spasm A: Anterior displacement without reduction on attempted mouth opening - Locking; treat with arthroscopic lysis & lavage, or open mobilization or removal with replacement A: Disc adhesion to the articular eminence - Limitation of mouth opening; treat with arthroscopic lysis of adhesions & lavage 3: Scale correlates with outcome prognosis; Earlier the click, the less displaced the disk

Embryologic spaces/structures of note in Glioma/Encephalocele formation

A: Anterior neuropore A: Foramen Cecum (between frontal and ethmoid) A: Prenasal Space (between nasal bones and cartilaginous septum) A: Fonticulus Nasofrontalis (between frontal and nasal bones)

Nerve ganglia for anterior tongue/base of tongue/hypopharynx involved in referred otalgia

A: Anterior tongue - V3, from Gasserian ganglion A: Posterior tongue - IX, from superior petrosal ganglion and inferior ganglion A: Hypopharynx - X, from superior (jugular) ganglion and inferior (nodose) ganglion

Surgical options for Macroglossia

A: Anterior wedge A: Tip amputation A: Dorsum and tip A: Central reduction A: Block excision central & tip or Keyhole A: Tip preservation - Kruchinsky A: Horizontal filleting A: Dorsal flap

Boundaries of Prussak's space

A: Anterior, Posterior and Superior - Lateral Malleal fold A: Inferior - Lateral process of Malleus A: Medial - Neck of Malleus A: Lateral - Pars Flaccida

Approaches to PCA muscle for Botox injection in Abductor spasmodic dysphonia

A: Anterior/transcricoid approach - via Cricothyroid membrane, then thru posterior cricoid ring A: Lateral/retrocricoid approach - through inferior constrictor 3: Confirmation of PCA location by EMG with sniffing

Boundaries of the Oral cavity

A: Anteriorly - Vermilion border A: Posteriorly - Posterior border of hard palate, intersection of retromolar trigone and anterior pillar, circumvallate line of the tongue (sulcus terminalis)

Three clinical indicators of a hyperfunctional laryngeal dysphonia on indirect laryngoscopy

A: Anteroposterior compression of the larynx A: Aryepiglottic fold contraction A: False vocal cord compression (lateral compression?)

Mechanism of action of Cetuximab (Erbitux)

A: Anti-EGFR Monoclonal Antibody, inhibits the receptor by Blocking the Ligand Binding site

Classes of Drugs causing Dizziness

A: Antiepileptics (Dilatin, Tegretol) - Cerebellar Toxicity A: Antihypertensives - Hypotension, Decreased CBF A: Anticoagulants - Hemorrhage in Inner ear/Brain A: Aminoglycosides, Cisplatin - Ototoxic A: Tranquilizers (BZDs, Barbiturates, TCA, Alcohol) - CNS Depression 3: "3 Aunties And me Sister Tranquie make me Dizzy!!!"

Treatment options Frey's syndrome

A: Antiperspirant A: Topical anticholinergic (Scopolamine cream, glycopyrrolate, or atropine) A: Botox A: Typanic neurectomy?

Opera singer with good vocal hygiene and Acute Laryngitis; 6 Recommendations

A: Antitussive agent (if cough is a problem) A: Use feel of singing rather than sound quality A: Sing in normal, unforced voice without whispering A: Educate about increased risk of vocal cord hemorrhage that could cause permanent damage A: Follow-up after performance, and counsel as to when normal activities can be restarted

Six categories of Medical Treatment for Sudden SNHL

A: Antivirals - Famvir 500 mg tid, Valacyclovir 1000 mg tid, Acyclovir, Amantidine x 7 days A: Antiinflammatories - Steroids 1 mg/Kg PO qd x 5days and taper (Consent, and give with PPI) A: Vasodilators - Histamine, Carbogen (95% O2 + 5% CO2), Niacin, Nicotinic acid, Papaverine, Procaine A: Rheologics - Dextran, Pentoxiphylline, Heparin, Warfarin A: Diuretics A: Triiodobenzoic acid - Assists Stria in maintaining Endocochlear Potential 3: Surgery - Perilymphatic Fistula closure if suspected; Mnemonic = "A Very Angry Rhino Talks Dirty!"

Nine indications for surgery with primary hyperparathyroidism (1990 NIH consensus statement)

A: Any overt Manifestations of hypercalcemia: Muscle weakness, Osteitis fibrosa cystica, Renal calculi +/- radiograph, Life threatening hypercalcemic crisis, Psychiatric problems A: Serum calcium >3 mmol/L or >0.25 mmol/L above normal (>12 mg/dL or >1 mg/dL above normal) A: Hypercalciuria (>400 mg/24h) A: Decreased creatinine clearance (>30%) (Cr Clearance/GFR < 60) A: Osteoporosis >2.5 SDs below normal on bone density scan A: Age <50 years A: Consistent followup unlikely A: Patient requests surgery A: Coexistent illness complicates management

Define Tumor

A: Any swelling from whatever cause

Definitions of Apnea, Hypopnea, AHI, RDI

A: Apnea - Cessation of ventilation ≥10 seconds that leads to an arousal; can be Obstructive (chest wall movement with above), Central (no chest wall attempted ventilation), or Mixed A: Hypopnea - Episode of Hypoventilation ≥10 seconds, with 30-40% decrease in airflow and ≥4% decrease in oxygen saturation, leading to an arousal A: Respiratory Effort-Related Arousal (RERA) - ≥10 seconds of progressive negative esophageal pressure leading to an arousal, but without an apnea or hypopnea A: Apnea Hypopnea index (AHI) - Average number of apneas and hypopneas in an hour period; >5 significant A: Respiratory distress index (RDI) - Number of apneas & hypopneas (also includes RERA's and in some sleep labs even includes other random things like PLM's etc so more nebulous) in an hour period; >10 significant (Baileys)

Criteria for abnormal polysomnogram in pediatric OSAS

A: Apnea index >1 (lasting longer than two consecutive breaths) A: Oxygen desaturation to <92%, >4% x >3 times/hour, or associated with a change in heart rate >25% A: End tidal CO2 >50mmHg >8% of total sleep time, or >45mmHg >60% of total sleep time

Criteria for 72 hour observation for AOM

A: Appropriate F/U A: Otherwise healthy children (see notes) A: Abx started if SSx persist/worsten 3: Healthy = no immune deficiency/craniofacial anomalies, treatment failures/relapse within 30 days, co-existing acute sinusitis/streptococcal pharyngitis, underlying chronic OME, complicated AOM

Treatment strategy for osteomyelitis

A: Appropriate culture directed antibiotics A: Debridement of foreign bodies/sequestra/necrotic tissue A: Adjuncts to increase oxygenation (trephination, decortication, HBO) A: Reconstruction after infection clears

Which has worse symptoms and prognosis: Congenital or Aquired SGS?

A: Aquired 3: Congenital tends to improve with growth of the child

Wavelengths of lasers

A: Argon 488 & 514 nm A: KTP 532 nm A: ND-YAG 510 & 532 nm A: Copper vapor 511 & 578 nm A: Flash lamp pumped dye laser (tunable yellow) 580 nm A: Sclerolaser 600 nm A: Argon tunable dye laser (tunable red) 633 nm A: Q-switch Alexandrite 755 nm A: Q-switch Ruby 694 nm A: ND-YAG 1064 nm A: Erbium-YAG 2940 nm A: CO2 10600 nm 3: "Are Katie ND Copper Flashing Scary Alex Ruby ND EbCOt?" 3: Vascular, Tattoos (Red, Orange, Green, Blue, Black), Hair, and Skin

Course of a persistent Stapedial artery

A: Arises from ICA in Hypotympanum A: Runs up between crura of Stapes then into Fallopian canal A: Continues to the Geniculate ganglion, supplies Dura

Three invasive localization studies for parathyroid adenoma

A: Arteriography A: Digital subtraction angiography - for multiple gland disease in revision cases, 60-70% sensitive A: Selective venous catheterization A: U/S-guided FNA

Neurovascular supply of the Lateral Thigh fasciocutanous free flaps

A: Artery - 3rd perforator of the Profunda Femoris (deep femoral artery), travels in intermuscular septum A: Nerve - Lateral Femoral Cutaneous nerve of the thigh

Neurovascular supply of the Scapula osseous composite free flaps

A: Artery - Circumflex Scapular artery off of Subscapular artery, divides into Transverse & Descending branches to supply two separate skin paddles (scapular & parascapular respectively) A: Venous - Vena comitantes A: Bone - Separate Thoracodorsal blood supply to bony component (Angular branch), 10-12cm length from inferior lateral aspect of bone 3: Not suitable for osseointegration

Neurovascular supply of the Iliac Crest osseous composite free flaps

A: Artery - Deep Circumflex Iliac artery off of External Iliac artery; (just above inguinal ligament), Internal Oblique supplied by ascending branch of DCIA A: Venous - Deep Circumflex Iliac vein 3: Natural shape conforms to that of native mandible (use ipsilateral hip)

Neurovascular supply of the Anterolateral Thigh fasciocutanous free flaps

A: Artery - Descending branch of the Lateral Femoral Circumflex artery, artery descends between the vastus lateralis and rectus femoris muscles

Neurovascular supply of the Dorsalis Pedis osseous composite free flaps

A: Artery - Dorsalis Pedis artery A: Nerve - Superficial Peroneal nerve 3: Thin sensate cutaneous flap from dosal foot; 2nd metatarsal included for osseocutaneous flap

Neurovascular supply of the Lateral arm fasciocutanous free flaps

A: Artery - Posterior Radial Collateral artery A: Venous - Posterior Radial Collateral vein A: Nerves - Posterior Cutaneous nerve of the Arm/Forearm

Neurovascular supply of the Radial Forearm fasciocutanous free flaps

A: Artery - Radial artery; perforators travel in lateral intermuscular septum (brachioradialis-flexor carpi radialis) A: Venous - Venae comitantes or Cephalic vein A: Nerve - Lateral Antebrachial Cutaneous

Neurovascular supply of the Gracilis myocutaneous free flaps

A: Artery - Terminal branch of Adductor artery, which arises from Profunda Femoris A: Vein - Venae comitantes, join or drain separately into Profunda Femoris vein A: Nerve - Anterior branch of the Obturator nerve (motor supply) 3: Primary use is for facial reanimation

Three methods of Voice post-laryngectomy

A: Artificial Larynx - Pneumatic vs. Electronic devices = Transcervical (Electrolarynx), Transoral (Cooper-Rand), or Intra-oral (Ultra Voice) A: Esophageal speech - Low fundamental frequency (65Hz), short duration (80cc air), effort to produce Injectional technique - air pushed into esophagus Inhalation technique - air inhaled (sucked) into esophagus A: Tracheoesophageal puncture - Primary (at time of resection), Secondary

Five components of the Cricoarytenoid unit

A: Arytenoid cartilage A: Cricoid cartilage A: Associated musculature A: Superior laryngeal nerve A: Recurrent laryngeal nerve for that unit

Five Most common feeding vessels of Paragangliomas (AAO MI)

A: Ascending Pharyngeal (most common) A: PostAuricular A: Occipitial A: Internal Maxillary A: Ipsi or Contralateral ICA (Caroticotympanic)

Four branches of the thyrocervical trunk

A: Ascending cervical - supplies prevertebral muscle region (and nasopharynx/adenoids) A: Transverse cervical - supplies the trapezius muscle (lower island flap) A: Inferior thyroid - to inferior pole of thyroid gland A: Suprascapular 3: Originates off of the 1st portion of the subclavian artery

What fungi are most commonly seen in allergic fungal sinusitis? (AABCCDEF)

A: Aspergillus (most common) A: Demitaceous species - Alternaria, Bipolaris, Curvularia, Cladosporium, Dreshleria, Exophilia, Fusarium

Name 5 side effects of Botox injection

A: Aspiration A: Weak breathy voice A: Sore throat A: Blood tinged sputum A: Hyperventilation and dizziness

Anticoagulation in free flaps

A: Aspirin - Inhibits cyclooxygenase, interferes with PGH2 synthesis; 81 or 325 mg A: Heparin - Binds to antithrombin III, inactivated thrombin, other esterases; increases electronegative potential of endothelium, thus decreases platelet adhesiveness A: Dextran - Antithrombin & antifibrin effects, Either 40 cc of Dextran 40 followed by 25 cc/h x 5 days; or 500 cc followed by 500 cc/d x 3 days A: Medicinal leech - Indication = venous congestion; produce Hementin (local anesthetic), Hirudin (inhibits fibrinogen to fibrin conversion); Risk = Aeromonas hydrophila - gram negative ß-lactamase producing commensal organism, causes necrotic soft tissue flap infection in 10-15%, cover with prophylactic Tetracycline?

In the Audiologic work up of a patient with Unilateral Tinnitus, 5 findings suggestive of Retrochoclear pathology

A: Asymmetric SNHL A: Disproportionate decrease in SDS A: Loss of Acoustic Reflexes or Positive Reflex decay A: Rollover A: Abnormal and Delayed wave V on ABR

Five Audiologic findings suggestive of a Retrocochlear lesion

A: Asymmetric SNHL on pure tone testing (>15 dB in 2 f except 6 and 8 KHz or >30 in 1 f) what about noise induced, can be asymmetrical cochlear loss? A: SDS asymmetry of 12-20% or unusually poor compared to PTA A: PiPb rollover >20% A: Absent stapedial reflex (with CN VII loss >40 dB) A: Positive reflex decay test 3: Other methods include ABR, Bekesy types 3 & 4 curves, Carhart tone decay >30 dB above threshold before properly hearing the tone

Three findings with SLN injury

A: Asymmetric VC Tension - Bowed cord A: Asymmetric VC Height - Inferior displacement A: Ipsilateral rotation of posterior glottis 3: PPP rule = Posterior commissure Points to Paralyzed side in unilateral SLN paralysis

Discuss Alternobaric trauma

A: Asymmetrically increased middle ear pressure due to decreasing Ambient pressure, most prominent in Ascent while diving or flying A: Relieved by equilibration of middle ear & ambient pressure A: Vertigo, Hearing loss & Tinnitus of short duration (10-15 minutes)

Where do Acoustic Neuromas Originate?

A: At Schwann cell-Glial junction (Obersteiner-Redlich zone)

Where is the lesion in Vertical Downbeating nystagmus?

A: At the Cranio-Cervical Junction

Where is the lesion in Periodic alternating nystagmus?

A: At the Dorsal medulla or cerebellum

Where is the lesion in Vertical Upbeating nystagmus?

A: At the Ponto-Medullary Junction or Fourth Ventrical

Indications for bone conducting hearing aids

A: Atresia/microtia A: Abnormally small EACs A: Chronic otorrhea 3: For situations where air conducting hearing aids do not fulfill the amplification needs for CHL

Seven strategies for voice therapy (ABPR4)

A: Attack A: Behavior A: Pitch A: Rest A: Relax A: Respiration A: Rate

Five Differences between Meniere's and Recurrent Vestibulopathy

A: Attacks of vertigo similar to those of Ménière's disease, however, without audiologic symptoms such as, tinnitus, hearing loss, or a sensation of fullness in the ear A: Mean age of onset is 37 years vs 45 years for Ménière's A: Sex distribution is equal vs 1.3x Female predominance in Ménière's A: Unilateral caloric weakness in 22% of these patients vs 50% in Ménière's A: Diuretics produce no improvement

Five Tests useful in the diagnosis of SCDS and their findings

A: Audiometry - Supranormal BC, Mild-Moderate CHL (mimicking otosclerosis), Normal discrimination A: VEMP - Depresses threshold stimulation to less than 80 dB (reflex at 70 shouldn't be there) A: ENG - Normal, performed to R/O other pathology A: Stapedial Reflex - Normal, performed to R/O otosclerosis or other CHL which would abolish the reflex A: High resolution CT temporal bone - 0.5-1.5 mm coronal slices, cannot differentiate dehiscence form thin bone (0.1 mm in 1.5%); Poschl view = Parallel to SSC, Stenver's view = Perpendicaular to SSC 3: Accurate diagnosis relies on CT AND Clinical findings

Methods of Voice Assessment

A: Audiotape & videotape Recording (speech therapist) A: Parent & child Questionnaire A: Fiberoptic laryngoscopy with video-stroboscopy A: Spectral voice Analysis = Multi-Dimensional Voice Program (MDVP)

Describe the 6 Nerves Innervating to EAC and Auricle (and Sources of Referred Otalgia)

A: Auriculotemporal nerve (V3) - Anterior auricle, Tragus, Anterior EAC (TMJ, Dental, Parotid path) A: Posterior Auricular nerve (VII) - Posterior surface of auricle, Posterior EAC (CPA tumors, VZV, Geniculate neuralgia) A: Jacobsen's nerve (IX) - Medial surface of TM, Promontory, Eustachian tube (Eagle syndrome, Pharyngeal tumor, Tonsillitis/Pharyngitis, Sinusitis) A: Arnold's nerve (X) - Lateral surface of TM, Floor of EAC, Concha (Laryngeal tumor, GERD, Thyroid path) A: Great auricular (C2-3) - Inferior preauricular, Medial postauricular, and Posterior auricle A: Lesser occipital (C2-3) - Mastoid region (Cervical spine degenerative disease, Whiplash/trauma, Cervical Meningioma) Jaber JJ et al. Otolaryngology-Head and Neck Surgery (2008) 138, 479-485

Ddx of Chronic Angioedema >6 weeks (ASTICA)

A: Autoimmune A: Systemic disease related A: Thyroid disease A: Idiopathic A: C1-esterase inhibitor deficiency A: Angioneurotic - Quinke's edema

Ddx of recurrent Parotid swelling (Bailey)

A: Autoimmune "pseudosialectasis" - A: Mikulicz's disease - Localized to parotid gland, adult and pediatric A: Sjogren's - Primary, Secondary A: Nonautoimmune - A: Recurrent sialoadenitis A: Sialosis/Sialoadenosis A: Mikulicz's syndrome A: Multinodular gland

Four indications for sialography

A: Autoimmune disease - Sjogren's, Mikulicz's syndromes A: Suspicion of sialoadenitis - chronic, recurrent or nonspecific (NOT ACUTE!) A: Sialolithiasis A: Postop or Posttraumatic fistula/stricture/cyst

Describe Gardner syndrome (FOLLE Thyroid & Colon)

A: Autosomal Dominant A: Fibromas A: Osteomas - Mandible and skull, maxilla and long bones; often first sign A: Lipomas A: Leiomyomas - Stomach & ileum A: Epidermal cysts A: Thyroid Ca - Well differentiated A: Polyposis of Colon and rectum - 40% malignant degeneration potential

Genetics & Epidemiology of Otosclerosis

A: Autosomal Dominant with Incomplete (40%) Penetrance A: 10% of Caucasians have Histologic changes, only 12% of these people are Symptomatic A: Clinical manifestations have 2:1 Female predilection; Histologic changes are 1:1 A: Contralateral ear affected in 80% of cases A: 60% of people report Family History

Describe Cowden's disease

A: Autosomal Dominant; Multiple hamartoma syndrome A: Thyroid malignancy - Well differentiated and Adenoma A: Greatly increased risk of Breast Cancer A: Hamartomas of skin, breast, thyroid & GI tract A: Trichilemmomas on face A: Gingival Fibromatosis; Oral papules involve lips, tongue (cobblestone appearance) 3: "The Beast Has Three Fingers"

Discuss Mobius syndrome

A: Autosomal Dominant; possibly some Recessive A: Bilateral Facial paralysis (Complete) A: Clinical - External Ear Deformities, Abducens (VI is second most common CN affected) & Tongue paralysis, other CN palsies, Hands/Feet sometimes missing, Clubfeet, MR, mixed HL A: Poland-Mobius syndrome - Congenital absence of Pectoralis Muscles

What is hyperparathyroidism-jaw tumor syndrome?

A: Autosomal dominant A: Hyperparathryroidism - solitary or multiple adenomas, 10% will present as parathyroid carcinoma; severe hyperCalcemia as a teenager A: Jaw - cemento-ossifying fibromas A: Renal - cysts, hamartomas, Wilm's tumor

Describe the Nevoid Basal Cell Carcinoma syndrome (Gorlin syndrome)

A: Autosomal dominant (9q), rare A: Multiple (hundreds) small cutaneous nodules appear in childhood, as child ages, multiple basal cell carcinomas develop and lesions become aggressive - invasion, destruction, mutilation A: Nine other characteristics - Frontal bossing, Falx cerebri calcification, Mental retardation, Cataracts, OKCs in 5%, Scoliosis, Bifid ribs, Pitting of the palms and soles

Describe the 3 common forms of Mendelian inheritance

A: Autosomal dominant inheritance A: Autosomal recessive inheritance A: Sex-linked inheritance

Discuss MEN I (Wermer syndrome)

A: Autosomal dominant, altered tumor suppressor gene (menin) found on chromosome 11 A: Pituitary lesions (prolactinoma), Parathyroid adenomas/hyperplasia, Pancreatic islet cell tumors (gastrinoma)

Discuss MEN II

A: Autosomal dominant, caused by RET mutation on chromosome 10 (tyrosine kinase), 5 different point mutations identified A: Medullary thyroid carcinoma, Pheochromocytoma (in ~50% of patients) plus: IIA (Sipple) - Parathyroid adenoma/hyperplasia & hyperparathyroidism (10-30% (KJ) or 85% of patients) IIB (Wagenmann-Froboese) - Marfanoid habitus, multiple mucosal neuromas, hyperplastic corneal nerves

Three approaches to Treatment of Allergy

A: Avoidance - Mattress/pillow covers are best measure for mites; removing carpets also useful A: Pharmacotherapy -Nasal Steroid (1st line therapy, spray or inferior turbinate injections), Antihistamines, Leukotriene Receptor Antagonists, Cromolyn (topical mast cell stabilizer, decreases influx of calcium), Decongestants (topical or systemic), Systemic steroids (short course) A: Immunotherapy

Management of SCDS and Complications

A: Avoidance of stimuli - Ear plugs; PET may be offered but generally not effective A: Subtemporal craniotomy - Weigh risks against severity; Carefully raise middle fossa dura and avoid suctioning to prevent trauma to exposed membranous labyrinth, Plugging of SSC with fascia or bone wax, Resurfacing with fascia or bone (ineffective without plugging) A: Complications of surgery - Epidural hematoma (use penrose), high frequency SNHL, vestibular dysfunction (can be prolonged in patients undergoing bilateral plugging - 30% of pts radiologically - so only do more severe side first)

Six treatment options for Lichen Planus

A: Avoidance of triggers A: Improve oral hygiene A: Steroids - oral or topical (Orabase/Kenalog) A: Cyclosporin 500 mg qd x 4-8 weeks A: Cryotherapy A: UV light A: Laser surgery

Five advantages of Z-plasty

A: Axis of scar changes A: Lengthens scar A: Tension redistribution A: Defect Closure A: Tissue brought in

Discuss humoral immunity

A: B-cells bear surface receptors (Fc) similar to Ig's A: Do not need joint recognition of self markers (MHCs) and antigen like T-cells A: T-dependent activation - Antigen is bound, processed and expressed with surface MHC II; T-cell expresses gp39 which binds to B-cell CD40, which causes cell proliferation and plasma cell differentiation A: T-independent activation - Inefficient, provides mainly IgM; antigens include carbohydrates from bacterial capsule & cell wall

General management approach for Aural Atresia

A: BC ABR/ECoG - For Unilateral to confirm other ear is normal, and for Bilateral to evaluate cochlear function A: High resolution CT of the temporal bone at age 5-6 A: Uniateral microtia & aural atresia - Do not reconstruct canal, as speech & learning will develop normally A: Bilateral aural atresia - Begin with the Better developed ear as child approaches school age, and depending on the results, may proceed with second ear within next few years A: Defer surgery to 6-8 years, allows growth of contralateral ear (template) and rib cage (cartilage graft source) A: Bone conduction hearing aids for Bilateral cases (start with soft band, dont' do BAHA until skull thick enough - at about 5 years old or skull 2.5mm thick)

What are the tumors treatable with Mohs' surgery?

A: BCC A: Cutaneous SCC A: Selected mucosal SCC A: Bowen's disease (Queyrats on the glans penis) A: Keratoacanthoma A: Verrucous carcinoma A: Microcystic or other adnexal neoplasms A: Dermatofibrosarcoma protuberans A: Malignant fibrous histiocytoma A: Atypical fibroxanthoma A: Extramammary Paget's disease A: Merkel cell carcinoma A: Cutaneous rhabdomyosarcoma 3: "BCCs, SCCs, Premalignants, Adnexals, MCC, DFSP, MFH, AFX, cRMS, EMP"

Six Initial Evaluations of Tinnitus

A: BP check both arms A: Bloodwork - CBC, Chem7, TFT (TSH etc.), Lipid profile A: FTA-ABS A: Audiometry 3: If unilateral, work up same as Asymmetric HL (MRI)

Condition in immunocomprimized patients with B. henselae infection

A: Bacillary angiomatosis (tumour like mass of blood vessels forming in the skin)

Best imaging study for Laryngeal Cleft

A: Barium swallow

Name four mechanisms of Innate immunity in the nose

A: Barrier - Epithelial tight junctions A: Enzymes/Peptide antibiotics - Locally secreted in mucus; Lactoferrin, Lysozyme, IgA A: Phagocytes - Neutrophils and macrophages A: PAMP Receptors - Expressed on epithelium and phagocytes, detect Pathogen Associated Molecular Patterns, cause secretion of immune mediators

Seven types of Skin Cancer, in descending frequency?

A: Basal cell cancer (75%) A: Squamous cell cancer (20%) A: Melanomas A: Cutaneous Lymphoma A: Kaposi sarcoma A: Adenexal Carcinoma A: Merkle cell cancer

Five common subtypes of Head & Neck SCC (BAG SVP)

A: Basaloid (poor prognosis) A: Adenosquamous A: Giant cell A: Spindle cell A: Verrucous A: Papillary

Four subsites of the oropharynx

A: Base of tongue A: Soft palate A: Tonsillar area (most common location) A: Posterior pharyngeal wall

Describe the nystagmus observed when all of the canals on one side become excited from their baseline firing rates

A: Beats to the Ipsilateral side both Horizontally and Torsionally A: The slow phase of the observed nystagmus has a horizontal component toward the contralateral side and a torsional component that moves the superior pole of the eye toward the contralateral side

Treatment Options for CSF leak

A: Bedrest with Head Elevation, avoid increased ICP (avoid straining, stool softeners) A: Compression dressing (Postmastoidectomy) A: Lumbar Drain or serial lumbar taps if conservative measures fail A: Open exploration 3: A 25-year Meta-analysis by Brodie (1997) suggested statistically significant reduction in meningitis using prophylactic antibiotics

Labyrinthine enhancement on MRI before and after contrast (in T1?)

A: Before - Blood (ie trauma) A: After - Inflammation (ie labyrinthitis)

Development of the Maxillary sinus

A: Begins at the 10th week of gestation, along with the hiatus semilunaris A: Present at birth, conspicuous growth by 3 years, second sinus to fully develop A: Inferior expansion starts with permanent dentition (7-8 years, overlies the 2nd bicuspid to 2nd molar) A: Reaches adult size by midadolescence; volume up to 15 cc 3: Remember "10 = 3 + 7"

Development of the Ethmoids sinus

A: Begins at the 14th week of gestation A: Present at birth, first sinus to fully develop A: Pneumatization begins significantly at 3-7 years, reaches adult form by 12-14 years, with ~14 cells (2-3 cc) 3: Remember "14"

Development of the Sphenoid sinus

A: Begins during the 3rd month A: Later becomes the Ossiculum of Bertini; third sinus to fully develop A: Does not pneumatize and become clinically significant until 4-5 years A: Growth complete by midadolescence, variable pneumatization 3: Remember "3-4-5"

3 types of behavioral auditory testing

A: Behavioral observation audiometry (BOA) A: Visual reinforcement audiometry (VRA) A: Conditioned play audiometry (CPA) 3: For infants/children from 0 to 5 years in age

Management of vocal cord Varices/Capillary Ectasias

A: Behavioural - Reduce voice training duration period, avoid sudden explosive vocals A: Medical - Hydration, stop anticoagulants/antiplatelet agents, PPI A: Surgical - Spot coagulation with CO2 laser to interrupt blood supply; visible spots should involute spontaneously

3 diagnostic categories of FNAB for thyroid (6 on bethesda)

A: Benign (60-75%) A: Suspicious (20%) A: Malignant (5%) A: Bethesda (non-diagnostic, benign, atypia, follicular lesion, suspicious for malignancy, malignancy) 3: 20% of suspicious biopsies are from malignant tumors

Ddx of a Sinonasal lesion

A: Benign Epithelial - Vestibular (squamous) papilloma, Schneiderian papilloma (inverted, fungifom, cylindrical), Adenoma A: Benign Nonepithelial - Fibroma, Osteoma, Chondroma, Hemangioma A: Malignant Epithelial - SCC (most common), Adenocarcinoma, Adenoid cystic, Olfactory neuroblastoma, SNUC, Melanoma A: Malignant Nonepithelial - Liposarcoma, Fibrosarcoma, Rhabdomyosarcoma, Leiomyosarcoma, Osteogenic sarcoma, Chondrosarcoma, Hemangiopericytoma, Neurogenic sarcoma, Lymphoma Extramedullary plasmacytoma, Giant cell tumor 3: Nonepithelial tissues are Fat, Fibrous, Muscle, Bone, Cartilage, Vessels, Nerve and White cells.

Ddx of Odontogenic tumors

A: Benign Epithelial tumors - Ameloblastoma, Ameloblastic fibroma, Ameloblastic fibroodontoma, Ameloblastic odontoma, Calcifying epithelial odontogenic tumor (Pindborg), Odontogenic adenomatoid tumor, Calcifying odontogenic cyst (Gorlin), Odontoma (complex, compound) A: Benign Mesenchymal tumors - Odontogenic fibroma, Odontogenic myxoma, Cementoma, Cementifying fibroma, Benign cementoblastoma, Periapical cementoosseous dysplasia, Dentinoma A: Malignant tumors - Malignant ameloblastoma, Ameloblastic fibrosarcoma, Ameloblastic dentinosarcoma, Ameloblastic odontosarcoma

Clinical features and treatment of Eosinophilic Granuloma

A: Benign, localized form of Histiocytosis X, chronic course A: Monoostotic or polyostotic A: Favors frontal & temporal bones; other sites femur, pelvis, vertebrae, ribs A: Treatment - Surgical excision; radiation for recurrence, inoperable, or high risk patients

Five Advantages of Preoperative Radiotherapy for SCC

A: Better Blood Supply preoperatively, thus are more Radiosensitive A: Unresectable tumors can be made Resectable A: Malignant cells at the periphery are destroyed, thus extent of surgical resection can be diminished A: Tumor Seeding at the time of resection may be decreased due to Decreased Viability A: Fewer and less viable cells Intravascularly & within Lymphatics at the time of surgery may Decrease Distant Metastases

Three Advantages of Smaller hearing aids

A: Better Cosmesis A: Better for less severe Hearing losses A: Uses Pinna anatomy for natural amplification (high frequencies) A: Decreased Occlusion effect

Three Advantages of Brachytherapy

A: Better dose Localization A: Continuous Fractionation A: Decreased dose to Adjacent normal tissue 3: Radioactive source placed in proximity to lesion

Five Advantages of Larger hearing aids

A: Better power for Amplifying severe hearing losses A: Larger Battery means better power supply A: Better for people with lower Dexterity A: Less Feedback A: Allow placement of Directional microphone

Five general Advantages of MRI

A: Better soft tissue definition than CT A: Multiplanar capability A: Clear delineation of arteries, veins, major cranial nerves A: Absence of ionizing radiation A: Absence of beam-hardening artifacts from dental implants

Three diagnostic signs of submucous cleft palate

A: Bifid uvula A: Muscular diastasis of the soft palate (zona pellucida) A: Notched hard palate

Diagnosis of NF Type II by 1 of the following 3 criteria

A: Bilateral CN VIII masses on MRI (seen with & without Gado, on Axial and Coronal cuts) A: Family history or 1st degree relative with confirmed NF2, and 1 CN VIII mass (as described above) A: Family history or 1st degree relative with confirmed NF2, and at least 2 of Meningioma, Schwannoma, Glioma, Juvenile Cataracts (Posterior Sub-Capsular)

Four Pediatric Cochlear Implantation criteria for Children 12-24 months

A: Bilateral Profound hearing loss (>90 dB) A: Lack of auditory skills development and minimal hearing aid benefit (documented by parent questionnaire) A: No medical contraindications A: Enrollment in a therapy of education program emphasizing auditory development

Head & Neck features of Polyarteritis Nodosa

A: Bilateral SNHL or vestiblar dysfunction A: CN palsy (VII most common) 3: Small-medium vessel vasculitis

Four findings on Bilateral Internuclear Ophthalmoplegia

A: Bilateral failure of Adduction A: Impaired vertical VOR A: Impaired vertical smooth pursuit A: Impaired vertical eccentric gaze holding

When would someone use a BiCROS hearing aid?

A: Bilateral hearing loss but one side worse than the other - Routes sound from poorer hearing ear

Four Pediatric Cochlear Implantation criteria for Children 25 months to 17 years 11 months

A: Bilateral severe to profound hearing loss (>70 dB) A: Lack of auditory skills development (parent questionnaire when < 5 yrs) and/or minimal hearing aid benefit (word recognition scores <30% correct when > 5 yrs) A: No medical contraindications A: Enrollment in a therapy of education program emphasizing auditory development

Seven Indications for Rotatory chair (BCC OCR)

A: Bilateral vestibular hypofunction A: Monitor Compensation after acute injury A: Monitor Changes in vestibular system over time A: Ototoxicity A: Cerebellar abnormalities A: Identify Residual labyrinthine function in patients with no caloric response A: Pediatrics - Postmeningitis, anyone with congenital hearing loss to examine vestibular component

Three steps in Botox toxin mediated paralysis

A: Binding A: Internalization (endocytosis) A: Inhibition of neurotransmitter release

Describe the pathology for Membranous inner ear malformations (3)

A: Bing-Siebenmann - Complete membranous labyrinthine dysplasia A: Scheibe - Most common histopathologic aplasia, limited membranous cochleosaccular dysplasia (pars inferioris) A: Alexander - Partial aplasia of cochlear duct at basal turn; patients have high frequency HL

Most common reconstruction after VPL

A: Bipedicled Strap muscle flap

Define Bipolar involvement, Biscut footplate, and Obliterative Otosclerosis

A: Bipolar involvement - Both Anterior and Posterior ends of Footplate involved A: Biscut footplate - Involvement Limited to the Footplate A: Obliterative Otosclerosis - Entire Footplate and Annular Ligament involved

Electron microscopy finding of Histiocytosis X

A: Birbeck granules 3: "Tennis-racket" or rod shaped cytoplasmic organelles with a central linear density and a striated appearance

Ten potential surgical complications in the resection of Carotid Body tumors

A: Bleeding A: Hematoma A: CVA A: Injury to CN XII A: Injury to CN X A: Injury to CN VII, IX, XI A: Horner's syndrome A: First Bite Syndrome (from resection of sympathetic chain, causes unopposed parasympathetic action) A: Cardiovascular collapse (secreting carotid body tumour) A: Death

Six Complications of Supraglottoplasty

A: Bleeding A: Temporary dysphagia A: Aspiration A: Temporary worsening of airway A: Supraglottic Stenosis (< 5% if bilateral, preserve islands of mucosa, esp. interarytenoid) A: Re-operation (15% if unilateral)

Define Meige's syndrome

A: Blepharospasm & Oromandibular dystonia syndrome

Pharmacologic action of Lidocaine

A: Blocks neuron Na channels, preventing action potential formation

Major landmarks along the medial orbital wall

A: Blood vessels found along the frontoethmoid suture, which divides the ACF from the ethmoid sinuses A: Anterior Ethmoid artery ~16 (14-22) mm posterior to the anterior lacrimal crest (maxillolacrimal suture) A: Posterior Ethmoid artery ~10 (10-12) mm posterior to the anterior ethmoid artery A: Optic nerve ~6 (4-7) mm posterior to the posterior ethmoid artery

Diagnostic Evaluation of Facial Palsy (guided by clinical judgement)

A: Bloodwork - CBC with Differential, Monospot/Heterophile Ab (Leukemia, Mono); ESR, Ca/ACE, ANA, RF (Sarcoid, Collagen Vascular, PAN), Cryoglobulins (Lyme disease), Glucose tolerance test (DM), FTA-ABS and TP-MHA (Syphilis) A: Radiographic - CT or MRI of Brain stem/CPA/TB/SB +/- Parotid; CXR (Sarcoidosis, Lymphoma, Carcinoma) A: Special tests - LP/CSF (Meningitis, Encephalitis, Guiliain-Barre, MS, Meningeal carcinomatosis); Bone marrow (Leukemia, Lymphoma) A: Urinary - Porphyrins and uPorphobilinogen (Acute Porphyria), uCalcium (Sarcoidosis), uGlucose (DM) A: Fecal - C. Botulinum toxin (Botulism)

Pathognomic histiologic sign in Otosclerosis (otospongiotic phase)

A: Blue Mantles of Manasse

Six tracheal lengthening techniques

A: Blunt dissection of larynx & trachea (anterior & posterior) - 3-5cm A: Incision of alternating annular ligaments - 2.5cm A: Suprahyoid laryngeal release - 5cm A: Infrahyoid laryngeal release (usually causes dysphagia, risk to SLNs) A: Release of inferior pulmonary ligament/hilum A: Cervical flexion suture A: Reanastomosis of left mainstem bronchus onto bronchus intermedius - 2.7cm 3: Airway 10-13cm long, 16-20 rings present, can excise 6.5 cm maximum Cavernous sinus contents, draw diagram

Embryologic derviations of the Incus

A: Body & short process - Meckel's cartilage (1st Arch) A: Long process - Reichert's cartilage (2nd Arch)

What 6 factors can affect ABR test results (TAG HIM)

A: Body Temperaure A: Age < 18 months A: Gender A: Hearing loss >50 dB (correction 0.1 ms/10 dB) A: Stimulus Intensity - Important: may affect Amplitude, but not Latency A: Medications - Diazepam, Phenytoin, Lidocaine

List 9 different types of wearable hearing aids

A: Body aid (obsolete) A: Eyeglass aid (obsolete) A: Behind the ear A: In the ear A: In the canal A: Completely in the canal A: Middle Ear Implants A: CROS (contralateral routing of signal) - Microphone on deaf side routes signal to contralateral side, which has normal hearing A: Bi-CROS - Asymmetric bilateral hearing loss present, both sides amplified with both signals going to better hearing ear

Two audiometric tests useful for bilateral aural atresia

A: Bone conduction ABR - Wave I generated at distal CN VIII, little crossover A: ECOG - For minor atresia, with transtympanic, TM, or EAC electrode

Seven Congenital causes of CSF Otorrhea

A: Bony Tegmen defect (most common congenital, but present later in life) A: Arachnoid granulations ("spontaneous", also present later in life) A: Mondini A: Enlarged Cochlear Aquaduct A: Enlarged Fallopian Canal A: Patent Hyrtl's fissure A: Petromastoid canal fissure 3: Preformed bony pathway with increased subarachnoid pressure transmitted; Generally present early in life with Meningitis or nonclearing SOM

Five histologic classes of Squamous Cell Carcinoma (BAG SVp)

A: Bowenoid - from Bowen's disease, SCCis A: Adenoid - Adenosquamous, pseudoglandular arrangement A: Generic - well-differentiated, actinic changes A: Spindle cell - pleomorphic, anaplastic, little/no keratinization, lymphoepithelioma-like carcinoma A: Verrucous - white cauliflowerlike lesion, HPV assoviated?

Twelve Craniofacial features of Down syndrome

A: Brachycephaly/Flat occiput A: Small ears with Narrow EACs A: Upslanting palpebral fissures A: Epicanthic folds A: Midface hypoplasia A: Small nose A: Narrow nasopharynx A: Large fissured lips A: Large fissured tongue A: Dental abnormalities A: Short neck A: Subglottic stenosis A: Atlantoaxial instability & subluxation

Three Emergenices with Middle and Inner ear Trauma

A: Brain herniation A: Massive hemorrhage - Angio first +/- Ballon occlusion 3: Another answer may be Vertigo with penetrating injury

Ddx of Pediatric Lateral neck mass (6)

A: Branchial anomaly A: Laryngocele A: Pseudotumor of infancy A: Hemagioma A: Lymphatic malformation A: Thymic cyst

Name some Autosomal Dominant syndromes associated with SNHL

A: Branchiootorenal syndrome (Melnick-Fraser) A: Stickler syndrome A: Waardenburg syndrome A: Osteogenesis imperfecta (van der Hoeve syndrome) A: Neurofibromatosis A: Treacher-Collins syndrome 3: "Although Dominant, Brian Stickler Was an Imperfect Neuro Teacher"

Side effects of 131I (BP BATCH)

A: Breast cancer A: Pulmonary fibrosis A: Bladder cancer A: AML A: Thyroid crisis/Thyroiditis (give steroids to avoid this) A: Chromosomal abnormalities/Gonadal dysfunction A: Hypoparathyroidism

Metastatic tumors of the orbit

A: Breast, lung, prostate, GI, renal cell, thyroid, melanoma 3: In other words, most common cancers; represent 8% of all orbital tumors, 25% will be the presenting manifestation of the original cancer

Seven events that occur during the pharyngeal phase of swallowing

A: Breathing cessation A: Palate elevation, closure of nasopharyngeal isthmus A: Glottic closure (3 levels) A: Laryngeal elevation A: Pharyngeal peristalsis A: Cricopharyngeal relaxation A: Dilation of pharyngoesophageal segment

Stapes reflex threshold using Pure tones vs Broadband noise

A: Broadband noise has 20-25 dB lower thresholds compared to pure tones A: This difference lessens as hearing worsens

Five signs & symptoms of chronic aspiration

A: Bronchorrhea A: Pneumonia A: Chronic cough A: Dysphagia A: Weight loss

Five Causes of False Positive Heterophil Antibody test for Infectious Mononucleosis (BRASH2)

A: Brucella A: Rheumatoid Arthritis A: Serum sickness A: Hodgkin's lymphoma A: Hepatitis

Six Indications for Tracheotomy

A: Bypass upper airway Obstruction (including Severe OSA) A: Prolonged intubation A: Protection from Aspiration (Inability of patient to handle own secretions) A: Assist with tracheo-bronchial suctioning (pulmonary toilet) A: Adjunct to H&N surgery with possible airway compromise A: Adjunct to management of severe facial fractures

Branches of Internal Carotid Artery

A: C1 - cervical segment = no branches A: C2 - petrous segment = vidian artery, caroticotympanic artery A: C3 - lacerum segment = no branches A: C4 - cavernous segment = meningohypophyseal artery, inferolateral trunk, capsular arteries A: C5 - clinoid segment = no branches A: C6 - ophthalmic segment = ophthalmic (which has ant/post ethmoid, supraorbital and supratrochlear) and superior hypophyseal artery A: C7 - communicating segment = posterior communicating, anterior choroidal, anterior/middle cerebral

11 lab tests and there association with childhood SNHL

A: CBC + Diff - Leukemia/lymphoma; Platelets - Fechner syndrome (Very rare, HF SNHL, Ocular disease, Proteinuria, Macrothrombocytopenia) A: BUN, CR, Urinanalysis - Alports syndrome (persistent microscopic hematuria) A: Glucose - Alstron syndrome (Obesity, Impaired glucose tolerance with insulin resistance, Retinal degeneration, Neurosensory deafness, Acanthosis nigricans, Hepatic dysfunction, and other Endocrine abnormalities) A: RPR, TTPA - Syphilis A: EKG - Jervell and Lange-Nielson syndrome A: GJB2 gene - Responsible for 50% of autosomal recessive nonsyndromic hearing loss, found to be positive in 35% of moderate to profound SNHL. A: CT scan - Michel aplasia, Mondini malformation, Enlarged VA A: MRI - Child with NF II, useful if progressive hearing loss, or vestibular symptoms, focal neurological symptoms A: ANA, ESR, RF - SLE, RA A: Thryoid function tests - Cretinism and Pendred syndrome (Pendred may have normal thyroid function tests and would therefore need perchlorate discharge test)

Primary immunodeficiency screening workup (10)

A: CBC and Differential A: T and B cell subsets A: T-cell stimulation tests - for candida A: IgM/A/G/E, with albumin & TP A: IgG subsets A: Specific antibodies - if titers low, Vaccine response A: C3, C4, CH50 A: Special tests - Phagocytic tests for CGD

Six Absolute Indications for FESS in children

A: CF A: Antrochoanal polyp A: Orbital abscess A: Intracranial complication A: Mucocele A: Fungal sinusitis 3: Relative indication = CRS exacerbation despite maximal medical management

Six Syndromes associated with Microtia/Aural Atresia

A: CHARGE A: Goldenhar's/Hemifacial macrosomia A: Treacher-Collins A: Crouzon's disease A: Pierre-Robin sequence A: VATER

Five conditions that would cause Bilateral Stapedial Reflex Absence

A: CHL >40 dB in reflex-eliciting ear or >10 dB in probe ear A: Cochlear HL >65 dB A: Bilateral CN VIII lesions A: Multiple sclerosis A: Brainstem lesions

Three indications for Vermillionectomy

A: CIS A: Diffuse premalignant disease (Actinic cheilitis) A: Multicentric disease

Twelve viruses involved in SNHL

A: CMV (#1 cause of Congenital viral deafness) A: Mumps (#1 cause of Acquired unilateral SNHL) A: EBV A: Rubella A: Hepatitis A: HSV I & II A: Adenovirus A: Polio A: Varicella A: Variola (Smallpox) A: Influenza A: Measles (Rubeola) - Warthin-Finkleday cells (multinucleated giant cells) 3: "See Muppets Eat Rubber Hippoes, Horses, And Ponies Very Voraciously In Meals"

Four neural systems of nose

A: CN 0 (nervus terminalis) - Loose plexus of fine nerve fibers throughout the nose; high GnRH content A: CN I A: CN V A: Vomeronasal Organ (VNO) - Rudimentary, non-functioning in humans

ENT manifestations of Arnold-Chiari malformation

A: CN IX-XII difficulties A: Bilateral vocal cord paralysis, respiratory distress A: Poor Feeding A: Aspiration

Subclassify Hypokinetic Neurologic disorders of the larynx (4)

A: CNS - Arnold-Chiari, MS, Parkinsons, Parkinson plus (Global atrophy, Shy-Drager, Diffuse Lewy-Body disease) A: Anterior Horn cells - ALS A: Neuromuscular junction - MG A: Myopathic

Xylocaine toxicity

A: CNS stimulation (3mcg/cc) A: CNS depression (5mcg/cc) A: Cardiovascular depression (8 mcg/cc)

Adenoid innervation

A: CNs IX & X

Depths of penetration of laser

A: CO2 = 30 um A: KTP = 0.9 mm A: Argon = 2.5 mm A: Nd YAG = 4 mm

Three broad repair options for Partial Thickness Lip defects

A: CO2 Laser lip shave and mucosalization by secondary intention A: Advancement flaps - Labial buccal or V-Y A: Two-stage techniques - Bipedicle visor flaps from opposing lips, ventral tongue, or cross-lip mucosal flaps

Six techniques for improving airway in bilateral vocal cord paralysis

A: CO2 laser cordotomy A: Laser arytenoidectomy and cordectomy A: Open arytenoidectomy A: Arytenoidopexy A: Arytenoid separation with cartilage grafting A: Laryngeal reanimation techniques (phrenic to RLN, phrenic to PCA, or omohyoid muscle pedicle)

Causes of Gradual Unilateral vestibular dysfunction (3)

A: CPA Tumor A: Ototoxicity A: Presbystasis

Twelve complications of External Frontoethmoidectomy

A: CSF leak A: Intracranial hemorrhage A: Bleeding/crusting A: Orbital hemorrhage A: Diplopia or blindness A: Telecanthus A: Epiphora A: Persistent or Recurrent disease A: Mucocele A: Stenosis/Synechiae of Frontal Recess with Frontal sinusitis A: Scarring/Keloid formation A: Forehead dysesthesia

Work up of paraganglioma

A: CT A: MRI A: 24 urine collection of vanillymandelic acid and metanephrines, if positive... A: Indium-111 pentetreotide (somatostatin/octreotide analog) or Iodide-131 metaiodobenzyl guanidine (MIBG) scan looking for multiple lesions; CT or U/S abdomen to R/O pheochromocytoma A: Angiography (and pre-op embolization)

What is the management of a frontal sinusitis and brain abscess in 10 year old?

A: CT A: Neurosurgical consult A: Surgical debridement of sinuses A: IV antibiotics with good CSF penetration

Radiographic features of Epidermoid

A: CT - Hypodense to CSF A: MRI - T1 Hypointense to brain, Isointense to CSF; T2 Hyperintense to brain, Isointense to CSF; FLAIR Intermediate, with Hyperintense foci; CISS, diffusion weighted A: General features - May Dumbbell into MCF or contralateral cistern, Cauliflower surface appearance, Similar to arachnoid cyst (FLAIR, CISS, DWI useful), T1 being low signal intensity differentiates from cholesterol granuloma in petrous apex lesions

Radiographic features of Arachnoid Cyst

A: CT - Isodense to CSF A: MRI - T1 and T2 Isointense to CSF, Homogeneous lesion A: General features - Smooth surface

Radiographic features of Meningoma

A: CT - Isodense to brain, may see calcifications A: MRI - T1 Isointense to brain, larger lesions may be heterogeneous with central hypointensity; T2 Hyperintense to brain, Hypointense to CSF; Gad Intense enhancement (but less than AN) A: General features - Obtuse angles to temporal bone, Dural tail present (50-75%), May herniated into MCF (50%), May show Calcification (25%), Pial blood vessels with Flow Voids, No widening of porus acousticus

Radiographic features of Acoustic Neuroma

A: CT - Non-contrast usually Isodense to brain, calcifications and central necrosis rare, larger lesions may be cystic; Contrast - 90% of non treated and not large tumors enhance homogenously A: MRI - T1 Isointense to brain, Hyperintense to CSF; T2 Hyperintense to brain, Iso/Hypointense to CSF; Gad Intense enhancement A: General features - Centered on porus acousticus, Acute angles to temporal bone, Homogeneous enhancement, No dural tail, Rare calcifications, Enlarged IAC (>2 mm compared to contralateral side), Rare extension anteriorly and superiorly, Almost never dumbbell into MCF

Radiographic findings of Cholesterol granuloma

A: CT - Soft tissue mass A: MRI - T1 and T2 Hyperintense, may show central Hypointensity, Very often confused with assymetric pneumatization of the petrous apex with bone marrow in the non-pneumatized side - differentiate using fat sat sequence which will make fat (marrow) dark but cholesterol granuloma will stay bright.

Medical management of hyperPTH

A: Ca, Cr, U/A, PTH q6month A: Bone density qyear A: Don't Restrict oral Ca intake, Cincalcet (calcimimetic) to lower serum Ca and bisphosphonates to lower calcium and slow bone turnover/slow osteoporosis

What are the clinical findings in CREST syndrome?

A: Calcinosis A: Raynaud's phenomenon A: Esophageal stenosis A: Sclerodactyly A: Telangiectasias

Four indications for parathyroidectomy due to CRF (CRRF)

A: Calciphylaxis (tissue calcification) unresponsive to hemodyalysis or meds A: Refractory bone pain/pruritis A: Renal osteodystrophy A: pathologic Fractures

Management of hyperPTH associated with renal failure

A: Calcium salts (Phosphate binder for hyperphosphatemia) A: Vit D (because hypovit D)

Six beneficial mechanisms of Chemoradiotherapy

A: Can act on Different subsets of tumor cells A: Recruit cells from Go into radiation sensitive phases A: Chemo Inhibits Repair of sublethal radiation injury A: Tumor shrinkage decreases Interstitial pressure, thus increases drug and O2 delivery A: Prevents Radiation Resistance A: Cell-cycle Synchronization increases the effectiveness of both therapies 3: "Different cells Go Repair the O-R Sink"

Discuss Ossifying Fibroma

A: Can be aggressive & locally destructive A: Osteoid rimmed by osteoblasts forming lamellar bone A: Round or oval, eggshell rims, central translucency A: Treatment - Complete removal

Discuss workplace noise allowances

A: Canada = 87 dB x 8 hours; Cut hours by ½ every 3 dB A: USA = 90 dB x 8 hours; Cut hours by ½ every 5 dB 3: Protection must be worn ≥85 dB

Etiology & 5 characteristics of peripheral nystagmus in BPPV

A: Canalithiasis - Loose otoconia, usually in the Posterior SCC, causes a vertical Torsional nystagmus (Geotropic), due to activation of the Ipsilateral Superior Oblique and Contralateral Inferior Rectus A: Latency - 5-15 sec A: Fatigable - on repeated testing A: Torsional, Geotropic - Direction does not change with repeated stimulation A: Reversability A: Brief duration - <1 minute

Most common cause of TEP failure after successfully establishing speech

A: Candidal infection of prosthesis A: Premature failure of prosthesis due to valve degradation with aspiration of liquids through prosthesis A: Treatment - Nystatin oral suspension

Three instances when median sternotomy will likely be necessary for a retrosternal goiter

A: Cannot feel the mass in the neck A: Goiter very low or POSTERIOR in the mediastinum A: Cannot finger dissect the retrosternal mass

Two instances when esophagoscopy should be aborted in esophageal burn injury

A: Cannot see the lumen A: Severe, Third Degree burn

From what cells do Meningiomas Arise?

A: Cap cells, around Tips of Arachnoid Villi

Subglottic Hemangioma - which type is safe to biopsy and use CO2 laser on

A: Capillary - Less colour to lesion, smaller vessel size, therefore able to use laser A: Cavernous - Dark red/blue, bleed with biopsy, and difficult to control with CO2 laser

Seven structures that can be injured during a neonatal tracheostomy

A: Carotid (and innominate) artery A: Jugular vein A: Recurrent laryngeal nerve A: Esophagus A: Lung - Pneumothorax A: Thymus A: Larynx A: Posterior tracheal wall

List the Paraganglia of the Head and Neck, beginning with the most common sites of Paragangliomas

A: Carotid body A: Tympanic A: Jugular bulb A: Intravagal A: Glossopharyngeal A: Superior laryngeal A: Inferior laryngeal A: Nasal A: Nasopharyngeal A: Orbital A: Subclavian A: Aortico-pulmonary A: Coronary

Most common sites of Paraganglioma occurrence

A: Carotid body (65%) A: Tympanic A: Jugular A: Intravagal A: Laryngeal A: Nasal A: Nasopharyngeal A: Orbital

Most common cause of chronic benign pediatric lymphadenopathy, diagnosis, and treatment

A: Cat Scratch Disease (Bartonella henselae) A: Serologic testing A: Intracellular gram-negative bacillus on Warthin-Starry stain A: Azithromycin (or self limiting) 3: Avoid I+D (draining tract)

Four Intraorbital Complications of Radiotherapy, Maximum Doses where applicable

A: Cataracts - as little as 6 Gy A: Lacrimal gland injury - ~35 Gy in 3.5 weeks A: Radiation Retinopathy - 50 Gy A: Optic Nerve injury - 50 Gy

Describe the four stages of Rhinoscleroma

A: Catarrhal - Foul smelling purulent rhinorrhea for weeks/months A: Atrophic - Large foul crusts/plaques simulating atrophic rhinitis A: Granulomatous - Large granulomas of URT A: Fibrosis - Up to complete stenosis of nares, can extend to NP and trachea

Describe the Impedance matching system of the Middle ear (3 lever systems)

A: Catenary lever - TM exerts force upon the malleus handle (minor input) A: Ossicular lever - When the Malleus and Incus rotate as a unit (length ratio 1.3:1) A: Hydraulic lever - Transmission of sound pressure collected over the TM (55 mm2) and transmitted to oval window (3.2 mm2) via piston-like stapes movement (area ratio averages 17:1) A: Overall Catenary-Ossicular lever ratio 22:1 A: Increased Gain in SPL from Middle ear and TM ~30 dB

Name 5 Nonviral vectors

A: Cationic Liposome A: Plasmid DNA A: Ballistic A: Calcium phosphate precipitation A: Electroporation - Cultured cells exposed to DNA in presence of strong Electrical Pulse 3: Nonspecific targeting, low transfection rate

Discuss Lyme disease

A: Cause - Borrelia Burgdorferi, transmitted by Ixodes ticks A: Stage I - Acute Erythema Migrans, influenzae-like Prodrome, Lymphadenopathy, Malaise A: Stage II - Weeks-months later, Meningitis, Cranial & Peripheral Neuropathies A: Stage III - Months-years later, chronic Arthritis, Neurologic deficits, recurrent Meningitis, Mental disorders A: Facial nerve palsy seen in 5% of patients, can be Unilateral or Bilateral, usually completely resolves A: Treatment - Ceftriaxone x 2 wks 3: Stages are PELM MCP MAN

Discuss Cavernous Sinus Syndrome

A: Cause - Ethmoiditis, 80% mortality rate A: Symptoms - Orbital pain (V1), Proptosis, Photophobia, Opthalomplegia (CN III, IV, VI involvement), Venous congestion of retina, lids, conjunctiva A: Treatment - Antibiotics, Anticoagulation 3: Similar to superior orbital fissure syndrome, except for additional involvement of venous system

Define Eagle syndrome

A: Caused by elongated styloid (normal is 2-3 cm) or calcification of stylohyoid ligament, causes irritation of V, VII, IX, X nerves; normal finding in 4% of population (Rule of 4's: 4 cm is considered long, 4% have it long {>3cm}, 4% of those are symptomatic) A: Symptoms - Throat pain, facial pain, carotodynia, otalgia, globus, dysphagia, increased salivation A: Trestment - Removal of styloid process via intraoral or external approach

Discuss Vocal Cord Hemorrhage

A: Causes - Following acute vocal trauma; predisposing factors are laryngitis, anticoagulants, varicosities/capillary ectasias A: Clinical - Arise from a varicosity on the medial vibrating edge of the vocal fold; Deeper bleeds may cause a blood blister (may stiffen the cord, lead to hemorrhagic polyp); On biopsy, may have hyalinization, Polyp may loose its vascular appearance and be pedunculated fall in and out of the airway A: Treatment - Voice rest, +/- steroids, +/- surgical evacuation

Discuss Vocal Cord Polyps

A: Causes - Vocal abuse/misuse A: Clinical - Usually unilateral, but can have a reactive lesion on the opposite cord, may occur anywhere on the vocal fold, stems from inflammation in superficial lamina propria space; variably affect the mucosal wave, more with larger polyps A: Types - Mucoid or Angiomatous A: Treatment - SLP, PPI, surgery (nearly all)

Discuss Vocal Cord Cysts

A: Causes - Vocal abuse/misuse A: Clinical - Usually unilateral, but can have a reactive lesion on the opposite cord; may occur anywhere on the vocal fold, usually deeper in the lamina propria/vocal ligament; variably affect the mucosal wave A: Types - Mucous retention or Epidermoid inclusion (keratin filled, more often associated with abuse) A: Treatment - SLP (more useful for Epidermoid inclusion cysts), PPI, surgery (nearly all)

Discuss Vocal Cord Nodules

A: Causes - Vocal abuse/misuse +/- LPR A: Clinical - ALWAYS bilateral, mid to anterior 1/3 of vocal fold, mid-membranous, affect primarily the epithelial basement membrane and superficial lamina propria A: Treatment - SLP, PPI, rarely surgery; avoid operating on singers -> correct underlying problem

Discuss Vocal Cord Pseudocyst/Fribrous Mass

A: Causes - Vocal abuse/misuse, previous trauma A: Clinical - Usually unilateral, but can have a reactive lesion on the opposite cord; may occur anywhere on the vocal fold, usually deeper in the lamina propria/vocal ligament; reduded mucosal wave A: Treatment - SLP, PPI, surgery (nearly all)

Describe mechanism of action of botulinum toxin (Botox), how does its effect wear off, and what doses are given in hyperfunctional laryngeal disorders

A: Causes irreversible blockage of receptors controlling presynaptic release of acetylcholine A: Wears off through regeneration of fresh synaptic terminals A: Dose in ADductor SD = 1 to 3.75 units A: Dose in ABductor SD = 0.5 to 5 units 3: Can say between 1 and 5 units in general

Most common benign and malignant orbital tumors in adults

A: Cavernous hemangioma A: Lymphoma

Four Medical Treatments of BPPV

A: Cawthorne Habituation exercises - Intensive, and provokes intense vestibular symptoms A: Brandt-Daroff exercises - Better tolerated A: Semont liberatory maneuver A: Particle repositioning maneuvers (Epley & Parnes)

Radiotherapy Pearls

A: Cell Death = Inability to Proliferate; both DNA strands must be knocked out A: Log cell kill = particular radiation Dose will kill the same Proportion of cells A: Therapeutic Window = Dose Response curves between Tumor cell & Tissue damage; relative positions of curves determine safety of tumor control A: Shrinking Field technique; now replaced by concomitant boost?

Define Repair

A: Cell proliferation to repair and restore toward normal structure and function

Histolopathologic characteristics of Basal Cell Carcinoma

A: Cell typically large, oval or enlongated nucleus, little cytoplasm A: Mucinous connective tissue stroma organized in parallel bundles around tumor masses, causes peripheral pallisading and stromal retraction (peritumoral lacunae)

Five groups of mediators important in CRS and asthma

A: Cells - Eosinophils, Th2 Lymphocytes A: Cytokines - Interleukins (IL-1B predominant, also -4, -5, -13), PAF, TNF A: Prostaglandins & Cysteinyl Leukotrienes A: Chemokines - RANTES, Eotaxin A: Adhesion Molecules - VCAM, ICAM, ELAM

Biocompatibility of facial implants

A: Cells don't adhere directly A: Substances in extracellular matrix bind the cells to the surface (fibronectin, vitronectin, globulin, proteoglycans) A: Extracellular matrix in bone must mineralize for compression

Three causes of Bony Resorption in cholesteatoma

A: Cellular - Osteoclasts play major role A: Biochemical - Bacterial endotoxins, Granulation tissue products, substances related to Cholesteatoma itself A: Mechanical

Six Radiographic signs of Nodal Metastasis

A: Central necrosis (most accurate CT criterion) A: Rim enhancement A: Spherical shape - LS ratio <2 (95% Accuracy, BJR 1995, Vol 68, Issue 807:266-70) A: Diameter >10 mm in short diameter (>15 mm for jugulodigastric) A: Multiple (matted?) nodes A: Extracapsular spread

What anatomic variations are there in the attachment of the upper and lower lateral nasal cartilages?

A: Cephalic end of lower lateral cartilage usually overlies the caudal end of the upper lateral cartilages A: Interlocking scroll 52% A: Overlapping 20% A: End-to-end 17% A: Opposed scroll 11%

Six indications for neck dissection for salivary gland neoplasms

A: Cervical metastasis A: Tumors >4 cm, or extraparotid extension of tumor (T3 or T4) A: High grade malignancies (mucoepidermoid carcinoma, primary SCC, adenocarcinoma, undifferentiated carcinoma) A: Facial paralysis A: Patient age >54 years A: Perilymphatic invasion

Pathophysiology and Course of an Abberant Internal Carotid Artery

A: Cervical portion of ICA fails to develop A: Inferior Tympanic Artery enters ME through Inferior Tympanic Canaliculus, joins Caroticotympanic artery and forms Horizontal Petrous portion of the ICA A: Commonly associated with Persistent Stapedial artery

Discuss the diagnosis of Basilar Impression

A: Chamberlain's line - Line between Hard Palate and posterior edge of Foramen Magnum (simpler variation is McGregor's line though the base of the Occiput) A: Wackenheim's Clivus-Canal line - Extension of Clival line through Foramen Magnum A: Extension of the Odontoid process above Chamberlain's line or posterior to Wackenheim's line indicates Impression likely exists

Staging of JNA

A: Chandler I-IV (based on NPC) A: Sessions I-III (lateral spread to PMF and ITF) A: Radkowski I-IIIB (Sessions + extent of SB erosion) A: Fisch CUMMINGS Table 73-1 -- STAGING SYSTEMS FOR JUVENILE NASOPHARYNGEAL ANGIOFIBROMA From Radkowski D and others: Arch Otolaryngol Head Neck Surg 122:122, 1996.

Define Dysplasia

A: Change affecting the size, shape and orientational relationship

Define Anaplasia

A: Change in a cell or tissue to a less highly differentiated form

Flap physiology - Define Strain

A: Change in length divided by the original length of the given tissue to which a force is applied

Define Parosmia/Cacosmia (cacosmia is all things changed to smell bad)

A: Change in the quality of the olfactory cue

What 4 mechanisms alter vocal pitch?

A: Changes in vocal cord length A: Changes in vocal cord tension A: Changes in vocal cord mass A: Changes in subglottic air pressure 3: Speed of vibration of the vocal folds (Pasha?)

Six complications of the Caldwell Luc procedure

A: Cheek edema & ecchymosis A: Dysesthesia of infraorbital n. distribution A: Epiphora A: Oroantral fistula A: Antral scarring A: Bone thickening

You see this (right Ménière's) patient five years later. She has continued to develop frequent severe attacks despite good compliance to maximal conservative therapy. She has seviceable hearing in her right ear. What are her 3 treatment options?

A: Chemical labyrinthectomy - Intratympanic gentamicin A: Endolymphatic sac decompression A: Vestibular neurectomy PEDIATRICS ANSWERS

Two Cell types, Histologic pattern, and positive IHC Stains present in Paraganglia

A: Chief cells - Granule storing A: Sustentacular cells - Schwann-like cells A: Zelballen pattern - Chief cell clusters enclosed in Fibrous Septa & Supporting cells within vascular network A: Immunohistochemistry - Chromogranin, Synaptophysin, NSE (Chief cells); S-100 (Sustentacular cells) 3: Neuroendocrine origin, nonchromaffin refers to no staining from chromium containing stains

Discuss Radiotherapy and Paragangliomas

A: Chief cells unaffected, but causes Obliterative Endarteritis of tumor vessels, controls rate of tumor growth in 90% A: Overall, not the management of choice, can reduce tumor mass; useful for management of Recurrences & Unresectable lesions

Normal EAC volumes

A: Children - 0.5-1 ml A: Adults - 0.6-2 ml

Ddx of aural polyp in pediatrics

A: Cholesteatoma A: Eosinophilic granuloma (langerhan's cell histiocytosis) A: Rhabdomyosarcoma

Most common primary lesion of Petrous Apex

A: Cholesterol Granuloma

Ddx of Petrous Apex lesions (5)

A: Cholesterol granuloma (20x more frequent than epidermoids in the petrous apex) A: Epidermoid A: Asymmetric pneumatization A: Retained mucus/mucocele A: Petrous apicitis +/- Osteomyelitis A: Petrous ICA aneurysm A: Meningioma A: Glomus tumors (Tympanicum, Jugulare) A: Chondrosarcoma A: Lymphoma A: Metastasis

Ddx of a Painful nodule on the Ear

A: Chondrodermatitis Helicis Nodularis - Necrotic, extruding auricular cartilage; can simulate BCC or SCC A: Skin cancer - BCC or SCC A: Furuncle - Staph aureus infection of hair follicle in outer EAC; treated with Abx ointment, warm compresses, +/- I&D A: Gout - Purine metabolic disorder, Uric Acid deposited in tissues; Acute pain treated with Colchicines; Allopurinol (overproducers) or Probenecid (underexcretors) A: Ochronosis - Alkaptonuria, Homogentisic acid metabolic disorder; turns black when oxidized, deposited in cartilage 3: Darwin's tubercle is a painless normal anatomic variation on the postero-superior helix

Parson's major criteria (7) for chronic pediatric sinusitis

A: Chonic nasal obstruction A: Nasal discharge A: Postnasal drainage A: Chronic cough A: Halitosis A: Headache A: Behavioral change

Location and type of mutation associated with VCF

A: Chromosome 22q11.2 A: Hemizygous Microdeletion

Seven most common GER related laryngeal disorders in pediatrics (CHARLES)

A: Chronic Cough A: Hoarseness A: Aspiration A: Recurrent Croup A: Laryngomalacia A: Episodic Laryngospasm A: Subglottic Stenosis

Six indications for Osteoplastic Flap of the frontal sinus

A: Chronic frontal sinusitis with persistent intractable symptoms, sepsis, or other complications despite previous intervention A: Mucocele with Orbital or Intracranial extension A: Osteomyelitis A: Frontal sinus tumor A: Frontal sinus fracture with comminuted anterior table or displaced posterior table A: CSF leak

Seven etiologic factors for Nasal Polyps

A: Chronic infection A: Allergy, including Fungal A: Samter's triad A: Cystic fibrosis A: Nasal mastocytosis A: Kartegener's syndrome A: Young's syndrome

Nine indications of the Caldwell Luc procedure

A: Chronic maxillary Sinusitis, or disease refractory to endoscopic surgery A: Maxillary sinus Foreign body, Tumor, Mycetoma, Multiseptate Mucocele, or Antrochoanal polyp A: Approaches to Pterygomaxllary space (Imax ligation, Vidian neurectomy, biopsy of skull base lesions) A: Repair of Oroantral Fistula A: Repair of Trauma A: Orbital Decompression of Grave's ophthalmopathy

Obstructive adenoid hyperplasia triad

A: Chronic nasal obstruction (obligate mouth breathing, snoring) A: Rhinorrhea A: Hyponasal speech

Five symptoms of Chronic Tonsillitis

A: Chronic sore throat A: Halitosis A: Tonsilloliths A: Peritonsillar erythema A: Persistent tender cervical adenopathy

Ten Intratemporal complications of AOM

A: Chronic suppurative OM A: Adhesive otitis A: Tympanic membrane perforation A: Cholesteatoma A: Tympanosclerosis A: Fixation and Discontinuity of ossicular chain A: Mastoiditis with or without Abscess (Postauricular, Bezold's, Zygomatic, Parapharyngeal, Retropharyngeal) A: Petrositis A: Labyrinthitis - Serous or Suppurative A: Facial palsy A: Labyrinthine fistula (Pasha) A: CHL or SNHL

What are the six cell types found in Olfactory epithelium?

A: Ciliated Bipolar receptor cells - Increase surface area, lack dynein arms and do not beat A: Microvillar cells - 1/10th as frequent as bipolar cells, may have a receptor function (unknown currently) A: Globose (light) Basal cells A: Sustentacular cells - Microvilli, insulate bipolar cells, deactivate odorants, protect epithelium from foreign agents, regulate mucus composition A: Horizontal (dark) Basal cells A: Lining epithelium of Bowman's glands & ducts 3: "Be My Big Sister HoBo!"

Three differences between mucosa of nose and sinuses

A: Ciliated cells more concentrated at the ostia A: Increased number of goblet cells present in the nose A: (No olfactory neuroepithlium in the sinuses)

Adenoid histology

A: Ciliated pseudostratified columnar epithelium A: Stratified squamous epithelium A: Transitional epithelium 3: Inflammation increases specialized squamous epithelium proportion and decreases respiratory proportion

Eight Contra-indications of endoscopic laser for SGS

A: Circumferential thick (cicatricial) scarring A: Length >1 cm A: Laryngotracheal stenosis A: Posterior glottic stenosis with arytenoid fixation A: Previous failure A: Previous severe bacterial infection associated with tracheostomy A: Exposure of cartilage during CO2 laser excision predisposing to chondritis A: Loss of cartilaginous framework

Define Hamartoma

A: Circumscribed overgrowth of tissues, normally present in that part of the body

Ddx of Sialosis/Sialoadenosis (recurrent bilateral non-tender parotid swelling)

A: Cirrhosis A: Diabetes mellitus (most common) A: Malnutrition (kwashiorkor, beriberi, bulemia) A: Ovarian, Thyroid, or Pancreatic insufficiency A: Drugs - antiHypertensive, Iodinated compounds, Catecholamines (also phenothiazine, ethambutol) 3: "Sir Desmond Morris Observes The Primates Hiding In Caves"

Five classes of Ototoxic Medications

A: Cisplatin A: Loop diuretics A: Salycilates A: Aminoglycosides A: Macrolides 3: "Cis Loop Sal Am Mac!"

Descibe the Koufman and Blalock classification system for muscular tension dysphonia

A: Class I - Increase in muscular tension manifested by an enlarged posterior glottal chink, an elevated larynx, and palpable neck tension or tenderness; may produce a breathy or strident voice and commonly occurs with vocal nodules A: Class II - Lateral to medial constriction with the false vocal folds adducted; in its most severe form, the false folds are used for phonation (plica ventricularis) A: Class III - Anterior to posterior constriction of the supraglottis when the epiglottis and the arytenoids obscure at least 50% of the laryngeal aditus A: Class IV - Epiglottis and arytenoids contact one another, lateral constriction is often seen

Classification of pituitary adenomas

A: Class I - Microadenoma (<10 mm) A: Class II - Macroadenoma (>10 mm) A: Class III - Partial destruction of sellar floor A: Class IV - Total destruction of sellar floor

Classification of Microtia (weerda)

A: Class I - Mild deformity, auricle decreased in size (can be big/protruding as well) A: Class II - Curving vertical ridge of tissue, all major structures present but with absolute deficiency of tissue A: Class III - Rudimentary soft tissue structure, no recognizable auricle or canal (peanut ear)

Describe the Sunderland Classification of nerve injuries

A: Class I - Neuropraxia; Conduction block, no disruption of axon continuity, recovery occurs quickly A: Class II - Axonotmesis; Wallerian degeneration of nerve from site of injury to motor endplate and back to a node of Ranvier, regenerates to original destination A: Class III - Neurotmesis; Wallerian degeneration, disruption of Endoneurium, Synkinesis may occur A: Class IV - Neurotmesis; Perineural disruption, potential for Incomplete/Abberant regeneration is greater A: Class V - Neurotmesis; Epineural disruption, complete transaction of nerve

Define Atrophic Rhinitis

A: Classified as a transformation of the respiratory pseudostratified columnar epithelium to a keritanized squamous epithelium that sloughs off

Name the WHO Histologic subtypes of thyroid Follicular Adenoma (CHAOS)

A: Clear cell A: Hyalinizing trabecular A: Atypical (insular) A: Oxyphilic/Oncocytic (Hurthle) cell A: Signet-ring

Four contraindications for Adenoidectomy

A: Cleft palate A: Submucous cleft A: Hypernasal speech A: Nasal regurgitation 3: All are signs of VPI

Tests to Identify CSF fluid

A: Clinical - Clear nonsticky nasal/aural discharge with Head Tilting, Straining A: Halo sign A: Test for Glucose and protein content A: B2 Transferrin - Disilated form of transferrin, almost exclusively found in CSF; also found in Perilymph, Aqueous Humor; Most sensitive measure of identifying CSF, controversial because of small volumes of fluid used for testing A: CT Cisternography A: Intrathecal Fluorescein (0.1 cc of 10% Fluorescein in 10 cc of CSF)

Diagnostic tests for CSF leak

A: Clinical - Halo sign A: Chemistry - Glucose (≥5 mg/dL), protein, beta-2 transferrin, beta-trace A: Endoscopy +/- Valsalva A: High resolution CT with coronal and sagittal reconstructions A: CT or MR Cisternography A: Intraoperative intrathecal fluorescein - 0.1 cc of 10% fluorescein in 10 cc of CSF, slowly infused over 10 minutes A: Radionucleotide scanning or scintiphotography

Discuss Junvenile Nasopharyngeal Angiofibroma (JNA)

A: Clinical - Males, second decade, unilateral nasal osbruction and recurrent epistaxis, centered at the PPF and usually extends into nasopharynx +/- the Pterygomaxillary fissure (PMF), Infratemporal fossa (ITF), Skull base A: Tests - CT & MRI, do Angio with Embolizayion prior to removal; If in a female, do Karyotyping to R/O androgen insensitivity/testicular feminization

Sensitivity & specificity for physical exam, CT, and combined for detecting neck disease in HNSCC

A: Clinical exam - sensitivity 74%, specificity 81%, accuracy 77% A: CT - sensitivity 83%, specificity 83%, accuracy 83% A: Combined - sensitivity 92% 3: Remember sensitivities in order = 74%, 83%, 92%; just add 9!

Five assistive listening devices (C FLAT)

A: Closed caption decoder A: FM radio A: Laser/Infrared A: Alarm (visual or vibratory) A: Telephone amplifier

Five mechanisms of the delay phenomenon in local flaps (typically done 10-21 days)

A: Closure of Arteriovenous Shunts - Due to development of autonomous tone or regrowth of sympathetics along flap base, or increased sensitivity to circulating catecholamines A: Reorientation of vessels along axis of the flap A: Increase in vessel Caliber A: Increase in vessel Numbers A: Conditioning of distal flap to ischemia

Ddx of early morning headache (7)

A: Cluster headache A: Glaucoma A: Sinusitis A: Bruxism in TMJ syndrome A: OSA A: Cervical spondylosis A: Diabetes mellitus 3: Think top down: Head, eyes, nose, teeth, throat, c-spine, pancreas!

Describe the 4 phases of wound healing (CIPRo)

A: Coagulation - Hemorrhage with exposure of platelets; formation of platelet plug & degranulation, release of mediators that cause initial vasoconstriction (TXA2), vasodilation (5-HT, Histamine), epithelial proliferation (EGF, TGF) and chemotaxis (PDGF) A: Inflammation - Infiltration of leukocytes which scavenge cellular debris, foreign bodies and other byproducts; PMNs in first 48 hrs, then transition to Monocyte/Macrophages predominance by days 3-4 A: Proliferative/Fibroplasia - Fibroblasts attracted to wound as of day 2 (max at day 15) and start extensive collagen III synthesis; re-epithelialization by basal cells at ~15 um/h (EGF); new capillary growth into matrix via angiogenesis (VEGF) A: Remodelling - Inflammation subsides, angiogenesis decreases, fibroplasia slows; dynamic balance reached between collagen I synthesis and lysis; by 6 months scar achieves 80% of original strength

Name 5 Ester anasthetics

A: Cocaine A: Procaine A: Tetracaine A: Benzocaine A: Chloroprocaine 3: Metabolized by plasma and liver cholinesterases; Higher pKa cf. amides, therefore decreased local uptake 4: "Ester Comes Prepared To Base Camp"

Give 5 Imaging abnormalities seen in X-linked Stapes Gusher Syndrome

A: Cochlear Modiolus deficient A: Lateral portion of IAC Bulbous A: Labyrinthine portion of Fallopian Canal dilated A: PSCC Dysplasia A: VA Enlargement

Three routes of access for Meningeal infection into the Inner ear

A: Cochlear aqueduct A: Internal auditory canal - Cribrose area of the cochlear nerve in the anterior inferior quadrant A: Spiral modiolar vessels - Arising from the Labyrinthine artery, off the AICA

Describe CHARGE syndrome

A: Coloboma A: Heart disease (endocardial cushion defect) A: Atresia (choanal) A: Retardation of growth, or mentation A: Genital defects (in males) A: Ear anomalies & deafness A: CHD7 on chromosome 8

Five useful signs for monitoring a cutaneous free flap

A: Color A: Temperature A: Turgor A: Capillary refill A: Palpable pulse A: Needle stick A: Other answers may include Swelling and Doppler probe

Nine methods of Increasing Tip Projection in rhinoplasty

A: Columellar strut grafts placed between the medial crura A: Shield or onlay grafts overlying the dome A: Plumping grafts to the columellar-labial angle A: Lateral crural steal (also increases rotation) A: Vertical dome division using lateral crural recruitment (Goldman tip) A: Transdomal suturing A: Tongue and groove (buried Septocolumellar sutures) A: Illusionary - Reduction of cartilaginous dorsum/enhancing supratip break A: Dynamic adjustable rotational tip tensioning (DARTT) 3: Any combination of above can be used; Preservation - Maintain tip support mechanisms; complete strip techniques with avoidance of complete transfixion incision

Superior Laryngeal innervation

A: Comes off of CN X at lower aspect of Nodose ganglion, ~36 mm from jugular foramen A: Internal branch - Sensory, enters larynx with superior laryngeal artery off of superior thyroid artery through Thyrohyoid membrane, supplies false cord, epiglottis, pyriform sinus A: External branch - Motor, supplies Cricothyroid

Name the 13 etiologies of "Other" rhinitis (non-Structural, non-Inflammatory, non-Allergic causes)

A: Compensatory hypertrophic rhinitis A: Non-Airflow rhinitis - Postlaryngectomy, choanal atresia, adenoid hyperplasia A: Temperature mediated A: Environmental/Irritative rhinitis A: Gustatory rhinitis A: Endstage vascular atony of chronic allergic or inflammatory rhinitis A: Hormonal - OCP, Puberty, Pregnancy (estrogen effect in 2nd trimester), Menopause, Hypothyroidism, Acromegaly A: Drugs - Antihypertensives, Topicals (cocaine & nasal spray abuse), NSAIDs, ASA, Psychotropics A: Non-Allergic Rhinitis with Eosinophilia Syndrome (NARES), or Basophilia - Similar to perennial allergic rhinitis, lacks the IgE mediated immunopathologic events, AND >20% eosinophils on nasal smear A: Recumbency rhinitis A: Paradoxic nasal obstruction and nasal cycle A: Emotional causes A: Idiopathic 3: "Compensating for Non Temperate Environments by Gustating Endstage Hormonal Drugs through my Nares Renders me a Paradoxically Emotional Idiot!"

Response Evaluation Criteria in Solid Tumors (RECIST)

A: Complete = Disappearance of all clinically detectable disease, ≥28 days A: Partial = >50% Reduction in tumor size, ≥28 days A: Minor response = <50% Regression in size A: Stable disease = No appreciable change in dimensions A: Progressive disease = Appearance of any new lesions or >25% increase in size of known lesions 3: Remember Complete, Partial, Minor, Stable, & Progressive = 100, >50, <50, 0, and new/>25, respectively From national cancer institute website: RECIST criteria offer a simplified, conservative, extraction of imaging data for wide application in clinical trials. They presume that linear measures are an adequate substitute for 2-D methods and registers four response categories: CR (complete response) = disappearance of all target lesions PR (partial response) = 30% decrease in the sum of the longest diameter of target lesions PD (progressive disease) = 20% increase in the sum of the longest diameter of target lesions SD (stable disease) = small changes that do not meet above criteria

Complete workup for vocal cord paralysis (Bailey p.851)

A: Complete H&P A: Flexible laryngoscopy +/- videostroboscopy A: Labs (6) - CBCwDiff, Fasting glucose, TFT, FTA-ABS, Lyme titers, Toxin screen (Lead, Arsenic) A: MBS and/or FEES - if suspicion of aspiration A: Ba Swallow - to R/O esophageal mass, vascular compression, or aspiration A: High resolution CT larynx - if suspicion of endolaryngeal pathology A: CT or MRI Skull base to upper chest (to aortic arch at least) A: CXR +/- CT chest - to R/O lung mass A: Laryngeal EMG (only useful between 1 and 6 months) - Paralysis vs. fixation, RLN vs. SLN, myopathy vs. neuropathy A: MRI Brain & Neuro consult - if suspicion of central/neurologic cause

Five things to do in surgery during Modified Radical Mastoidectomy to ensure Dry ear

A: Complete Saucerization of mastoid bowl (remove all mucosa) A: Lower the Facial Ridge to remove any Dependent Spaces A: Take down the Mastoid Tip A: Cavity Obliteration with Fat or Muscle (Palva flap or Temporalis Muscle) A: Wide Meatoplasty

Laryngeal atresia types

A: Complete absence of laryngeal lumen A: I - Supraglottic + Infraglottic A: II - Infraglottic A: III - Glottic

Eight Poor Prognostic indicators in Bell's palsy

A: Complete loss of function A: Hyperacusis A: Decreased Lacrimation A: Hypertension A: Severe aural, facial, radicular Pain A: Age >60 years A: Recovery Time >3 months A: Diabetes Mellitus 3: "Completely Hyper Lackies Have Produced A Terrible Mess!"

Two Contraindications to cochlear implantation on temporal bone imaging

A: Complete otic capsule aplasia (Michel) A: Narrow IAC syndrome (<3 mm wide with normal facial nerve function)

Six methods of Decreasing Tip Projection in rhinoplasty

A: Complete transfixion A: Resection of caudal septum A: Vertical dome division with suture reapproximation of medial crura A: Excision & suture part of medial & lateral crura of alar cartilage A: Dettachment of ULC & lateral attachements of LLC A: Nasal spine reduction A: Lateral crural overlay (also increases rotation)

Most common cause of unilateral proptosis in children

A: Complication of sinusitis

Innominate artery compression pattern and specific sign to look for

A: Compresses anterior tracheal wall (R more than L) A: Pulsatile, should affect the R arm pulses???

Classification of Traumatic TM Perforations (4)

A: Compression A: Penetrating A: Thermal A: Lightening A: Chemical

What is the Toby-Ayer Queckenstadt test

A: Compression of IJV has no effect on CSF pressure in presence of ipsilateral lateral sinus thrombosis

State the Resonance frequencies of the External and Middle ear components

A: Concha = ~5000 Hz A: External auditory canal = ~2500 Hz A: Tympanic membrane = ~1200 Hz A: Ossicular chain = 500-2000 Hz A: Middle ear = 800 Hz

Describe the Pinna and EAC's contribution to sound amplification

A: Concha's shape funnels sound into EAC A: Resonance frequency of EAC increases gain A: Overall 10-15 dB increase in gain in 3-5 kHz range

Describe the 6 testing conditions in the Sensory organization test of Dynamic Posturography

A: Condition 1 - Eyes open, Surround stable, Support fixed A: Condition 2 - Eyes closed, Surround stable, Support fixed A: Condition 3 - Eyes open, Surround sway, Support fixed A: Condition 4 - Eyes open, Surround stable, Support sway A: Condition 5 - Eyes closed, Surround stable, Support sway A: Condition 6 - Eyes open, Surround sway, Support sway

In Sensory organization testing in Dynamic Posturography, abnomal findings in which test conditions are suggestive of vestibular hypofunction?

A: Condition 5 - Eyes closed, Sway referenced platform movement A: Condition 6 - Sway referenced movement of both the visual surround and the platform 3: In these conditions, you are taking away Proprioceptive and Visual inputs, leading patient to rely solely on vestibular input - suggests vestibular in this situation but also suggests well compensated given it can only be brought out when all compensatory mechanisms removed (if bilateral loss especially)

What are the 9 complications of COM with cholesteatoma in order of frequency?

A: Conductive HL - Ossicular chain disruption in up to 30% A: Inner Ear Fistula - 10% of cases, mainly HSCC, rarely Cochlea A: Extradural or Perisinus Abscess A: Labyrinthitis - Serous or Suppurative A: Facial Nerve Paralysis - Acute (infection) or Chronic (slow expansion) A: Meningitis secondary to Tegmen Erosion A: Subdural or Intraparenchymal Brain Abscess A: Sigmoid Sinus Thrombosis/Phlebitis A: Subperiosteal Abscess/Bezold's Abscess due to erosion of Mastoid Cortex A: Recurrent Cholesteatoma 3: "Condy Fought Extra Long For Men Bored of Sitting in Subs to Come home"

Reconstruction options for TMJ

A: Condyle Allograft - Condyle prosthesis Autogenous bone (iliac) A: Glenoid fossa Preservation of disc Fossa prosthesis Temporalis flap A: Costochondral graft (growth center in kids)

Ddx of Unilateral Facial Paralysis (CLINTONS)

A: CongenitaL Congenital Unilateral Lower Lip Palsy (CULLP) Mobius syndrome Myotonic Dystrophy A: Infectious & Idiopathic Bell's palsy (Idiopathic; 48%) Meningitis Herpes Zoster Oticus/Ramsay Hunt syndrome (5-9%) Lyme disease EBV HIV TB Syphilis Clostridial (Tetanus, Botulism) A: Neurologic Cerebrovascular disorder - Central or Peripheral Guillain-Barre syndrome Myasthenia Gravis Benign Intracranial Hypertension Multiple Sclerosis ALS A: Trauma Temporal Bone trauma Birth trauma Iatrogenic Barotrauma A: Otitis Acute bacterial Otitis Media (most common in pediatrics) Chronic bacterial Otitis Media Cholesteatoma Necrotizing Otitis Externa A: Neoplastic Malignant - Primary or Metastatic Parotid tumor Benign - Schwannoma (~20%/segment between Labyrinthine and Mastoid), CN VII Hemangioma (90% between IAC and 2nd Genu), Glomus tumor Leukemia/Lymphoma Cholesteatoma A: Systemic (PHD MASK) Pregnancy Hyperthyroidism Diabetes Mellitus Melkerson-Rosenthal Autoimmune disease Sarcoidosis (Heertford's disease) Kawasaki disease Collagen Vascular diseases Polyarteritis Nodosa

Ddx of Bilateral Facial nerve palsy (CITTENS)

A: Congenital Mobius Syndrome Syphilis A: Infectious & Idiopathic Bacterial Meningitis Brainstem Encephalitis Lyme disease Bell's palsy - Bilateral in 0.3% of cases A: Trauma Bilteral Temporal Bone trauma Birth trauma Iatrogenic A: Tumors Leukemia/Lymphoma A: Endocrine Diabetes Mellitus Hyperthyroidism A: Neurologic Gullain-Barre syndrome Myasthenia Gravis Multiple idiopathic cranial neuropathies Benign Intracranial Hypertension Multiple Sclerosis ALS A: Systemic Sarcoidosis (heerfort's syndrome) Melkersson-Rosenthal syndrome Osteopetrosis (Albers-Schoenberg) - Hereditary, bony obliteration of foramina/compression of CNs; decompression rarely indicated Collagen Vascular diseases Polyarteritis Nodosa

Ddx of Laryngotracheal stenosis

A: Congenital - Tracheomalacia Laryngomalacia VC paralysis Laryngeal cleft Congenital cysts Extrinsic compression, Vascular (innominate artery, right sided aortic arch, aberrant left pulmonary artery), or Mass (teratoma, lymphatic malformation, hemangioma) A: Infectious/Inflammatory - Croup Tracheitis Epiglottitis Retropharyngeal abscess GER A: Traumatic - External compression Foreign body A: Neoplastic - Subglottic hemangioma RRP

Ddx of salivary gland cysts

A: Congenital - Branchial cleft, Epidermoid, Dermoid A: Acquired - Benign lymphoepithelial lesion (AIDS) Mucus retention cyst/mucocele, Sialocele (pseudocyst, associated with trauma)

Classification of Local causes of Macroglossia (CITeN)

A: Congenital - Hemangioma, Lymphangioma, Lingual thyroid A: Inflammatory/Infectious - Angioedema, TB, Actinomycosis, Dental infection, Syphilitic gumma, Riga disease, Ranula A: Traumatic - Dental irritation, Hematoma, Postoperative edema, Sublingual calculus A: Neoplastic, Benign - Granular cell tumour, Neurofibroma, Leiomyoma, Lipoma; Malignant - Carcinoma, Sarcoma

Ddx of Olfactory dysfunction

A: Congenital - Kallman's, Familial anosmia (AD, premature baldness, vascular headaches) A: Obstructive nasal disease (23%) - Polyps, edema, tumors, nasal deformity A: Idiopathic (21%) A: Postinfectious (19%) - Viral injury to olfactory neurons A: Head Trauma (15%) - Shearing of filaments, olfactory bulb contusion, frontal lobe injury; CN I is the most commonly damaged CN, followed by VIII, X, and VII A: Neurologic - Parkinson's, Alzheimer's, Multiple Sclerosis A: Psychogenic (schizophrenia) A: Toxins/medications (3%) - Smoking, Formalin A: Aging A: Neoplastic - Foster-Kennedy syndrome A: Other

Classification of Generalized causes of Macroglossia (CITEn)

A: Congenital - Primary idiopathic macroglossia, Beckwith-Wiedemann syndrome, Down syndrome, Trisomy 4P syndrome, Triploid syndrome, Gangliosidosis syndrome, Mucopolysaccharidoses, Robinow syndrome A: Inflammatory - Chronic glossitis A: Toxic - Amyloidosis, Lipoid proteinosis, Chronic steroid therapy A: Endocrine - Acromegaly, Myxedema, Cretinism

What is CHAOS and two example of lesions that can cause conditions

A: Congenital High Airway Obstruction Syndrome A: Lymphangiomas and Teratomas located in the anterior compartment

Define Desmoplasia

A: Connective tissue reaction to tumor

Treatment options for OSA

A: Conservative measures A: Nasal optimization A: Oral appliances - Tongue retaining or Mandibular advancement/dental devices A: CPAP A: Procedures / Surgery

Black child for T and A - what need to consider first, simple test and preoperative management

A: Consider Sickle Cell anemia A: Simple test - CBC (anemia), Sickle cell test, then Hb electrophoresis A: Preop - Consult hematology, transfusion to achieve a ratio of 60:40 for sickle blood and normal blood (if I remember correctly), avoid hypoxia as a precipitate a sickle cell crisis

Lab finding with common variable hypoglobulinemia

A: Consistently low total immunoglobulins

What does a CROS hearing aid stand for?

A: Contralateral Routing Of Sound

Describe the p53 gene

A: Controls cell cycle by binding to Cyclin-dependent Kinins and arrests cell replication in G1; can induce Apoptosis if DNA Repair mechanisms fail 3: Located on chromosome 17p13.1

Agents responsible for caustic ingestion and their damage pattern

A: Corrosives/Acids - Coagulative necrosis, superficial coagulum prevents deeper damage A: Caustics/Bases - Liquefactive necrosis, early disintegration of tissues with deep penetration, significant damage at pH >12 A: Bleaches - pH ~7, esophageal irritants

What are the three types of autogenous bone grafts used in secondary mandibular reconstruction?

A: Cortical - Lamellar plates of bone, provides the least viable cells, requires long periods for revascularization, provides best rigid support for mandible reconstruction; excellent source of BMP for transformation of pluripotential cells into osteoblastic cells (good for phase II) A: Cancellous - Medullary bone and bone marrow, provides the highest concentration of viable cells which can produce new bone, requires shortest period for revascularization (good for phase I) A: Particulate cortical bone and cancellous marrow contains both medullary and cortical bone, which provides a tradeoff between support and a viable transferable cell population (best osteogenic potential); requires a crib for stability

Three typical osseous findings of mandibular ORN on CT scans

A: Cortical disruption A: Disorganization of trabeculation A: Osseous fragmentation 3: Radiolucency, demineralization, sequestrum

Mechanism of action and 5 examples of DNA Binding chemotherapeutic agents

A: Covalent DNA Intercalators, interfere with normal DNA function, alter Replication 3: Cisplatinum, Carboplatin, Adriamycin, Bleomycin, Mitomycin

Describe the cover-body theory (Fujimura) and the corresponding vocal cord layers

A: Cover - Mucosa (nonkeratinized stratified squamous epithelium) and superficial lamina propria (Reinke's space) A: Transition - Middle & Deep layers of lamina propria A: Body - Vocalis muscle

Risk factors for cardiorespiratory complications in patients with airway obstruction (OSA?)

A: Craniofacial malformation A: Abnormal pharyngeal musculature A: Congenital heart disease A: Susceptibility of pulmonary vasculature A: Obesity A: Concurrent URTI

Give 6 etiologies of Basilar Impression (CRROOP)

A: Cretinism A: Rheumatoid Arthritis A: Rickets A: Osteomalacia A: Osteogenesis Imperfecta (van der Hoeve syndrome) A: Paget's disease

Contraindication to any organ preserving laryngeal surgery

A: Cricoarytenoid joint involvement

Four anatomic structures are seen on Barium swallow causing compression

A: Cricopharyngeus (C6) A: Aortic arch (T4) A: Left mainstem bronchus (T6) A: Lower esophageal sphincter 3: Common sites of injury in caustic ingestion

Five variants of Meniere's

A: Crisis of Tumarkin - Acute Utriculosaccular dysfunction, leads to sudden falls without warning due to acute vestibular asymmetry & loss of vestibulospinal efferents (loss of postural tone); 2-6% of patients A: Lermoyez attacks - Increasing tinnitus, ear fullness and hearing loss, all suddenly resolve with the onset of vertigo A: Cochlear hydrops - Hearing loss, Tinnitus, and Aural fullness without vertigo A: Vestibular hydrops - Episodic Vertigo without hearing loss or tinnitus A: Delayed Endolymphatic hydrops - Hearing loss followed later by typical Meniere's symptoms

Mechanism of action and 3 examples of Alkylating agents

A: Crosslink DNA, interfere with DNA Replication 3: Cyclophosphamide, Chlorambucil, Nitrogen mustard

Syndromes are associated with Choanal Atresia (50% of all cases, CAT CTV)

A: Crouzon syndrome A: Apert syndrome A: Treacher-Collins syndrome A: CHARGE syndrome A: Trisomies 18, 21 A: Velocardiofacial syndrome

Two conditions with congenitally shallow orbits

A: Crouzons A: Aperts

Eight local factors that increase risk for laryngeal damage from ETT

A: Cuff pressure too high A: Too large an ETT for the larynx A: Tube movement A: Length of intubation greater than 7 days in adults, ~3-4 weeks in pediatrics A: Blind intubation/poor visualization A: Preexisting laryngeal anatomic abnormality A: Presence of GERD A: Presence of local Infection

Three reasons for Age-related olfactory changes

A: Cumulative damage to the olfactory epithelium from viral and other insults A: Ossification/closure of the foramina of the cribriform plate A: Pathologies such as Alzheimer's and Parkinson's

Describe Odontogenic Keratocyst

A: Cystic neoplasm that causes bone destruction, frequent recurrence A: Treatment - Enucleation & curettage

Identify the Histochemical Markers for SCC

A: Cytokeratin autoantibodies

Esophageal carcinoma

A: DDx - A: Diagnosis - Brush cytology or biopsy A: Causes - see risk factors A: Clinical - Nonpainful dysphagia & weight loss common; odynophagia, dysphagia, anemia, hemorrhage, aspiration pneumonia, VC paralysis, adenopathy A: Complications - Obstruction, hemorrhage, airway compromise, perforation A: Types - SCC most common; Adenocarcinoma less common A: Tests - CXR, Esophagoscopy with brush cytology or biopsy, Endoscopic ultrasound, CT A: Treatment - Radiochemotherapy or surgery; dismal prognosis

Ten Environmental risk factors for AOM

A: Daycare attendance (2.6x) A: Season (Fall/Winter) A: URTIs A: Older siblings A: Parental history of OM A: Passive smoking A: Low S/E status (overcrowding, poor sanitation) A: Lack of breastfeeding A: Night-time bottle (horizontal position) A: Pacifier use

What are the components of the Classical Rubella Triad?

A: Deafness - SNHL typically in a Cookiebite configuration A: Cataracts - also Glaucoma, Microphthalmia A: Heart defects - PDA, Pulmonary Artery Stenosis 3: Expanded syndrome includes Encephalitis, Microcephaly, Mental Retardation, Hepatosplenomegaly at birth, Thrombocytopenia, Radiolucencies in the long bones, Interstitial pneumonitis, Low birth weight

What are the 4 categories of Central Auditory Processing disorders

A: Decoding - Impairment of breakdown of auditory processing at the Phonemic level A: Integration - Difficulty integrating auditory information with other functions, such as Visual and Nonverbal aspects of Speech A: Tolerance Fading Memory - Poor auditory memory or difficulty understanding speech under adverse conditions A: Organization - Reversals and Sequencing errors 3: "Donkey Irritates Tall Ogers!"

Flap physiology - Define Stress Relaxation

A: Decrease in stress when skin is held in tension at a constant strain/length for a given time; This occurs over a matter of days to weeks and is due to an increase in skin cellularity and the permanent stretching of skin components

Six effects of corticosteroids in Allergic Rhinosinusitis

A: Decreased Arachidonic Acid metabolism (PG/LT/TX) A: Decreased secretion of Mediators of inflammatory cell Proliferation A: Decreased Influx of Eosinophils, Basophils & T-lymphocytes into the nasal epithelium A: Decreased Capillary Permeability and promotes Vasoconstriction A: Decreased glandular response to cholinergic stimulation, Decreases Mucus production A: Stabilize Lysosomal membranes

Define Hyposmia/Microsmia and Hyperosmia

A: Decreased ability to smell and hypersensitivity to odors, respectively

Three effects of steroids on wound healing

A: Decreased leukocyte & monocyte migration & phagocytosis A: Inhibition of keratinocyte & fibroblast mitosis (slows reepithelialization) A: Vasoconstriction, results in decreased oxygen & nutrient delivery

Mechanisms of Dextran

A: Decreased viscosity leads to improved capillary blood flow A: Decreased platelet adhesiveness A: Alters fibrin structure A: Antithrombotic effect starts 2-4 hours after infusion starts 3: Adverse reactions include anaphylaxis (test preop), pulmonary edema due to fluid overload, ARDS and nephrotoxicity

Describe the blood supply to the middle ear (9)

A: Deep Auricular artery - from the Internal Maxillary, supplies the lateral surface of the TM and the inferior portion of the medial surface A: Anterior Tympanic artery - from the Internal Maxillary, 3 branches supply the lateral wall of the Epitympanum and the Ossicles A: Superior Tympanic artery - from the Middle Meningeal artery, supplies the Epitympanum, Tensor Tympani, portion of Stapes A: Superficial Petrosal artery - from the Middle Meningeal, supplies the Dura & Perigeniculate Facial nerve A: Inferior Tympanic artery - from the Ascending Pharyngeal artery, supplies Floor of middle ear, Promontory A: Posterior Tympanic artery - from the Stylomastoid artery, supplies the Chorda Tympani A: Stylomastoid artery - from the Posterior Auricular artery, supplies bone & mucosa of Mastoid region, Floor & inferoposterior wall of Middle ear, Facial nerve, Stapedius A: Tubal artery - from the Accessory Meningeal artery, supplies the Eustachian tube A: Caroticotympanic arteries - from the ICA, extend from the carotid canal to supply the anterior wall of the Middle ear

Triad of symptoms in Gradenigo syndrome

A: Deep Facial/Retroorbital Pain (CN V) A: Diplopia (CN VI Palsy) A: Otorrhea 3: Others - Fever, Headache, CN VII, VIII, IX involvement

Ddx of parapharyngeal space

A: Deep lobe parotid tumors (45%) A: Neurogenic tumors (30%) - Schwannoma, Neurofibroma, Paraganglioma A: Lymphoma (25%) A: Vascular (3%) - Hemangiopericytoma, Hemangioendothelioma, AVM, IAC aneurysm A: Minor salivary gland tumors - most frequently malignant, occasionally pleomorphic adenoma A: Other - Brachial cleft cyst, Lipoma, Teratoma

Discuss Histiocytosis X

A: Definition - A: DDx - A: Diagnosis - Birbeck granules (laminar rod-shaped organelles within the nuclear cytoplasm) and X bodies (cytoplasmic inclusions) within the histiocyte are pathognomonic; CD1 antigen present A: Causes - A: Clinical - Involves temporal bone in 20-60% of cases; 25% present with initial otologic symptoms A: Complications - A: Types (3) - Eosinophilic Granuloma, Hand-Schuller-Christian, Letterer-Siwe disease (see details below) A: Tests - Biopsy A: Treatment - Radiation, surgery, chemotherapy, or combination, depending on type

Discuss Trigeminal neuralgia

A: Definition - A: DDx - Sluder syndrome, Cluster, post-herpetic neuralgia A: Diagnosis - Clinical A: Causes - Vascular loop... A: Clinical - Repeated brief attacks of severe, sharp, jabbing or lancinating unilateral facial pain, 5-7th decades; Typically in mandibular trigeminal division, followed by maxillary division; trigger points common; No CN impairment (facial numbness/weakness, loss of corneal reflex, change in taste or smell, or other) A: Complications - A: Types - A: Tests - MRI with contrast, LP for patients with negative MRI or other symptoms A: Treatment - Medical - Baclofen, Anticonvulsants (Carbamazepine, Gabapentin, Phenytoin, Sodium valproate); Surgical - Percutaneous rhizotomy (surgical severance of nerve roots to relieve pain), Microvascular decompression

Discuss Cluster headache

A: Definition - A: DDx - Sluder syndrome, Trigeminal neuralgia, Migraine, Tension A: Diagnosis - Clinical A: Causes - Strong male predominance A: Clinical - Explosive onset of unilateral periorbital/retroorbital pain, constant with boring/burning quality, lasts 15-180 minutes; Associated autonomic symptoms (nasal stuffiness, lacrimation, conjunctival injection), Lacks nausea/vomiting features A: Tests - CT, or MRI A: Treatment - Prophylactic = Antihistamines, Calcium channel blockers, Ergotamine, Lithium, Methylsergide, Valproic acid, Ranitidine; Abortive therapy (SILO) = Sumatriptan, Intranasal lidocaine, Oxygen

Common features of Migraine headache

A: Definition - A: DDx - Tension, Cluster A: Diagnosis - Exclusion A: Causes - Strong female predominance, family history common; precipitating factors (see below) A: Clinical - Recurrent attacks of moderate to severe unilateral throbbing pain, sudden onset and limited duration; Associated symptoms common (Photophobia, phonophobia, nausea & vomiting) A: Complications - A: Types - with aura (classic), without aura (common), basilar (complicated), mixed (with tension) A: Tests - MRI, or CT (to R/O SAH) A: Treatment - Abortive, acute, preventative and prophylactic (see below)

Discuss Rhinoscleroma

A: Definition - A chronic infectious granulomatous disease of the nose A: DDx - A: Diagnosis - Hallmark histologic sign = Mikulicz cells (large foamy histiocytes), Russell bodies (eosinophilic plasma cells) A: Cause - Klebsiella rhinoscleromatis A: Clinical - A: Complications - A: Types - 4 stages = Catarrhal, Atrophic, Granulomatous, Fibrosis A: Tests - Biopsy, culture? A: Treatment - Debridement; long term Streptomycin plus Tetracycline 3: "Cat Ate Grannies Finger, Stupid Tetrapod!"

Describe Heerfordt's disease (Uveoparotid fever)

A: Definition - A variant of sarcoidosis characterized by Parotitis, uveitis, CN paralysis A: Diagnosis - H&P, High ACE level & Ca, SPEP, CXR A: Causes - Idiopathic A: Clinical - Prodrome of fever, malaise, weakness, nausea, night sweats; Parotitis, Uveitis, CN paralysis in 5% of patients (CN VII in 50% of those with paralysis), SNHL, and facial swelling; seen in 3-4th decades, self-limiting A: Complications - SNHL, facial paralysis, blindness A: Tests - ACE level A: Treatment - Corticosteroids, if nonresponsive, use methotrexate or azathioprine; ocular care (artificial tears, ointment, night patch)

Discuss Polypoid Degeneration of the Vocal Cords

A: Definition - AKA Reinke's edema; accumulation of gelatinous mucoid material in the superficial lamina propria A: Cause - Women>Men, Tobacco, Vocal abuse, LPR (not mentioned in Cummings)? A: Clinical - Marked edema of the superficial layer of the VC, may prolapse inward during inspiratory phonation; may have marked decrease in fundamental frequency A: Smoking cessation, PPI, SLP, but do not usually go away without Surgery

Discuss Alport syndrome

A: Definition - AR or XR, abnormal Type IV collagen in the glomerular basement membrane A: Diagnosis - Renal failure and SNHL A: Causes - Degeneration of organ of Corti and Stria vascularis A: Clinical - Progressive SNHL (usually 2nd decade), varying degrees of Renal disease (mild renal Dysplasia to renal Agenesis identifiable on ultrasound), gross Hematuria associated with UTI A: Complications - Renal failure, Death in males by age 30 A: Types - 6 subtypes, 3 AR, 3 X-linked A: Tests - BUN, Creatinine, Urinalysis A: Treatment - Dialysis/renal transplant important therapies (more so in males)

Discuss Myasthenia Gravis

A: Definition - Acquired autoimmune disease with autoantibodies targeting the nicotinic ACh receptor at the motor end-plate A: DDx - MS, ALS (remember EOMs preserved!), basilar artery thrombosis, Lambert-Eaton (Abs against presynaptic Ca Channels releasing ACh, can be associated with lung cancer), thyroid disease A: Diagnosis - History, Tensilon test (edrophonium), 85% will have positive anti-AchR antibodies A: Causes - Idiopathic in most, many Drugs can cause MG (penicillamine, Abx, BB, Li, ...) A: Clinical - Weakness & fatigue of striated muscles (ptosis, diplopia, dysphonia, dysarthria, dysphagia, VPI, aspiration) A: Complications - Myasthenic crisis (with intercurrent illness or meds) and rapid respiratory collapse A: Types - MGFA Class I (only ocular) to Class V (intubated) A: Tests - Tensilon test (Edrophonium), Anti-AchR antibodies A: Treatment - Thymectomy, Acetyhcholinesterase inhibitors - Edrophonium (Tensilon), Neostygmine, Pryidostigmine (Mestinon)

Kawasaki disease: 5 investigations, 2 therapies

A: Definition - Acute febrile illness of childhood, small vessel vasculitis, most common cause of acquired heart disease in children A: DDx - Infectious lymphadenopathy A: Diagnosis - Clinical A: Causes - A: Clinical - Usually presents <5 years of age; Fever, nonsuppurative conjunctivitis, red dry lips, oral ulcers & erythema, erythematous desquamative rash of fingers and toes, polymorphous truncal rash, nonsuppurative cervical adenopathy > 1.5cm A: Complications - Coronary artery aneurysms & MI within 2-12 weeks of disease onset (7-20%) A: Types - A: Tests - CBC, Lytes/BUN/Creat, Acute phase reactants (ESR, CRP, etc.), CXR, Echocardiogram (pericardial effusion, coronary artery dilation), Abdominal U/S (acalculous cholecystitis) A: Treatment - Immunoglobulin, Aspirin

Discuss Supraglottitis (Epiglottitis)

A: Definition - Acute inflammation of the supraglottis A: Ddx - URTI, croup, bacterial tracheitis, retropharyngeal abscess A: Diagnosis - History, clinical presentation; radiographics only if diagnosis is in question A: Causes - HiB, also GABHS, Staph, Pneumococcus, Klebsiella, H. parainfluenzae A: Clinical - Mild URI prodrome, rapid onset of high fever, toxic symptoms, drooling, dysphagia; affects children from 1 year to adulthood, peak is 2-6 years A: Complications - Airway obstruction, death A: Tests - Lateral soft tissue neck x-ray, "thumbprint sign", can culture epiglottis once airway is secure A: Treatment - Do not agitate child, OR intubation, rigid/flexible bronchoscopy, IV antibiotics (ceftriaxone, cefotaxime, ampicillin/sulbactam), extubation usually within 48 hours once swelling down and air leak present

Discuss of Vestibular Neuronitis

A: Definition - Acute, peripheral and unilateral vestibular disorder NOT associated with hearing loss A: Clinical - Single episode of severe prolonged vertigo, typically occurs in middle age; Complete resolution of symptoms within 6 months A: Tests - ENG = Decreased caloric response in affected ear

Describe Churg-Strauss syndrome

A: Definition - Allergic granulomatous vasculitis; characterized by fibrinoid, necrotizing epithelioid eosinophilic extravascular granulomas A: DDx - WG, PAN, Goodpasture's, Hypereosinophilic syndrome A: Diagnosis - 4/6 Criteria = Asthma, Nonfixed pulmonary infiltrates, Eosinophilia >10%, Extravascular eosinophilia, Sinus abnormality, Neuropathy A: Clinical - Triad of asthma/allergic rhinitis, eosinophilia, systemic vasculitis of small-medium vessels; 70% have nasal involvement (allergic rhinitis, polyps, obstruction, rhinorrhea, crusting), asthma A: Complications - A: Types - 3 phases = 1) Prodromal (allergic rhinitis, nasal polyps, asthma), 2) Hypereosinophilia, in peripheral blood and tissues, 3) systemic necrotizing vasculitis A: Tests - CBC, ESR, CRP, pANCA (+ in 70%), biopsy A: Treatment - High dose corticosteroids; can try adding cyclophosphamide but does not respond as well as WG

Describe the Kuhn classification of frontal cells

A: Definition - An anterior ethmoid cell above the agger nasi that can obstruct the frontal recess or the frontal sinus A: Type I - Single frontal recess cell above agger nasi but below the floor of the frontal sinus A: Type II - Multiple cells in frontal recess above agger nasi but below floor of frontal sinus A: Type III - Single cell pneumatizing cephalad into frontal sinus A: Type IV - Single isolated cell completely within the frontal sinus, not located within the frontal recess

Discuss Osler Weber Rendu syndrome/Hereditary Hemorrhagic Telangiectasia

A: Definition - Autosomal dominant disorder characterized by ectatic vessels of the skin, mucous membranes, and viscera A: Diagnosis - Clinical A: Cause - Mutation in the Endoglin protein, a receptor for TGF-beta, which has a role in tissue repair and angiogenesis, leading to the development of abnormal vasculature A: Clinical - Triad of Telangiectasias, recurrent Epistaxis, and a positive Family History for the disorder; may also have pulmonary, GI & CNS bleeds A: Complications - Morbidity and mortality due to multiorgan arteriovenous malformations, and associated hemorrhages A: Treatment - Manage anemia/acute bleeds; Septal dermoplasty for epistaxis

Describe Waardenburg's syndrome and the 4 types

A: Definition - Autosomal dominant except type IV = AR A: Clinical - Pigment abnormalities (white forelock, premature graying, vitiligo, heterochromia iridis), craniofacial abnormalities (dystopia canthorum, broad nasal root, synophrys), unilateral or bilateral SNHL, +/- vestibular Sxs A: Type I - Presence of Dystopia Canthorum, SNHL occurs in 20%, PAX3 gene on chromosome 2 Type II - Absence of dystopia canthorum, SNHL occurs in 50%, MITF (microphthalmia transcription factor) on chromosome 3 Type III - Klein-Waardenburg, features of WS1 plus Blue eyes, Hearing impairment, upper limb Skeletal dysplasias, muscular hypotonia, unilateral ptosis? (Pasha) Type IV - Waardenburg-Shah, AR, phenotype similar to WS2 plus Hirschprung megacolon

Describe Jervell Lange-Nielsen disease

A: Definition - Autosomal recessive A: Causes - Linked to a Potassium channel gene (KVLQT 1) on 11p15.5 A: Clinical - Profound bilateral congenital SNHL (high frequencies worse), Heart disease (prolonged QT, large T waves) A: Complications - Stokes-Adams attacks, recurrent syncope, usually terminates fatally with sudden death A: Tests - ECG, Audiometry A: Treatment - ß-blockade, Amplification

Describe Pendred syndrome

A: Definition - Autosomal recessive A: Causes - Tyrosine Iodination defect leading to Euthyroid goiter A: Clinical - Goiter, Mondini or Enlarged VA's A: Tests - Perchlorate discharge test (shows abnormal organification of nonorganic iodine) A: Treatment - Exogenous T4

Describe Sjogren's syndrome

A: Definition - Chronic autoimmune disease characterized by lymphocytic infiltration of exocrine glands and epithelia in multiple organs A: DDx - A: Diagnosis - Minor salivary gland biopsy, Focus score ≥1 in a 4 mm2 field (scanning power), Focus = 50+ lymphocytes, must have normal mucosa and ≥5 MSGs separated by connective tissue; Blood tests for Primary Sjogren's = SS-A/Ro 60%, SS-B/La 30%, HLA-DW3, HLA-B8; for Secondary Sjogren's = HLA-DW4; other factors = RF, ANA, ANCA; Sialography = punctate (≤1 mm), to globular (1-2 mm) contrast collections, progresses to cavitary or destructive when superinfected A: Causes - unknown, Autoimmuity (HLA-B8 and HLA-DR3)? Multifactorial? A: Clinical - Triad of xerophthalmia/keratoconjunctivitis, xerostomia (sicca complex), association with other autoimmune disease; altered taste, intermittent unilateral/bilateral SG enlargement A: Complications - Dental caries, oral candidiasis, angular cheilitis; epistaxis, hyposmia, chronic sinusitis; 40-fold increased risk of lymphoma (greatest with primary, constant parotid swelling, and lymphadenopathy, splenomegaly, decreased IgM) A: Types - Primary or Secondary (RA #1, SLE, sceroderma, PBC) A: Tests - Minor salivary gland biopsy (lip, septum, hard palate); Anti-Ro/SS-A, Anti-La/SS-B, ANA, RF; Radiology (sialography); IgM (low = increased risk of progression to malignancy) A: Treatment - Symptomatic, hydration, artificial saliva and tears, pilocarpine, antifungals

Discuss Actinomycosis

A: Definition - Chronic granulomatous and suppurative disease A: DDx - Nocardia, KITTENS for neck mass... A: Diagnosis - Histopathology = Multifilamented, anaerobic gram positive rods; Sulfur granules (collections of Actinomyces organisms), Granuloma formation A: Cause - Actinomyces species, israelii most common A: Clinical - Palpable purple mass most common H&N manifestation, 61% have visible sinus tracts, 40% have lymphadenopathy; often have concurrent dental, sinus or perimandibular disease A: Complications - A: Types - A: Tests - FNAC? A: Treatment - Surgical debridement, Penicillin G IV for 2-6 weeks; Tetracycline or Erythromycin if pen allergic

Discuss Bacterial tracheitis

A: Definition - Complication of laryngotracheobronchitis A: Ddx - of URTI, croup, supraglottits, retropharyngeal abscess A: Diagnosis - Clinical +/- bronchoscopic A: Causes - S. aureus, also S. pyogenes, H. influenzae, M. catarrhalis A: Clinical - URI prodrome, rapid onset in children 6 months - 8 years; high fever, hoarseness with cough, dysphagia, toxic symptoms, no drooling A: Tests - CBC, ABG, croup series, irregularity of airway on CXR A: Treatment - OR intubation, bronchoscopic suction and cultures of airway exudates, extubation when normothermic, decreased secretions, air leak present

Discuss Multiple Sclerosis

A: Definition - Demyelinating disease of young adults (esp women) A: DDx - ALS, MG, HIV encephalopathy, Lyme disease, TIA/RIND/CVA A: Diagnosis - MRI findings of demyelinated foci in white matter, abnormal auditory & visual evoked responses, elevated CSF protein content (oligoclonal bands) A: Causes - Unknown, genetic? Caucasian, living in higher latitudes are more at risk A: Clinical - Vertigo, nystagmus, deafness, diplopia, bilateral INO (attacks MLF because it is one of the most highly myelinated areas in the body); Charcot's triad - nystagmus, scanning speech, intention tremor A: Complications - Seizures, falls, pressure ulcers, aspiration, pneumonia A: Types (4) - Primary progressive, secondary progressive, relapsing progressive, relapsing remitting A: Tests - MRI brain, CSF cytology and chemistry (glucose, protein, IgG), evoked potentials A: Treatment - Interferon beta-1a/b, steroids,...

Discuss Amyloidosis

A: Definition - Disease that is characterized by the extracellular deposition of fibrillar proteins A: Diagnosis - Biopsy, light microscopy - Primary & myeloma = light chain Ig, Secondary = Amyloid associated protein; confirmed by electron microscopy A: Causes - Idiopathic (2/3), multiple myeloma (1/4), TB, RA, OM (1/10) A: Clinical - Primary = Tongue most commonly, then Orbit; Larynx (TVC, Ventricle, and FVC) most common site in the respiratory tract then trachea A: Complications - A: Types (4) - Primary systemic (56%, mesenchymal organs = heart, tongue, GIT), Myeloma associated (26%), Primary localized (9%), Secondary systemic (8%, due to chronic destructive disease = TB, RA, osteomyelitis; seen in kidney, adrenals, liver, spleen), or Secondary localized (Alzheimer's, Parkinson's Huntington's) A: Tests - Biopsy, shows apple green birefringence with Congo Red staining, reversal with potassium permanganate infers secondary amyloid; Multiple airway lesions are the rule, so perform a bronchoscopy on everyone; Need to Biopsy Bone Marrow where Plasma cells = Multiple Myeloma A: Treatment - Conservative removal of deposits, Steroids; Antimetabolites not helpful

Discuss Relapsing Polychondritis

A: Definition - Episodic recurrent inflammation of cartilages and other tissues high in glycosaminoglycans, eventually replaced by granulation and fibrosis A: DDx - A: Diagnosis - Three of the clinical features in the absence of histologic confirmation, two of these features with response to steroids or dapsone, or any one of these features with histologic confirmation A: Causes - Predilection for whites A: Clinical - Recurrent chondritis of the auricular (with sparing of the lobule), nasal, laryngeal or tracheal cartilages; seronegative nonerosive inflammatory polyarthritis, ocular inflammation, and cochlear (SNHL or tinnitus) or vestibular damage (vertigo) A: Complications - Nasal deformity, serous otitis media; death may result from tracheal collapse or CV disease; perform tracheotomy for severe glottic/subglottic edema, or laryngeal collapse A: Types - A: Tests - Increased ESR, Biopsy A: Treatment - Steroids, NSAIDs (arthritis), colchicine (auricular chondritis), dapsone; methotrexate or azathioprine (for recalcitrant)

Discuss Sulcus Vocalis

A: Definition - Epithelium-lined pocket whose lips parallel the free edge of the folds, may represent an epidermoid cyst that has spontaneously emptied A: Causes - History of voice overuse A: Clinical - Chronic hoarseness, diplophonia, aphonia at certain frequencies A: Types - Congenital or Acquired A: Treatment - SLP for overusers, Surgical options include injection, medialization thyroplasty, or microlaryngeal excision

Discuss Lemierre syndrome

A: Definition - IJV thrombophlebitis, usually due to dental or pharyngeal infections A: Diagnosis - Clinical & CT A: Causes - Fusobacterium necrophorum A: Clinical - Spiking fever, engorged optic disks, increased CSF pressure, SCM tenderness, neck stiffness, metastatic lung abscesses, septic arthritis; Griesinger's sign A: Complications - Retrograde spread of thrombus, intracranial hypertension A: Types - A: Tests - CT neck, Toby-Ayer/Queckenstadt test A: Treatment - Drainage of infection, ligation of IJV; anticoagulation controversial

Discuss Wegener's Granulomatosis

A: Definition - Idiopathic autoimmune disease, a triad of upper/lower airway necrotizing granulomas, systemic vasculitis, focal glomerulonephritis A: DDx - Churg-Strauss, NK/T cell or DLBC lymphoma; Goodpasture's A: Diagnosis - Clinical, Pathologic (nasal biopsy), lab findings (cANCA) A: Clinical - Pulmonary involvement >95%, Nasal involvement 90%, Renal involvement >85%; see remainder of H&N manifestations below A: Complications - A: Types (3) - 1) Limited, or E for ENT, 2) Systemic, or EL for Lung, 3) Widely dissemited, or ELK for Kidney A: Tests - CBC (anemia), increased ESR, CRP, c-ANCA (specificity 90% in systemic vasculitis stage, 65% in granulomatous phase, 30% in remission patients); Urine sediment, CXR, Nasal biopsy A: Treatment - "PCMA Sept-Iles" Prednisone (1mg/kg/day x 4 weeks, then taper), Cyclophosphamide (2mg/kg/day for 6-12 months); if hemorrhagic cystitis occurs may use Methotrexate or Azathioprine; Septra, IVIG in immunosuppresion nonresponders

Discuss Sarcoidosis

A: Definition - Idiopathic systemic noncaseating granulomatous disease, more common in African-American women A: DDx - A: Diagnosis - Biopsy = Noncaseating granulomas composed of Laghans giant cells containing laminated mucopolysaccharide Schaumann bodies and collagen Asteroid bodies; must R/O Infectious etiology A: Causes - Unknown etiology A: Clinical - Cervical adenopathy most common presentation (40%), uveoparotid fever, supraglottic mass, VC paralysis, nasal or orbital mass, septal perforation; Incidental hilar pulmonary LNs most common presentation overall, Darier-Roussy (subcutaneous) nodules, hepatic, renal, splenic, cardiac, bone, or nerve involvement A: Complications - SNHL, facial palsy, VC paralysis, blindness, lung disease, airway obstruction A: Types - Heerfordt's syndrome (Uveoparotid fever), Lupus pernio A: Tests - Biopsy, ACE & Ca levels, SPEP (hypergammaglobulinemia), CXR A: Treatment - Steroids for acute exacerbations; conservative surgery for obstructing laryngeal lesions

Discuss Laryngeal Diphtheria

A: Definition - Inflammatory reaction with vascular dilation A: Cause - Corynebacterium diphtheriae, a gram-positive pleomorphic aerobic bacillus A: Clinical - Exudative membranous tonsillitis with fever, dysphonia; Diphtheria exotoxin can cause Myocarditis and Neurologic sequelae (so get cardio consult) A: Treatment -Tracheostomy (not intubation, could dislodge exudate); Antitoxin; High dose Penicillin, Erythromycin or Tetracycline

Describe Granular cell tumors

A: Definition - Large polygonal cells in nests/strands/sheets; infiltrative, no distinct capsule; defining characteristic are abundant eosinophilic cytoplasm filled with granules of varying sizes (lysosomes in varying stages of fragmentation); keratin pearl formation in some cases (pseudoepitheliomatous hyperplasia) A: Diagnosis - Endoscopy with biopsy A: Causes - Arise from Schwann cells A: Clinical - Slow, painless growth; well circumscribed; Typically solitary, polypoid, sessile, papillary or cystic; Commonly found at the posterior aspect of the true VC, but Tongue is most common site of origin A: Complications - Recurrence (8%), malignant transformation (5%) A: Types (3) - Congenital, Mucosal, Gingival A: Tests - Biopsy stain positive for S-100 and PAS A: Treatment - Complete excision (endoscopic or open via laryngofissure or conservative resection)

Discuss Laryngocele/Saccular cyst

A: Definition - Laryngocele = Air-filled dilatation of the appendix of the ventricle; Laryngopyocele = Infected pus-filled laryngocele; Saccular cyst = Fluid-filled dilatation of the saccule without communication with the laryngeal lumen (can be subclassified as Anterior, Lateral, or Ductal) A: DDx - A: Diagnosis - Flexible laryngoscopy, CT neck A: Causes - Congenital or acquired expansion of the laryngeal saccule (in ventricle) from increased intraglottic pressure A: Clinical - Lateral compressible neck mass, increases in size with intralaryngeal pressure (external); Dysphonia and cough (internal) A: Complications - Secondary infection, rare malignancy, airway obstruction A: Types - Internal (remains within the thyroid cartilage, less common), External (protrudes through the thyrohyoid membrane, presenting as a mass in the neck), or Combined A: Treatment - Marsupialization, complete endoscopic (laser) removal for internal, open approach for external

Describe Carcinoid syndrome

A: Definition - Most common APUDoma, found usually in ileum & bronchi, high synchronous & metachronous rate, secretes serotonin A: DDx - A: Diagnosis - 24hr urine collection for 5-HIAA (serotonin metabolite) A: Clinical - Flushing, diarrhea, cardiac valve disease, wheezing A: Tests - 24hr urine collection for 5-HIAA, CT or MRI depending on site A: Treatment - Surgery, pharmacotherapy

Describe Usher syndrome and the 4 types

A: Definition - Most common cause of congenital deafness; Autosomal recessive except type IV = XR, 10% of hereditary deafness A: Diagnosis - Eye changes detected on Electroretinography even before funduscopic changes A: Clinical - Retinitis pigmentosa (progressive visual loss), atrophy of organ of Corti/Scheibe aplasia (congenital deafness), Ataxia and vestibular dysfunction common A: Complications - Deafness, blindness A: Type I - Profound congenital deafness, RP onset by age 10, no vestibular response, 90% of cases Type II - Moderate/severe congenital deafness, onset of RP in teens/twenties, normal or decreased vestibular response, 10% of cases Type III - Progressive HL, RP begins in puberty, <1% of cases Type IV - X-linked inheritance, similar to type II A: Tests - Electroretinography (measures potentials of retinas from light & visual stimuli), Opthalmology consult essential A: Treatment - Amplification

Discuss Acute Laryngotracheobronchitis (Croup)

A: Definition - Most common cause of stridor in children A: Ddx - URTI, supraglottitis, bacterial tracheitis, retropharyngeal abscess A: Diagnosis - Clinical and radiographic A: Causes - Parainfluenza type 1 (most common), 2 & 3, Influenza A, RSV, Rhinovirus, Measles, Adenovirus A: Clinical - URI prodrome, slow onset, affects 6 months - 3 years, variable/minimal fever, hoarse with barking cough, can develop respiratory difficulty with inspiratory stridor, better in supine position A: Complications - Obstruction, pulmonary edema, pneumonia, bacterial tracheitis A: Tests - Croup series, "steeple sign" on AP views A: Treatment - Expectant, humidification, racemic or levo-epinephrine (0.5 ml of 2.25% solution in 3cc NS), steroids controversial (decadron 0.6-1 mg/Kg), intubation if medical therapy fails, use ETT 0.5 mm smaller than estimated, extubate when air leak detected

Discuss the Mumps virus

A: Definition - Most common viral disorder of the salivary gland and most common cause of parotid enlargement A: Diagnosis - Blood tests, saliva, urine, and CSF viral cultures A: Causes - Paramyxovirus A: Clinical - Fever, malaise, myalgia, headache, either unilateral or bilateral parotid swelling; peak incidence 4-6 years, self-limiting; transmitted by respiratory droplets, incubation of 14-25 days after which time prodromal symptoms occur and last anywhere from 3-5 days A: Complications - uncommon, sudden SNHL, facial paralysis, VCP, meningitis, encephalitis, pancreatitis, orchitis, nephritis, arthritis A: Tests - Antibodies to hemagluttinin, mumps S & V antigens; virus isolation from urine A: Treatment - Supportive, vaccine for prevention

Discuss Achalasia; what infection causes the same symptoms

A: Definition - Neuromuscular degenerative disease of Auerbach's plexus, leading to non-relaxation/functional obstruction of the LES and esophageal distention A: DDx - Chaga's disease = systemic parasitic infection that destroys Auerbach's plexus, similar presentation as achalasia; Esophageal stricture/mass, Scleroderma, Zenker's A: Diagnosis - Radiographic esophageal distension, air-fluid level, aperistalsis, absence of LES relaxation, retention of barium, administration of smooth muscle relaxant to differentiate stricture A: Causes -Degeneration of inhibitory ganglion cells of Auerbach's plexus A: Clinical - Aperistalsis, esophageal dilation, failed LES relaxation; progressive disease leads to fibrosis & atrophy of muscle A: Complications - Retention esophagitis, perforation, esophageal carcinoma A: Types - A: Tests - Contrast esophagogram +/- smooth muscle relaxants (methacholine, nitroglycerin), esophagoscopy, esophageal manometry A: Treatment - GI consult; Early symptomatic = Ca-blockers or nitrates to stimulate LES relaxation, esophagoscopy with pneumatic dilation common procedure; Surgery = Heller myotomy for failed pneumatic dilatation or those at increased risk of dilation (previous perforation, epiphrenic diverticula, adjacent aortic aneurysm)

Discuss Schneiderian papilloma

A: Definition - Proliferation of squamous epithelium through fingerlike projections into underlying stroma A: Ddx - Benign vs. Malignant, Epithelial vs. Non-epithelial tumors A: Diagnosis - Biopsy A: Causes - HPV 6 & 11 suspected (inhibit p53) A: Clinical - White, males 3:1, 5th-7th decade; Nasal obstruction (87%), rhinorrhea, facial pain/pressure, epistaxis, frontal headaches, epiphora; Exophytic, fleshy, sessile vs. pedunculated, bony destruction/erosion common, tendency to recur A: Complications - Recurrence, Malignant change (~10%, lateral wall > septal) A: Types - Inverting (50%, lateral wall), Fungiform (47%, septal), Cylindrical/Oncocytic (3%) A: Tests - FOB scope, CT, MRI (iso-hyperintense on T2?), Biopsy A: Treatment - Total surgical removal with medial maxillectomy, either via Lateral rhinotomy or Transnasal/ESS approach

Discuss Retropharyngeal abscess

A: Definition - Purulent collection originating from the necrotic degeneration of a retropharyngeal node of Rouvier A: Ddx - URTI, croup, bacterial tracheitis, supraglottitis, parapharyngeal abscess, PTA A: Diagnosis - Clinical & radiographic A: Causes - Mixed bacteriae (Streptococci, S. aureus, H. influenzae, Bacteroides, Peptostreptococci, Fusobacteria) A: Clinical - URI prodrome in children usually <6 years, fever, sore throat, progressive dysphagia, drooling A: Complications - Airway compromise, mediastinitis, rupture & aspiration pneumonia, sepsis, arterial rupture, arterial septic emboli, jugular vein thrombosis, vertebral osteomyelitis, death A: Tests - Lateral soft tissue neck x-ray = Subcutaneous gas, widening of prevertebral tissues (>2x diameter of C2 body 90% sensitive, or >6 mm at C2 and/or >2 cm at C6), CT scan A: Treatment - Secure airway, IV antibiotics, possible OR drainage (transoral vs. transcervical)

Behcet's syndrome

A: Definition - Relapsing/remitting autoimmune disease triad = Recurrent aphthous ulcers of mouth, Recurrent painful ulcers of genitals, Uveitis or conjunctivitis A: DDx - IBD, Reiter's, Pemphigus, Infectious (HSV, EBV, HIV, Cox), Major apthous ulcers, Trauma, SCC A: Diagnosis - A: Causes - A: Clinical - Ulcerations of orogenital mucus membranes (painful puched out lesions, typically first symptom), Iritis/uveitis (in 60%, loss of sight in 25%), Progressive SNHL A: Complications - A: Types - A: Tests - A: Treatment - Corticosteroid cream for ulcers, Colchicine or Dapsone PO, myadriatics and corticosteroid drops for eyes

Discuss Fibrous Dysplasia

A: Definition - Replacement of Normal Medullary bone with Fibroosseous tissue A: Clinical - Presents before age 30; 25% occurs in H&N (mainly Maxilla); Lesions typically present as Painless enlarging bony swelling A: Types - Monoostotic (80%), Polyostotic (17%), McCune-Albright syndrome (5Ps, 3%, Polyostotic disease associated with triad of HyperPigmentation (cafe au lait spots), Precocious puberty & Endocrinopathy-Thyroid/Parathyroid) A: Treatment - Surgical Recontouring

Common features of Tension-type headache

A: Definition - Stress related bilateral continuous nonpulsatile headache A: DDx - Migraine, Cluster A: Diagnosis - Clinical A: Causes - Associated/precipitated by physical or psychological stress A: Clinical - Bilateral squeeking feeling, lasts days to weeks, rare associated symptoms (nausea/vomiting); can continue ADL's with headache A: Treatment - Somatic therapy (exercise, biofeedback), Medications (NSAID's, acetaminophen, NSAID combinations), Preventive (antidepressants, muscle relaxant)

Discuss Cat Scratch disease

A: Definition - Suppurative & necrotic granulomatous lymphadenitis caused by an intracellular pleomorphic gram negative bacillus, the most common cause of chronic lymphadenopathy in children A: DDx - KITTENS for neck mass... A: Diagnosis - 5 criteria = 1) History of cat exposure, 2) Presence of inoculation site, 3) Regional adenopathy, 4) Histologic features on excisional biopsy (suppurative & necrotic granulomatous lymphadenitis), 5) Bacteria seen on Warthin-Starry silver staining A: Cause - Bartonella henselae A: Clinical - Cutaneous papules at primary sites, tender lymphadenopathy that later is painless and lasts for months, mild fever/malaise, pustulous lesion tend to ulcerate/fistulize; Self-limited A: Complications - Fistula formation (avoid I&D), Parinaud Oculoglandular syndrome (unilateral conjunctivitis and regional lymphadenitis), Bacillary angiomatosis (also caused by B. quintana, in immunocompromized) A: Tests - Excisional biopsy with Warthin-Starry staining; Cat scratch antigen test not useful A: Treatment - Supportive, Azithromycin in immunocompromized patients

Discuss Plummer-Vinson syndrome

A: Definition - Syndrome seen in Caucasian females typically of Scandinavian descent A: Clinical - Dysphagia, esophageal stricture, glossitis, iron deficiency anemia, cheilosis, koilonychia (nails are flattened and have concavities), splenomegaly, SCC of the postcricoid region & cervical esophagus A: Tests - CBC, contrast esophagogram, esophagoscopy,... A: Treatment - Endoscopic web rupture, iron replacement

Discuss Otopalatodigital syndrome

A: Definition - XR A: Causes - Mutation localized to Xq28 A: Clinical - CHL due to ossicular malformation, Cleft palate, Stubby/clubbed digits, Craniofacial deformities (hypertelorism, supraorbital deformity, flat midface, small nose), short stature, wide space between 1st & 2nd toe

Discuss Norrie syndrome

A: Definition - XR A: Causes - Mutation of NDP gene on Xp11.4, produces norrin protein, structurally similar to TGF-ß A: Clinical - Progressive SNHL (2nd/3rd decade), occasional progressive mental deterioration, congenital/rapidly progressive Blindness due to pseudoglioma, exudative vitreoretinopathy, opacification, ocular degeneration A: Complications - A: Types - A: Tests - A: Treatment -

Describe Wildervank syndrome

A: Definition - XR, Klippel-Feil malformation, Brevicollis, mode of inheritance unclear A: Clinical - SNHL or mixed loss, due to bony inner ear (Mondini) and middle ear malformation, CN VI paralysis with eye retraction on lateral gaze (Duane retraction syndrome), short neck/fused cervical vertebrae, Assimilation of the Atlas (Basilar impression), Spina bifida

Three Indications for Conservative Treatment in CN VII nerve injury with temporal bone Fracture

A: Delayed onset - Documented normal facial nerve function after injury, regardless of progression A: Incomplete paralysis at presentation that doesn't progress to complete A: Complete paralysis (either from outset or progression to) with <90% degeneration on ENoG

Causes of congenital hypoparathyroidism (due to agenesis)

A: Deletion 22q11.2 spectrum - DiGeorges, Velocardiofacial syndrome

Cri-du-Chat syndrome

A: Deletion chromosome 5p (1/50 000 births) A: CNS - Microcephaly, 1% profound MR, hypotonia, CVD? A: ENT - High pitched stridor (cat cry), Hypertelorism, Broad nasal root, Cleft lip/palate A: Laryngoscopy - Elongated/narrowed diamond-shaped endolarynx, interarytenoid muscle paralysis, Cleft?

Five reasons for Angiography in the workup of Carotid Body tumors

A: Delineate the primary blood supply to the tumor A: Identify any collateral feeding vessels A: Evaluate the feasibility of pre-op embolization A: Evaluate for the presence of carotid involvement A: Evaluate for the presence of multifocal disease

Describe the Approaches used in surgery of the nasal tip

A: Delivery - For more abnormal or asymmetric tip anatomy, allows presentation of the alar cartilages as bipedicled chondrocutaneous flaps for surgical modification A: Nondelivery - For conservative/minimal tip refinement, disturbs little of the normal anatomy (Cartilage splitting, or Retrograde) A: Open - External

Histiologic classification of Adenocarcinoma

A: Depends on degree of glandular structure cellular differentitation A: Grade I - Well formed ductal structures A: Grade III - Solid growth pattern with few glandular characteristics

Pharmacologic action of Mucolytics (N-acetylcysteine)

A: Depolymerizes mucopolysaccharides, making them more soluble

Four muscles innervated by marginal mandibular nerve

A: Depressor anguli oris A: Depressor labii inferioris A: Inferior Orbicularis oris A: Mentalis

Condition of positive Skin Endpoint Titration with negative control solution

A: Dermatographia 3: Can do RAST testing for these patients (not affected by anti-histamines)

Most common benign and malignant orbital tumors in children

A: Dermoid A: Rhabdomyosarcoma

Ddx of pediatric midline nasal mass

A: Dermoid cyst (most common) A: Neurogenic - Glioma, Encephalocele, Neurofibroma A: Hemangioma

Define the Jod-Basedow phenomenon

A: Development of overt hyperthyroidism in subclinical patients due to exogenous iodide administration

Seven Characteristics of the unilateral Cleft Lip Nasal deformity

A: Deviation of nose to affected side A: Lower Dome A: Flattened or crimped Lateral Crus A: Caudal displacement of Alar Rim A: Horizontal nostril axis A: Loss of nasal Sill A: Depressed or missing Bony Floor

What are the equivalent doses for Steroids

A: Dexamethasone (Decadron) 0.75 mg A: Triamcinolone (Kenalog) 4 mg A: Methylprednisolone (Medrol, Solumedrol) 4 mg A: Prednisone/Prednisolone 5 mg A: Hydrocortisone (Solucortef) 20 mg A: Cortisone 25 mg A: Remember "DexTer Might Predate His Chief" and 0.75, 4, 4, 5, 20, 25

Discuss Hereditary Angioedema

A: Diagnosis - Clinical, Family history, Lab tests A: Ddx - Non-hereditary angioedema = Allergy, Infection, Drug reaction, Autoimmune, Systemic disease, Thyroid, Idiopathic, Angioneurotic edema A: Cause - Autosomal dominant C1 esterase inhibitor deficiency A: Clinical - Episodic facial/pharyngeal/laryngeal angioedema WITHOUT urticaria, abdominal pain common; Triggers = stress, infection, oral/dental manipulation A: Types - Acquired C1 esterase inhibitor deficiency (elderly, haematologic malignancies, autoimmune, estrogen?) A: Tests - C4 and C1e inhibitor levels +/- C1e inhibitor functional assay A: Treatment - ABC, C1 esterase inhibitor replacement (Berinert P or FFP) for acute; Tranexemic acid and Danazol for long-term prevention (raises C1 esterase inhibitor level)

Discuss Eosinophilic Esophagitis

A: Diagnosis - Esophagoscopy (furrows, rings/trachealization, exudates), Biopsy proven local eosinophilic infiltrate (>24/HPF), and Allergy testing A: Clinical - Young atopic teens/adults, 70% males, refractory GERD, dysphagia with sensation of food getting stuck A: Treatment - Food allergy evaluation, Hypoallergenic diet (trial elimination diets to try to figure out aggravators but is difficult), swallowed topical steroids (swallow inhalers, also trying liquid budesonide in viscous gel) , treat other allergies

What 3 Drugs can alter the ABR? (DPL)

A: Diazepam A: Phenytoin A: Lidocaine

Five treatment options for Vocal cord granuloma

A: Dietary and lifestyle modifications - Cut out coffee A: Vocal hygiene A: Voice therapy A: Antireflux medications - longterm, PPIs bid A: Botox injection - 12 U on side of lesion, 3-5 units contralaterally to prevent overcompensation A: Intralesional vs. systemic corticosteroids (controversial) A: Microlaryngeal Surgery - Cold steel vs. Endoscopic CO2 excision; rarely indicated, only if very symptomatic, may recur

Criteria defining a Second Primary

A: Different Histology A: Different Location A: Each one has its own Metastases 3: Overall incidence of 2nd primary cancer is ~10-15%, of which 80% are metachronous and 50% present in the first 2 years

Define Metaplasia

A: Differentiation of one cell into another

Four benefits of MRI for sinuses

A: Differentiation of soft tissue involvement A: Differentiation of soft tissue from fluids A: Multiplanar capabilities with minimal patient movement A: No exposure to radiation

Graves disease triad

A: Diffuse toxic goiter A: Infiltrative ophthalmopathy A: Infiltrative dermopathy (pretibial myxedema)

Name the four muscles controlling laryngeal elevation

A: Digastric - Anterior (V) & Posterior (VII) bellies A: Mylohyoid (V) A: Stylohyoid (VII) A: Geniohyoid (C1)

Therapy for Severe Acute Vertigo

A: Dimenhydrinate 50mg IM q6h A: Lorazepam (Ativan) 0.5-2 mg PO x 1 - Can be abortive for early attack of Meniere's

Define Atrophy

A: Diminution in the size of cells, organ, or tissue after a stage of full development

List 3 First generation H1 Antihistamines

A: Diphenhydramine/Benadryl A: Hydroxizine/Atarax A: Chlorpheneramine/Chlortripolon 3: Sedating; "Diphen-Hydroxy-Chlorphen"

Radiotherapy mechanisms of cell injury (2)

A: Direct injury - Electron from x-ray absorption causes DNA Damage (1/3) A: Indirect injury - Electron from x-ray creates an Oxygen Free Radical which then damages the DNA (2/3)

Microphones options for Hearing Aids

A: Directional A: Omnidirectional

What are the 3 important aspects to informed consent as per the Royal College?

A: Disclosure - Information about the risks and benefits of the proposed test or treatment as well as any alternatives, presented in language that the patient can understand A: Capacity - The ability to understand and appreciate the consequences of a particular decision or lack of decision; If in doubt, consultation from a psychiatrist, hospital attorney, or ethicist may be helpful, the ultimate judge of a patient's capacity is court; If the patient is incapable, seek consent from the appropriate substitute decision maker A: Voluntariness - The ability to make treatment choices without undue external coercion

Most common site of CN VII Damage from Temporal bone Trauma and 3 reasons for it

A: Distal Labyrinthine segment and Geniculate Ganglion A: Small size & lack of fibrous Supporting tissue A: Traction between GSPN and Geniculate Ganglion A: Watershed areas of vascularization

Define Dysosmia

A: Distorted or perverted smell perception

Describe the blood flow to the nasal tip

A: Dorsal nasal artery (2) - terminal branch of the Ophthalmic artery, runs along dorsum near the midline A: Lateral nasal artery (2) - branch of the Facial artery A: Columellar artery (2) - branch of the Superior Labial artery (side note: septal branch of sup labial supplies septum just behind columella)

Nasal tip defect of less than 1.5 cm - alternative to paramedian forehead

A: Dorsal nasal flap - Advancement flap of dorsal nasal skin based on the angular artery (dorsal nasal artery?); can have an element of rotation from the nasal sidewall

List the 6 aesthetic subunits of the nose (3 are paired)

A: Dorsum A: Sidewalls (2) A: Tip A: Nasal ala (2) A: Soft triangle (2) A: Columella

Most common vascular malformation causing tracheal compression with stidor

A: Double aortic arch

What is the Anastamotic Vein of Labbe?

A: Drains the Temporal Lobe A: Communicates the Superficial Middle Cerebral vein with the Transverse Sinus medial to Superior Petrosal sinus A: Occlusion will cause Temporal Lobe Edema, resulting in speech and language deficits, possibily followed by coma and death

Environmental factors contributing to cleft palate

A: Drugs - Phenytoin, Thalidomide, Vitamin A derivatives, Folic acid antagonists A: Smoking & Alcohol use in 1st trimester A: Amniotic band syndrome, maternal diabetes

Tests for GERD

A: Dual pH probe - Short term or Long term (gold standard) A: Barium swallow A: Radionuclide Reflux scan A: Esophagoscopy & biopsy A: Manometry (although 30% will be normal) A: Lipid laden macrophage test - BAL A: Acidification tests - Rarely used; Bernstein test instills N/S for 15 minutes via NGT, then 0.1M HCl @ 6cc/min until symptomatic or for 45 minutes; Tuttle test in children is similar, uses age-depedent HCl instillation, pH <3 x 2 occasions is positive

Appearance of Oral hairy leukoplakia, what is the cause, chance of developing AIDS, 4 other oral findings in AIDS?

A: Due to EBV infection and typically seen in immunocompromised individuals A: Chances of developing AIDS with HIV positivity = 50% at 16 months, 80% at 30 months, 100% at 60 months A: White, vertically corrugated asymptomatic lesion on anterolateral tongue A: Oral candidiasis, Kaposi's sarcoma (HHV-8), HSV, CMV infections 3: Can treat with Acyclovir if symptomatic, but usually recurs (mainly asymtomatic and don't require treatment - it is more of a barometer signaling declining immune status especially in HIV)

Discuss Atmospheric Inner ear Barotrauma

A: Due to significant sudden pressure changes placed on the inner ear A: Seen either with Implosion (RW/OW displaced into labyrinth) or Explosion (increased ICP transmitted to inner ear or ME pressure drops suddenly and RW/OW sucked out) A: Hearing loss, Tinnitus most common complaints, Vertigo occurs less frequently; often permanent or long lasting A: Conservative treatment as spontaneously resolves most of the time (bedrest, head elevation); explore for HL progression or failure of symptom resolution in 3-5 days

Classic shape of Epidermoid

A: Dumbbell (into MCF) A: Can also extend into contralateral Cistern

The 2003 Task Force criteria for the Diagnosis of Chronic Rhinosinusitis

A: Duration >12 weeks AND EITHER OF... A: Discolored discharge, polyp, polypoid swelling on anterior rhinoscopy or endoscopy A: Edema/erythema of the MM on endoscopy A: Edema/erythema/granulation in the nasal cavity, but if somewhere else than the MM, imaging is required A: Imaging confirmation (plain films or CT)

Embryology of the tracheoesophageal system

A: During the fourth week (26 days) A: Laryngotracheal groove appears as median outgrowth from the caudal end of the ventral wall of the primitive pharynx A: Laryngotracheal diverticulum forms by 28 days A: Tracheoesophageal folds fuse into a septum that separates the laryngotracheal tube A: Arytenoid swellings from neural crest cell-derived mesenchyme of the 4th and 6th arches A: Laryngeal epithelium occludes lumen at 8th week, and recanalization occurs by 10th week.

How to Audiologically determine whether a lesion is Cochlear in origin (tests of Recruitment)

A: Dynamic range between SRT and Discomfort level with speech normally = 95 dB A: Alternate Binaural Loudness Balance (ABLB) A: Short Increment Sensitivity Index (SISI) A: Metz recruitment test <60 dB between PTA threshold & acoustic reflex threshold; normally = 85 dB A: Carhart tone decay 0-25 dB above threshold A: Bekesy type 2 curve 3: OAEs are also useful to detect cochlear loss

Ddx of Variable CHL

A: EAC Collapse A: Increased Perilymph Pressure A: Inner ear conductive hearing loss A: Ossicular Chain Discontinuity A: Superior Semicircular Canal Dehiscence Syndrome

Five complications in Atresia surgery

A: EAC stenosis A: Recurrent/persistent CHL A: SNHL A: Facial nerve injury A: Chronic infection 3: Risks minized by near normal CN VII course, middle ear and mastoid at least 2/3 normal size

Indications for postoperative Chemo with XRT

A: ECS A: Positive margins A: Multiple positive nodes A: T3 or T4 primary A: Perineural or vascular invasion A: Oral/Oropharyngeal primaries with level IV or V nodes

Ten parameters possibly measured in polysomnography

A: EEG A: EOG A: ECG A: EMG - Submental and tibial A: O2 Saturation A: Nasal/Oral airflow A: Respiratory movement A: Position A: BP A: Esophageal pressure (+/-, usefull for detection of RERAs)

Which 4 tests should be routine when no other etiology of childhood SNHL is found?

A: EKG A: RPR A: CT A: GJB2 3: In addition to CBC, SMA7 (may also detect leukemia, Fechner, Alstron, and Alport)

Vestibular testing in Acoustic Neuromas

A: ENG Abnormal in ~80%, usually Unilateral Weakness A: Tumors arising from Inferior Vestibular nerve will be missed

Width of face divided into fifths, where are vertical lines placed?

A: Each fifth = ~1 eye width, and ~ nasal width A: At the edge of each eye, from medial canthus to lateral canthus A: Medial canthus roughly equal to lateral edge of nasal alae

General advice on Hearing Protection

A: EarPlugs - Reduce noise by 15-30 dB, work best in 2-5 kHz range A: EarMuffs - More effective protectors, reduces noise by 30-40 dB, works best in 500-1000 Hz range 3: Must be worn at all times ≥85 dB; removal even for short periods severely reduces their Effective Cumulative Attenuation Capability (eg. 30 dB effective attenuation reduced to 13-15 dB if earmuffs removed for 5% of an 8 hour day)

Six Audiogram pattern/findings of Otosclerosis

A: Early - Low frequency loss due to Stiffness effect A: Late - Flattening of AC line due to Mass effect A: Later - May see involvement of Cochlea with High frequency SNHL (Basal turn most commonly afftected) A: If primarily cochlear otosclerosis, "cookie bite pattern" with a Mixed hearing loss pattern A: Carhart notch A: Typical Max CHL = 50 dB

Management of hypopharyngeal carcinoma

A: Early - XRT, Surgery, or both A: Late - Surgery + postop Chemo-XRT

Ddx of a Reverse-Sloping (Rising, or Low frequency) Hearing Loss (2)

A: Early Menieres A: Sudden SNHL

Side effects of Interferons

A: Early constitutional - Fatigue, headache, nausea, fever, chills A: Delayed - Leukopenia, alopecia, neurotoxicity, polymyalgia, arthralgia, growth retardation A: Rare - Spastic diplegia

Treatment of Hodgkin's lymphoma

A: Early disease (I & IIA) - XRT, 10 year survival 90%, 10 year relapse free survival 75-80% A: Advanced disease - Combination chemotherapy; complete response rate 44-87%, long term disease free survival rate 50% A: Children - Combined modality therapy equally effective while causing less growth impairment

Describe the allergic reaction at the cellular level

A: Early response (~5 minutes post exposure) - mediated by Mast cells & Basophils; IgE binds to FceR on mast cell, crosslinking of IgE triggers activation = Degranulation of mast cell's preformed mediators (histamine, heparin, tryptase, beta-glucosaminidase, eosinophil & neutrophil chemotactic factors), and Synthesis of mediators from membrane bound phospholipids (PGs, LTs, PAF) A: Late response (~4 hours post exposure) - mediated by neutrophils & eosinophils, reaction secondary to cytokines

Five Indications of Laser for SGS

A: Early stenosis A: Grade I, II A: Granulation tissue A: Thin webs A: Crescent-shaped bands 3: "Early mild soft thin crescents"

What is Greisinger's sign

A: Edema at mastoid tip due to thrombus of sigmoid sinus

CT and MRI criteria for diagnosis of Carotid invasion in HNSCC

A: Effacement of the Fascial plane surrounding greater than 25% of the vessel Circumference A: Tumor involvement of 75% or more of the Circumference of the carotid 3: Best modality between CT, MRI, or U/S remains controversial; MRI = 100% sensitive, 94% specific

Three options for Sclerotherapy

A: Effective for lymphatic and venous malformations A: Ethibloc (95% ethanol with starch) A: Sodium tetradecyl sulphate A: OK-432 (lyophilized low-virulence Strep Pyogenes, macrocystic = 92% response microcystic = 44%)

Five Indications for Radiotherapy in Melanoma

A: Elderly poor candidate for operative resection A: Extensive Facial LMM which would require significant resection A: Postop adjuvant for High risk lesions, defined as >1.5 mm depth or ulcerated lesion A: Positive Regional Metastasis A: Desmoplastic histology

Four lab findings with tertiary hyperparathyroidism

A: Elevated PTH A: Low or normal Ca A: Low Vit D A: Hyperphosphatemia

Name the important muscles of the nose (NB exam asked for 4 elevators and 4 depressors)

A: Elevators Procerus Levator Labii Superioris Alaque Nasi Anomalous Nasi A: Depressors Alar Nasalis Depressor Septi Nasi A: Compressors Transverse Nasalis Compressor Narium Minor A: Minor dilators Dilator Naris Anterior & Posterior

Three treatment strategies for JNA

A: Embolization (ideal within 48 hours of excision) A: Surgery (mainstay) A: XRT

What is the Definition of Congenital Cholesteatoma as per Levenson? (NOPE)

A: Embryonic rest of epithelial tissue in the ear without tympanic membrane perforation and without a history of ear infection A: Normal Pars Flaccida and Pars Tensa A: No history of prior Otorrhea A: No history of prior otologic Procedures 3: Prior episodes of otitis media without otorrhea are not grounds for excluding congenital origin

Discuss Antiemetics

A: Emetic center in medulla (reticular formation); ACh & Histamine receptors present A: Chemoreceptor trigger (CRT) zone in 4th ventricle; Dopamine receptors present A: Phenothiazines work well - Antidopaminergic and anticholinergic A: Metoclopramide, prochlorperazine, haldol - Antidopaminergic A: Odansetron & granisetron - 5HT3 receptor antagonist, found on vagus in CRT zone

Contents of EMLA

A: Emulsified Prilocaine and Lidocaine, reliability increases if applied for >1hr

Histologic subtype of PTC with better prognosis

A: Encapsulated 3: Others include papillary microcarcinoma (<1 cm), follicular, diffuse follicular & macrofollicular

Ddx of Unilateral Pediatric Nasal Mass

A: Encephalocele A: Glioma A: Neurofibroma A: Nasolacrimal duct cyst A: FB/Rhinolith A: JNA A: Hemangiopericytoma A: Hemangioma A: AVM A: Polyp

Name the 3 measurable Potentials in Electrocochleography (ECoG), their source and there clinical utility

A: Endocochlear Potential - DC response from the Stria Vascularis maintaining the electrolyte balance in the scala media, producing a resting potential around +80 mV A: Cochlear Microphonic - AC response from hair bearing surface of Hair Cells (outer hair cells), mimics waveform of acoustic signal; not clinically useful (useful for diagnosing ANSD when OAE's not present) A: Summating Potential - DC response from Hair Cells, a direct current shift, due to asymmetry in Basilar membrane movement (essentially inner hair cells) A: Compound Action Potential - Averaged response of discharge pattern of Auditory Neurons (Similar to ABR wave 1) 3: Ratio of SP:AP increases >50% with Hydrops; 2/3 of Meniere's patients will have an increased SP:AP ratio; also seen in Syphilis

Describe the Endocochlear and Hair cell potentials

A: Endocochlear potential - Produced by Stria Vascularis, resting potential of endolymph, +80-90 mV relative to Otic Capsule bone (perilymph) A: IHC potentials ~ -35 mV A: OHC potentials ~ -70 mV

Nine Surgical options of Tracheal Stenosis

A: Endoscopic (Balloon or serial dilatations, Cold knife, Laser, Stents) A: Augmentative (Tracheoplasty with Rib cartilage, Pericardium) A: Resection and Anastomosis (Wedge resection, Segmental resection <50% tracheal length, Slide tracheoplasty) A: Cadaveric Homograft with Dumon stent

Ten possible Surgical approaches for JNA excision, from least to most invasive

A: Endoscopic transnasal A: Transmaxillary (Transantral?) A: Transpalatal A: Lateral rhinotomy A: Medial maxillectomy A: Midfacial degloving +/- LeFort I A: Facial translocation (Maxillary swing?) A: Infratemporal fossa (Fisch C?) A: Subcranial

Age 1 and recurrent croup - what investigation

A: Endoscopy - R/O subglottic stenosis

Characteristics of the Carbon dioxide laser

A: Energy strongly absorbed by water independent of tissue color A: Minimal thermal effect on surrounding tissues A: Disadvantage - cannot pass through fiberoptics

Four symptoms of Obstructive Tonsillar hyperplasia

A: Enlarged tonsils A: Snoring A: Obstructive disturbances A: Dysphagia and voice changes

Seven concerns on history, physical, or testing for diagnosis of thyroid cancer

A: Enlarging A: Size >3 cm A: Dysphagia A: Hoarseness A: Cervical adenopathy A: Fixation to skin A: Microcalcifications on U/S

Eight Indications for open biopsy of lymphadenopathy in HIV = FNA that suggests Malignancy, or FNA negative for malignancy with any of:

A: Enlarging node A: Nodes >2 cm A: Asymmetric, localized, unilateral adenopathy A: Significant mediastinal or abdominal lymphadenopathy A: Failed antibiotic trial A: Low CD4 with new lymphadenopathy A: B symptoms - Fever, night sweats, weight loss

Seven Histopathologic findings in CRS

A: Eosinophilic and Lymphocytic infiltration A: Major Basic Protein deposition A: Basement membrane thickening A: Subepithelial edema/fibrosis A: Goblet cell hyperplasia A: Mucous hypersecretion A: Submucosal gland formation

Four histopathologic findings of Eosinophilic Mucin in fungal sinusitis?

A: Eosinophils A: Charcot-Leyden crystals (byproduct of eosinophil degranulation) A: Fungal elements & Hyphae A: Necrotic inflammatory cellular debris

Name 8 topical decongestants

A: Ephedrine A: Epinephrine A: Phenylephrine A: Oxymetazoline A: Xylometazoline A: Cocaine A: Naphazoline A: Tetrahydrozoline A: Propylhexedrine

Skull Base lesions associated with Facial Twitching/Tics

A: Epidermoid A: Facial nerve Schwannoma A: Facial nerve Hemangioma

Seven Infectious causes of laryngeal stenosis

A: Epiglottitis A: Croup A: RRP

Ddx of orbital tumors, primary and secondary

A: Epithelial - Dermoid cyst, simple epithelial (conjunctival) cyst, mucocele, lacrimal (50% benign = PA, 50% malignant = ACC, MEC, mixed, AdenoCA), SCC, BCC (90% of eyelid tumors), mets A: Mesenchymal - Fibrous dysplasia, rhabdomyosarcoma, osteoma, osteosarcoma A: Vascular - Hemangioma (cavernous or capillary), lymphangioma A: Neural - Meningioma, schwannoma, optic nerve glioma A: Hematopoietic - Lymphoma, leukemia (Chloroma), histiocytosis A: Inflammatory - Pseudotumor, Grave's (most common in adults), infection (viral, bacterial, fungal), vasculitis (WG, PAN, TA, SLE) 3: Epithelial Fat Fibrous Muscle Bone Cartilage Vessel Nerve and White cells

Dermoid

A: Epithelium lined, contains skin appendages, sinus tract leading to the skin A: Dural connection in 30%

Seven Etiologies of pseudomembranous tonsillitis

A: Epstein-Barr virus (mononucleosis) A: Group A ß-hemolytic Streptococcus A: Corynebacterium diphtheriae A: Neisseria gonnorheae A: Syphilis A: Candidiasis A: Vincent's angina

Five symptoms of Acute Tonsillitis

A: Erythematous exudative tonsils A: Sore throat A: Dysphagia A: Tender cervical adenopathy A: Fever Centor Criteria: tonsillar exudate, tender cervical adenopathy, fever, absence of cough

Ddx of a red lesion in the oral cavity

A: Erythroplakia A: Pyogenic granuloma A: Papilloma A: Inflammatory papillary hyperplasia A: Pigmented nevi A: Kaposi's sarcoma A: Leukemia A: Hemangiosarcoma A: Mycosis fungoides A: Polycythemia rubra vera

5 TEF types in descending order of frequency

A: Esophageal atresia (EA) & distal TEF (85%) A: Isolated EA (10%) A: Isolated true TEF (H-type, 4%) A: EA with proximal TEF (0.5-1%) A: EA with double TEF (0.5)

Anomalies associated with Laryngeal Clefts

A: Esophageal atresia +/- TEF (#1, 6% of TEFs have clefts, continued aspiration) A: Cleft lip/palate A: CHD, GI anomalies A: G syndrome (Opitz-Friass) - Hypertelorism, Hypospadias, Cleft lip/palate A: Pallister-Hall syndrome - Bifid epiglottis, Hypothalamic hamartoblastoma, Hypopituitarism, Imperforate anus, Postaxial polydactyly

Three situations of Cupula becoming affected by gravity

A: Ethanol intoxication - PAN 1 = Geotropic; PAN 2 = Apogeotropic A: Heavy water poisoning = Apogeotropic A: Cupulolithiasis - Apogeotropic

Etiology and types of RRP

A: Etiology - HPV infection (6 & 11) A: Juvenile onset (<12 years) - More common & more aggressive, typically diagnosed by 2-4 years, vertical transmission from mother during parturition (risk 1:80-1:500) A: Adult onset (>12 years) - Peaks between 20-40 years

Discuss Bell's palsy

A: Etiology - Idiopathic disease, attributed to Viral (most likely), Vasa vasorum failure, Ischemic neuropathy, or Autoimmune A: Clinical - Complete unilateral paralysis of Sudden onset (usually over 2 weeks); often viral Prodrome, Hyperacusis, decreased Tearing, Numbness or pain of ear/face/neck; Dysgeusia A: Pathology - Diffuse Demyelination throughout Intratemporal course, worst at Meatal & Labyrinthine portions A: Diagnosis - One of exclusion, rule out any Trauma, Infection, Neoplasm A: Tests - See list for Facial palsy W/U A: Prognosis - Incomplete paralysis (30%) have ~95% complete recovery without intervention; Complete paralysis (70%) prognosis depends on speed of degeneration(overall 60% have good recovery); >90% degeneration by 14 days will recover to HB Grade I/II 50% of time (Fisch) without intervention A: Medical Treatment - If within 14 days of onset Steroids 1 mg/kg qd over 10 days with taper (CONSENT for side effects, and give with PPI) Valacyclovir 1000 mg or Famvir 750 mg, tid x 7 days; reduces degeneration and lower incidence of unsatisfactory recovery (alt Acyclovir 800 mg 5x/day x 7 days) A: Surgical Treatment - Indications for decompression = >90% Degeneration of motor fibers on ENoG by 14 days; Maximal nerve injury occurs at Meatal foramen/Labyrinthine segment, best approached through Middle Fossa, unroofing labyrinthine segment +/- incising perineureum

Distinguish euthanasia and assisted suicide from decisions to forego treatment

A: Euthanasia and assisted suicide involve the injection of a lethal substance or the provision of a lethal overdose A: Decisions to forego treatment involve the non-initiation or discontinuation of a life-sustaining treatment such as CPR, ventilator, tube feeding, etc. The law permits discontinuation, even though it leads to death, under defined conditions

Four indications for imaging in pediatric hearing loss

A: Evaluation for cochlear implantation A: Recurrent meningitis A: Sudden or progressive SNHL (especially with head trauma) A: Impact on counseling (atresia?)

Define EXIT procedure

A: Ex utero intrapartum treatment procedure for babies with airway compression, partially delivered through a C-section but remain attached by their umbilical cord to the placenta, while a pediatric or neonatal general surgeon establishes an airway so the fetus can breathe, the umbilical cord is cut and clamped, and the infant is fully delivered. Then the remainder of the C-section proceeds

Describe Videostroboscopy

A: Examines vibratory nature of vocal cords during phonation A: Strobe controlled by pitch of phonation or a frequency generator

A 43 year old woman has recent onset of episodic vertigo associated with Tinnitus, ear fullness and fluctuating hearing loss in the right ear. She is seen in the Emergency Room with right beating nystagmus. Bithermal caloric testing shows right directional prepoderence of 32 % and 8% left canal weakness. What is the pathophysiologic explanation of this finding?

A: Excitatory phase of right Ménière's disease

Discuss Tissue expansion

A: Expanders - Round, Rectangular, Crescentic; Match geometric shape of expander to the shape of the expansion site & planned flap A: Area of expander ~2.5x that of the defect to be closed; amount of flap advancement either half the base diameter of the expander, or half the diameter of the expansion dome A: Expansion starts ~3 weeks after implant insertion, 6-12 weeks to reach maximum expansion A: Titrate to slight blanching

Describe Furstenburg's sign

A: Expansion of a nasal mass with compression of the IJV's, associated with encephalocele, but not glioma or dermoid

Internal Nasal valve & Cross sectional area

A: Extends from caudal end of upper lateral cartilages to anterior end of inferior turbinates A: Influenced by nasal septum, upper lateral cartilage, pyriform aperture, anterior inferior turbinate A: Cross sectional area 0.73 cm2 3: Nasal valve competency tested with the Cottle maneuver

9 anatomic relationships of a 2nd branchial arch anomaly

A: External opening along lower third of SCM A: Internal opening associated with posterior pillar in tonsillar fossa A: Deep to platysma, CN VII, external carotid A: Superficial to stylopharyngeus, CN IX, X, XII, & internal carotid

Five causes of mandible ankylosis

A: External trauma A: Infectious arthritis A: Rheumatoid arthritis A: Condylar degeneration A: Osteoradionecrosis

Diagnosis of Macroglossia - any of 3 following criteria

A: Extravasation of lingual apex or lingual border onto or outside the dentition A: Impression of ≥ 1 teeth on lingual border visualized when mouth is open A: Following Sx for correction, a relapse of increased interdental space, open bite deformity, and/or jaw deformation with malocclusion occurs

Head and Neck manifestations of Sarcoidosis

A: Eye - Episcleritis, uveitis A: Salivary glands - Parotid swelling A: Neck - Cervical lymphadenopathy A: Oropharynx - Tonsillar hypertrophy A: Nose - Lupus pernio, inflammation, septal perforation A: Larynx - Epiglottic swelling and subglottic stenosis A: Neuropathies - CN VIII (sudden SNHL), CN VII (unilateral or bilateral facial palsy), CN X (VC paralysis)

Five Indications for surgeries for severe laryngomalacia (10%)

A: FTT, weight loss, feeding difficulty A: Life-threatening episode, cyanotic attacks, respiratory distress, documented desaturation A: Cor pulmonale, pulmonary hypertension A: Obstructive sleep apnea A: Severe chest deformity

Three Cranial Nerves most often involved in Necrotizing OE

A: Facial (75%) A: Vagus (70%) A: Accessory (56%) 3: "7-10-11"

Name and describe 4 Brainstem Nuclei associated with CN VII

A: Facial Motor nucleus - Sends SV Efferents to muscles of Facial expression, Stylohyoid, Posterior digastric, Stapedius A: Nucleus Tractus Solitarius - Receives SS Taste Afferents from oral tongue A: Superior Salivatory nucleus - Sends GV Efferents to Lacrimal, Sublingual, Submandibular, and Nasal Glands A: Spinal Nucleus of Trigeminal - General sensory afferents from EAC/Conchal skin

Looking at the CPA, starting superiorly, what nerves do you see and which are the most superficial?

A: Facial N and Nervus intermedius (both of these are deep to Cochlear and vestibular N A: Vestibular N (separated by a blood vessel from the Cochlear N) A: Cochlear N 3: Starting superiorly: Face, Vest, Cock (sorry again girls)

Tonsil blood supply

A: Facial artery (Tonsillar & Ascending Palatine branches) A: Dorsal lingual A: Internal maxillary artery (Descending/Greater palatine branch) A: Ascending pharyngeal artery

Name 4 other clinical uses for Botox in the head and neck

A: Facial dystonias (eg. blepharospasm, torticollis) A: Forehead rhytids A: Ptyalism (drooling) A: Frey's syndrome

Seven signs or symptoms which would support a malignant diagnosis for parotid mass

A: Facial nerve paralysis/paresis A: Fixation to overlying/underlying structures A: Overlying skin ulceration/infection A: Localized pain or trismus A: Cervical adenopathy A: Metastatic disease A: Large size (>5cm)

What sequences of surgeries are used when considering facial nerve reanimation?

A: Facial nerve reanastomosis (best <3 days, acceptable <30 days) A: Cable grafting (same time frame as anastamosis) A: Nerve Crossover techniques A: Dynamic Muscle Slings A: Free Muscle Transfer A: Static Slings - Fascia, Dermis, Silastic, Goretex, Rubber

Six complications of Medial Maxillectomy

A: Facial neuralgia/Dysesthesia A: Epiphora/Dacryocystitis A: Telecanthus A: Diplopia (may be transient) A: Nasal collapse A: Mucocele

What is the main Theory of origin of Congenital Cholesteatoma?

A: Failure of involution and continued growth of the Epidermoid Formation, derived from 1st branchial groove ectoderm (Teed, Michaels) A: Found at junction of Eustachian tube orifice and middle ear near anterior tympanic annulus A: Normally disappears in 33rd week of gestation 3: Other theories include Ectodermal migration, and Metaplasia of the middle ear mucosa

Potential complications of radiotherapy for management of JNA?

A: Failure of treatment A: Induction of malignancy A: Failure of facial growth centers A: Cataract formation 3: Radiotherapy used in past successfully for primary therapy, recurrences, inoperable tumors

Ten complications of Septoplasty

A: Failure/persistent obstruction A: CSF leak A: Synechia A: Anosmia A: Hemorrage A: Septal Hematoma/Abscess A: Perforation A: Saddle nose deformity A: Toxic-shock syndrome A: Aspiration

JCIH (joint committee on infant hearing) Position Statement 1994 for neonatal hearing screening (FAT MOH BA)

A: Family history of hereditary childhood SNHL A: Craniofacial Abnormalities and/or external ear malformations A: Findings associated with syndrome involving HL A: TORCHS infections A: Bacterial Meningitis A: Ototoxic medications A: Hyperbilirubinemia requiring exchange transfusion A: Birth weight under 1500g A: APGAR 0-4 at 1 minute or 0-6 at 5 minutes (Anoxia/Admission to ICU/Afterward...) A: Mechanical ventilation >5 days

Six patient risk factors for diagnosis of thyroid cancer

A: Family history of thyroid cancer A: History of radiation exposure A: Male gender A: Age >45 A: Previous history of thyroid cancer A: Ethnicity (Hawaiian, Philipino)

Discuss HIV

A: Family retroviridae A: Strong disease correlation with CD4 counts A: HIV diagnosis made by ELISA first (rule out test), then confirmed with Western blot (rule in test) A: Three clinical categories A - Asymptomatic, PGL (persistent generalized lymphadenopathy), or Acute HIV infection B - Symptomatic condition attributable to HIV infection and associated defects in CMI, but not in A or C C - AIDS (diagnosis = CD4 <200 cells/microliter (uL), or HIV+ with AIDS defining disease, or CD4/CD8 ratio <1.5) A: Stratification by CD4 counts - >500 cells/uL, 200-499 cells/uL, <200 cells/uL

Four Material Options for Static Slings in Facial Paralysis

A: Fascia A: Dermis A: Silastic A: Goretex A: Rubber

Your postop ESS patient develops a postoperative orbital hematoma. What are your management considerations?

A: Fast (arterial) vs. slow (venous) hematoma formation; Proptosis, pupil changes, vision loss indications for immediate surgical intervention A: Medical treatment (5) Urgent Ophthalmology consultation and assess vision Remove packings and suction at bleeding site Mannitol 1-2 g/kg in 20% IV infusion (100 g in 500 cc bag) over 20 minutes Acetazolamide 500 mg IV q4h prn Timolol 0.5% ophthalmic drops (only if slow bleed) Steroids (controversial) Orbital massage (controversial) A: Surgical treatment (4) Lateral canthotomy/cantholysis Medial external (Lynch) decompression Endoscopic decompression 3: If anterior or posterior Ethmoid artery is suspected, Kennedy recommends postop CT sinus to R/O concomitant skull base injury

Five ENG test findings suggestive of a Peripheral disorder

A: Fatiguing positional nystagmus A: Direction-fixed nystagmus A: Unilateral caloric weakness A: Bilateral caloric weakness with a history of labyrinthine disease or administration of ototoxic drugs A: Intact fixation suppression response

CT & MRI findings for Fibrous Dysplasia

A: Featureless trabecular (ground glass) pattern on CT A: Hypo to Hyperintense on T1, T2 homogenous (like air)

Symptoms typical of acute tonsillitis (2 or more for dx)

A: Fever greater than 38.5 A: Erythematous or exudative tonsils A: Tender cervical lymphadenopathy > 2cm A: Positive GABHS culture

Pathway of Crocodile Tears (Bogorad's syndrome) when due to temporal bone fracture

A: Fibers crossinnervate from LSPN to GSPN after trauma to the superior surface of the temporal bone A: Inferior Salivatory nucleus A: CN IX A: Jacobsen's nerve A: Lesser Superficial Petrosal nerve A: Greater Superficial Petrosal nerve A: Sphenopalatine ganglion A: Zygomaticotemporal branch of V2 A: Lacrimal gland

Five Differences between Fibrous Dysplasia & Paget's Disease

A: Fibrous Dysplasia - Age of onset <30, Monostotic most common, occurs in Ribs or Femur, in H&N Maxilla often involved, treated by Surgical Recontouring A: Paget's Disease - Age of onset >40, Polyostotic most common, occurs in Lumbar Spine, in H&N Skull often involved, treated with Calcitonin

List 5 Dysplasias affecting the Temporal Bone

A: Fibrous dysplasia A: McCune-Albright Syndrome A: Hyperparathyroidism A: Pagets disease A: Osteogenesis imperfecta (van der Hoeve syndrome) A: Osteopetrosis (Albers-Schoenberg)

Name the four types of tongue papillae and their innervation

A: Filiform - Nonsensory A: Fungiform - CN VII A: Foliate - CN IX A: Circumvallate - CN IX

Hemangioma appearance, Gender predilection, percent Multiple, and Cellular findings

A: First 6 weeks A: Female:Male = 3-6:1 A: 20% Multiple; half of children with a Subglottic hemangioma will also have a cutaneous lesion, but the converse in only true in 1% A: Proliferating Endothelial cells and increased Mast cells A: Predilection for the Left Posterolateral subglottis

Three degrees of Endoscopic Appearance for esophageal burn injury

A: First Degree - Nonulcerative esophagitis, mild erythema, edema of mucosa A: Second Degree - Whitish exudate, erythema, underlying ulceration that may extend into the muscularis A: Third Degree - Dusky or blackened transmural tissue, deep ulcerations that may extend into periesophageal tissue, lumen may be obliterated

Triad of juvenile onset RRP risk factors

A: First born (75% of cases) A: Vaginal delivery A: Teenage mother

Antibiotic therapy for community-acquired bacterial sinusitis

A: First line - Amoxicillin, Erythromycin-sulfisoxasole, Keflex & Septra A: Second line - Clavulin, Extended macrolides (clarithromycin, azithromycin), 2nd Gen Cephalosporins (cefpodoxime, cefuroxime, cefprozil), Quinolones (ciprofloxacin, levofloxacin, moxifloxacin)

Branches of the internal maxillary artery

A: First part (5) - lies medial to the mandible, all branches go through foramina = Anterior tympanic, Deep auricular, Middle meningeal, Accessory meningeal, Inferior alveolar A: Second part (5) - lies lateral to lateral pterygoid, all branches supply muscles = Deep temporal (anterior & posterior), Medial pterygoid branch, Lateral pterygoid branch, Massseteric artery, Buccal artery A: Third part (6) - lies within pterygomaxillary fossa = Posterior superior alveolar, Infraorbital, Descending palatine (forms greater and lesser palatine), Vidian/pterygoid canal, Pharyngeal, Sphenopalatine

Dexcribe the Access provided by the Fisch A, Fisch B, and Fisch C Approaches

A: Fisch A - Access to Jugular Bulb, Vertical Petrous Carotid, Posterior Infratemporal Fossa A: Fisch B - Access to Petrous Apex, Clivus, Superior Infratemporal Fossa A: Fisch C - Access to Nasopharynx, Peritubal space, Rostral Clivus, Parasellar area, Pterygopalatine Fossa, AnteroSuperior Infratemporal Fossa

What anatomic Locations does Otosclerosis occur most often?

A: Fissula Ante Fenestram A: Round Window Niche A: Anterior Wall of IAC A: Medial wall of Cochlear Apex A: Area posterior to Cochlear Aqueduct A: Region adjacent to SCC's A: Posterior Stapes footplate

What are the Fissures of Santorini?

A: Fissures present in the Lateral Cartilaginous EAC which allows passage of Infection or Neoplasm into the Preauricular Parotid tissues and Glenoid fossa

Risk factors for postoperative major vessel rupture (FEARR!)

A: Fistula/infection A: Exposure of vessel A: Adventitia removed A: Recurrent tumor A: Radiation

Nine Contraindications for VPL/laryngoplasty in early glottic carcinoma

A: Fixed vocal cord A: Cartilage invasion - Thyroid or Cricoid A: Posterior commissure involvement A: Involvement of Both arytenoids A: Bulky transglottic lesions A: Lesion extending above the free edge of the false vocal cords superiorly A: More than 10 mm of subglottic extension anteriorly and 5 mm posteriorly A: More than 5 mm (1/3) of contralateral true vocal cord involvement A: Poor pulmonary reserve (FEV1/FVC <50%)

Three abnormal wheeling patterns on Skin Endpoint Titration

A: Flash Day 1 - 5-5-5-13-18-21 Day 2 - 5-5-5-5-7-9-11 A: Plateau - 7-7-9-9-11-11 Confirmatory here is fifth injection with fourth injection the SET A: Hourglass - 9-7-5-5-7-9-11

Five characteristics of the ideal scar

A: Flat, level with skin A: Narrow A: Parallel to RSTLs A: Same Color A: No long unbroken segments

Investigations for Laryngeal Web

A: Flexible laryngoscopy A: +/- Rigid laryngoscopy + bronchoscopy (site, thickness, horizontal and vertical extent) A: +/- X-ray (Sail sign = Persistent tissue between VCs + subglottis, R/O SGS) A: +/- FISH for anterior glottic web (22q11.2 deletion)

Management of post-obstructive pulmonary edema

A: Fluid restriction A: Diuretics A: CPAP

Clear cells on FNAB, 3 possibilities

A: Follicular adenoma A: Follicular carcinoma A: Metastatic Renal Cell carcinoma 3: RCC can be distinguished by lack of Colloid on PAS, Tg and TTF-I stains negative, CD10 and RCC marker stains positive

Hurhtle cells on FNAB, 4 possibilities

A: Follicular adenoma A: Follicular carcinoma/Hurthle cell carcinoma A: Hashimotos thyrroiditis A: Papillary carcinoma 3: Other lesions include Grave's, post-XRT, Aging, Papillary, and Medullary carcinoma (Wenig, p. 894)

Three thyroid tumors that are hard to diagnose with the best FNA

A: Follicular carcinoma A: Lymphoma A: Poorly diff/Met?

Histological features of Hurthle cells

A: Follicular cells with rich granular eosinophilic cytoplasm

Six Histologic patterns of Ameloblastomas

A: Follicular or Simple (most common) A: Plexiform A: Acanthomatous A: Desmoplastic A: Granular cell A: Basal cell 3: Histologic findings include cystic spaces lined with tall columnar epithelium, filled with stellate reticulum, surrounded by a dense fibrous stroma

Waves of the Auditory Brainstem Response (ABR)

A: Follows the pathway of the auditory CNS (ECOLI) A: I - Distal CN VIII A: II - Proximal CN VIII A: III - Cochlear nucleus A: IV - Superior Olivary complex A: V - Lateral lemniscus A: VI-VII - Inferior Colliculus A: Other waves beyond this - Medial Geniculate ganglion, Auditory radiation (Brodmann's area 41)

Four Indications for LSCC Fenestration

A: Footplate Refixation after Stapedotomy A: Facial Nerve Overhang A: Persistent Stapedial Artery A: Absent Oval Window (?)

Conditioned play audiometry (CPA)

A: For children 2-5 years A: Child performs a task in response to an auditory stimulus A: Play activity changes to keep the child's interest

Behavioral observation audiometry (BOA)

A: For infants from 0 to 6 months A: Based on reflex responses & state changes A: Unconditioned responses, variable & imprecise, habituate rapidly

Why can't EMG be used at the outset of facial nerve injury, and what is the lag between EMG recovery and clinical return?

A: For the first 10-14 days, do not see muscular Fibrillation Potentials indicating muscular Degeneration A: Polyphasic Potentials are seen 6-12 weeks prior to clinical Recovery

Visual reinforcement audiometry (VRA)

A: For toddlers 6 months - 2 years A: Operant conditioning - Look for auditory stimuli (lights, motion)

Flap physiology - Define Stress

A: Force applied per cross-sectional area

Define Myospherulosis

A: Foreign body reaction to the petroleum ointment used in packing

Discuss Inner ear Decompression sickness and Isobaric Gas Counterdiffusion sickness

A: Formation of gas bubbles in the body due to rapid reduction in Atmospheric pressure or too rapid an exchange of Inspired gas mixtures under Isobaric conditions (as in deep sea diving) A: Tinnitus, SNHL, Vertigo most common symptoms; Vertigo sole complaint in IEDS (50%) A: Treatment is Rapid Recompression 3: A 2008 Royal college oral exam question was about a diver suffering from hearing loss due to either Atmospheric Inner ear Barotrauma vs. Inner ear Decompression sickness; what features on history would allow you to distinguish them? The former occurs on descent while the latter occurs on ascent.

Describe the anatomy of the Tympanic membrane

A: Four layers - Squamous epithelium, Radiating fibrous layer, Circular fibrous layer, Mucosal layer A: Width = 8-9 mm A: Height = 9-10 mm A: Surface area = 70-80 mm2 A: Vibrating surface area = 55 mm2 3: Remember "Inner circular" just like in esophageal muscle

Four methods of reconstructing the cervical esophagus, advantages & disadvantages

A: Free jejunum - Adv = mucosalized tubed structure that reestablishes conduit; Disadv = requires laparotomy, requires microvascular anastomosis, often unable to achieve TE speech. Preferred for total pharyngoesophageal defects, not ideal if significant base of tongue resected A: Colonic interposition (not a first choice reconstructive alternative) - Adv = mucosalized tube structure that reestablishes food conduit; Disadv = requires laparotomy, high morbidity/mortality from postoperative infection A: Gastric pullup/transposition - Adv = single anastamotic line, can perform total esophagectomy for margins; Disadv = morbidity with the pullup (pneumothorax), postoperative early satiety/dumping/emesis, mortality is 5-15%. Preferred for resections that extend into the cervical esophagus A: Free microvascular transfer (radial forearm, lateral thigh)

Adult onset RRP risk factors

A: Frequent oral sex A: Multiple sex partners

Describe Temporary Threshold Shift (TTS)

A: From exposures to Moderately intense sounds A: Elevated Thresholds, esp. 3-6 kHz, with Tinnitus, Loudness Recruitment, Diplacusis A: Recovery occurs over Minutes, Hours & Days

Name the Plain Film projections useful for each Sinus

A: Frontal - Lateral and Caldwell A: Ethmoid - Lateral and Caldwell A: Maxillary - Waters A: Sphenoid - Lateral and Submentovertex

Three branches of V1, what these pass through, what they provide innervation to

A: Frontal - enters the superior orbital fissure (SOF) lateral to tendon of Zinn, divides into supratrochlear and supraorbital nerves to supply forehead and scalp A: Lacrimal - enters the SOF lateral to tendon of Zinn, supplies lacrimal gland (postsynaptic autonomic efferents) and lateral eyelid A: Nasociliary - enters the SOF medial to tendon of Zinn, supplies tip of nose through external branch of anterior ethmoidal nerve, root of nose through infratrochlear nerve, corneal surface sensation

Four bony components of the Medial Orbital Wall

A: Frontal process of the Maxilla A: Lacrimal bone A: Lamina papyracea of the Ethmoid A: Sphenoid bone (just anterior to the optic canal) 3: Orbital process of palatine bone is also near the apex (more inferior than lateral?)

Describe the Lund-Mackay CT Staging system

A: Frontal sinus R & L A: Maxillary sinus R & L A: Anterior Ethmoid R & L A: Posterior Ethmoid R & L A: Sphenoid sinus R & L A: OMC R & L 3: Scoring is 0 = clear, 1 = partial opacification, and 2 = total opacification; except for the OMC which is 0 = clear, and 2 = occluded, total score is 24.

Seven areas to evaluate in preoperative CT scan

A: Frontal sinus/recess and Agger nasis A: Skull base (Keros classification) A: Anterior ethmoid artery A: Lamina papyracea/Uncinate process A: Maxillary sinuses (Haller cells) A: Vertical height of the posterior ethmoids A: Sphenoid sinus (Onodi cells, ON, ICA)

Six indications of External Frontoethmoidectomy

A: Frontoethmoid mucopyocele A: Orbital complication of sinusitis A: Revision surgery with absent or distorted landmarks A: CSF leak repair A: Biopsy of Anterior skull base lesion - Frontal, orbital, or ethmoid lesion A: Access to the anterior ethmoid artery

Evaluated criteria on videostroboscopy (remember F-SCAMP)

A: Fundamental Frequency A: Symmetry (note: asymmetry in amplitude is ALWAYS pathologic) A: Glottic Closure A: Amplitude of vibration A: Mucosal wave A: Periodicity 3: Can also comment on Adynamic Segments, Phase Closure, and Vertical Closure Level

Bacterial cause of Lemierre's Syndrome

A: Fusobacterium necroforium

Name and describe the 5 phases of the cell cycle

A: G1 - 1st Growth phase in preparation for DNA synthesis A: S - DNA Synthesis A: G2 - Cell Growth, duplication of cell proteins & structures A: M - Mitosis A: G0 - Resting phase

Four Rotatory chair testing characteristics measured

A: Gain - Ratio of maximum slow phase eye velocity to maximum chair velocity; should be equal but opposite (Gain = 1) A: Phase - Temporal shift between eye velocity relative to chair velocity, can lead or lag; normally eye velocity leads chair velocity (Phase-lead); Most reproducible A: Symmetry - Comparison between peak slow-wave velocity to the left and right; will show weakness on an affected side A: Bias velocity - A spontaneous nystagmus, if present, will add to the rotational response & shift it towards the direction of the slow phase 3: More physiologic test conditions, acceleration frequency range from 0.01 to 1.25 Hz

Three ANSI measures in the electroacoustic assessment of hearing aids

A: Gain = Output SPL - Input SPL; 3 measures: Full-on gain (Maximum volume), Reference test gain (HFA = OSPL90 - 17 dB), and Use gain (Volume set to usual as-worn level) A: Output SPL with a 90 dB input (OSPL90) = Maximum amplification A: Frequency response = Range of Frequencies intersected by a line drawn at 20 dB below the HFA calculated in the Reference test gain 3: High Frequency Average (HFA) = Gain at 1000, 1600, & 2500 Hz A: Clinical Indications for BAHA A: Intolerance to wearing conventional H.A. - Chronic Dermatitis of the external ear, Chronic Otorrhea from COM or Chronic Mastoiditis, Mastoid cavity (feedback) A: Inability to wear conventional H.A. - Congenital malformations of the External & Middle Ear (Microtia/Atresia, Syndromic/Sporadic) A: Unilateral Dead Ear +/- CHL in only hearing ear A: Fourth option in Otosclerosis

Name two syndromes associated with Papillary thyroid ca

A: Gardner A: Cowden

Six Evaluations for Laryngopharyngeal Reflux (LPR)

A: Gastric emptying scan (milk scan) technetium 99 m A: 24 hour double-probe pH monitoring A: Barium swallow A: Broncho-alveolar lavage for lipid laden macrophages (LLM, 70% needed) A: Esophagogastroduodenoscopy with Biopsy (EGD, suspected eosinophilic esophagitis) A: Diagnostic markers pepsin & carbonic anhydrase isoenzyme III (CA-III)

What is the VOR?

A: Gaze stabilizing system that produces eye movement equal in magnitude but opposite in direction of a head movement A: Maintains binocular fixation and stable binocular foveal images during head movements; complements the optokinetic system A: Di or Trisynaptic pathway involving Scarpa's ganglion, the vestibular nuclei, the MLF and bilateral nuclei of CN III and VI; latency is quick (12 msec) A: Can be Angular (canals), Linear (otolithic), or Torsional (both)

Three examples of small molecular Tyrosine Kinase Inhibitors and side effects

A: Gefitinib (Iressa), Erlotinib (Tarceva), and Sunitinib (Sutent) A: Diarrhea, and Acneiform rash (in 50%) which is predictive of better Response Rate and progression-free Survival

Seven injectable materials for vocal cord medialization (Bailey, p.857)

A: Gelfoam -If VC recovery expected, lasts 4-6 weeks, saline resorption so overinject 30% A: Zyplast - Bovine collagen, up to 3 years? Risk of host reaction A: Alloderm - Acellular dermis A: Perlane - Hyaluronic acid A: Radiesse - Ca Hydroxylapatite, permanent A: Fat - ≥6 months, 50% resorption so overinject 40% A: Polytef - Teflon; permanent, if no recovery expected; discontinued due to granuloma formation, irreversibility, risks of migration, stiffening of mucosal wave

Discuss connexin 26 (CX26)

A: Gene GJB2 product (on CC 13) A: DFNB1 mutation Most common association with Genetic Nonsyndromic hearing loss (30% of Sporadic HL, >50% of cases with an affected Sibling) A: Six connexins form a connexon

Six risk factors for complications in FESS

A: General anesthesia - lack of patient feedback A: Extent of disease A: Amount of Bleeding A: Right side for a right-handed surgeon A: Revision FESS A: Expertise of the surgeon 3: "this General Extent of Bleeding is Right for a Revision Expert"

Medical treatment strategy for Meniere's disease (6)

A: General measures - Salt restriction (1200 mg/day), avoidance of caffeine, etc. A: Diuretics - Thiazide, Acetazolamide (CAI, theoretic decrease in endolymph production) A: Vestibular suppressant - Betahistine (Serc), Histamine agonist; also a Vasodilators, theorized improvement in Strial ischemia & Bloodflow to labyrinth, direct inhibition of neurons in the Vestibular Nuclei; starting dose 8-16mg/kg po tid (now going up to 48 mg/kg po tid in europe with better results) A: Benzodiazapenes - Anti-GABA-ergic, give Ativan or Valium as ABORTIVE agents, not regular use A: Corticosteroids - Anti-inflammatory, immunomodulatory A: Antihistamines/Antinauseants - For acute symptoms, eg Dimenhydrinate

What methods are used to introduce genes in gene therapy?

A: General methods - In vivo (direct introduction of vector with gene into tissues), Ex vivo (removing target tissue/cell portion, Transduction with vector, Reimplantation into patient) A: Viral vectors A: Nonviral vectors - Nonspecific targeting, low transfection rate

List 5 Complications of intrathecal fluorescein

A: Generalized Seizures A: Opisthotonus (spasmotic hyperextension of the back) A: Lower limb weakness/paresthesias A: Headache A: Cranial nerve deficits

Etiologies of pediatric hearing loss and percentage of cases?

A: Genetic = 50% (30% part of an identifiable Syndrome, AR ~75%, AD ~20%, XL ~3-4%, Mitochondrial <1%) A: Environmental = 20-25% (pre-, peri-, postnatal) A: Unknown etiology = 25-30%

Risk factors for NPC

A: Genetics - HLA-A2, B17, Bw46 (only in Orientals) A: Geographic - southeast Asia (Kwantung, Hong Kong, Taiwan) A: Environmental - Nitrosamines, salted preserved foods A: Infectious - EBV

A 50 year old man presents with a six month history of right pulsatile tinnitus and progressive hearing loss. He has no past history of ear problems and denies trauma to his head or ears. He recently developed some difficulty swallowing. Examination reveals a large red mass behind an intact tympanic membrane. His audiogram shows mostly conductive hearing loss. What is the most likely diagnosis? Excluding the possiblity of associated tumors, what other manifestations do you expect to see on H&N examination? What is the best investigation to establish a diagnosis?

A: Glomus Jugulare tumour A: Paralysis of the soft palate on the right side A: Right vocal cord paraslysis A: Right sternomastoid (trapezius) paralysis A: Test = Carotid Angiography

Ddx of a Soft Tissue Mass overlying the Promontory (7)

A: Glomus tympanicum A: High riding jugular bulb (defined so if superior limit above the floor of the IAC) A: Congenital cholesteatoma A: Schwannoma A: Adenoma A: Persistent stapedial artery A: Aberrant carotid artery

Anatomic associations between the superior & middle constrictors

A: Glossopharyngeal nerve (IX) A: Stylopharyngeus A: Stylohyoid ligament A: Lingual artery

Two essential parameters of normal vocal cord vibration and voice production

A: Glottal gap - Interarytenoid & lateral cricoarytenoid muscles both close and appose the vocal cords A: Normal vocal cord stiffness - Thyroarytenoid controls stiffness, which increases pitch and resistance; Cricothyroid antagonizes thyroarytenoid, thins & lengthens vocal cord which increases tension and pitch while decreasing the area of contact

Relative frequency of Laryngeal Webs by location

A: Glottic = 75% A: Subglottic = 7% A: Supraglottic = 2%

Four objective voice measures

A: Glottography - Photo or Electroglottography A: Videostroboscopy A: Airflow measures A: Acoustic measures

Three Diagnostic tests in Meniere's disease

A: Glycerol test - 1.5 g/kg mixed 1:1 with juice, serial audiograms over next 3 hours to see if SDS & PTA improves (>16% & 25 dB in 3 consecutive frequencies a positive test) A: ENG - 48-74% of patients will have significant caloric reduction; complete loss noted in 6-11% A: ECoG - Elevated SP:AP ratio (>50%) due to hydrops causing altered basilar membrane mechanics

Monomorphic adenoma Histology

A: Glycogen rich A: Basal cell (most common) A: Myoepithelioma (Pasha) A: Canalicular A: Clear cell (R/O metastatic renal cell CA) A: Membranous (Pasha) A: Sebaceous A: Gross = Well circumscribed and encapsulated A: Microscopic = Basal cell, rows of peripheral pallisading cells 3: "M.A. Gladly Backed My Canadian Club Membership"

Characteristics of Electron Beam XRT

A: Good for Superficial lesions, deep tissue sparing A: Range of Penetration (cm) = MeV/3

Four Options for Free Muscle Transfer Facial Paralysis

A: Gracilis A: Serratus A: Latissimus dorsi A: Extensor digitorum brevis 3: Two stages - 1st is Sural nerve graft to the contralateral normal side, 2nd stage 9 months later is the transfer of muscle fibers suspended from Zygoma to Commissure

What does the GRBAS scale in voice assessment stand for?

A: Grade A: Roughness A: Breathiness A: Asthenia (essentially weakness) A: Strain 3: Score 0 for normal, and 3 for severe

Decribe the House-Brackman Grading of Facial Paralysis

A: Grade 1 - Normal in all areas A: Grade 2 - Mild dysfunction; Slight weakness (+/- Synkinesis) on close inspection, complete eye closure with minimum efffort A: Grade 3 - Moderate dysfunction; Obvious but not disfiguring Asymmetry/weakness, Synkinesis, Contracture, or Hemifacial spasm, Normal at rest, eye closure with effort A: Grade 4 - Moderately-Severe dysfunction; Obvious and/or Disfiguring Asymmetry/weakness, Normal at rest, Incomplete Eye closure, No Forehead movement A: Grade 5 - Severe dysfunction; Barely perceptible motion, Asymmetry at rest, Slight mouth movement A: Grade 6 - Total paralysis

Staging of Esophageal Burn injury

A: Grade 1 - mucosal erythema (superficial) A: Grade 2 - mucosal erythema with noncircumferential exudate (transmucosal) A: Grade 3 - circumferential exudates (transmucosal) A: Grade 4 - circumferential exudates with perforation (transmural)

Define Broder's Classification of SCC of the Oral cavity

A: Grade I = 75-100%, Well differentiated A: Grade II = 50-75%, Moderately Well differentiated A: Grade III = 25-50%, Moderately differentiated A: Grade IV = 0-25%, Poorly differentiated 3: Based on cellular differentiation, expressed in percentage of total cellular elements

Two indications for Four Quadrant Cricoid Split

A: Grade III & IV A: Congenital elliptical cricoid 3: Procedure - Division of ant. + post. walls (+/- grafts) + lateral walls of cricoid anterior to inf. cornu of thyroid & extraperichondrial externally to avoid RLN, Aboulker or Cotton-Lorenz Stent for 6 months (very unstable airway)

Six pseudoepitheliomatous lesions of the larynx

A: Granular cell tumor A: Rhinoscleroma A: Necrotizing sialometaplasia A: Pachydermia laryngis A: Histoplasmosis A: Blastomycosis A: Aspergillus/Candida

Ddx of Multinodular Parotid gland

A: Granulomatous diseases - TB, Sarcoid A: Lymphoproliferative disorders (lymphoma) A: Warthins Tumor A: Other tumors

Most common cause of unilateral or bilateral proptosis in adults

A: Grave's orbitopathy

Definition of a Gray and Rad

A: Gray (Gy) = The absorption of one joule of radiation energy by one kilogram of tissue A: Rad = Radiation absorbed dose; 100 rad = 1 Gy

Describe the Venous drainage of the nose

A: Greater Palatine vein - Posterior Facial vein, into External Jugular vein A: Septal and Angular veins - Anterior Facial vein, into Internal Jugular vein A: Anterior and Posterior Ethmoidal veins - Ophthalmic vein, to Cavernous sinus, into Internal Jugular vein A: Sphenopalatine vein - Maxillar vein, to Cavernous sinus, into Internal Jugular vein

Discuss options for Cable grafting of Facial nerve, Lengths and Advantages

A: Greater auricular nerve (5-10 cm) - Advantages = Within surgical field, Minimal morbidity, Similar fascicular pattern; Harvest from contralateral neck if malignant tumour case?(should just go to sural) A: Sural nerve (30? cm) - Advantages = Can use 2 team approach, Minimal donor morbidity; Found ~2 cm posterior to the lateral maleolus, and between heads of Gastrocnemius A: Median Antebrachial Cutaneous nerve (20 cm) - Advantages = Similar branching pattern as facial nerve, Minimal donor morbidity, found in Bicipital groove next to Basilic vein 3: Can clip branches to the digastric, posterior auricular, plastysma and buccal muscles to direct neural regeneration to upper and lower face

Describe Bullous Pemphigoid

A: Greater cutaneous involvement, less frequent oral ulcers A: Seen in elderly

Pleomorphic adenoma Histology

A: Gross = well encapsulated, smooth, rubbery A: Microscopic = incomplete encapsulation, pseudopod extensions A: Mixture of epithelium, myoepithlium and mesenchymal stroma A: Epithelial growth patterns - Cystic, Papillary, Small nests, Solid sheets, Ductal structures, Anastamosing Trabeculae A: Myoepithelial cells - Spindle shaped, hyperchromatic nuclei, may be multiple cell layers thick A: Mesenchymal stroma morphologies - Fibroid, Myxoid, Mucoid, Chondroid, Osteoid

Define Carcinoma in situ

A: Growth disturbance in which there is sufficient atypicality of the epithelial cells and their arrangement to warrant the diagnosis of cancer in the absence of invasion

Five Oncogenes classified according to Protein's Function

A: Growth factor - int-2 A: Growth factor receptors - erbB/neu A: Signal transducers for surface growth factor receptors - ras A: Protein kinases - b-raf, c-raf A: Transcription regulator - myc

Discuss the clinical assessment for non-Hodgkin's lymphoma

A: H&P with direct laryngoscopy A: CBC/Diff, LFT's, LDH A: CT neck, chest, abdomen & pelvis A: Barium swallow (3-11% of patients with Waldeyer ring lymphoma will have an associated GI lesion) A: Bone marrow biopsy (18% of patients with extranodal H&N lymphoma involved) A: Lumbar puncture (at risk of CNS involvement: High grade lymphoma, Intermediate grade lymphoma of the paranasal sinuses, bone marrow, testes, paraspinal areas)

Organisms seen in orbital/periorbital cellulitis

A: H. influenzae A: S. aureus A: S. pneumoniae A: S. pyogenese A: Bacteroides species A: Peptostreptococci A: Veionella

Audiometric results of Aural atresia repair

A: HL <30 dB in 50-75% A: HL <20 dB in 15-50%

Discuss Horizontal Canal BPPV

A: HSCC affected structure in ~17% of cases A: Dix Hallpike variant maneuver - Bring patient from sitting to supine with head straight, then turn rapidly to test side with ear to the table A: Nystagmus - Horizontal, either Geotropic (Canalithiasis) or Ageotropic (Cupulolithiasis); Shorter latency, Increased in magnitude while in test position, and Slower to Fatigue on repeat testing compared to PSCC symptoms

Three causes of Pseudo-Macroglossia

A: Habitual tongue posturing A: Retrognathia/Micrognathia A: Hypotonia of tongue

Complications of Adenotonsillectomy

A: Halitosis (most common) A: Dehydration A: Postoperative hemorrhage - 0.5-1% A: Pulmonary - postoperative edema & hypoxemia (loss of auto-PEEP, hypercapneic respiratory drive) A: Airway obstruction A: Velopharyngeal insufficiency A: Nasopharyngeal stenosis A: Atlantoaxial subluxation (Grisel's syndrome) - Down syndrome at higher risk A: Eagle syndrome A: Death - 1/25,000

Five postural effects on swallowing

A: Head rotation to side of VCP closes off that pyriform sinus to the bolus A: Chin tucking brings anterior wall of pharynx posterior, airway entrance narrowed (good if premature spillage and aspiration occurring before the pharyngeal phase) A: Head tilt to side opposite of VCP directs food down that side of the oral cavity & pharynx A: Supine position causes residua to stick to pharyngeal walls rather than fall down airway A: Lifting chin decreases oral transit time via gravity

Acoustic properties of the Head that assist in sound localization

A: Head shadow effect - Head attenuates sound waves with wavelengths shorter than the width of the head (>2 kHz) by 5-15 dB A: Interaural time difference - Maximum ~0.6 msec time difference for sound hitting one ear to reach the other ear A: Another answer may be Spectral acoustic characteristics of sound (to be verified)

Describe the Ocular Tilt Reaction

A: Head tilt toward the lesioned side A: Skew deviation - Disconjugate deviation of the eyes such that the pupil on the intact side is elevated and the pupil on the lesioned side is depressed A: Static conjugate counter roll - Rolling the superior pole of each eye away from the intact utricle 3: Occurs as a sign of acute unilateral vestibular loss, due to loss of utricular nerve activity

Embryologic derivations of the Malleus

A: Head, Neck, & Anterior Mallear ligament - Meckel's cartilage (1st Arch) A: Manubrium - Reichert's cartilage (2nd Arch) A: Anterior process - Process of Folius (Mesenchyme)

Bacteriology of bacterial Meningitis

A: Heamophilus influenza A: Streptococcus pneumonia A: Nisseria meningitidis

Pathogen causing bilateral, dull TM, non-mobile, T > 99.0°

A: Heamophilus influenzae

Definition of Idiopathic Sudden SNHL, and percentage of recovery?

A: Hearing loss ≥30 dB in 3 Contiguous Frequencies in <3 days A: ~33% recover without therapy

Five head & neck signs & symptoms of hypothyroidism

A: Hearing loss: sensorineural, mixed, conductive A: Tinnitus A: Vertigo A: Enlarged tongue A: Hoarseness (mucopolysaccharide infiltration) A: Blurred vision

Five Surgical management options for Meniere's disease

A: Hearing preserving nonablative procedures - Endolymphatic sac decompression (50-75% success rate, 1-2% postop HL); Cochleosacculotomy (historical) A: Hearing preserving vestibular ablative procedures - Vestibular Neurectomy (complete resolution in 95%; Middle fossa, Retrolabyrinthine, or Retrosigmoid appoaches); Chemical vestibular intratympanic ablation (Gentamycin, ~90% success rate, 6-30% rate of HL); Intramuscular Streptomycin (for Bilateral Meniere's disease) A: Hearing and vestibular ablative procedures - Labyrinthectomy (when no serviceable hearing, >60 dB PTA, <50% SDS) 3: Surgical management is for patients who fail medical management (~10%)

Ear analysis

A: Height ~6cm A: Width 0.55-0.6 of the Height A: Top of Helix at same level as lateral Brow A: Helical Root at same level as lateral Canthus A: Inferior attachment at same level as Alar-Facial junction A: Long axis (posterior) parallels the Nasal Dorsum A: Short axis (anterior) is inclined ~20 degrees A: Protrusion from posterior skull 20 degrees, approximately 20 mm

Ddx of Intraaxial tumors (4)

A: Hemangioblastoma A: Medulloblastoma A: Brainstem Glioma (most common Pediatric CPA lesion) A: Malignant Choroids Plexus Papilloma & Ependymomas

Most common Benign Salivary gland tumors in Children

A: Hemangioma A: Pleomorphic adenoma A: Lymphangioma A: Neurogenic

Sinusitis pathways of spread

A: Hematogenous spread - Retrograde thrombophlebitis through valveless veins (veins of Breschet) A: Direct extension - Preformed pathways (eg. natural dehiscence of lamina) A: Direct extension - Traumatic/surgical pathways (eg. traumatic dehiscence of lamina papyracea) A: Direct extension - Osteomyelitis (Pott's puffy tumor)

Five complications/emergencies of tissue expanders

A: Hematoma - truly emergent; return to OR for evacuation, stop bleeding, possibly reinsert expander A: Skin necrosis A: Dehiscence/Implant exposure (most common) A: Extrusion A: Bone remodeling/neurovascular compromise - remove expander or perform surgery ASAP

What are the two signs associated with Otosyphilis?

A: Hennebert's sign - Positive fistula test with an intact tympanic membrane A: Tullio phenomenon - Vertigo & Nystagmus precipitated by loud Noise exposure

Ddx of a Cookie-bite Hearing loss (1)

A: Hereditary A: Cochlear otosclerosis

CT & MRI findings for Mycetoma

A: Heterogeneous (double density) on CT A: Hypointense on T1/T2

Six indications for systemic therapy in Melanoma

A: High Risk patients finished Primary Therapy - primary Ulcerated lesions, >4 mm depth or Clark level IV, Satellitosis, In-Transit Metastases, Nodal metastases A: Distant Metastases on presentation

Warthin's tumor (papillary cystadenoma lymphomatosum, adenolymphoma) Histology

A: High density of mitochondria on EM A: Gross = Smooth with well defined capsule A: Microscopic = Biphasic, epithelial cells forming Papillary projections into Cystic spaces with a background of Lymphoid stroma A: Epithelium = Double cell layer of Oncocytic cells with nuclei of cystic side (columnar luminal cells) toward cystic space and nuclei of basement membrane (cuboidal basal cells) side toward basement membrane (i.e. nuclei away from each other) A: Lymphoid stroma = Lymphocytes with follicles and germinal centres

Three things must rule out in parotid SCC

A: High grade Mucoepidermoid ca A: Metastatic SCC to intraglandular nodes A: Direct extension of cSCC to parotid 3: Treatment - Surgical resection with ND, with post op XRT

Seven Indications for Radiotherapy in Salivary gland malignancies

A: High grade tumors = SCC, AdenoCA, Undifferentiaed, High grade mucoepidermoid, Malignant mixed A: Positive margins - Unresectable or incomplete resection A: Extraparotid spread - T3 or T4 A: Lymph node mets A: Extracapsular spread A: Facial nerve dysfunction caused by cancer (Perineural spread) A: Recurrent tumor 3: All malignant tumors except T1N0 or T2N0 disease with low grade histology (low grade mucoepidermoid, acinic cell) and negative resection margins 3: Use Neutron beam for primary site in Adenoid Cystic carcinoma

What is the "H" zone of the face?

A: High risk sites for cutaneous malignancy 3: Junction of the ala with the nasolabial fold, nasal ala, the septum, inner canthi & lower eyelids, periauricular region extending to the temple, certain scalp regions

Four points about Combined Chemotherapy

A: Higher Response Rates with some combinations A: Overall survival not improved A: Toxicity is More severe A: Cisplatinum & 5FU the "gold standard", act synergistically; Carbo-Taxol more recently

Six advantages of Rigid endoscopes

A: Higher resolution A: Brighter, clearer pictures A: Contrast is better A: Large selection of viewing angles A: Image is more accurately magnified than with a flexible endoscope A: The examination is simple and does not usually require topical anesthetic

Define the Tip Defining Point

A: Highest, medial and cephalic portion of the lateral crus that corresponds to the light reflex externally

List Cahan's 4 criteria in the diagnosis of Post-Radiation Sarcoma (PRS)

A: Histologic features of the original lesion and PRS are completely different A: PRS is located within the field of irradiation A: Patients with cancer syndromes such as Li-Fraumeni and Rothmund-Thomson are excluded A: Latent period (period between initiation of radiotherapy and histologic diagnosis of second neoplasm) is >4 years

Nine characteristics of high risk cutaneous SCC (HIGHER RISC)

A: Histology - poorly differentiated A: Invasion to reticular dermis or adipose tissue A: rapid Growth A: Host immunocompromise A: Ear or lip A: Radiated skin A: Recurrent tumor A: Invasion - perineural or lymphatic A: Size >2 cm A: Chronic ulcer/sinus tract, or scar

Three hints on Clinical History suggestive of Pseudohypoacusis (Malingering)

A: History Inconsistent with that of hearing loss recorded on testing A: Referral source is for a Compensation case; may name the specific incident that caused the loss A: Behaviors Suspicious for invalid responding during testing

Four Indications for Mastoidectomy in Facial nerve Palsy secondary to Otitis Media complication

A: History of COM A: Onset of paralysis >2 weeks after onset of AOM A: ENOG showing >90% Degeneration after 6 days A: Failure of paralysis to resolve after appropriate Medical management

Risk factors for VPI after adenoidectomy

A: History of nasal fluid regurgitation A: Occult submucus cleft A: Family history of clefts A: Neuromuscular problems (CNS)

Four clinical Signs & Symptoms of Presbylaryngeus

A: Hoarse, breathy voice A: High pitch voice A: Vocal fatigue A: Tremor A: Apiration

Define the Reflux Symptom Index (RSI)

A: Hoarseness A: Clearing A: Excess mucus/ Postnasal drip A: Dysphagia A: Cough after eating/lying down A: Dyspnea/Choking spells A: Troublesome cough A: Globus sensation A: Heart burn, indigestion, pain 3: Nine symptoms worth up to 5 points each = Max 45; >15 is suggestive, controversial

Surgical landmarks of CN VII through the Middle Ear

A: Horizontal SCCs A: Cochleariform process - VII runs above this A: Short process of the Incus - Marks the beginning of the Second Genu A: Fossa incudis A: Digastric Ridge - Anterior extent of ridge leads to Stylomastoid foramen A: Chorda Tympani A: Oval Window A: Cog A: Pyramidal eminence

Most aggressive WDTC

A: Hurthle cell (50-60% five year survival)

Management of Peritonsillar Abscess (infection of a peritonsillar salivary/Weber gland)

A: Hydration A: Analgesia A: Incision & drainage or needle aspiration (75% effective), culture A: Antibiotics, parenteral, enteral, or combined A: If unsuccessful - CT scan, reincision & drainage, Quinsy tonsillectomy

What are the equivalent doses for Opioids

A: Hydromorphone (Dilaudid) 1 mg PO A: Oxycodone 2.5 mg PO (no IV form) A: Morphine 5 mg PO A: Empracet (Codeine) 15 mg PO 3: Remember "HOME" is 1, 2.5, 5, 30; IV/SQ doses are generally half

Opera singer with VC edema, wants to sing, what to suggest (2)

A: Hygiene - Voice rest and Hydration A: Meds - Steroid and PPI

Which cranial bones are formed by Endochondral ossification (i.e. the others are all intramembranous)

A: Hyoid bone A: Inferior turbinate A: Styloid process A: Petrous Temporal A: Occipital A: Ethmoid A: Mastoid A: Sphenoid A: "HIS POEMS"

Contraindications for SCPL-CHP in supraglottic carcinomas

A: Hyoid invasion A: Massive preepiglottic space invasion with vallecula involvement A: Tongue base invasion A: Arytenoid fixation A: Subglottic extension to cricoid A: Pharyngeal or interarytenoid involvement A: Extensive thyroid cartilage invasion A: Inadequate pulmonary reserve (FEV1/FVC ratio<50%) A: Resectable by supraglottic laryngectomy

What is Pathognomic for Graves orbitopathy on physical exam?

A: Hyperemia over the Lateral Rectus muscle

CT & MRI findings for Allergic Mucin

A: Hyperintense/heterogenous on CT A: Hypointense on T1/T2 - lack of water

Five categories of early laryngeal squamous malignancy

A: Hyperkeratosis A: Hyperkeratosis with atypia (?same as dysplasia) A: Carcinoma in-situ - chance of developing SCC with CIS or severe dysplasia ~16% if left untreated A: Superficial invasive carcinoma A: Invasive carcinoma

Ddx of Leukoplakia on oral mucosa

A: Hyperkeratosis A: Lichen planus A: Leukoedema A: Hairy leukoplakia A: White spongy nevus A: Candidiasis A: SLE A: Psoriasis A: Secondary syphilis A: Geographic tongue A: Koplik spots (Measles) A: Fordyce granules

Six histologic features of Leukoplakia

A: Hyperkeratosis - Increased Thickness of keratinized layers A: Parakeratosis - Retention of Nuclei in cells attaining the level of the Stratum Corneum A: Dyskeratosis - Production of Keratin at lower layers A: No Pleomorphism - More than one form of a single cell type A: No Anaplasia - Change in a cell or tissue to a Less highly differentiated form A: No Desmoplasia - Connective tissue Reaction to tumor

Four contraindications for Osteoplastic Flap Obliteration of the frontal sinus

A: Hyperpneumatized supraorbital ethmoid cells A: Fungal sinusitis (mainly AFRS) A: Inverting papilloma or other frontal sinus tumor A: Posterior table dehiscences

Ten Complications and Contraindications of Steroid sprays

A: Hypersensitivity A: Atrophic rhinitis A: Epistaxis A: Septal perforation A: Glaucoma A: Cataracts A: Ocular herpes A: Systemic fungal infections A: TB A: Adrenal suppression

Three possible Donor nerves for Nerve Crossover techniques in Facial Paralysis

A: Hypoglossal - All or a portion of the nerve A: Spinal Accessory - SMC branch, Trapezius branch, or a portion of each A: Contralateral Facial nerve

Name the 5 structures that cross over the internal carotid lateral to it

A: Hypoglossal nerve A: Occipital artery A: Posterior auricular artery A: Posterior belly of digastric A: Stylo-everything - Stylopharyngeus, styloglossus, stylohoid muscle and ligament

Eight Reasons why Downs are susceptible to OSA

A: Hypoplastic midface and cranium A: Narrow nasopharynx A: Macroglossia A: Muscular hypotonia A: Obesity A: Increased susceptibility to upper respiratory tract infections A: Small larynx A: SGS 3: UPPP may be useful in this patient population

What 2 systemic diseases should be considered in the differential of a vocal cord lesion?

A: Hypothyroidism A: Acromegaly 3: S, A, W???

Benjamin & Inglis classification of Laryngeal Clefts

A: I - Interarytenoid A: II - Partial cricoid (below VCs) A: III - Through entire cricoid into cervical trachea A: IV - Distal (thoracic) trachea

Describe the Fisch Classification of JNA

A: I - Limited to nose and/or nasopharyngeal vault A: II - Extension to one or more sinuses, or the Pterygomaxillary Fossa A: III - Invades the Infratemporal fossa, orbit, or parasellar areas A: IV - Extends into cavernous sinus, optic chiasm, or pituitary fossa

Management of Laryngeal Clefts by grade

A: I - Observation, anti-reflux & thickened feeds, vs. Endoscopic repair (2 layer closure) A: II or III - Laryngofissure for precise multilayered anatomic closure A: IV - Laryngofissure & Sternotomy vs. Lateral pharyngotomy & Thoracotomy (postop ECMO, no ETT)

4 types of Chiari malformations

A: I - Protrusion of cerebellar Tonsils A: II - Protrusion of cerebellar Vermis, lower Pons and Medulla (Arnold-Chiari malformation) A: III - Herniation of Cerebellum (high cervical meningocele) A: IV - Cerebellar Hypoplasia (Dandy-Walker syndrome)

Sade & Berco's stages of middle ear atelectasis

A: I - Retraction A: II - Severe retraction (against ISJ) A: III - Atelectasis (against Promontory) A: IV - Adhesive Otitis

Staging of non-Hodgkin's disease

A: I - Single LN region or ES site with exclusion of mediastinum and abdomen A: II - Single ES site with regional node involvement; 2+ LN regions or ES sites on same side of diaphragm A: III - Nodal regions or ES sites on both sides of diaphragm; any intrathoracic, paraspinal, epidural tumor A: IV - Disseminated disease or any of above with initial involvement of CNS, bone marrow, or both

Ann Arbor staging of Hodgkin's lymphoma

A: I - Single LN region or extralymphatic site (IE) A: II - 2+ LN regions or EL sites (IIE) on same side of diaphragm A: III - Nodal regions, EL sites (IIIE), or spleen involvement (IIIS) on both sides of diaphragm A: IV - Disseminated disease A: A (absence) or B (presence) of unexplained weight loss >10% of total body weight, unexplained fever >38, night sweats

Lymphatic malformation staging

A: I - Unilateral infrahyoid (macrocystic) A: II - Unilateral suprahyoid (microcystic) A: III - Unilateral infra and suprahyoid A: IV - Bilateral suprahyoid A: V - Bilateral infra and suprahyoid A: VI - Bilateral infrahyoid 3: "U In the U.S., cross-border (hyoid), across states (bilateral), and back again"

Cotton-Myer grading of SGS

A: I) 1-50% A: II) 51-70% A: III) 71-99% A: IV) 100%

Describe the Radkowski staging of JNA

A: IA - Limited to nose and/or nasopharyngeal vault A: IB - Extension to one or more sinuses A: IIA - Minimal extension to Pterygomaxillary Fossa (PMF) A: IIB - Full occupation of PMF with or without erosion of orbital bones A: IIC - Infratemporal fossa with/without cheek, or posterior to pterygoid plates A: IIIA - Erosion of skull base; minimal intracranial A: IIIB - Erosion of skull base; extensive intracranial with/without cavernous sinus

Rates of Cholesteatoma Recurrence after ICW and CWD Mastoidectomy

A: ICW - Residual = 20-35%, Recurrence = 5-20% A: CWD - Residual = 2-17%, Recurrence = 0-10% 3: Just remember "30 & 15%, and 10 & 5%"

ABR findings in Acoustic Neuromas

A: ILDV (Interaural difference) >0.2 msec in 40-60% A: Absent Waveforms in 20-30%, Wave I only wave present 10-20% A: Abnormal Morphology A: Absolute wave V latency >5.7 msec A: Interwave latencies I-III (>2 msec) or I-V (>4 msec) - These are most specific A: Normal ABR in 10-15% (especially if tumour less than 1 cm, if >2cm then ABR basically 100% sensitive) A: 18-30% False Positive rate for small intracanalicular tumors

Ddx of CSF rhinorrhea

A: Iatrogenic A: Blunt trauma (closed HI or skull fractures) A: Increased ICP - Tumors, Post-infectious, Post-trauma, Hydrocephalus A: Arachnoid granulations A: Encephalocele A: CSF otorrhea via ET 3: Consult Neurosurgery for all cases of spontaneous CSF leak in order to R/O elevated Intracranial Pressure

Ddx of Bilateral vocal cord Immobility

A: Iatrogenic A: Malignancy A: Neurologic A: Endotracheal intubation A: Idiopathic/Infectious 3: "I M Not En Idiot!" 3: Important to differentiate bilateral VC paralysis from posterior glottic stenosis

Etiologies of esophageal perforation

A: Iatrogenic (most common, usually in cervical esophagus) A: Trauma (blunt, penetrating) A: Toxic - caustic ingestion A: Inflammatory A: Neoplastic A: Spontaneous - Mallory-Weiss, Boerhaave

What are the boundaries of Lore's triangle, and what does it identify?

A: Identifies location of RLN in the neck during thyroidectomy A: Medial - tracheal sidewall A: Lateral - carotid sheath A: Superior - surface of retracted thyroid pole inferiorly

Five Etiologies of BPPV (dislodgement of otoconia)

A: Idiopathic (48%) A: Closed head injury A: Vestibular neuritis A: Otologic infections (postsurgery & nonpostsurgery) A: Prolonged bedrest

Lip adhesion

A: If necessary, done @ 2-4 weeks with definitive repair at 4-6 months of age A: For unilateral, bilateral, or asymmetric wide complete cleft lip and palate

Five factors present in nasal secretions which fight infection

A: IgA A: IgG A: Lysozyme A: Lactoferrin A: Interferon

9 possible mechanisms of Immunosuppression in HNSCC

A: IgA-blocking antibodies A: Circulating immune complexes A: Antigenic modulation of tumor A: TGF-b A: Suppressor T cells A: Histamine activation of Suppressor T cells A: Exogenous INF A: Tumor production (PGs, p15e, VEGF, GM-CSF, IL-1/10) A: Exogenous Immunosuppressive agents (chemo, XRT) 3: "I C Ant (Modulate) Transform or Suppress His Infinite Tumor Immunosuppression"

Immune workup for recurrent sinusitis

A: IgG subclasses A: IgM A: IgA A: IgE A: Ability to respond to polysaccharide antigens of S. pneumoniae, and H. flu

Most Sensitive tests for Mononucleosis

A: IgM and IgG for EBV (VCA and EBNA) 3: Monospot/Heterophile Ab test is negative in early disease (for 2-3 weeks)

What is Talbot's law?

A: Images linger on the retina for 0.2 seconds, and only five distinct images can be viewed per second; if more than five per second are presented, the viewer perceives the images as connected and sees the result as a smooth motion

Define the Donaldson line

A: Imaginary line along the axis of the lateral SCC, bisecting the posterior SCC; a landmark identifying the superior aspect of the endolymphatic sac

Three phases of skin graft survival

A: Imbibition - Absorption of nutrients from underlying recipient bed A: Inosculation - Growth of graft's blood vessels to meet preexisting blood vessels of recipient bed A: Neovascularization - New blood vessels form & bridge graft to underlying tissues

Treatment of Ottitic Hydrocephalus

A: Immediate Neurosurgical consultation A: CT Brain A: Medical - Initially Acetazolamide (500 mg bid), Steroids, repeated LP's (Drain?); next Furosemide & Mannitol (0.5 g/Kg IV) if not controlled; VP shunt A: Surgical - Mastoidectomy, expose diseased Dura, Debride Granulations

Patients who may benefit from early Surgical intervention in Facial nerve injury & other indications

A: Immediate paralysis with no return of function & absent responses >1 week A: Immediate paralysis with progressive decline on ENoG to <10% within 6 days of injury and surgery preformed within 14 days of injury A: Immediate paralysis with CT evidence of significant temporal bone disruption, Bony Impingement, or obvious Transsection of Fallopian canal A: Exploration at time of surgery for other reasons

Success rates of LAUP/UPPP procedures for snoring

A: Immediate results = 85-90% success A: At 12 months = 45% success (all comers) A: Success is higher in thin patients, with only retropalatal collapse, and have tonsillectomy performed

Nine Advantages of free flaps over local or regional flaps

A: Immediate single stage reconstruction A: Two team approach A: Potential for sensate, motor & secretory function A: Improved vascularity and healing A: Large number of donor sites available A: Unrestricted positioning and reach A: Large amount of composite tissue A: Can cover any defect (larger tumor free margins) A: Low rate of resorption A: Permits primary placement of osseointegrated implants

Six contraindications for Type I Isshiki medialization thyroplasty

A: Immunodeficiency A: Coagulopathy A: Neoplasm A: Previous Teflon A: Previous surgery or trauma A: Previous XRT

Four areas of research in Gene therapy for Neoplastic disease

A: Immunomodulation - Modification of TIL's, enhance immunogenicity in vivo & ex vivo A: Cytotoxic gene therapy - Introduction of Enzymes that convert a Prodrug into a toxic compound HSV-tk (thymidine kinase phosphorylates gangcyclovir into its active compound) Cytosine deaminase (converts prodrug 5-fluorocytosine into 5-fluorouracil) A: Antisense gene therapy - Introduction of Blocking RNA against RNA or DNA targets A: Suppressor gene therapy

Discuss Interferons

A: Immunomodulatory secretory proteins with a broad range of effects Antitumor cytotoxicity Inhibition of cell proliferation Gene activation Modulation of cell surface antigens Stimulation of other cytokines & immune modulators Immune cell activation A: Type I interferons - IFN-α produced by leukocytes, IFN-ß produced by connective tissue cells, in response to viral infection or exposure to ds-RNA; significant antiviral effects = decreased viral replication, increased cell membrane proteins, decreased lymphocyte mitogenesis A: Type II interferons - IFN-γ produced by T-lymphocytes in response to antigenic stimuli; more potent immunomodulatory effect than type I interferons; increases expression of cell membrane antigens (including MHC I/II & Fc receptors)

Three reasons for surgical excision of a benign salivary gland lesion diagnosed on FNAB

A: Impact on cosmesis & continued growth of lesion A: Definitive diagnosis A: Chance of malignant transformation over time

Indications for treatment of Hemangioma (VASCO)

A: Impaired Vision or Hearing A: Airway compromise A: Impaired Swallowing A: Cosmesis (massive, ulcerating, disfiguring) A: High Output Cardiac failure

Top 3 reasons for failure of Type I Isshiki medialization thyroplasty

A: Implant to High A: Implant to Anterior A: Undercorrection of the glottic gap posteriorly 3: Others include Extrusion, Infection, Hematoma

Eight indications for use of in-vitro allergy testing include

A: Impracticality of skin testing due to Skin disorder, Drug inhibition or Uncooperative patients A: Clarification of Bizarre or Borderline results from skin testing A: Prevention of systemic reactions in patients with a prior history of or suspected Anaphylactic reactions, Asthma, or when testing for Stinging hypersensitivity A: Convenience of in vitro testing 3: "Impractical Clarifications Prevent Convenience"

Four Indications for Treatment of Localized Kaposi sarcoma

A: Improve local control A: Reduce symptoms A: Cosmesis

Five Advantages of Binaural Hearing Aid use

A: Improved sound Localization (Microphone placed closely to each middle ear) A: Increased Loudness (Binaural summation) A: Release from Masking (Binaural squelch) - Helps tune out unwanted noise, Improves speech intelligibility in noise due to phase differences of signal and noise A: Elimination of head Shadow effect (which normally reduces High Frequency cues necessary for hearing Consonants) A: Avoidance of sensory Deprivation in children

Describe the Moffett position

A: Improves efficacy of nasal steroid application, especially for reaching the frontal sinus A: Performed by laying upside down with one's head hanging over the side A: Hwang, Oto clinics 2001 says: kneeling with forehead against the floor, although many modifications exist - this is actually the mecca position

Overall survival for NPC postradiotherapy

A: In general, 5 year survival ~60%, worse for type I and better for types II/III A: Survival continues to decreased past 10 years

Define the Gonzalez-Ulloa line

A: In the facial profile view, it is perpendicular from the Frankfort line, passing through the Nasion A: Midforehead, Subnasale, upper/lower Lips, Pogonion all lie on the line

What hearing aids allows use of concha bowl and pinna

A: In-the-canal (ITC) A: Completely-in-canal (CIC)

Define Olfactory Agnosia

A: Inability to recognize an odor

What are the questions on the Epworth Sleepiness Scale, and how to score them? (IPWORTh SLeepiness): How likely are you to fall asleep...

A: Inactive in a public place A: Passenger seat for 1 hour A: Watching TV A: Lying dOwn in the afternOOn A: Sitting and Reading A: Sitting and Talking A: Stopped in traffic for a few minutes A: Sitting quietly after Lunch 3: Score each from 0 to 3; >6 Abnormal, >10 Very sleepy, >16 Dangerous

Five Reasons for Alaryngeal TE speech failure

A: Inadequate air supply - Decreased respiratory support, improper stoma occlusion A: Puncture site closure A: Prosthesis failure - Position, patency, size, type, degradation, infection A: Reflex pharyngeal constrictor spasm - Constrictors reflexively contract when dilated A: Nonvibrating pharyngoesophageal segment - Postradiation, reconstructed segment

Epidemiology of Choanal Atresia

A: Incidence 1:5000-8000 births A: F/M = 2/1 A: 50% have other anomalies (75% of bilateral cases) A: 60% mixed bony-membranous, 30% bony, 10% membranous A: 70% unilateral (60% of which are right-sided)

Two Factors affecting impedance tympanography in neonates

A: Incomplete ossification of the EAC causing greater compliance A: Persistence of Middle ear fluid or Mesenchyme

Define Hypertrophy

A: Increase in individual cell size

Flap physiology - Define Creep

A: Increase in strain seen when skin is under constant stress A: Mechanical Creep - Occurs over a matter of minutes and is due to an displacement/extrusion of fluid & MPS ground substance from the dermis, and a breakdown of the dermal framework; realignment of collagen, microfragmentation of elastin, migration of undermined tissue into field defect by stretching force A: Biologic Creep - Seen in pregnancy, lymphedema, morbid obesity, gradual stress stretches skin, muscles, nerves, blood vessels, lymphatics; increase lengths by cell division and collagen & elastin synthesis

Define Hyperplasia

A: Increase in the number of cells per unit of tissue or organ of origin

Two histological findings with Allergic Rhinitis (from inferior turbinate)

A: Increased ratio of columnar epithelium to goblet cells (4:1 instead of 5:1) A: Increased number of eosinophils (>20% of granulocytes is suggestive of inhalant allergy)

Five effects resulting from an arytenoid adduction

A: Increased stiffness at the vocal process A: Medializes and stabilizes the vocal process A: Increased length of the vocal cord A: Decreased difference in height between the 2 vocal cords (lowers vocal process) A: Decreased malrotation of the arytenoid 3: Always admit patient postop due to risk of hematoma, or airway obstruction

Define Hyperkeratosis

A: Increased thickness of keratinized layers

Define Acanthosis

A: Increased thickness of prickle cell layer

Six mechanisms of action of Hyperbaric oxygen (HBO)

A: Increases amount of oxygen dissolved in solution A: Increases diffusion distance from capillaries (ie the distance Oxygen can diffuse) A: Increases neovascularization A: Increases fibroblast proliferation A: Improves leukocyte oxidative killing A: Antiobiotic synergy - Fluoroquinolones, Ampho B, Aminoglycosides

Five prognostic factors in oral cavity cancer imparting a worse prognosis

A: Increasing Stage A: Increasing tumor Thickness A: Perineural invasion A: Angiolymphatic invasion A: DNA ploidy

Blood supply to the Incudostapedial joint

A: Incudal artery - Terminal branch of Ossicular branch of Anterior Tympanic artery

Four most common Ossicular injuries in Temporal bone Trauma (Bailey)

A: Incudostapedial Joint dislocation (most common, 82%) A: Complete Incus dislocation (57%) A: Stapedial Crural fracture (30%) A: Epitympanic ossicular Fixation (25%) A: Malleus fracture (11%) 3: "Joint In Cruci Fix Malleable"

Least secure ossicle and most susceptible to shear force

A: Incus

Discuss 3 Indications for Dynamic Muscle Slings in Facial Paralysis

A: Indicated when neural techniques are unsuitable A: No intact facial neuromuscular system due to Absence (Mobius) A: Loss of motor endplates from longstanding facial nerve interruption (>3 years) A: Sacrifice of other Cranial nerves 3: Steristrips and overcorrection a must

Describe Ewald's Second Law

A: Indicates an excitation-inhibition asymmetry; Movement of endolymph in the "on" direction for a canal produced greater nystagmus than an equal movement of endolymph in the "off" direction

Indication and 4 Contraindications for Medial Maxillectomy

A: Indication - Benign or low grade malignant tumors confined to the lateral nasal wall, maxillary antrum, and/or ethmoid sinus A: Contraindications - Invasive malignancies extending intracranially, to the pterygoids, palate, or extensively into the orbit

Indication and Contraindication for Exploratory Tympanotomy and Ossicular Reconstruction in Temporal Bone Trauma

A: Indication = >30 dB CHL persisting >2 months postinjury A: Contraindication = Only hearing ear

Indications & Contraindications for Anterior Cricoid Split

A: Indications - Failure of extubation 2 times in neonate/young child, Congenital small cricoid in older child A: Contraindications - Short duration of extubation before reintubation (hours), Peak airway pressure > 35 mm Hg

Indications for Adenoidectomy (ION)

A: Infection - Recurrent acute/chronic Adenoiditis, recurrent acute/chronic Otitis Media with or without Effusion (kids >4 years benefit from adenoidectomy with the Second set of PETs), chronic Sinusitis A: Obstruction - Adenoid hyperplasia with chronic Nasal Obstruction or obligate Mouth Breathing, Sleep-related disordered breathing (OSAS, OSHS, UARS), Cor pulmonale, FTT, Orofacial growth/dental/speech/swallowing abnormalities A: Neoplasia - Suspected, benign or malignant, Lymphoproliferative disorder 3: Absolute indications for adenoidectomy = Obstruction with secondary Cardiopulmonary complications, suspected Malignancy, and FTT Clinical indicators for adenoidectomy as recommended by the AAO-HNS in 2000 are: Four or more episodes of recurrent purulent rhinorrhea in prior 12 months in a child <12. One episode documented by intranasal examination or diagnostic imaging. Persisting symptoms of adenoiditis after 2 courses of antibiotic therapy. One course of antibiotics should be with a beta-lactamase stable antibiotic for at least 2 weeks. Sleep disturbance with nasal airway obstruction persisting for at least 3 months. Hyponasal or nasal speech Otitis media with effusion >3 months or second set of tubes Dental malocclusion or orofacial growth disturbance documented by orthodontist. Cardiopulmonary complications including cor pulmonale, pulmonary hypertension, right ventricular hypertrophy associated with upper airway obstruction. Otitis media with effusion over age 4.

Indications for Tonsillectomy (ION)

A: Infection - Recurrent acute/chronic Tonsillitis, Halitosis, Complications (Cervical Abscess, Airway obstruction Cardiac valve disease, recurrent febrile Seizures, tonsillar Hemorrhage), streptococcus Carrier unresponsive to medical treatment, recurrent or unresponsive Peritonsillar Abscess A: Obstruction - Tonsillar hyperplasia with obstruction, Sleep-related disordered breathing (OSAS, OSHS, UARS), Cor pulmonale, FTT, Orofacial growth/dental/speech/swallowing abnormalities A: Neoplasia - Suspected, benign or malignant, Lymphoproliferative disorder 3: Absolute indications for tonsillectomy = Suspected Malignancy, FTT, OSA with Cardiopulmonary complications, Acute tonsillitis with Febrile convulsions, tonsillar Hemorrhage Clinical indicators for tonsillectomy as recommended by the AAO-HNS in 2000 are: Patient with 3 or more infections per year despite adequate medical therapy. Hypertrophy causing dental malocclusion or adversely affecting orofacial growth documented by orthodontist. Hypertrophy causing upper airway obstruction, severe dysphagia, sleep disorders, or cardiopulmonary complications. Peritonsillar abscess unresponsive to medical management and drainage documented by surgeon, unless surgery performed during acute stage. Persistent foul taste or breath due to chronic tonsillitis not responsive to medical therapy. Chronic or recurrent tonsillitis associated with the streptococcal carrier state and not responding to beta-lactamase resistant antibiotics. Unilateral tonsil hypertrophy presumed neoplastic

Ludwig's angina

A: Infection of sublingual, submandibular, submental spaces A: Usually mixed flora A: Airway distress secondary to posteriorly displaced tongue, kinks the supraglottis A: Treatment - Airway control, I&D of abscess/phlegmon

Ddx of oral cavity vesiculobullous lesions

A: Infectious (3) and Autoimmune (6) A: HSV stomatitis (primary, secondary on hard palate/attached gingival or vermillion border) A: VZV stomatitis (primary, secondary) A: Coxsackie (Herpangina) A: Lupus (DLE > SCLE > SLE) A: Erythema multiforme A: Cicatricial pemphigoid (mucosal involvement > cutaneous involvement overall) A: Bullous pemphigoid (cutaneous involvement > mucosal involvement overall) A: Pemphigus vulgaris A: Lichen planus (erosive or bullous form)

Enlarging Lacrimal gland pathology

A: Infectious - Acute bacterial (Staph, Strep), chronic bacterial (trachoma, syphilis, TB), viral (HSV, EBV, mumps) A: Epithelial - Pleomorphic adenoma most common (50%); other 50% malignant (adenoid cystic, mucoepidermoid, malignant mixed, adenocarcinoma) A: Hematopoietic - Benign lymphoid hyperplasia (20% get systemic lymphoma), atypical lymphoid hyperplasia (40% get systemic lymphoma), lymphoma (50% localized), leukemia A: Idiopathic - Sarcoid

Ddx of Granulomatous diseases of Head & Neck

A: Infectious - Bacterial Cat scratch disease/Bacillary angiomatosis (Bartonella henselae) Rhinoscleroma (Klebsialla) TB Non-tuberculous mycobacterium Leprosy Syphilis (Treponema pallidum) Actinomycosis A: Infectious - Fungal Histoplasmosis (Mississippi & Ohio river valleys, rarely can cause Fibrosing mediastinitis, and Mediastinal granulomatosis) Blastomycosis (East of Mississippi & Central America) Coccidiomycosis (San Joaquin valley, California) Rhinosporidious Candidiasis Aspergillus/Mucorales A: Infectious - Parasites Leishmaniasis A: Trauma Intubation Teflon granuloma A: Neoplastic Histiocytosis X (Langerhans Cell Granulomatous, Hand-Schuller-Christian) Pyogenic granuloma (Lobular capillary hemangioma) T-Cell lymphoma A: Idiopathic Sarcoidosis Necrotizing Sialometaplasia A: Autoimmune/vasculitis Wegeners Granulomatous Churg-Strauss syndrome Relapsing Polychondritis Systemic lupus Erythematous Sjogrens Syndrome

Causes of Sudden Unilateral vestibular dysfunction (IDIOT VI)

A: Infectious - Labyrinthitis (Viral or Bacterial), Vestibular neuronitis, Ramzay-Hunt, Otosyphilis A: Degenerative - Fistula, Cholesteatoma A: Iatrogenic - Labrynthectomy, Vestibular nerve section A: Ototoxicity - Gentamicin, Streptomicin A: Trauma - Blunt, Barotrauma, Labyrinthine concussion A: Vascular - Labyrinthine apoplexy A: Idiopathic - Meniere's

Etiologic factors for the development of sinusitis, classified into 3 main categories

A: Infectious - Viral, Bacterial, Fungal A: Immune - Congenital, Acquired, Allergic A: Local - Craniofacial anomalies (Choanal atresia, VPI, Cleft palate), Trauma, Surgery, Dental, Ciliary, Anatomic

Eight openings of the pterygopalatine fossa and connections where applicable

A: Inferior orbital fissure - Orbit A: Pterygomaxillary fissure - Infratemporal fossa A: Sphenopalatine foramen - Nose A: Foramen rotundum - Middle cranial fossa A: Pterygoid canal (Vidian N) A: Pharyngeal canal A: Greater palatine canal - Oral cavity A: Lesser palatine canal - Oral cavity

What are the 2 Extraocular muscles most commonly affected in Grave's ophthalmopathy?

A: Inferior rectus A: Medial rectus

Pathway for Frey's syndrome

A: Inferior salivatory nucleus A: CN IX A: Jacobson's nerve (leaves skull base via inferior tympanic canaliculus) A: LSPN (enters skull base via foramen ovale) A: Otic ganglion A: Auriculotemporal nerve A: Parotid and sweat glands

Innervation of the Vestibular organs

A: Inferior vestibular nerve - PSCC (via Singular nerve) and Saccule A: Superior vestibular nerve - HSCC, SSCC, and Utricle

EPOS 2007 guidelines for diagnosis of Rhinosinusitis

A: Inflammation of the nose and sinuses with 2 or more symptoms, one of which is either nasal congestion, obstruction, or anterior or posterior nasal discharge, AND +/- A: Facial pressure/pain A: Hyposmia/Anosmia AND EITHER A: Endoscopic finding of polyps, middle meatal edema, mucosal obstruction A: CT evidence of mucosal changes in the OMC or sinuses

Describe Orbital Pseudotumor

A: Inflammation of unknown etiology, diffuse or localized (muscle, lacrimal gland, sclera, optic nerve) A: Symptoms - Pain increased with movement, Proptosis A: Signs - Eyelid edema, chemosis, diplopia, vision loss (20%) A: Radiographic - CT enlargement of involved structures; EOM tendons involved unlike in Graves ophthalmopathy; MRI T1 iso to muscle with gad enhancement, T2 hyper to fat A: Treatment - Corticosteroids, rarely cyclophosphamide, cyclosporine, or XRT; biopsy treatment failures!

Ddx of Maxillary & Mandibular cysts

A: Inflammatory (85%) - Periapical/Radicular cyst Lateral periodontal cyst A: Developmental Odontogenic (10%) - Dentigerous/Follicular cyst Odontogenic keratocyst Eruption cyst Alveolar cyst of infants Gingival cyst of adults A: Nonodontogenic (5%) - Nasopalatine duct cyst Nasolacrimal duct cyst Midpalatal cyst of infants Nasolabial cyst Aneurysmal bone cavity Stafne lingual cortical defect Idiopathic (Traumatic) bone cyst

Five foramena of maxilla

A: Infraorbital A: Superior alveolar canal(s) A: Incisive canal A: Maxillary ostium A: Accessory maxillary ostium

Pharmacologic action of Cocaine

A: Inhibits Norepi & Dobutamine reuptake, which is responsible for catecholamine side effects

Seven Physiologic alterations to the aerodigestive tract with a tracheostomy tube

A: Inhibits vertical motion of the larynx during swallowing A: Reduced subglottic pressure A: Dyscoordination between breathing and swallowing A: Elimination of vocal cord closure reflex while swallowing A: Physiologic obstruction when swallowing d/t esophageal compression A: Tracheomalacia A: Interferes with Ciliary function 3: Potential for Tracheoesophageal fistula in long term

Standard results for Stapedectomy, Initial & Revision

A: Initial Stapedectomy - 90% have closure of ABG <10 dB, 10% no change, 1-2% Profound SNHL A: Revision Stapedectomy - 65% have successful result, 7x increased risk of Profound SNHL

How many rows of OHCs, IHCs? Which are more important and why?

A: Inner Hair cells - 1 row, flask shaped, few; 90% of neurons innervating the cochlea synapse on the IHC's (type I neurons, bipolar, myelinated), ~20 neurons synapse on each IHC A: Outer Hair cells - 3 rows of OHC's, cylindrical, many; 10% of neurons innervating the cochlea synapse on the OHC's (type II neurons, pseudomonopolar, unmyelinated), 1 neuron innervates ~10 OHC's 3: ~2000 Efferent neurons from Superior Olive synapse on the hair cells (LSO to IHC's, MSO to OHC's)

Seven most common congenital Vascular anomalies that can cause Tracheomalacia?

A: Inominate artery compression (most common, arises more medially than normal, anterior compression) A: Double Aortic arch (most common true vascular ring) A: Right Aortic arch with left ligamentum arteriosum A: Anomalous left Carotid artery A: Anomalous right Subclavian artery (Dysphagia lusoria: arises from left descending aorta, passes behind E (85%) or between T&E (15%), associated with a non-recurrent right RLN) A: Pulmonary artery sling (left pulmonary artery arises off the right pulmonary artery, passes between T&E) A: Pulmonary artery dilation 3: 2 aortic, 2 pulmonary, 1 carotid, 1 innominate, and 1 subclavian

Location of stridor by its pattern

A: Inspiratory = dynamic supraglottis and glottis A: Biphasic = subglottis and cervical trachea A: Expiratory = fixed intrathoracic trachea

Describe Paradoxical Vocal Fold Motion

A: Inspiratory vocal fold adduction, absent during sleep, and with normal voice A: Seen in young women (6:1); poorly understood A: Can temporize with Heliox, sedatives; DO NOT TRACH!

What are 2 prerequisites for central adaptation of peripheral vertigo?

A: Intact visual system A: Intact proprioceptive system

Reference levels for the Intensity Level (IL) and Sound Pressure Level (SPL) referents

A: Intensity (I) = Pressure Squared (P2) A: Bell B= log (Is/Ir) A: Decibel dB = 10 log (Is/Ir) A: Sound pressure dB = 20 log (Ps/Pr) A: Intensity level - 10-12 watts/m2 or 10-16 watts/cm2 A: Sound pressure level - 20 μPa or 0.0002 dynes/cm2

Dose limiting factors for Intensity and Total dose

A: Intensity - Acute toxicity = Mucositis A: Total dose - Late toxicity = Soft tissue Fibrosis

Bogdasarian classification of adult posterior glottic stenosis

A: Interarytenoid adhesion with posterior sinus tract A: Posterior commisure stenosis A: Posterior commissure stenosis with unilateral cricoarytenoid ankylosis A: Posterior commissure stenosis with bilateral cricoarytenoid ankylosis

Describe the Incisions used in surgery of the nasal tip

A: Intercartilaginous A: Intracartilaginous A: Marginal

Neurovascular supply of the Rib osseous composite free flaps

A: Intercostal vascular pedicle; Marginal blood supply to skin

Mechanism of action and 3 examples of Vinca Alkaloid chemotherapeutic agents

A: Interfere with Mitotic Spindle formation 3: Vincristine, Vinblastine, Vinorelbine

Describe the ARIA (Allergic Rhinitis and its Impact on Asthma) Classification of Allergic Rhinitis

A: Intermittent - <4 days/week OR <4 weeks/year A: Persistent - >4 days/week AND >4 weeks/year A: Mild - Normal sleep, no impairment in ADLs/Work/School, and no troublesome Symptoms A: Moderate-Severe - Any of Abnormal sleep, impairment in ADLs/Work/School, or troublesome Symptoms

ABR findings in Multiple Sclerosis

A: Interpeak interval longer, with normal hearing

Six methods of Increasing Tip Rotation in rhinoplasty

A: Interrupted strip technique - Fosters cephalic tip rotation, especially if performed lateral to domes A: Residual complete strip - Conservative cephalic or caudal trim of lateral crura A: Caudal septal shortening +/- high septal transfixion A: Shorten lateral crura A: Shorten overlong upper lateral cartilages A: Reduction of the membranous columellar skin A: Reduction of convex caudal medial crura A: Illusionary - Cartilage grafts in the lobule, columella, nasolabial angle (plumping grafts) A: Minor - Resection of excess vestibular skin A: Minor - Cutting of depressor septi muscle A: Minor - Proper taping 3: Volume reduction of alar cartilages with complete strip or incomplete strip (more pronounced with incomplete); Can control complete strip resection with base up triangle resection laterally); If using complete strip, use adjuctive procedures to augment

Brachytherapy source placement (3 types)

A: Interstitial A: Intracavitary A: Surface mold

Describe the characteristics of Cogan's Syndrome

A: Interstitial Keratitis - 90% of patients with typical syndrome; Atypical syndrome patients have Scleritis, Episcleritis, Papilledema, Retinal detachment A: Meniere-like Hearing Loss - Peak shaped initially, but Bilateral and Progressive, can become Profound A: Vestibular symptoms - Sudden true Vertigo, Ataxia, Vegetative symptoms A: Systemic Inflammatory involvement - Heart and Lungs involved in typical form; more Systemic involvement seen in Atypical form A: Patients typically have URTI within 7-10 days of initial onset A: Untreated may lead to Profound SNHL and Loss of Vestibular function A: Therapy - Corticosteroids (systemic & topical), Cyclophosphamide

Minor FESS complications

A: Intracranal - CSF leak A: Orbital - Emphysema, Fat herniation, Ecchymosis A: Bleeding - Small amount, not requiring transfusion A: Other - Hyposmia, mild Asthma exacerbation, MRSA infection, Hypesthesia of the infraorbital nerve or teeth, Synechiae, Myospherulosis, Atrophic rhinitis, Osteitis

Three approaches for CSF leak repair

A: Intracranial A: Extracranial external A: Endoscopic 3: Can be Overlay or Underlay (>5 mm) 3: Postop care = 24 hrs bed rest, HOB elevated, lumbar drain, no nose blowing, no straining, stool softeners lumbar drain at 10 cc/h for 24-48 hrs may be used

Branches of the Maxillary division of trigeminal

A: Intracranial Middle meningeal N A: Pterygopalatine/Sphenopalatine Zygomatic N (Zygomaticofacial, Zygomaticotemporal) Pterygopalatine N - Contains postganglionic fibers to lacrimal gland (via zygomatic br) Orbital branches Nasopalatine N Greater Palatine N Lesser Palatine N Pharyngeal branch Posterior Superior Alveolar N Infraorbital N (Middle superior alveolar, Anterior superior alveolar) Facial (Inferior palpebral, External nasal, Superior labial branch)

Major FESS complications

A: Intracranial (7) - CSF leak, Tension pneumocephalus, Meningitis, Abscess, Hemorrhage, Encephalocele, Direct brain injury A: Orbital (5) - NLD injury, Enophthalmos, Diplopia (medial rectus injury), Hematoma, Blindness (optic nerve injury) A: Bleeding (4) - Damage to Anterior ethmoidal, Sphenopalatine, or Internal carotid arteries; any requirement of a transfusion A: Other - Anosmia, Asthma exacerbation, Toxic Shock, Death

Surgical Approaches to the Lateral compartment of the Middle Cranial Fossa

A: Intracranial - Temporal craniotomy A: Transtemporal - Lateral, mainly Extradural; Anterior limit is Intrapetrous ICA; adjuncts used in combination with other MCF approaches A: Infratemporal A: Transfacial - Facial Translocation

Lengths of the Facial nerve segments (Bailys/Ben's note in the brackets - essentially all multiples of 4)

A: Intracranial = 23-24 mm (24 mm) A: Meatal = 8-10 mm (8 mm) A: Labyrinthine = 3-5 mm (4 mm) A: Tympanic = 8-10 mm (12 mm) A: Mastoid = 10-14 mm (20 mm) A: Extratemporal = 15-20 mm to pes A: Narrowest portion of the Facial nerve A: At fundus of IAC = 0.68 mm Labyrinthine segment?

Complications of tracheotomy

A: Intraoperative Great vessel injury Laryngeal damage (improper placement) Injury to tracheoesophageal partition Pneumothorax Pneumomediastinum Thymus injury (in children) Death A: Early postoperative Tracheostomy tube obstruction (mucus plug) Tracheostomy tube displacement Infection Pulmonary edema A: Late postoperative Tracheal stenosis Granulation tissue Subglottic stenosis Tracheal-inominate fistula Tracheoesophageal fistula

Twelve Complications of Cochlear Implantation

A: Intraoperative complications - Partial implant insertion, Perilymph gusher, Traumatic implantation A: Early postoperative (CLIT, sorry ladies) - Cross stimulation of CN VII, Leak (CSF or Perilymph), Infection (Wound, Flap necrosis, Meningitis), Tinnitus/Vertigo A: Late postoperative - Device failure, Extrusion

Eight uses on laryngeal EMG

A: Intraoperative nerve Monitoring A: Diagnose Neurologic disorders (MG, ALS,...) A: Localization of lesion (RLN vs. SLN) A: Establish Prognosis (correlate in 60-70% after 4 weeks) A: Biofeedback A: Differentiate between paralysis and arytenoid Fixation A: Synkinesis A: Muscle Localization for Botox injection 3: "Many Nerds form Local Programs use Biofeedback to Fix Sinking Boats"

Describe Masking

A: Introduction of noise via air conduction into the nontest ear during audiometric testing to eliminate it from testing (prevent crossover) A: Narrow band noise used for pure tones; Speech noise for speech signals A: Used when air conduction of test ear exceeds bone conduction of nontest ear by 40 dB, or when air-bone gap exceeds 0 dB in the test ear (can always use it for bone)

Etiology/predisposing factors for acquired SGS in adults (10)

A: Intubation-related (>90%, 1-8% incidence): oversized, repeated, shearing motion (agitation), route, duration A: Iatrogenic trauma (laser surgery, high tracheotomy, cricothyrotomy) A: External laryngeal trauma A: Burn (inhalational/thermal/chemical/radiation) A: Gastroesophageal reflux A: Infection (Primary or Superimposed bacterial or Fungal infection) A: Neoplasms (Benign or Malignant, Intrinsic or Extrinsic) A: Autoimmune (Wegeners, Sarcoidosis, SLE) A: Inflammatory disease (Sarcoidosis, Relapsing Polychondritis) A: Idiopathic SGS

Four Theories of Acquired Cholesteatoma formation

A: Invagination theory - Main modality of Primary acquired cholesteatoma; infection has not given rise to the cholesteatoma, Occurs in Pars Flaccida A: Epithelial Invasion - Main modality of Secondary acquired cholesteatoma, due to meatal epithelium invading middle ear through a Marginal or Attic Perforation (contact inhibition lost); Postero-superior cholesteatoma A: Squamous Metaplasia theory - Secondary to chronic or recurrent Infection (not thought to be significant) A: Implantation - Iatrogenic implantation into middle ear or eardrum from Surgery, Foreign body, Blast injury 3: Two other theories = Basal cell hyperplasia theory, and Otitis media with effusion theory (as a means to create a retraction pocket)

Significant prognostic factors for Follicular thyroid ca

A: Invasion of blood vessels (has significant effect the prognosis in the first 10 years)

Four Stages of Intracranial Abscess Formation (InLET)

A: Invasion/Encephalitis - Fever, malaise, drowsiness, headache A: Localization/Latency - Clinically silent A: Enlargement/Expansion - Mass effect, seizures, LOC seen A: Termination - Rupture into ventricle or subarachnoid, usually Fatal

Schneiderian papilloma types and locations

A: Inverted (50%) - Lateral wall A: Fungiform/Everted (47%) - Septal A: Cylindrical/Columnar/Oncocytic (3%) - Lateral wall 3: Bony destruction common even though benign, malignant change seen in ~10%, low for septal, highest for Lateral wall 3: Therapy - Total surgical removal, either Medial Maxillectomy (recurrence rate ~9%), or Transnasal/ESS

Describe Foster Kennedy syndrome

A: Ipsilateral anosmia/hyposmia A: Ipsilateral optic atrophy A: Central/contralateral papilledema 3: Due to tumors of the olfactory groove or sphenoid ridge (e.g. meningiomas)

Five Requirements for Nerve Crossover techniques in Facial Paralysis

A: Irreversible facial nerve injury A: Intact proximal donor nerve A: Intact distal facial nerve A: Intact motor endplate function A: Intact mimetic function

Describe the Lateral Pontomedullary syndrome, the symptoms/signs, the vessel and cranial nuclei affected

A: Ischemia in AICA territory; No CN VI, IX, X, or Horner's as in Wallenberg, just CN V, VII, VIII nuclei, Cerebellum, and descending Spinothalamic tracts A: Ipsilateral loss of Pain & Temperature sensation on the Face - Nucleus & Descending tract of CN V A: Ipsilateral Facial weakness - CN VII Nerve & Nucleus A: Ipsilateral Hearing loss & Tinnitus - Labyrinth & Cochlear nerve A: Spontaneous Vestibular nystagmus - Labyrinth & Vestibular nerve A: Ipsilateral Cerebellar dysfunction - Anteroinferior Cerebellum A: Contralateral loss of Pain & Temperature sensation - Descending Spinothalamic fibers 3: "No Horny 6-9 on 10 year olds"

Biopsy of which 4 nasal masses are at significant risk of hemorrhage?

A: JNA A: Hemangioma A: Hemangiopericytoma A: AV malformations A: Also some mets ie. renal cell and thyroid

Scheibe malformation is associated with which 4 syndromes?

A: Jervell-Lange Nielsen A: Refsum A: Usher A: Waardenburg 3: "JR Ushers and Wards Sheep"

Three Risk factors for increased spread of RRP

A: Juvenile A: Young age A: HPV type 11

Kasabach-Merritt phenomenon

A: Kaposiform hemangioendothelioma or tufted angioma A: Sequestration of platelets, ecchymoses A: Supportive care, IFN-A2a, PRBCs for anemia, limit blood products unless bleeding

Four non-infectious non-neoplastic causes of lymphadenopathy

A: Kawasaki A: Kikuchi -histiocytic necrotizing lymphadenopathy, Lymphocytosis, fever, splenomegaly (like lymphoma), self-limiting, can use steroids, need to biopsy to make dx and rule out cancer A: Kimora - Idiopathic unilateral?, usually submandibular adenopathy in Asians, High IgE, Eosinophilia, Painless, needs bx to r/o Ca, treat with conservative surgical excision but typically recur. A: Sarcoid

NPC histology and clinical features WHO

A: Keratinizing (Type I) - Squamous, 25% in nonendemic areas, 1-2% in endemic areas; distinct intercellular bridges, keratin production; worst prognosis = 10% 5 year survival A: Nonkeratinizing (Type II) - Squamous/Transitional, 12% in nonendemic areas; little/no keratinization present; papillary morphology; ~60% 5 year survival A: Undifferentiated (Type III) - Lymphoepithelioma, 63% in nonendemic areas, 95% in endemic areas; clear, enlarged nuclei; ~60% 5 year survival

Describe the two nasal vascular arterial plexuses and their blood supply

A: Kiesselbach's plexus - Little's area, found on the anterior septum (Anterior ethmoid, Sphenopalatine, Greater palatine, and Superior labial arteries) A: Woodruff's plexus (naso-nasopharyngeal plexus) - Venous, located in the posterior lateral nasal wall inferior to the Inferior Turbinate

Esophageal triangles

A: Killian's dehiscence or triangle - Space between inferior constrictor and cricopharyngeus (Baileys) A: Killian-Jamison area - Space between pars obliquus and pars fundiformis (horizontal) portions of the cricopharyngeus Laimer's triangle - Between the cricopharyngeus and uppermost circular fibers of the esophagus (Cummings)

Blood supply to the Facial nerve

A: Labyrinthine (AICA origin) - IAC portion A: Superficial Petrosal (Middle Meningeal origin) - Perigeniculate region A: Stylomastoid (Posterior Auricular origin) - Mastoid & Tympanic regions 3: Rich anastamoses between Superficial Petrosal and Stylomastoid; poor in Labyrinthine segments

Seven Indications for CWD Mastoidectomy (FUSION)

A: Labyrinthine Fistula or other Complication A: Unfit for second look, Unsuccessful previous mastoidectomy (residual/recurrent cholesteatoma) A: Sclerotic Mastoid A: Involved of the posterior EAC wall (Automastoidectomy) A: Only Hearing ear A: Non-compliant patient and uncertain followup

Blood supply to Inner ear

A: Labyrinthine artery branches off of AICA (80%; or from AICA 45%, Superior cerebellar artery 24%, basilar artery 16%?) A: Anterior Vestibular artery - Supplies Utricle & part of Saccule, Horizontal and Anterior SCC A: Common Cochlear artery - Proper Cochlear artery/Spiral Modiolar artery, supplies Cochlea except basal turn A: Vestibulocochlear artery - Off the Common Cochlear, further splits; Cochlear ramus (supplies Basal turn of cochlea), and Posterior Vestibular artery (supplies Posterior SCC, Saccule)

Jongkee's formulas and their levels of significance

A: Labyrinthine preponderance/Unilateral Weakness - Comparison of each vestibular organ's responsiveness to caloric stimulation; >20% difference considered significant (decreased responsiveness of the HSCC or ampullary nerve) A: Directional preponderance - Comparison of the total right beating responses and total left beating responses; >25% (Cummings) or >30% (Bailey's) considered significant (in Spontaneous nystagmus, also in Central asymmetric sensitivities to inhibitory-excitatory stimuli)

Six possible findings in Immotile cilia

A: Lack of Dynein arms A: Lack of central core structures A: Radial spoke defect A: Microtubule translocation A: Microtubule altered in length A: Ciliary aplasia

What are the 3 phases of irradiated tissue response to HBO

A: Lag phase - First 8 treatments; Increase in collagen synthesis and capillary budding; Little change in O2 tension (30-50%) A: Rapid Response - 8-22 treatments; Neoangiogenesis, fibroplasias, increase cellularity, increase collagen; Log increase in O2 tension (plateau at 82%) A: Plateau phase - Greater than 22 treatments; Loss of hypoxia, therefore less stimulation for angiogenesis; O2 tension plateaus at 85% 3: "Lag 8 Rapid Response 22 Plateau"

What are the four sides of Bolger's parallelogram?

A: Lamina papyracea A: Skull base A: Superior turbinate A: Basal lamella of SUPERIOR turbinate

Define Keratoacanthosis

A: Large acanthoma, surface keratosis

Most common causes of stridor in children Infants?

A: Laryngeal (60%) = Laryngomalacia (60%), Subglottic stenosis (20%), VC palsy (13%), Others (7%) A: Tracheal (15%) = Tracheomalacia (45%), Vascular compression (45%), Stenosis (5%) A: Bronchial & tracheal infections 5% each; miscellaneous 15% 3: Croup most common in children?

Define Pallister-Hall syndrome

A: Laryngeal (clefts or bifid epiglottis) A: Hypothalamus abnormalities (hypothalamic hamartoblastoma, hypopituitarism) A: Postaxial polydactyly A: Imperforate anus

What is the suspected mechanism of SIDS and how is placement in the supine position protective?

A: Laryngeal chemoreflex (LCR) A: Decreasing the hypercarbia associated with the prone position which potentiates the LCR

2 alternatives to ETT in the airway management of known SGS cases

A: Laryngeal mask airway A: Heliox

Ddx of congenital supraglottic abnormalities (8)

A: Laryngomalacia A: Hemangioma A: Lymphatic malformation A: Laryngocele A: Saccular cyst A: Anomalous cuneiform cartilage A: Bifid epiglottis A: Supraglottic web

What 4 pediatric airway abnormalities are improved in the prone position?

A: Laryngomalacia A: Pierre-Robin sequence A: Vascular compression A: Mediastinal mass

Ddx Congenital Laryngeal Stridor

A: Laryngomalacia A: VC paralysis A: Stenosis (subglottic, tracheal) A: Web A: Clefts A: Cysts (valecular, saccular, subglottic) A: Rings (complete tracheal, vascular)

Top 3 congenital laryngeal anomalies

A: Laryngomalacia A: Vocal cord paralysis A: Subglottic stenosis

Name 10 associations of laryngopharyngeal reflux (LPR)

A: Laryngotracheal stenosis (SGS most common association = 92%, Bailey p.833) A: Laryngomalacia A: Laryngospasm/Cough/Asthma/Recurrent Croup/SIDS A: Vocal cord Nodules A: Vocal cord Polyps A: Otitis media A: Vocal process (contact) ulcers/Granulation/Granulomas A: Carcinoma A: Chronic laryngitis/Mucosal thickening/Pachyderma laryngis A: Dental cares and erosion

Development of the Frontal sinus

A: Last sinus to fully develop A: Not present at birth, begins growth in the 3rd-5th year, continues into adolescence A: Pneumatization highly variable; volume up to 7 cc; 5-10% are atretic 3: Remember "3-5 years"

Briefly describe the plain radiographic views of the head

A: Lateral A: Lateral oblique - A: Towne's - Half-axial antero-posterior, 30 degrees down A: Caldwell - Occipitofrontal or posteroanterior A: Waters - Head tipped back A: Submentovertical - Base projection

Describe the borders of Level Ib in the neck

A: Lateral - body of mandible A: Medial - anterior belly of digastric A: Posterior - posterior belly of digastric A: Floor - myelohoid and hyoglossus

Boundaries of the Paraglottic space

A: Lateral - thyroid cartilage (anterior) and mucosa over medial wall of pyriform fossa (posteriorly) A: Superior - quadrangular membrane A: Inferior - conus elasticus

Six CT findings in Temporal Bone Infection Contraindicating Lumbar Puncture

A: Lateral Shift of midline structures A: 4th Ventricle Obstruction A: Loss of Cisterns (Basal, Suprachiasmal, Superior Cerebellar, or Quadrigeminal Plate Cisterns)

Tensor veli palatini anatomy

A: Lateral head origin from scaphoid fossa & greater sphenoid wing, swings anterior, lateral & inferior, tendon around hamulus, inserts onto posterior hard palate and palatine aponeurosis

Describe Wallenberg syndrome, the symptoms/signs, the vessel and cranial nuclei affected

A: Lateral medullary syndrome, obstruction of Vertebral artery or rarely PICA, involves CN V to X nuclei, Cerebellum, and descending Spinothalamic tracts A: Ipsilateral Horner's syndrome - Involves preganglionic sympathetic fibers from the Hypothalamus A: Ipsilateral loss of Facial Pain & Temperature sensation - Nucleus & descending tract of CN V A: Ipsilateral Lateral Rectus and Facial weakness - CN's VI & VII A: Spontaneous nystagmus - Vestibular nucleus A: Ipsilateral paralysis of Palate, Pharynx, Larynx - Nucleus Ambiguus and exiting fibers of CNs IX & X A: Ipsilateral Dysmetria, Dysrhythmia, Dysdiadochokinesia - Cerebellum A: Contralateral loss of Pain & Temperature sensation - Descending Spinothalamic fibers

Five changes to the vocal cord that occur after severing the RLN

A: Lateral position/Immobile A: Decreased length A: Decreased height A: Atophy/bowing A: Yellow discoloration

Blood supply to tympanic membrane

A: Lateral surface - Deep Auricular artery A: Medial surface - Anterior Tympanic artery 3: Both arise from Internal maxillary

Approaches to post Traumatic CN VII Repair

A: Lateral to geniculate ganglion - Transmastoid +/- Facial Recess A: Medial to geniculate with No Hearing - Translabyrinthine A: Medial to geniculate with Hearing - Combination Middle Cranial Fossa, Transmastoid and Transepitympanic (supralabyrinthine)

Most common benign esophageal lesions

A: Leiomyoma (most common) A: Polyps A: Hemangioma A: Adenoma A: Neurofibroma A: Granular cell tumor

Ho & Neel's 5 adverse prognostic indicators for NPC

A: Length & symptomatology of disease (>2 months) A: Extension beyond the nasopharynx A: Low neck disease (Levels IV/V) A: Keratinizing histology A: Distant metastasis

Relevant eustachian tube anatomy

A: Length = ~36 mm in adults, ~18 mm long in infants A: 1/3 bony, 2/3 cartilaginous (opposite of EAC which is 1/3 cartilaginous, 2/3 bony) A: Wide at both ends, narrow in midportion at the isthmus (1 x 2 mm) A: Lateral end 4 mm above floor of epitympanum, relatively horizontal, meets cartilaginous portion at a 160 degree angle, cartilaginous portion then descends at a 40-45 degree angle, 45 degrees off sagittal, into the nasopharynx A: Torus Tubarius - Formed by soft tissue overlying the medial cartilaginous end of the tube A: Rosenmuller's fossa - Nasopharyngeal mucosal fold found posterior to torus

What is a Hutchinson's freckle?

A: Lentigo Maligna, or melanoma In Situ 3: Associated with Solar damage & Atypical melanocytes, Radial spread along dermal/epidermal Junction, focal Nests of cells 3: Overall probability of developing into melanoma ~5%; presence of a Nodule carries likelihood of invasion of 100%

Discuss Superior Canal BPPV

A: Less common that PSCC & HSCC A: Nystagmus is Vertical (Downbeating) and Torsional (Ageotropic), due to activation of Ipsilateral Superior Rectus & Contralateral Inferior Oblique

Theoretic increases in length with varying Z-plasty angles

A: Less than 30 degrees - Tip necrosis A: 30 degrees = 25% A: 45 degrees = 50% A: 60 degrees = 75% A: Greater than 75 degrees = Redundant standing cones

End to side anastamosis - maximum angle of anastamotic vessels

A: Less than 60 degrees

Four cutaneous branches of the cervical plexus

A: Lesser occipital (C2,3) A: Greater auricular (C2,3) A: Transverse cervical (C2,3) A: Supraclavicular (C3,4; divided into anterior, middle, and posterior)

Four contraindications for Tonsillectomy (LASH CPA)

A: Leukemias A: Agranulocytosis A: Systemic disease that is uncontrolled (DM, TB) A: Hemophilias 3: Relative contraindications = Cleft Palate, and Acute infection

Identify the Histochemical Markers for Lymphoma

A: Leukocyte Common Antigen (LCA)/Panleukocyte autoantibodies A: CDs

Describe the Clark classification

A: Level I = Within Epidermis only A: Level II = Invasion into Papillary dermis A: Level III = Invasion to Papillary-Reticular dermis A: Level IV = Invasion into Reticular dermis A: Level V = Invasion into Subcutaneous tissues

N+ disease in WDTC - what type of neck dissection

A: Level II-VI A: Level I only if clinically positive A: N0 - Central compartment (upper mediastinum to hyoid, laterally to Carotids)

What is the Kobner Isomorphic Phenomenon?

A: Lichen planus lesions provoked by physical trauma (eg: scratching)

Name 7 Amide anasthetics

A: Lidocaine, bupivicaine, mepivicaine, prilocaine, ropivicaine, etidocaine, dibucaine 3: Metabolized by liver dealkylation

Conservative/Medical therapy for OSA

A: Lifestyle modifications - Weight loss, Smoking cessation, Sleep hygiene (alcohol, sedatives, caffeine, supine posture) A: Progesterone - Respiratory stimulant A: Acetazolamide - Increases hydrogen concentration in blood A: Theophylline - Increases hypoxic ventilatory drive A: Protryptiline A: Oxygen therapy

Six disadvantages of Flexible endoscopes

A: Light transport is inferior to the rigid endoscope A: Magnification of the image is inferior to the rigid endoscope A: Distortion of the periphery of the image (wide-angle distortion) A: Trade-off between adequate focus and light fiber mismatch causing a moiré or honeycomb effect that is enhanced by the edge-detection software of digital imaging systems A: Many patients find the flexible examination to be more invasive than the rigid A: Risks of nosebleed, adverse reactions to the anesthetic, and vasovagal reaction 3: Many of these issues are resolved with newer "chip-tip" scopes

Discuss Titanium as a facial implant material

A: Lightweight A: Resists Corrosion A: Soft, deformed by loading forces A: Most Biocompatible - forms titanium oxide on surface (10 nm), interacts with protein for binding (osteointegration)

Eight Common sites of lesion for RRP

A: Limen Vestibuli (junction of the nasal vestibule and nasal cavity proper) A: Nasopharyngeal surface of Soft Palate A: Midline of laryngeal surface of Epiglottis A: Upper & lower Ventricle margins A: Undersurface of the Vocal Cords A: Carina A: Bronchial Spurs A: Tracheostomy site (iatrogenic junction) 3: Preferentially junction of squamous epithelium with respiratory epithelium

Four Disadvantages of Smaller hearing aids

A: Limited Amplification, so for less severe hearing losses A: Increased Feedback A: Requires better Dexterity A: More Fragile & Expensive

Three disadvantages of local flaps

A: Limited length with random pattern blood supply A: Not enough bulk for deep defects A: Distortion surrounding structures due to donor site closure in some instances

Define Ohngren's line

A: Line between Medial Canthus and Angle of Mandible that divides Maxillary sinus for staging

Define the Frankfurt line

A: Line drawn from the lower margin of the orbit to the superior border of the external auditory meatus A: AKA the anthropological base line, or Reid's line

Pathway of afferent taste fibers from the oral tongue

A: Lingual nerve A: Chorda tympani A: CN VII A: Geniculate ganglion (cell bodies present anteriorly) A: Nervus intermedius A: Nucleus tractus solitarius (Pons) A: Ventroposteromedial nucleus of the Thalamus (VPMT) A: Cortex (Operculum & Anterodorsal Insula)

Four sensory nerves that supply tongue

A: Lingual nerve (V3) A: Chorda tympani (Facial nerve) A: Glossopharyngeal nerve A: Vagus nerve

Ddx of Posterior Tongue Mass & investigations

A: Lingual thyroid A: TGDC A: Cystic hygroma A: Valecular cyst A: Dermoid/Teratoma 3: Investigations - CT scan to define margins of the mass; TSH, T3/T4 establish thyroid function; I-123 scan or ultrasound to rule out lingual thyroid, identify other foci of functioning thyroid tissue

Name all 6 components of Walderyer's ring

A: Lingual tonsils A: Palatine tonsils A: Adenoid A: Gerlach's tonsils (Tubal, posterior to ET opening) A: Lateral bands A: Posterior pharyngeal wall

Treatment options for Systemic Kaposi sarcoma

A: Liposomal Doxo- or Daunorubicin +/- Combination Vinblastine-Bleomycin A: Systemic Etoposide and Paclitaxel

Name the six sphincters regulating normal swallowing

A: Lips/oral commissure A: The tongue with hard palate A: Soft palate with the tongue base (Palatoglossus) A: Velopharynx A: Larynx (true cords first, false cords next, then epiglottis & aryepiglottic folds) A: Cricopharyngeus (upper esophageal sphincter)

Describe the Histology of Meningiomas

A: Lobulated groups of cells reminiscent of normal Arachnoid Granulations A: Psammoma bodies often present A: Hyperostosis of surrounding Bone in 25%

Medical management of Grave's orbitopathy

A: Local Eye care - Eyedrops, nocturnal eye taping, sunglasses A: Thyroid therapy - Antithyroid drugs (PTU, Methimazole), radioiodine ablation, thyroidectomy A: Steroid therapy - For acute vision loss in Graves orbitopathy, Prednisone 80-120 mg qd x 2 weeks with slow taper following this A: Immunosuppressives - Cyclosporine, Cyclophosphamide experimental to for autoimmune aspects A: Radiotherapy - Indicated for late sequelae of stable thyroid orbitopathy, 20 Gy in 10 fractions (2 weeks), good-excellent response in 35-92%, not for acute/subacute phases, for those who can't tolerate, refuse, or fail surgery 3: Beta-blockers - Propranolol can be used as adjunct for severe symptoms in Grave's

Nasolacrimal Duct Cyst location, pathophysiology, and treatment

A: Location - Below inferior turbinate anteriorly A: Pathophysiology - Proximal and distal obstruction of the nasolacrimal duct with fluid accumulation and cyst formation; 85% resolve by 9 months A: Treatment - Medical = Nasal decongestants and feeding modifications A: Treatment - Surgical = Endoscopic transnasal marsupialization, CO2 laser may be used, ophthalmology consult for intraoperative nasolacrimal duct probing stenting 3: Indications for surgery (ION) - Infection, respiratory Obstruction, Nutrition (feeding difficulties)

Adenocarcinoma

A: Location = MSG > parotid A: Aggressive tumors, likely to recur & metastasize A: Gross - firm/hard, attached to surrounding tissue A: Micro - cylindric cells of variable height, form papillae, acini, solid masses; mucin producing A: Treatment - Aggressive surgical resection, with post op XRT, elective ND if extensive local disease or high grade lesions

Mucoepidermoid carcinoma

A: Location = Parotid > MSG (palate) > SMG A: Low, Intermediate, or High Grade = Ratio of Mucinous to Epidermoid cells A: Treatment - Stage I and II = Wide local excision; Stage III and IV = Radical excision, possible neck dissection and post op XRT A: Mucoepidermoid Histiologic Grading A: Low grade - More Mucous cells, fewer epidermoid cells, with prominent cysts and mature cellular elements; behave like benign neoplasm but capable of local invasion & metastasis A: Intermediate grade - Mucous cells = Epidermoid cells, fewer and smaller cysts, increasing pleomorphism and mitotic figures A: High grade - More Epidermoid cells, fewer mucous cells; requires IHC staining for Mucin to differentiate from SCC

Ten clinical signs of Marfan's syndrome

A: Long spidery fingers, scoliosis, hammer toe, pigeon breast, dolichocephaly, low hairline, tall & thin body structure, lens displacement, aortic root dilatation, may have SNHL, CHL or mixed HL A: Autosomal dominant

List 4 Second generation H1 Antihistamines

A: Loratidine/Claritin A: Desloratidine/Aerius (3rd gen?) A: Certrizine/Reactine A: Fexofenadine/Allegra (3rd gen?) 3: "Lora-Deslora-Certri-Fexo"

Six criteria to call SCC Poorly Differentiated

A: Loss of Intercellular Bridges A: Loss of Keratinization A: Increased Number of Mitoses A: Abnormal Mitoses A: Multinucleated cells A: Nuclear and cellular Polymorphism

Define Anosmia

A: Loss of ability to smell A: Can be specific, partial, or total depending on whether certain odors or no odor can be detected

Define Attenuation

A: Loss of auditory sensitivity

Identify 6 Histologic features found in Dysplasia of the oral cavity

A: Loss of cellular Polarity A: Cellular/nuclear Pleomorphism A: Enlarged Nucleoli A: Increased Nuclear:Cytoplasmic Ratio A: Increased Mitoses/Mitotic spindling A: No penetration through the Basement Membrane

Characteristics of Megavoltage X-rays/Gamma rays (photons)

A: Low (4-6 MeV) or high (15-25 MeV) energy A: Skin-sparing properties A: Depth-dose properties (Penetration) A: Isodose distribution (Beam Uniformity)

CT characteristics of deep space neck infections/abscesses

A: Low attenuation A: Contrast enhancement of abscess wall A: Tissue edema A: Cystic/loculated appearance

CT & MRI findings for Polyps

A: Low density on CT A: Hypointense on T1, Hyperintense on T2

Classification of vascular malformations

A: Low flow (capillary, venous, lymphatic, combinations) A: High flow (arterial, arteriovenous)

Working classification of non-Hodgkin's

A: Low grade - Small lymphocytic, Follicular small cleaved, Follicular mixed A: Intermediate grade - Follicular large, Diffuse small cleaved, Diffuse mixed, Diffuse large cell (most common in H&N) A: High grade - Small noncleaved (Burkitt), Immunoblastic (Large cell), Lymphoblastic

Small TM Perforations selectively reduce which Frequencies

A: Lower more affected

Six functions of saliva

A: Lubrication/moisturization A: Digestion A: Taste modulation A: Buffering (pH 6-7, HCO3, PO4) A: Dental protection - via inorganic ions (Ca, F, PO4, Mg) A: Antibacterial properties - Secretory IgA, Lactoferrin, Lysozyme, Salivary peroxidase

Most common metastatic sites for head and neck Rhabdomyosarcoma

A: Lungs A: Bone A: Bone marrow

Four Indications for Neck dissections in Salivary gland malignancies

A: Lymph node mets A: >T3 = Size >4 cm, Extension to extraglandular tissues A: High grade tumors = SCC, AdenoCA, Undifferentiaed, High grade mucoepidermoid A: Transcervical approach

Histopathology of Sjogren's syndrome

A: Lymphocyte & histiocyte infiltration A: Acinar atrophy A: Ductal epithelial hyperplasia & metaplasia A: Prominence of myoepithelial components as ductal lumens disappear

Antigen specific tests for Autoimmune Inner Ear Disease

A: Lymphocyte Transformation Test (LTT) - Targets Type II Collagen, predictive value 70%, sensitivity 70-80% & specificity 93% in detecting organ specific AIED A: Lymphocyte Migration Inhibition Test (LMIT) - Sensitivity & specificity of test results questionable A: Western Blot Testing - Detection of Autoantibodies against Inner Ear Antigen epitope with a 68 kD molecular weight; possibly Best Evidence of Organ-Specific AIED; 68 kD Antigen either a Heat Shock Protein (HSP-70) or an Antigen bound to it

Ddx of nonepidermoid oral cavity cancer

A: Lymphoma A: Melanoma A: Kaposi's sarcoma A: Minor salivary gland cancer 3: 10% occurrence

60 year old female with blocked nose, and nasopharyngeal mass. Biopsy is LCA positive and cytokeratin negative. What is this? What is ruled out?

A: Lymphoma A: SCC (NPC) ruled out by negativity of Cytokeratin

Most common pediatric H & N malignancy

A: Lymphoma (60% NHL and 40% HL) Table 183-2 -- AGE-INCIDENCE RANKING FOR HEAD AND NECK TUMORS IN CHILDREN

Three approaches to a frontal mucocele, advantages & disadvantages of each

A: Lynch frontoethmoidectomy - Ethmoidectomy plus removal of entire frontal sinus floor A: Osteoplastic flap - Advantage = Eradication of disease and mucosa; Disadvantage = Requires coronal or mid-forehead approach, disfiguring, high rate of mucocele A: Endoscopic - Advantage = All intranasal; Disadvantage = Cannot access sinuses to eradicate mucosa, limited access laterally

Timing of thyroidectomy in children whose genetic screen is positive for MEN type 2

A: MEN 2A (Sipple) around 6 years of age A: MEN 2B (Wagenmann-Froboese) around 6 months of age 3: Risk of developing MCT during lifetime >90%

Definition of Phonation quotient

A: MPT/VC 3: Maximum phonation time divided by patient's vital capacity

What are Langerhan's cells?

A: Macrophage/Dendritic cell (APC) that normally resides in the epidermis of skin

Name of Neuroepithelial component of Otolith organs and significance of Striola

A: Macula A: Striola - Central line through otolith membrane A: Cilia movement Towards striola for Utricle causes Excitation A: Cilia movement Away from striola for Saccule causes Excitation

Primary Indication for a Fisch A Approach

A: Main indication is for Paragangliomas of the Temporal Bone

5 measure in initial management of xylocaine toxicity

A: Maintain airway A: Maintain ventilation A: Give oxygen A: Place IV A: Fluids/pressors prn

Discuss cell mediated immunity

A: Major Histocompatibility Complex proteins Class I - present on all cells, binds to CD8 Class II - present on APC's and B-cells, bind to CD4 A: T-cell subclasses Helper cells (TH) - Express CD4, recognizes foreign antigen and class II MHC's required IL-1 for activation; 2 subsets = TH1 stimulates IgM, IgG and IgA & increases CMI response, TH2 more effective in stimulating IgE antibody secretion Cytotoxic (TC) - Express CD8, recognizes antigen expressed with Class I MHC Suppressor (TS) 3: Delayed Hypersensitivity (TDH) Class II MHC activation, requires IL-2

Five proteins found in eosinophils

A: Major basic protein (MBP) A: Eosinophil cationic protein (ECP) A: Eosinophil peroxidase (EPO) A: Charcot Leyden crystal A: Neurotoxin

Ten indications for external (open) rhinoplasty (NOT OPEN REDUCTION!)

A: Major reconstruction A: Extensive Tip work A: Non-Caucasian nose A: Crooked nose/Eccentric anatomy A: Revision rhinoplasty A: Saddle nose deformity A: Closure of septal Perforation A: Cleft lip nasal deformity A: Benign nasal Tumors A: Teaching

What is the significance of Xanthine oxidase in reperfusion injury?

A: Major source of free radical in ischemic tissues, responsible for conversion of hypoxantine to uric acid after restoration of oxygen to hypoxic tissue, produces superoxide ions

Ddx of Unilateral vocal cord Immobility (MINTS, Bailey p.849)

A: Malignancy (25%, primary pulmonary malignancies and mediastinal adenopathy most commonly) A: Iatrogenic (24%) A: Idiopathic (20%, 70% of which will recover, may take 12 months) A: Infectious - Lyme, Syphilis, TB, EBV, other viral A: Neorologic - Poliomyelitis, Pseudobulbar palsy, MG, MS, ALS, CVA, Wallenberg A: Neurotoxic medications - Cisplatinum, Vincristine, Streptomycin, Quinine, Lead, Arsenic A: Trauma A: Thoracic aneurysm of cadiomegaly (Ortner syndrome) A: Systemic disease - Sarcoid, DM

Describe the difference between Malignant Mixed and CA ex-pleomorphic

A: Malignant mixed tumors display malignant transformation in both epithelial and stromal components, whereas CA ex-pleomorphic only displays malignant transformation in its epithelial component A: Their should be evidence of underlying benign pleomorphic adenoma in CA ex-pleomorphic

Types of Ossicular Joints

A: Malleo-Incudal joint - Diarthrodial (synovial) A: Incudo-Stapedial joist - Diarthrodial (synovial) A: Stapes footplate annular ligament - Syndesmotic

Management of carotid blowout

A: Manual pressure to stop bleeding A: Blood products/IV fluids A: OR ligation of carotid +/- Fogarty catheterization to help control bleed

What creates endolymph? How absorbed?

A: Marginal cells of Stria Vascularis (Cochlea) A: Dark cells of the Macula and Cristae (Utricle and Saccule, and SCC) A: Absorbed in Endolymphatic sac

Three physical properties that alter frequency of vocal cord vibration (what is the difference with the pitch question?)

A: Mass A: Stiffness A: Viscosity (what about length?)

8 muscles from 1st arch

A: Masseter A: Temporalis A: Medial pterygoid A: Lateral pterygoid A: Anterior belly of digastric A: Tensor tympani A: Tensor veli palatini A: Mylohyoid

Describe the Masticator space contents

A: Masseter, temporalis, pterygoid muscles A: Internal maxillary artery A: Mandibular nerve A: Trismus hallmark symptom A: Important in infections of Maxillomandibular molar origin

Eleven limits of Endoscopic Resection of Sinonasal tumors

A: Massive Dural/Intradural extension A: Massive Skull Base erosion A: Frontal sinus extension (especially if lateral to the mid pupillary line) A: Nasal pyramid extension A: Intraorbital extension A: Lacrimal tract extension A: Maxillary wall involvement (except medial) A: Nasopharyngeal extension A: Hard palate extension A: Abundant scar tissue from previous surgery A: Associated Squamous cell carcinoma

Five indications for Radiation for Skin tumors

A: Massive tumors (SCC >2 cm) A: Very aggressive (Merkels cell) A: Invasive (bone, muscle, cartilage, and nerve) A: Resection margins uncertain or positive A: Multiple recurrences A: Marjolin's ulcer A: To the neck - Multiple positive nodes, extracapsular spread and lymphovascular invasion

Eight functions of the oral cavity

A: Mastication A: Taste A: Deglutition A: Oral competence A: Articulation A: Respiration A: Airway protection A: Oral hygiene

Four criteria important in the De la Cruz classification of congenital aural atresia

A: Mastoid pneumatization A: Oval window footplate A: Footplate/CN VII relationship A: Inner ear 3: Normal in Minor, abnormal in Major

Name the 11 Intratemporal Complications of Otitis Media

A: Mastoiditis with or without Abscess (Postauricular, Bezold's, Zygomatic, Parapharyngeal, Retropharyngeal) A: Petrositis A: Facial Paralysis A: Labyrinthitis - Serous or Suppurative A: Labyrinthine Fistula (Pasha) A: TM Perforation A: Cholesteatoma A: Tympanosclerosis A: Fixation and Discontinuity of ossicular chain A: CHL or SNHL A: Chronic suppurative OM A: Adhesive otitis

Definition of Maximum phonation time

A: Maximum period a subject can maintain audible phonation of a vowel at comfortable pitch and loudness A: Women = 16-25 sec (15-25 sec easier to remember) A: Men = 22-34 sec (25-35 sec is easier to remember) A: Less than 10 sec is bad and reason to do something in VC paralysis for example

Describe the Reflex Decay test

A: Measured at 500Hz, 10 dB above stapedial reflex threshold for 10 seconds A: Abnormal decay = failure to maintain 50% of original amplitude in 10 seconds A: Indicates probable retrocochlear lesion (CN VIII or Brainstem)

Describe Electrocochleography (ECoG)

A: Measures Far Field (think it is actually near field) potentials arising from distal auditory pathway (Cochlea and Auditory nerve) A: Needle electrode placed either transtympanically onto Promontory or Transcanal A: Wide band click most commonly used stimulus

Discuss Rhinomanometry

A: Measures the resistance to airflow in the nose (transnasal pressure and air flow), but cannot identify specific sites of obstruction; 3 types A: Anterior Rhinomanometry - Occlusion of one nostril with a inflatable cuffed pressure sensor at the nasal orifice measures nasopharyngeal pressure; nonphysiologic, as all airflow happens through the other unoccluded nostril (can't be used in patients with septal perf's) A: Posterior Rhinomanometry - Placement of pressure catheter in mouth which can accurately measure nasopharyngeal pressure; more physiologic measure of nasal resistance through both nostrils simultaneously A: Postnasal (Pernasal) Rhinomanometry - Tube is placed in the posterior nose through one of the nostrils and measures pressure at both nasopharynx and nasal entrance and measures the pressure difference between the two 3: Total nasal resistance >0.3 Pa/cm2 usually is symptomatic 3: >35% reduction in resistance after decongestion infers mucosal disease; <35% reduction in resistance infers a structural cause

Describe Carhart's notch and provide a Theory for its appearance

A: Mechanical artifact that Reverses with stapes mobilization A: Hallmark audiologic sign of Otosclerosis characterized by a Decrease in Bone Conduction thresholds of A: 5 dB at 500 Hz A: 10 dB at 1000 Hz A: 15 dB at 2000 Hz A: 5 dB at 4000 Hz 3: Proposed theory - Stapes fixation disrupts the Normal Ossicular Resonance (2000 Hz), Possible relation to Resonant Frequency of EAC (2-3 kHz), Normal Compressional Mode of bone conduction is disturbed because of relative Perilymph Immobility

Borders of the CPA

A: Medial - Brainstem A: Lateral - Petrous bone A: Superior - Middle Cerebellar Peduncle A: Inferior - Arachnoid over lower Cranial Nerves A: Anterior - Petrous bone A: Posterior - Cerebellar Peduncle/Cerebellum

Define the Buccal space boundaries & source of infection

A: Medial - Buccinator & Buccopharyngeal fasica A: Lateral - Cheek skin A: Superior - Zygomatic arch A: Inferior - Inferior border of mandible A: Anterior - Lip muscles A: Posterior - Pterygmandibular raphe A: Important in maxillomandibular bicuspid & molar infections 3: Urgent external drainage as can spread to orbit, cavernous sinus, and intracranially via angular veins

Medial Maxillectomy is the En-bloc resection of what 5 structures?

A: Medial Maxillary sinus wall A: Ethmoid sinuses A: Lamina papyracea A: Medial Orbital floor A: Lacrimal bone

Describe the Mucociliary flow in the frontal sinus (60% recirculation at ostium-according to Amin)

A: Medial to the ostium - Flows superiorly then laterally along the roof A: Lateral to the ostium - Flow medially and inferiorly toward the ostium

Parasympathetics autonomic innervation of the major salivary glands

A: Mediates overall secretion of saliva A: Submandibular & sublingual glands - Superior Salivatory nucleus, Nervus Intermedius, Facial nerve, Chorda Tympani (Preganglionic), Submandibular ganglion, Lingual nerve (Postganglionic) A: Parotid gland - Inferior Salivatory nucleus, CN IX, Jacobsen's nerve, Lesser Superficial Petrosal nerve (Preganglionic), Otic ganglion, Auriculotemporal nerve (V3, Postganglionic)

Osteoradionecrosis, 4 treatment modalities

A: Medical - Local topical care, biopsy, culture & IV antibiotics A: Hyperbaric oxygen A: Debridement & partial mandibular resection, and removal of foreign bodies/sequestra A: Vascularized tissue transfer (local/regional/free)

Management options for respiratory distress in Pierre-Robin patient

A: Medical - Prone position, McGovern nipple, Nasopharyngeal airway, Intubation (difficult) A: Surgical - Tracheostomy, Subperiosteal Floor of mouth release, Glossopexy, Tongue-lip adhesion (Routledge), Distraction osteogenesis

What are the 7 CanMEDs roles? (MCC Has PMS)

A: Medical expert - Diagnostic and Therapeutic skills, effective patient care, access and apply information, consultation A: Communicator - Therapeutic relationship, relevant history, listen effectively, discuss appropriate information A: Collaborator - Consult effectively, multidisciplinary team A: Manager - Utilize and allocate resources, health care organization, use information technology A: Scholar - CME, facilitate learning, critically appraise the literature, contribute to new knowledge A: Health advocate - Identify determinants of health, for patients and community, recognize and respond A: Professional - Integrity, honesty, and compassion, interpersonal behaviours, practice medicine ethically consistent with obligations

Four therapeutic approaches for Chronic Aspiration

A: Medical management A: Adjunctive procedures A: Reversible surgical procedures A: Irreversible surgical procedures

Ddx of chronic drooling

A: Medications A: Neurologic A: Indirect causes - Nasal obstruction, head position, malocclusion, tongue size, sitting position, and emotional state

Ddx of small round cells on nasal biopsy (MRS SLEEPI)

A: Melanoma A: Rhabdomyosarcoma A: Small cell neuroendocrine tumor (Carcinoid?) A: Sinonasal undifferentiated carcinoma (SNUC) A: Lymphoma A: Esthesioneuroblastoma A: Ewing's sarcoma/PNET (Carcinoid?) A: Plasmacytoma A: Immature teratoma

Describe the classification for congenital inner ear malformations

A: Membranous inner ear malformations (~80%) A: Bony & Membranous malformations (~20%) A: Labyrinthine anomalies (40% of radiologically abnormal Cochleas will have SCC abnormality) A: Aqueductal anomalies A: IAC abnormalities

What is the human vocal fundamental frequency?

A: Men ~125 Hz; increases with age A: Women ~225 Hz; decreases with age

Reverse-Cookie bite (Tenting) Hearing loss (1)

A: Menieres A: Cogan (bilateral)

Ddx of Bilateral absent calorics (6)

A: Menieres A: Meningitis A: Ototoxic Medications A: Sepsis A: Autoimmune A: Bilateral temporal bone trauma 3: "Many Men Medidate Simply About Trucks"

Six main branches of the vagus nerve in the neck and subdivisions

A: Meningeal A: Auricular (Arnold's, exists tympanomastoid suture line, supplies posterior and lower EAC, and lower TM, causes referred otalgia!) A: Pharyngeal (from Nodose ganglion, runs between ICA & ECA, to join pharyngeal branch of CN IX) A: Superior laryngeal (External and Internal) A: Cervical cardiac A: Recurrent laryngeal

Five most common Primary Orbital Tumors overall

A: Meningioma (30% of all cases primary) A: Hemangioma A: Lymphoma A: Inflammatory tumors A: Optic nerve glioma 3: "Men Have Lived In Orbit" 3: Hemangioma & Dermoid most common tumors in kids, followed by Rhabdomyosarcoma

Eight Radiographic Differences between Meningiomas and Acoustic Neuromas

A: Meningiomas more Dense & Homogenous A: Meningiomas cause More Hyperostosis of surrounding bone A: Meningiomas Sessile, Broad base, Obtuse Angle to Petrous bone A: Meningiomas Eccentric over IAC A: Meningiomas Uncommonly involve the IAC, whereas ANs have Intracanulicular component A: Meningiomas frequently have a Dural Tail A: Meningiomas frequently have Calcifications A: Acoustic Neuroma exhibit Greater, More Homogeneous Gadolinium Enhancement

Eight causes of Labyrinthitis Ossificans

A: Meningitis A: Labyrinthitis (serous or suppurative) A: Labyrinthectomy A: Cochlear implantation A: Temporal bone trauma A: Advanced Otosclerosis A: Autoimmune A: Vascular occlusion

6 Intracranial complications of AOM

A: Meningitis (#1, HiB > Pneumococcus, Mondini) A: Epidural/subdural/cerebral abscesses A: Focal encephalitis A: Lateral/sigmoid sinus thrombosis (Tobey-Ayer/Quesckenstedt's test) A: Otitic hydrocephalus A: Blindness with optic neuropathy

Name 6 Intracranial Complications of Otitis Media

A: Meningitis (#1, HiB > Pneumococcus, Mondini) A: Epidural/subdural/cerebral abscesses A: Focal encephalitis A: Lateral/sigmoid sinus thrombosis (Tobey-Ayer/Quesckenstedt's test) A: Otitic hydrocephalus A: Blindness with optic neuropathy

Six Histological types of Meningioma

A: Meningotheliomatous (most common) A: Angioblastic A: Fibroblastic A: Psammomatous A: Transitional A: Malignant

Five factors affecting olfactory testing (MASSAM)

A: Mental State A: Age A: Satiety A: Sex A: Adaptation/Cross-adaptation A: Masking

Seven Uncommon CPA lesions

A: Metastatic malignant tumor A: Lipoma A: Dermoid A: Teratoma A: Chordoma A: Chondrosarcoma A: Giant cell tumor

Side effect of Prilocaine?

A: Methemoglobinemia with >600 mg

Describe the pathology for Bony & Membranous malformations (3)

A: Michel aplasia - Complete labyrinthine aplasia (arrest at end of 3rd week prior to Otic Capsule formation) A: Cochlear anomalies - Common cavity (4th week), Cochlear Aplasia (5th week), Cochlear Hypoplasia (6th week), A: Mondini - Incomplete bony and membranous labyrinth, cochlea may be present as a curved tube (7th week)

Indications for near total thyroidectomy

A: Microcarcinomas with multifocality A: Local lymph node mets (will get I-131 post op)

Name the WHO Architectural (growth patterns) subtypes of thyroid Follicular Adenoma

A: Microfollicular (fetal) A: Macrofollicular (colloid) A: Normofollicular (simple) A: Trabecular (embryonal?) A: Solid (embryonal?) A: Atypical 3: "My MaNor Troubles Solemn Atheists"

Triad seen in Pierre-Robin

A: Micrognathia A: Cleft palate A: Glossoptosis

Surgical Treatment for Intracordal Hematoma

A: Microlaryngoscopy and evacuation of hematoma

Name the components of a Cochlear Implant

A: Microphone A: External Processor A: External Transmitter A: Internal Receiver-Stimulator A: Electrode array

Dose of XRT for Micropscopic disease and N0 neck

A: Micropscopic dz = 65 Gy A: N0 neck = 55 Gy

Describe the Fisch Classification for Glomus Tumors

A: Middle ear only A: Middle ear and Mastoid A: Infralabyrinthine, toward the Petrous apex A: Intracranial extension <2 cm

Classification of EAC/ME Deformity (Ombredanne)

A: Minor - EAC may be mildly stenotic, ME space & TM normal or slightly small, conductive hearing loss secondary to ossicle absence or fixation A: Major - EAC & TM usually absent, decreased or absent middle ear space, ossicles rudimentary or absent, fused or fixed to atretic plate, CN VII displaced

Recurrent Apthous Stomatitis

A: Minor aphthae - <1 cm, heal in 7-10 days, without scarring A: Major aphthae - 1-3 cm in size or >1 ulcer, can leave scarring, bx to R/O SCC A: Herpetiform aphthae - 1-3 mm, about 150, unrelated to herpes virus

Describe 5 non-Mendelian inheritance patterns

A: Mitochondrial inheritance - maternal mitochondria inherited from ovum A: Dynamic mutation (trinucleotide repeats) A: Genetic imprinting - expression depends on parent of origin A: Mosaicism - presence of both normal and abnormal cells within the same organism, can be somatic or germline A: Uniparental disomy - offspring inherits both copies of a chromosome pair from a single parent

Bacteriology Chronic Otorrhea with or without cholesteatoma

A: Mixed flora including Streptococcus pneumonia and Anaerobes (including Bacteroides) A: Pseudomonas (most common aerobe) A: Staphylococcus aureus and epidermidis A: Gram negatives - Klebsiella, E. coli, Proteus

Eight Criteria for Hemilaryngectomy for recurrent cancer after radiotherapy

A: Mobile cord A: Lesion limited to one cord, may involve anterior commissure A: Body of arytenoid free of tumor A: Subglottic extension no more than 5mm A: The lesion must extend no higher than the lateral wall of the ventricle A: No cartilage invasion A: Recurrence correlating with initial tumor A: The entire area of pre-XRT tumor involvement must be encompassed in the resection

Mondini malformation is associated with which 6 syndromes?

A: Mobius syndrome A: Branchiootorenal A: Waardenburg A: Pendred A: Treacher-Collins A: Klippel Feil (Wildervanck)

Sympathetics autonomic innervation of the major salivary glands

A: Modulates composition of saliva A: Nucleus - Sympathetic chain T1-5 A: Preganglionics - Sympathetic chain A: Ganglion - Superior cervical A: Postganglionics - Run with blood vessels supplying the glands; Parotid = External carotid artery, Submandibular = Facial artery, Sublingual = Lingual artery

List 4 Intranasal corticosteroids

A: Mometasome/Nasonex A: Fluticasone/Flonase AND Avamys A: Triamcinolone/Nasacort A: Budesonide/Rhinocort AND Pulmicort

Describe the AAO-HNS Hearing Handicap

A: Monoaural Impairment (MI) = 1.5(PTA - 25) A: Hearing Handicap (HH) = [5(MIb) + (MIw)]/6 3: Assumptions - Hearing loss does not begin handicapping until the PTA (0.5, 1, 2, 3 kHz) exceeds 25 dB; Handicap grows at rate of 1.5% per dB of HL beyond 25dB; Unilateral deafness only a Mild handicap, 2 ears should not be equally weighted

Describe the 4 qualities of laser light that differentiate it from radiant light

A: Monochromatic - Radiant energy emitted all of one wavelength A: Collimated - Unidirectional light travel A: Coherent - Temporal (waves oscillate in phase) & Spatial (photons equal & parallel along wavefront) A: High power density

What are the 5 antigen presenting cells of the body?

A: Monocyte A: Macrophage A: Dendritic cell A: Langerhans cell A: B-cell 3: Also, Microglia (brain) and Kupfer cells (liver)

Parotid tumors arising from Myoepithelial cells

A: Monomorphic adenoma A: Pleomorphic adenoma A: Adenoid cystic

Eight indications for permanent tracheostomy in OSA

A: Morbid obesity A: Severe apneas with desaturation A: No response to weight loss and avoidance of sedatives and alcohol A: No response/tolerance to CPAP trial A: Cardiac arrhythmia with apneic events A: Cor pulmonale A: Chronic alveolar hypoventilation A: Disabling hypersomnolence

Six Advantages of Digital Hearing Aids

A: More Flexible programming strategies A: Multiple programs for different listening Environments A: Useful for Unusual hearing loss configurations A: Anti-Feedback mechanisms A: Improved Sound quality A: Higher Fidelity (no internal noise for circuitry) 3: "Flexible Environments provide Unusual Feedback for Sound Fidelity"

2 ways pediatric allergic fungal sinusitis is different from adult

A: More likely to facial skeleton abnormalities A: More likely unilateral

Clinical features and treatment of Hand-Schuller-Christian disease

A: More severe chronic disseminated form of Histiocytosis X, 30% overall mortality A: Classic triad in 10% - Diabetes insipidus, exophthalmos, skull lesions A: Treatment - Radiation, surgery, chemotherapy, or combination

Pemphigus Vulgaris

A: More severe, B-cell mediated autoimmune disease with IgG auto-antibodies against intercellular bridges A: Intraepithelial bullae, acantholysis A: Positive Nikolsky sign A: Asboe-Hansen sign (direct pressure over bulla causes extension) A: Tzanck cells are pathognomic A: Poor prognosis

Define Pleomorphism

A: More than one form of a single cell type

Diagnostic Criteria of Rheumatoid Arthritis

A: Morning stiffness ~1 hour A: Swelling in 3 or more joints A: Swelling in hand joints A: Symmetrical joint swelling A: Subcutaneous Rheumatoid nodules A: Serum RF A: Radiologic evidence of erosions or osteopenia in hand joints 3: Need 4 out of 7; criteria 1 through 4 must be >6 weeks and must be observed by a physician

Neonatal CMV infection and SNHL

A: Most Common cause of Viral Congenital deafness A: Unusual for CMV infection acquired after birth to cause hearing loss A: 1% of all children born; 95% clinically silent infection, 1-5% symptomatic with CID A: Cytomegalic Inclusion Disease - Hemolytic anemia, Hepatosplenomegaly, Jaundice, Purpura, Intracerebral calcifications & Microcephaly; 30-65% of surviving neonates will have Severe, Symmetric SNHL mostly in High Frequencies A: Asymptomatic CMV infection - Remainder of CMV infected children; 8-15% will have Mild-Moderate SNHL A: Treatment - Acyclovir 1500 mg/m2/q8h (Gangcyclovir not in peds?); no real clinical improvements seen

Discuss primary salivary gland pathologies within parapharyngeal space

A: Most common PPS tumor (45%), seen in pre-styloid space A: Pleomorphic adenoma most common, and mucoepidermoid most common malignant A: Can be parotid or minor salivary gland (look for fat plane between parotid and tumor); either displace carotid posteriorly

Discuss Atypical mycobacterial infection

A: Most common are M. Avium intracellulare, M. kansaii, M. scrofulaceum A: Childhood disease A: Nontender, slowly enlarging, skin fixation common A: Corneal ulceration is most common H&N manifestation A: Few systemic effects, no pulmonary involvement A: Antibiotic triple therapy may be effective - Clarithromycin, Rifampin, & Ethambutol A: Incision and drainage may cause fistulization A: Excision with skin for neck masses

Paraganglioma Pearls

A: Most common benign neoplasm of the temporal bone in adults A: 10% Familial, with Multiple lesions in 26% A: 10% of patients have Multiple lesions overall A: 5% Malignant - Clinical Signs of Invasion of surrounding structures & Distant Metastasis A: 3% active Secretors - Clinical Symptoms = Flushing, Diarrhea, Palpitations, Headache, Hypertension, Perspiration, Orthostasis; treated with α & ß blockade A: Tests - Serum Catecholamines, Urine VMA/Metanephrines/5-HIAA A: Radiography - CT shows irregular destruction of Jugular Foramen & Temporal Bone; MRI shows Salt & Pepper mix on T1 & T2 A: Treatment - Primarily Surgical with preop Embolization; Radiotherapy indicated for High risk patients, Incompletely excised or Recurrent lesions, Bilateral lesions, Metastases; Observation in select tumors that are small, stable, asymptomatic, in older patients 3: "Families with Multiple Malignant Secrets"

Discuss Achondroplasia

A: Most common cause of short limb dwarfism A: AD, most sporatic, mutation of FGFR-3 gene, chromosome 4p16.3 A: Short limbs, genu varum, limited elbow extension, trident hand, long trunk, lumbar lordosis, frontal bossing, midface hypoplasia A: Normal cognition

Adenoid cystic carcinoma

A: Most common malignancy of SMG, SLG, and MSGs; second most common malignancy overall A: Gross - Monolobular, either no capsule or partially encapsulated; infiltrates surrounding tissue A: Micro - Basaloid epithelium arranged in cylindric formations, eosinophilic hyaline stroma A: Histologic subtypes - Solid (worst prognosis), Tubular (best prognosis), Cribriform (intermediate prognosis, most common subtype) A: Grade depends on percentage of Solid elements; low grade <30% solid, high grade >30% solid A: Perineural invasion a typical feature A: Treatment - Surgery with postop XRT, neck dissection only if N+

Describe Periapical/Radicular cyst

A: Most common maxillary/mandibular cyst, arise from nonvital root/pulp infection A: Treatment - root canal

Discuss Sinonasal SCC

A: Most common sinonasal malignancy (85%) A: Maxillay > Ethmoid > Sphenoid > Frontal A: Incidence of Regional and Distant mets = 10 and 7% respectively A: Regional lymph nodes = Facial, Parotid, Submandibular A: Treatment - Surgical resection +/- Chemo/XRT A: Prognosis - 50% at 5 yrs

Discuss Osteoma

A: Most common tumor of the paranasal sinuses A: Clinical - Males affected 3:1; R/O Gardner's syndrome (with soft tissue tumors/cysts, and colonic polyps) because 100% risk of colorectal cancer by age 40 A: Causes - Developmental (at the junction of membranous frontal bone and endochondral ethmoid bone), but can also be Tramatic, or Infectious (osteitis) A: Histopathology - Eburnated (dense), Mature (cancellous), or Mixed A: Types (FEMS) - Frontoethmoidal (95%) > Frontal (80%) > Ethmoid (25%) > Maxillary > Spehoid A: Complications - Sinusitis (28%), Orbital, or Intracranial A: Treatment - Complete removal, only if symptomatic

Endotracheal tube pearls & complications

A: Most commonly made of polyvinyl chloride A: Cuff pressure >25 cm H2O will compromise mucosal capillary blood flow and cause necrosis

Goldenhar syndrome (oculoauriculovertebral spectrum)

A: Most sporadic, some AD A: Unilateral facial asymmetry, Hemifacial microsomia A: Ocular - Upper lid coloboma A: Otologic - Mildly deformity to Anotia, EAC atresia, ossicular abnormalities A: Underdevelopment of Orbit, Facial muscles, Mandible, may have Hemivertebrae

Difference between a Mucocele and a Mucus Retention Cyst

A: Mucocele - A chronic, cystic lesion of the paranasal sinuses lined with pseudostratified or low columnar epithelium, containing occasional goblet cells; presents radiologically with complete sinus opacification, with rounded thinning pushing margins A: Mucus retention cyst - Retained mucus within a blocked goblet cell, lined by sinus mucosa rather than a true epithelium 3: These definitions do not apply to salivary pathology, where only the mucus retention cyst is the true cyst, and mucocele refers to mucus extravasation

Parotid tumors arising from Excretory duct cells

A: Mucoepidermoid A: Squamous

Most common Malignant Salivary gland tumors in Children

A: Mucoepidermoid carcinoma A: Acinic cell A: Adenocarcinoma A: Undifferentiated carcinoma A: Adenoid cystic

Name 4 genera of Mucormycosis

A: Mucor A: Rhizomucor A: Rhizopus A: Absidia

Characteristics of Fungal Hyphae in Invasive fungal sinusitis

A: Mucor - Broad & ribbonlike (10-15 um), irregular/90 degree branching, rarely septated; order Mucorales in Zygomycetes class, most virulent and common is Rhizopus oryzae; more commonly seen in diabetic ketoacidotic patients A: Aspergillus - Narrow hyphae, regular septations, 45 degree branching; most virulent & frequent species found in those with invasive fungal sinusitis; typically A. fumigatus in North America; more commonly seen in immune compromised patients with neutropenia

Diabetic patient with poor glycemic control, acute onset sharp R facial pain, opacification of right maxillary sinus with bony destruction; what is dx and what would you see on anterior rhinoscopy?

A: Mucormycosis A: Necrosis of nasal mucosa, ischemic infarction, granular serosanguinous rhinorrhea; fungal hyphae may be seen A: Stains - Gomorri methenamine silver (GMS), KOH

Describe the 9 layers of the cheek from medial to lateral

A: Mucosa A: Pharyngobasilar fascia A: Buccinator muscle A: Buccopharyngeal fascia A: Buccinator fat pad A: Masseter muscle A: SMAS/muscles of facial expression A: Subcutaneous tissue A: Skin

Five levels of vocal cord

A: Mucosal layer - Stratified Nonkeratinizing squamous epithelium A: Lamina propria, Superficial layer (Reinke's space) - Few fibroblasts, scant elastic and collagen fibers A: Lamina propria, Intermediate layer - Mainly Elastic fibers, few fibroblasts A: Lamina propria, Deep layer - Collagenous fibers, fibroblasts A: Thyroarytenoid/Vocalis Muscle

Histology of Papillary thyroid carcinoma

A: Multicentricity A: Papillary formations with fibrovascular cores A: Elongated Follicles A: Psammoma bodies - Laminated calcific densities A: Nuclei - Enlarged, Irregularly shaped, Overlapping/Crowding, Ground glass chromatin (Optically cleared/Orphan Annie eyes), Margination of chromatin along nuclear membrane, abundant nuclear Grooves, intranuclear Cytoplasmic inclusions/Vacuoles, loss of basal Polarity, and prominent Nucleoli? A: Inspissated colloid A: Intratumoral fibrosis

Maffucci syndrome

A: Multiple Cavernous Hemangiomas, occasional Visceral vascular lesions A: Dyschondroplasia & shortening/deformity of involved bones A: Chondrosarcoma in 25%

Four skin lesions associated with a higher risk of melanoma?

A: Multiple benign nevi (>20 increases relative risk to 12:1) A: Congenital nevi (especially giant types >20 cm) A: Junctional nevi A: Dysplastic nevi (dysplastic nevus syndrome) 3: 50-70% of all melanoma arise from preexisting benign nevi 3: "Multiple CJDs"

What do you want to R/O in a patient with laryngeal amyloidosis

A: Multiple myeloma (very low percentage) A: Need bone marrow biopsy to rule this out, SPEP and UPEP (may also be useful)

Five viral causes of sialoadenitis

A: Mumps A: Coxsackie A A: Influenza A: HIV A: CMV A: Echovirus

Ddx of Headache

A: Muscular Tension A: Vascular - Migraine - with aura (classic), without aura (common), basilar (complicated) Cluster Mixed Migraine/Tension A: Inflammatory/Infectious - Sinusitis Dental A: Intracranial - Traction - Space-occupying lesion Vascular - SAH Infectious - Meningitis, Encephalitis A: Ocular - Oculomotor imbalance Glaucoma A: Trigeminal neuralgia (Tic douloureux) A: TMJ dysfunction

Subclassify Hyperkinetic Neurologic disorders of the larynx (7)

A: Muscular tension dysphonia A: Spasmodic dysphonias A: Essential tremor A: Tic (Tourette) A: Myoclonus A: Functional dysphonia/aphonia A: Paradoxical movement of the vocal cords

Discuss optimal Facial nerve Reanastomosis techniques

A: Must be Tension free A: Within temporal bone - End to end Apposition appropriate A: Extratemporally - Use 8-0 or smaller nylon for Epineurial repair

Describe Neurofibromatosis Type I (von Recklinghausen's disease)

A: Mutation of nerve growth factor Neurofibromin on chromosome 17 A: CNS involvement may lead to MR, blindness, SNHL, acoustic neuromas present in only 5%

Describe Neufibromatosis type II

A: Mutation of tumor suppressor gene Merlin on chromosome 22q A: Autosomal dominant, but 50% of cases due to spontaneous mutations A: 95% incidence of Bilateral Acoustic Neuromas often before age 21 A: Central Meningiomas, Gliomas, Schwannomas, early lens opacifications (cataracts) A: Fewer café au lait spots & cutaneous nodules than NF I

Side effects of 5-FluoroUracil

A: Myelosuppression, Mucositis, Dermatitis, Diarrhea, Cardiotoxicity 3: 13% single agent partial response

Side effects of Methotrexate

A: Myelosuppression, Mucositis, Dermatitis, N/V/D, Hepatic fibrosis A: Leucovorin (Folinic Acid) rescue 3: 30% partial response rate as a single agent

Side effects of Cisplatin

A: Myelosuppression, Nephrotoxicity, Ototoxicity, Neurotoxicity, N/V A: Decreased ototoxicity, neurotoxicity & nephrotoxicity with Carboplatin, myelosuppression main side effect 3: 15-30% single agent partial response

Describe the 2 Theories of Vocal Cord vibration

A: Myoelastic Aerodynamic theory A: Collapsible tube theory - Air velocity exceeds tissue wave velocity; Bernoulli effect?

Parts of a salivary gland tubuloacinar unit (MAISE)

A: Myoepithelial cell A: Acinus A: Intercalated duct A: Striated duct A: Excretory duct

Ddx of Multiloculated Bone/Jaw Cyst (MACHO)

A: Myxoma A: Ameloblastoma A: Aneurysmal bone cyst A: Cherubism A: Central giant cell granuloma A: Hemagioma A: OKC

Management of neck disease for Medullary thyroid ca

A: N0 - central compartment (upper mediastinum to hyoid, laterally to Carotids) A: N+ - include II-VI

Seven Medical management options for Chronic Aspiration

A: NPO A: Tube feeds, either NG or PEG A: Oral hygiene A: Speech therapy - Swallowing retraining, Postural Compensatory manoeuvers, Head position manoeuvers, Oral motor exercises A: Optimization of pulmonary status A: LPR prophylaxis A: Anti-sialogogues

Sleep stages

A: NREM I - Light stage, short duration, theta waves on EEG A: NREM II - Defined by sleep spindles/K-complexes on EEG (50% of sleep) A: NREM III & IV - Delta sleep, defined by high voltage, slow wave activity A: REM sleep - Beta waves on EEG (20% of sleep), Total body paralysis during REM and therefore OSA worse during REM 3: "At night, BATS Drink Blood" 3: Normal progression is Stage I - II - III - IV - III - II - REM...

Treatment options for SLE

A: NSAIDs, Antimalarials, Glucocorticoids A: Azathioprine & Cyclophosphamide for resistant cases 3: "The old NAG with lupus is resistant: Cycle her Az!"

Seven surgical options for Grave's orbitopathy

A: Naffziger (superior orbital plate decompression into anterior cranial fossa) - Through anterior craniotomy A: Kronlein (lateral orbital plate) - Through coronal direct rim incision or lateral canthotomy A: Hirsch (inferior orbital plate) - Through subciliary and/or Caldwell-Luc incision A: Sewall (medial orbital decompression) - Through coronal incision or external ethmoidectomy approach A: Walsh-Ogura - Transantral decompression of medial & inferior orbit, most widely used, as it is extracranial, decompresses two orbital walls into the largest empty space, allows gravity to aid in the expansion A: Orbital fat removal - Through upper lid crease and subciliary approaches; ~6 mm maximum reduction in proptosis possible; caution around inferior oblique A: Endoscopic orbital decompression 3: Remember superior, lateral, inferior, medial, inf+med in order = Naffziger, Kronlein, Hirsch, Sewall, Walsh-Ogura.

Describe the pathology for IAC abnormalities

A: Narrow IAC <3 mm, if facial function present then CN VIII likely Absent A: Widened IAC >10 mm, associated with stapes Gusher

Four parts to the anatomic deformity in Choanal Atresia

A: Narrow nasal cavity A: Lateral bony obstruction from Pterygoid plate A: Medial bony obstruction from Vomer A: Membraneous obstruction

Head & Neck symptoms of Wegener's granulomatosis

A: Nasal (90%) - Crusting, epistaxis, obstruction, septal perforation, saddle nose A: Otologic (25%) - CHL, suppurative OM, SNHL (can be bilateral & profound) A: Orolaryngeal - Gingival hyperplasia & gingivitis, laryngeal ulceration & edema (25%), subglottic stenosis (8.5%)

Name the surgical procedures done for OSA

A: Nasal - Septoplasty, nasal valve reconstruction, turbinectomy/reduction, RFA, adenoidectomy A: Palatal - Tonsillectomy, LAUP, UPPP, RAUP, lateral pharygoplasty, transpalatal advancement pharyngoplasty A: Tongue base - Genioglossal advancement, Hyoid myotomy and suspension, tongue base suspension, tongue base reduction/lingual tonsillectomy, laser midline glossectomy, thyrohyoidopexy A: Maxillofacial - Mandibular advancement, maxillary advancement, MMOA A: Tracheotomy 3: Anesthetic considerations - Absolutely no preoperative sedation, both anesthetist and surgeon must be present at induction, availability of equipment for jet ventilation, bronchoscopy, oral/nasal airways, tracheotomy

Nose analysis

A: Nasal Width from alar groove to alar groove = 70% the Length of the nose from nasion to tip defining point; nasal width equal to intercanthal distance A: Four profile measures Contour - should be relatively straight Projection - extent of tip protrusion from the anterior facial plane; tip defining point to base of the ala Rotation - angle of inclination of the nasolabial angle Length - dorsum measured from nasion to tip defining point

Six Major signs/symptoms of Rhinosinusitis

A: Nasal obstruction A: Nasal discharge/PND A: Facial pain/pressure A: Hyposmia/Anosmia A: Purulence on examination A: Fever (only in ARS)

Define Samter's triad

A: Nasal polyposis, asthma, and ASA sensitivity A: Thought to block oxidative phosphorylation; anti-inflammatory actions due to blockage of cyclooxygenase enzyme; polyp formation influenced by stimulation of 5-lipoxygenase, which causes leukotriene overproduction

Nasal optimization for OSA

A: Nasal topical corticosteroids - 6 month trial A: Nasal splints A: Turbinate reduction A: Septoplasty

Identify the 5 profile angles for facial analysis

A: Nasofrontal angle - Vertex at the nasion, formed by lines through the glabella and nasal dorsum; measures 115-135 degrees A: Nasofacial angle - Angle defined by intersection of a line drawn through the Glabella and Pogonion and 2nd along the nasal dorsum; measures 36 degrees ideally A: Nasolabial angle - Vertex at the subnasale, formed by lines through the columellar point and the upper vermilion border; measures 90-105 degrees in men, 100-120 degrees in women; this angle defines the amount of tip rotation A: Nasomental angle - Vertex at tip defining point, tangents drawn through nasion and pogonion; 120-132 degrees A: Mentocervical angle - Measures the angle between a line from the Glabella to the Pogonion and a line from the Menton to the Cervical Point; measures 80-95 degrees

What 4 angles make up the aesthetic triangles of Powell & Humphries?

A: Nasofrontal angle = 115-135o A: Nasolabial angle = 95-115o A: Nasofacial angle = 30-40o A: Nasomental angle = 120-132o

Four H&N manifestations of Rheumatoid Arthritis

A: Neck - Cervical pain, Decreased ROM A: TMJ dysfunction - Pain/tender joint or overlying muscles A: Ossicular - Tympanometric alterations A: Cricoarytenoid joint involvement (86%), findings include: Rheumatoid nodule deposition into cords Acute inflammation/edema over arytenoids Arytenoid process tenderness on palpation Decreased mobility & fixed fold in adducted position Subluxed CA joint on CT can be seen

What are the 6 top signs & symptoms of NPC?

A: Neck mass (60%) A: Ear plugging/fullness (41%) A: Conductive hearing loss (37%) A: Epistaxis (30%) A: Nasal obstruction (29%) A: Cranial nerve palsy (18%), usually VI, or V initially

Three Early findings Audio/Impedance in Otosclerosis

A: Negative On/Off effect - A diphasic Stapedial Reflex (ie Increase Compliance at the Onset and Cessation of the sound stimulus) A: Carhart's notch - BC (Sensorineural) dip seen at 2000 Hz which resolves after stapedectomy A: Low Frequency Conductive hearing loss - Stiffness effect

Five Acoustic Reflex findings seen in Otosclerosis as disease progresses

A: Negative On/Off effect - Diphasic Stapedial Reflex; seen early, even before any CHL A: Reduced Acoustic reflex Amplitudes A: Elevated Ipsilateral thresholds A: Then Elevated Contralateral thresholds A: Finally, Disappearance of the Reflexes

Four Agents used for Otitis Externa (NPPC)

A: Neomycin - Aminoglycoside, 6-8% hypersensitivity to topical application A: Polymyxin B A: Polymyxin E (Colistin) A: Cipro

Four clinical applications of OAEs

A: Neonatal hearing screening A: Patients that are Difficult to test/Pseudohypoacusis A: Auditory neuropathy A: Noise-induced hearing loss A: Ototoxicity monitoring A: Helpful in tinnitus patients when audio is normal

Danger of Primary hyperparathyroidism in pregnancy

A: Neonatal tetany 3: Ca crosses placenta, PTH doesn't; newborn's PTH is suppressed and serum Ca drops soon after delivery

Causes of Unilateral Tonsillar hyperplasia

A: Neoplastic - Lymphoma A: Infectious - Mycobacteria (tuberculosis, atypical), Actinomycosis, Fungal

What is the Babysitter procedure in Facial Paralysis

A: Nerve Crossover techniques that uses 40% of CN XII to maintain tone in the Facial musculature A: Requires a second stage crossover graft from the contralateral CN VII, as reinnervation is too weak

Four Electrophysiologic tests of Facial nerve function

A: Nerve excitability test (NET) - Compare thresholds to cause Minimal muscle contraction on each side of face; >3.5 mA difference = reliable sign of progressive Degeneration, indication for Decompression A: Maximal stimulation test (MST) - Uses the threshold to Maximal muscle stimulation without pain on normal side and Compares the difference with paretic side (Subjective test), Rank the difference as Minimal, Moderate, Severe or No response; If MST remains normal for 10 days, 88% of patients have complete return A: Electroneurography (ENoG) - Objective analysis of extent of muscle electrical response, records facial muscle Compound APs; >90% Degeneration within 14 days of Poor prognosis for immediate/complete restoration of function; useful in following progression in acute period q1-2d A: Electromyography (EMG) - Determines level of muscle activity, does not provide quantitiative estimate of the state of the nerve; only useful after 14 days, Polyphasic potentials (spontaneous or voluntary) suggests return of function 3: Indicated only if complete paralysis, not paresis because integrity not in doubt 3: MST, NET, ENoG not useful for prognosis until Wallerian degeneration has occurred (3 days)

Ddx of Poststyloid parapharyngeal space tumors (8)

A: Nerve sheath tumors A: Paraganlioma A: LN (met) A: Meningioma A: Hemangioma A: Chondrosarcoma A: Rhabdomyosarcoma A: Perineural mets 3: Prestyloid almost all salivary; look for fat plane

Histology of Medullary thyroid carcinoma

A: Nests of small round cells with abundant granular cytoplasm A: Presence of Amyloid A: Dense, irregular areas of calcification A: IHC - Calcitonin (most sensitive & specific), NSE, Chromogranin, Synaptophysin, CEA (sensitive, not specific)

Ddx of small blue cell malignancies of childhood (5)

A: Neuroblastoma A: Undifferentiated soft-tissue sarcomas A: Lymphoma A: Primitive NeuroEndocrine Tumor (PNET)/Ewing's sarcoma A: Rhabdomyosarcoma

Diagnosis of NF Type I by 2 of the following 7 criteria

A: Neurofibromas (2 or more cutaneous, or 1 plexiform) A: Café au lait spots (6 or more, >15 mm in adults, >5-10 mm in children) A: Axillary or inguinal freckling A: Lisch nodules (2 or more iris hamartomas) A: Optic glioma A: Bone - pseudoarthrosis, scoliosis, or thinning of the tibia; sphenoid wing dysplasia A: Family history or 1st degree relative with confirmed NF1

Ddx of pediatric vocal cord paralysis (NATIVE)

A: Neurologic (brainstem tumor, CP, hydrocephalus, meningomyelocele, hypoxic encephalopathy, hypotonia) A: Arnold-Chiari malformation (ALWAYS consider in neonate) A: Birth Trauma (C-section, nuchal cord, Recovery >9 mos) A: Iatrogenic injury (CVS, PDA, TEF) A: Idiopathic (47%, especially bilateral) A: Infectious - Syphilis A: Vascular anomalies A: Everything else - Mobius, Charcot-Marie Tooth 3: >50% Bilateral

Six IAC lesions that light up with Gadolinium

A: Neuromas - Acoustic or Facial nerve A: Meningiomas A: Inflammatory nerve lesions A: AVM's A: Vascular loops A: Petrous bone metastasis

What are the most common causes of Severe Aspiration?

A: Neuromuscular - MS, ALS, PD, CVA A: Head injury

Side effects of Taxanes

A: Neuropathy, Muscular pain, Alopecia, Allergic reactions 3: 30-40% Single agent Partial Response rate of Taxanes

Good prognostic EMG findings

A: No Spontaneous activity A: Some Recruitment A: Normal/bi/triphasic, or low amplitude polyphasic motor unit potentials

Define Ottitic Hydrocephalus

A: No clear definition; Increased ICP secondary to Acute or Chronic Middle Ear disease, with or without Lateral/Sigmoid Sinus Thrombosis, WITHOUT evidence of Meningitis or Abscess

Definition of Congenital SGS (vs Aquired SGS)

A: No history of ETT or laryngeal trauma

Rye classification of Hodgkin's lymphoma

A: Nodular sclerosis (60%) - nodules of lymphoid infiltrates, lacunar variants of RS cells A: Mixed cellularity (30%) - pleomorphic lymphocytes, more numerous RS cells A: Lymphocyte depleted (6%) - paucity of lymphocytes, diffuse fibrosis and bizarre RS cells; worse prognosis A: Lymphocyte predominant (3%) - rare Reed-Sternberg cells, favorable prognosis

Five clinical types of Basal Cell Carcinoma (No More ****ing Sun Please)

A: Nodular/noduloulcerative (most common) and Micronodular A: Morphealike/sclerosing (worst prognosis) A: Pigmented - similar appearance & behaviour as noduloulcerative except for brown pigment A: Fibroepithelioma - often seen on back; R/O Cowden's syndrome A: Superficial multicentric 3: "No More Pigs For Supper"

Six Pathologic conditions that Lower fundamental frequency

A: Nodules A: Polyps A: Tumors (load-producing) A: Renke's edema A: Hypothyroidism A: Aging in women

Five Factors influencing success of OAE's for hearing screens

A: Noise level in test environment A: Vernix/debris in the EAC A: EAC collapse A: Middle ear fluid/mesenchyme/dysfunction A: Decreased responses in low birth weight infants & preemies

Definition of Mikulicz Syndrome

A: Non autoimmune recurrent parotid swelling with non-specific lymphocytic infiltration

Management of Epistaxis

A: Non surgical (5) Topical decongestion Cauterization - silver nitrate, electrocautery Packing - Absorbable vs. nonabsorbable, anterior, posterior, combined, balloon packs Greater palatine canal injection Radiologic Embolization of internal maxillary & facial arteries A: Surgical (4) Sphenopalatine artery ligation - Transantral vs. Endoscopic Ethmoid artery ligation - Lynch frontoethmoidectomy approach vs. Endoscopic Maxillary artery ligation - Transantral External carotid artery ligation

Management of velopharyngeal insufficiency (VPI)

A: Non-surgical - Speech therapy, Prosthetics (palatal lift or obturator), Biofeedback with nasometry A: Surgical - Pharyngoplasty, Pharyngeal flaps, Posterior pharyngeal wall augmentation

Five Characteristics of the Decibel scale

A: Nonlinear, Logarithmic scale A: Relative measure, Incorporates a ratio A: Expressed with different Reference levels

Classification of Benign Thyroid Disease

A: Nontoxic - Diffuse, Nodular Goiter A: Toxic - Toxic multinodular goiter, Toxic adenoma, Graves Disease A: Inflammatory

Three findings (normal, fibrillation, and polyphasic potentials) on laryngeal EMG and significance with clinical setting of vocal cord immobility

A: Normal - Joint fixation or just paresis A: Fibrillation - Denervation A: Polyphasic potentials - Reinnervation and Synkinesis

Discuss the Xenon Cerebral Blood Flow study

A: Normal CBF = 50 ml/100g/min; <20 ml/100g/min produces failure of brain function A: Indications - En bloc resection requiring ICA resection; Tumor encases ICA on imaging; Contour irregularities of ICA on angio A: Results of Xenon-CBF study - 15 minute balloon occlusion Group I - No CBF side difference Group II - Mild symmetric CBF decrease Group III (10-15%) - Marked decrease in CBF (<30 ml/100g/min) Group IV (5%) - Neurologic deficit, CBF <20ml/100g/min A: Preop permanent Balloon Occlusion for groups I & II A: Prophylactic or Intraoperative Revascularization for group III, or for groups I & II if patient Young or if Contralateral disease present A: No surgery or prophylactic revascularization for group IV patients

What 4 types of Potentials are seen on EMG?

A: Normal Voluntary Action Potentials - Functioning axons are connected & stimulating motor units A: Polyphasic Potentials - Seen with Reinnervation of muscle ~4-6 weeks post-injury A: Denervation/Fibrillation Potentials - Seen in Denervated muscle ~10-14 days post-injury A: Electrical Silence - Denotes Atrophy or congenital Absence of muscle

Define Ectopic

A: Normal appearing tissue in an abnormal location

Discuss Absent Spontaneous OAE

A: Normal variant (~50%) A: Mild hearing loss (>30 dB) A: If present can be used to follow for ototoxicity as would be first to go

ABR findings in Retrocochlear pathology

A: Normal wave I but all else delayed (classic)

Describe Stapedial Reflex testing

A: Normally occurs at 70-100 dB SL for pure tones A: Usually measured at 500, 1000, and 2000 Hz A: Absent Ipsilaterally with minimal CHL, or CN VII pathology proximal to stapedial branch A: Absent Contralaterally with a brainstem lesion (would have to be in the center) A: Absent Bilaterally with a CHL or CN VIII loss >40 dB, or a SNHL >65 dB HL A: Latency of 10 ms; will not protect cochlea from unanticipated sounds A: Incidence of absent stapedius tendon, muscle & pyramidal eminence - 1% A: Incidence of absent stapedius reflex = 10?% 3: Stapedial reflex = 10 ms; Acoustic reflex decay = 10 sec

When and for which levels to do an Elective Neck Dissection (or Irradiate) according to Primary Tumors

A: Nose and Paranasal Sinuses - Never A: Glottis - >T3, Unilateral levels II-IV A: Oral cavity - ≥T2, Unilateral levels I-III, unless approaching Midline, Anterior tongue, or FOM A: Supraglottis - ≥T1, Bilateral levels II-IV A: Subglottis - ≥T1, Bilateral levels II-VI +/- V, & Total Thyroidectomy A: Tonsil - ≥T1, Unilateral levels I-III, & Retropharyngeal nodes A: Base of Tongue - ≥T1, Bilateral levels I-V A: Hypopharynx - ≥T1, Bilateral levels II-V, & Retropharyngeal nodes A: Nasopharynx - ≥T1, Bilateral levels II-V, & Retropharyngeal nodes

RRP prophylactic Cesarean section recommedations

A: Not routinely recommended (1 in 400 risk of getting it when mother has active genital condylomata) A: Should be strongly considered in Young, Primiparous mothers with recent HPV infection and genital warts

Describe Hitselberger's sign

A: Numbness of Posterior EAC from Compression of Sensory branches of Facial Nerve seen with acoustic neuroma's

Twelve possible causes of Burning Mouth syndrome (patients may have multiple causes)

A: Nutritional deficiencies - Being deficient in nutrients, such as iron, zinc, folate (vitamin B-9), thiamine (vitamin B-1), riboflavin (vitamin B-2), pyridoxine (vitamin B-6) and cobalamin (vitamin B-12) A: Allergies - Reactions to foods, flavorings, additives, fragrances, dyes or other substances A: Gastroesophageal reflux disease A: ACE inhibitors A: Neuropathy A: Endocrine disorders - Hyperglycemia in diabetes, and hypothyroidism A: Hormonal imbalances - Occurs most commonly among postmenopausal women, although it affects many other people as well; Changes in hormone levels may affect the composition of saliva A: Xerostomia - Can be related to certain medications, including tricyclic antidepressants, central nervous system depressants, lithium, diuretics and antihypertensives; It can also occur with aging or Sjogren's syndrome A: Other oral conditions - Thrush, Diabetes, Denture use and Geographic tongue A: Irritating dentures - Muscle stress and mucosal irritation A: Excessive irritation - Excessive brushing of tongue, overuse of mouthwashes, or consuming too many acidic drinks A: Oral habits - Often-unconscious activities, such as tongue thrusting and bruxism A: Psychological factors - Anxiety, Depression, Extreme fear of cancer

Define Charcot's triad

A: Nystagmus A: Scanning speech A: Intention tremor 3: Seen in multiple sclerosis

Define Nystagmus, and Vestibular nystagmus

A: Nystagmus - The to and fro motion of the eyes A: Vestibular nystagmus - Bilateral, conjugate, comprised of a slow phase (image on fovea) and a fast phase (saccade, correction of image) - Saw toothed pattern

Person with otomycosis, 6 classes of topical agents used (first 6 from KJ Lee)

A: Nystatin A: Clotrimazole cream or drops A: Chloromycetin-sulfanilamide- Ampho B or Tonalfate A: Locacortin vioform A: 1.5% Acetic acid A: 95% Isopropyl alcohol A: Merthiolate A: Metacresyl-acetate (Cresylate) A: Gentian violet A: Aluminum Sulfate-Calcium Acetate (Domboro) A: Drying agents - Boric acid, Hydrogen peroxide

List the sites of Ototoxic activity for Anti-Inflammatories

A: OHC main site, Salicylates can also affect Cochlear Blood Flow A: All cause Reversible High Frequency Tinnitus, and Mild-moderate Flat/ High Frequency Hearing Loss A: Aspirin, NSAIDs, Quinine

Pediatric norms

A: OSAS = Apnea index >1 A: CO2 50 mmHg > 10% of sleep time (obstructive hypoventilation) A: CO2 45 mmHg > 60% of sleep time A: Minimum O2 sat <92% A: Primary snoring - no symptoms A: BMI - weight in kg/height in meters squared A: Multiple sleep latency test - naps during day, time to fall asleep measure A: AHI >5, with 20 being extremely abnormal

Severity of OSAS

A: OSAS = RDI > 5 A: Mild OSAS = RDI 5-14, sats >85% A: Mod OSAS = RDI 15-29, sats 65-84% A: Severe OSAS = RDI ≥30, sats <64% A: Obstructive sleep hypopnea syndrome = Hypopnea index >15 A: Upper airway resistance syndrome - Daytime sleepiness with RDI <5, but with ≥15 RERAs per hour

Outline the steps in the surgical resection of Carotid Body tumors

A: Oblique neck incision made anterior to SCM, subplatysmal flap elevation to provide wide exposure of the carotid system A Limited selective neck dissection is performed; the IJV is retracted sampling of regional nodes for metastasis, CNXI, CNX, and CNXII identified and preserved; division of ansa cervicalis, CN XII can be retracted superiorly A: Common, external and internal carotid arteries proximal and distal to the tumour are controlled; if the external carotid is involved within the tumour, it can be sacrificed A: Subadventitial dissection in the plane of Gordon-Taylor; tumour is dissected away from the surface of the internal carotid, excessive bleeding may be avoided by ligation of the small tumour-feeding vessels on the posterior aspect of the carotid bifurcation; the tumor freed from the bifurcation is retracted superiorly to allow dissection of the superior laryngeal nerve away from the deep surface of the tumour A: If more exposure is needed superiorly, the digastrics is taken down along with the stylohyoid muscle A: If the carotid is involved with the tumour, it is sacrificed and replaced with either a venous or synthetic graft

Five causes for false positive retropharyngeal abscesses on LNXR

A: Oblique view A: Neck flexion A: Crying A: Swallowing A: Expiration

Outline the management options for Cutaneous malignancies

A: Observation A: Medical - Topical 5-FU, Retinoic acid, Interferon-α A: Surgical - Electrodessication & curettage, Excision, Cryosurgery, Photodynamic therapy, Mohs surgery A: Radiotherapy - Generally not indicated due to duration, complications, carcinogenesis, lack of margin control, increased aggressiveness of recurrences

What is the management of an uncomplicated TM perforation after barotrauma (3)?

A: Observation A: Minor patch (paper patch) A: Myringoplasty/tympanoplasty

Four treatment options for macroglossia

A: Observation A: Orofacial therapy (for hypotonia, or lingual mal-position) A: Surgery A: Submucosal minimally invasive lingual excision (SMILE)

Treatment options for Hemangioma

A: Observation A: Standard of care = Tracheostomy and wait for involution A: Propanolol (start 0.5-1 mg/kg/day then increase to 2 mg/kg/day) A: Oral steroids (2-3 mg/Kg/d PO x 7 days then R/A, if responsive, taper over 4-6 weeks up to 10 months) A: Intralesional steroids (Triamcinolone 40 mg or Betamethasone 6 mg q4-6 weeks x 1-5, avoid periorbital) A: IFN-Alpha 2a or IFN-Alpha 2b (unresponsive to steroids, can be daily S/Q x 6+ months) A: Surgery (cold knife, CO2/KTP laser) A: Photocoagulation (q4-6weeks, early proliferative phase, superficial lesions) A: Cryotherapy A: Tracheostomy or Laryngotracheoplasty

Management of Otosclerosis

A: Observation A: Trial of Amplification A: Medical - Sodium Fluoride 20-120 mg/d, creates Fluoroapatite bone (replaces Hydroxyl group) which is Resistant to Absorption; taken with Vitamin D and Calcium Gluconate Indications - Presence of SNHL or Vertigo; patients who are Unfit or Refuse Surgery; Schwartze's sign or Otospongiosis seen intraop Contraindications - Pregnancy Fosamax has also been tried with variable results (small risk of Osteonecrosis of the Mandible) A: Surgery - An elective surgery, should have Negative Rinne with 512 Hz fork; Procedures include Stapes Mobilization Stapedotomy (better High Frequency) Stapedectomy (better Low Frequency)

Therapeutic options for unilateral vocal cord paralysis

A: Observation A: Voice therapy A: Injection techniques (percutaneous, transoral, laryngoscopically) A: Type I Isshiki thyroplasty (medialization thyroplasty) A: Type I Isshiki + Arytenoid adduction A: Laryngeal reinnervation (ansa cervicalis transfer)

Non-surgical management of laryngomalacia

A: Observation (90%) A: Anti-reflux A: Position A: Thickened feeds A: Frequent smaller meals A: CPAP

Management strategy for Acoustic Neuromas

A: Observation - 3 categories of growth Slow/no growth (40%) = <0.2 cm per year Limited growth = ~0.2 cm per year More Rapid growth = ~1 cm per year A: Surgery - Criteria for hearing preservation = 30 dB PTA, 70% SDS, <2 cm extension into CPA, Fundus of IAC free A: Complications - SNHL, CN VII palsy, Hemorrhage, Meningitis, Air Embolism, CSF leak, Cerebellar Ataxia, Headache A: Stereotactic Radiosurgery - Gamma-knife or LINAC (best if less than 2 cm and should be less than 3 cm? according to ben's notes) - if bigger than 3 cm then can get significant problems from immediate post-radation tumour edema

Management of unexpected facial weakness Post-Tympanoplasty

A: Observation - Allow local anesthetic to wear off A: Decompression - Remove packing A: Consultation with 2nd surgeon present A: Reexplore with 2nd surgeon present

Management of Zenker's

A: Observation - Alternate enteral feeding via PEG, NG, kaofeed, etc.; monitoring for aspiration A: Surgery Open - CP Myotomy and either Diverticulectomy, Diverticulopexy, or Diverticular Inversion Endoscopic - CP Myotomy via Cautery (Dohlman), Laser, or Stapling

Management options for SGS

A: Observation - Grade I-II with minimal symptoms & reliable follow up, especially congenital, repeat bronch q3-6 months A: Medical - Anti-reflux A: Tracheotomy A: Endoscopic procedures - Balloon dilatation, Laser, mitomycin C A: Open reconstructive procedures - Expansion procedure (LTP/LTR, single stage or with trach and Stent: Anterior w/wo Posterior w/wo Lateral cricoid split, anterior and/or posterior cartilage Graft) Segmental resection (Cricotracheal resection & anastomosis, primary, salvage, extended with expansion, arytenoid lateralization or arytenoidectomy, stents)

Three management options for Paragangliomas

A: Observation - Small, stable, asymptomatic, elderly A: XRT/Gamma-knife A: Surgery - Resection in subadventitial plane +/- Vascular replacement (Saphenous, Synthetic, or STA to MCA bypass) +/- Embolization +/- Regional neck dissection if malignant

Treatment of Glottic Webs by type

A: Observation, if possible delay surgery to >3 yo A: Endoscopic repair - Lysis via laser/cold knife, can be staged (one side then other), serial Dilations, +/- Stent, Suturing of free edges, Local flaps, Mitomycin C A: Type I - Divide with laser/cold steel at age 3-4 A: Type II - Incise along one cord then serial dilations or incise along other cord 2 weeks later; If keel required, trach needed A: Type III/IV - Trach, corrective procedure at age 3-4, Laryngofissure with Stent/Keel or LTR

Pathogenesis of Cholesterol Granulomas

A: Obstruction of air cell drainage pathways A: Inflammation and Hemorrhage A: Subsequent Breakdown of RBC's & Foreign Body Reaction to Cholesterol Crystals A: Incites more Inflammation and a Vicious Cycle

Adult PSG results (3 categories)

A: Obstructive Sleep Apnea is diagnosed as an RDI >5, SpO2 <90%. A: Upper airway resistance syndrome is diagnosed as a RDI <5, and SpO2 >90%, A: Primary snoring when RDI <1 and SpO2 >90%.

Classification of congenital Encephaloceles

A: Occipital - Most common, ~75% of cases A: Sincipital/frontoethmoidal, ~15% - nasoFrontal (most common subtype), nasoEthmoidal, nasoOrbital A: Basal, ~10% - Transethmoidal (most common subtype), Sphenoethmoidal, Transsphenoidal, Sphenoorbital

Three general classes of papillary thyroid ca

A: Occult (less than 1.5 cm) - associated with good prognosis A: Intrathyroidal A: Extrathyroidal

Describe the Occlusion Effect

A: Occurs with larger hearing aids that block the EAC A: Shifts the Resonance Frequency lower, which increases low frequency Gain A: Troublesome in people with normal low frequency hearing; Improved with Smaller hearing aids

Diagnostic criteria of Sjogrens

A: Ocular symptoms - Dry eyes, or sensation of gravel in eyes more than 3 months, or use of tear substitutes >3x/day A: Oral symptoms - Dry mouth more than 3 months, or recurrent or persistent swelling of salivary glands A: Ocular signs - Schirmer test < 5 mm in 5 min, or Rose-bengal score >4 A: Histopathology - Focus score ≥1 = ≥50 lymphocytes in 4 mm2 of glandular tissue A: Salivary gland involvement - Unstimulated flow of <1.5 mL in 15min, Abnormal parotid sialography (diffuse sialectasia), or Abnormal salivary scintigraphy (reduced uptake, concentration, or excretion of tracer) A: Serology - Presence of Anti-Ro/SS-A or Anti-La/SS-B antibodies

Define Phatosmia

A: Odor sensation in the absence of an olfactory stimulus (olfactory hallucination)

Describe the olfactory transduction mechanism

A: Odorants are solubilized in mucus, or hydrophobic odorants by Odorant Binding Protein (OBP) A: G-Protein coupled receptors (~1000, 1% of expressed genes) A: Second messenger = cAMP

Risk factors for hypothyroidism

A: Old age A: Female sex A: Grave's disease A: Drugs - lithium iodide, amiodarone, iodide containing drugs A: Surgery - postthyroidectomy A: Hashimoto's disease A: Autoimmune disease A: Euthyroid goiter A: Laryngectomy with/without irradiation A: Prior head & neck radiotherapy management 3: "Old Female Grave Diggers Smoke Hash And Eat Lotsa Peanuts!"

Risk factors for worse prognosis for well differentiated thyroid cancer

A: Older age (men >40, women >50) has worse prognosis A: Size of primary - >5cm worse, <1.5cm improved prognosis; thus size >1.5cm consider worse A: Extrathyroidal extension A: Distant metastases adversely affects prognosis A: Unifocal more likely to be malignant versus multifocal disease; mutifocality increases with age A: Cervical metastases - does not impact overall survival but does increase chances of local recurrence

Two surgical treatment options for Parosmia/Phantosmia

A: Olfactory bulbectomy A: Endoscopic removal of olfactory neuroepithelium

Three differences between Olfactory and Respiratory epithelium

A: Olfactory epithelium is thicker (60-70 nm) than respiratory epithelium (20-30 nm) A: Olfactory epithelium has cilia that lack dynein arms A: At physiologic flow rates olfactory epithelium gets ~15% of nasal airflow, 50% flows through middle & inferior meati

List the central olfactory connections

A: Olfactory tubercle A: Lateral Entorhinal cortex A: Prepyriform cortex A: Periamygdaloid cortex A: Amygdaloid nucleus A: Nucleus of the Terminal Stria A: Dentate and Semilunate gyri

Three Advantages of local flaps

A: One stage in most cases A: Better donor site match A: Low donor site morbidity

Six Disadvantages of Stereotactic Radiosurgery for Acoustic Neuromas

A: Only Small Tumors can be radiated A: Tumor is not excised, Stops tumor Growth, doesn't shrink it A: Delayed Facial Weakness A: Not Hearing Preserving (Delayed hearing loss = Rate for surgery) A: Trigeminal Neuropathy A: Postradiation Scarring makes Salvage Surgery more difficult A: Radiation-induced Malignancy of concern in younger patients

Contraindications for surgical intervention for Peripheral Vestibular disease

A: Only hearing ear A: Bilateral disease A: Central pathology A: Suspicion of Impaired Central Compensation following surgery

AAO-HNS indications for coagulation studies

A: Only if indicated by history or genetic information is unavailable

Three things to do to a hearing aid mold to decrease low frequency gain in patients with High Frequency SNHL

A: Open Venting - Allows low frequency sounds to escape A: Shorten the earmold canal A: Enlarge the Sound Bore

Macroglossia triad

A: Open bite deformity A: Mandibular prognathism A: Malalignment/malocclusion

Incidental findings of sphenoid sinusitis on imaging - what should you consider?

A: Optic complications - 25% of patients with sphenoid sinusitis have neurophthalmic dysfunction

Name the 4 structures visible on the walls of the sphenoid sinus

A: Optic nerve - Projects into sinus less commonly, ~50%; bone dehiscent in ~4% A: Internal carotid artery - Medial deflection into lateral wall called the carotid sulcus, present 65-98% of time; bone dehiscent in ~8-25% A: Optico-carotid recess A: Cavernous sinus

Eight functions of the Oral cavity

A: Oral competence A: Mastication A: Taste A: Deglutination A: Airway protection A: Respiration A: Oral hygiene A: Articulation

Name the four stages of swallowing

A: Oral preparatory A: Oral phase A: Pharyngeal phase A: Esophageal phase

Most common primary sites in the head and neck for Rhabdomyosarcoma in descending order (ONES)

A: Orbit A: Nasopharynx A: middle Ear/mastoid A: Sinonasal cavity 3: 35-40% of all cases occur in the head and neck

Describe the 7 features of superior orbital fissure syndrome

A: Orbital pain A: Photophobia A: Proptosis A: Ophthalmoplegia A: Failure of accommodation A: Upper eyelid paralysis A: Forehead paresthesia/hypoesthesia 3: Caused by sphenoid sinusitis, neoplasm, trauma; Involves CN III, IV, V1, VI; Differs from Orbital apex syndrome in that CN II usually not involved as it is in its own canal

What is Tonotopic organization?

A: Organ of Corti ~35 mm long from base to apex A: Travelling wave from middle ear causes simultaneous fluid wave in the Perilymph A: Each place on the Basilar membrane responds best to a specific frequency A: High frequency sounds cause maximal displacement of the basilar membrane in the Basal region A: Low frequency sounds cause maximal displacement of the basilar membrane in the Apex A: Tonotopic organization preserved in higher brainstem nuclei

Discuss Hemangiopericytoma

A: Originates from the capillary Pericyte of Zimmermann A: Occurs wherever there are capillaries; MSK and Skin predilection A: Histologically consist of packed ovoid/spindle cells, and staghorn vessels displaying perivascular hyalinization A: Unpredictable, considered Malignant & infiltrative A: Treatment - Wide surgical excision, may require preoperative embolization; Radiotherapy generally for palliative cases, although adjuvant XRT has been recommended for high-grade features or positive margins; Neck dissection not necessary as lymphatic spread is rare; 5-year survival rate near 70%, and distant metastases usually portend recurrence at the primary site

Two Symptoms of Bilateral Vestibular loss

A: Oscillopsia A: Difficulty with balance in the dark (with eye closure)

Ddx of Otosclerosis (Fixed CHL) (9)

A: Ossicular Discontinuity - Incus necrosis due to Recurrent OM; 60 dB HL and type Ad tympanogram A: Fibrous Union of the ISJ - Greater CHL in the Higher frequencies A: Tympanosclerosis A: Malleus Head Fixation - If congenital, high incidence of Aural Atresia; Type As tympanogram (only seen in the most advanced otosclerosis); Presense of Tympanosclerosis A: Congenital Stapes Fixation - Usually diagnosed in first Decade of life; Non-progressive HL; Only 10% Family Hx (vs 66% with Otosclerosis) A: Otitis Media - Can be Recurrent, Chronic +/- Cholesteatoma, or with Effusion A: Middle Ear Neoplasm - Glomus or Facial nerve A: Pagets disease - Increased Alk Phos; CT findings seen on Bilateral Petrous bones (demineralization); Typically causes Crowding of the Ossicles in the Epitympanium A: Osteogenesis Imperfecta - Blue sclera, History of multiple bone fractures

Three cell types and their Chemical mediators in Cholesteatoma

A: Osteocytes - BMP-2, TGF beta A: Osteoclasts - Acid Phosphatase, Acid Proteases, Collagenase A: Macrophages - ILs -1, -6, -11; TNF-α, TGF-α, PGs, LTs, PTH-rP, CSF-1, OPF

Predisposing factors for osteomyelitis of the maxilla & mandible (OSTEITIS)

A: Osteodystrophies (Paget's, fibrous dysplasia) A: Systemic (Steroids, Bisphosphonates) A: Trauma A: Endosteal lesions (eg. Cementoma) A: Immune compromise (HIV, DM, leukemia, chemo) A: radioTherapy A: dental Implants (plates, and other FBs) A: Surgery

Ddx of Fibroosseous nasal lesions (4)

A: Osteoma A: Osteochondroma A: Ossifying fibroma A: Fibrous dysplasia

Not including polyps, what are the 4 most common benign nasal cavity lesions in order of frequency? (OHPA!)

A: Osteoma (most commonly in the frontal sinus) A: Hemangioma A: Papilloma A: Angiofibroma

What is the difference between Osteomas and Exostoses?

A: Osteoma - Solitary, Pedunculated, related to Suture lines, More often lateral in bony EAC; Histo = Lamellar bone around trabecular cancellous bone with marrow and fibrovascular tissue A: Exostosis - Multiple/Bilateral, Sessile/Broad based, Medial EAC, Overall more common, Cold water exposure correlation, Found along tympanomastoid and tympanosquamous suture lines; Histo = Parallel layers of subperiosteal bone, no or poorly developed trabeculated fibrovascular channels 3: Radiographically, both show an intact cortex

Seven Indications for HBO

A: Osteoradionecrosis A: Osteomyelitis A: Necrotizing Fasciitis A: Necrotizing Otitis Externa A: SSNHL A: Preop dental extraction/implant prohylaxis in irradiated patient (20 preop and 10 postop dives) A: Flap failure

Three important factors for normal nasal function

A: Ostial patency A: Mucociliary function A: Quantity/quality of nasal secretions

Four cardinal features of Necrotizing OE

A: Otalgia >1 month A: Otorrhea/Granulation tissue A: Advanced age, Diabetes or Immune suppression A: Cranial neuropathy

Ten Contraindications for Stapedectomy/Stapedotomy

A: Otitis Externa/Media of Chronic nature (Absolute) A: Middle Ear Effusion (Absolute) A: Unrepaired Tympanic Membrane Perforation (Absolute) A: Only Hearing ear A: Family history of Stapes Footplate Fixation or Stapes Gusher syndromes A: History of Cholesteatoma A: Repetitive Barotraumas A: Severe ET Dysfuncion A: Employment requiring excellent balance A: Downsloping SNHL loss Preop or Postop in first ear - Often hearing will deteriorate postop A: Active Meniere's disease or other uncompensated Vestibular disorder A: Extremes of Age 3: Avoid Otospongiosis Phase of otosclerosis

Ddx of Subjective Tinnitus

A: Otologic - Presbycusis, NIHL, Meniere's; 75% have >30 dB HL >3 kHz A: Neurologic - Skull trauma, Whiplash, MS, Meningitis A: Metabolic - Hyperthyroidism, Hypothyroidism, Hyperlipidemia, Decreased zinc, Hypovitaminosis A: Psychiatric - Depression, anxiety A: Dental - TMJ disorders A: Drugs - ASA, NSAIDs, Aminoglycosides, Heterocyclic antidepressants, Caffeine, Heavy metals 3: "ON MetaPsychic Dental, Drugs"

Nine complications of Tympanostomy tubes

A: Otorrhea: ~3.5% rate of persistent drainage A: Granulation tissue formation A: Myringosclerosis A: TM perforation A: TM atrophy, retraction, atelectasis A: Cholesteatoma A: Loss of tube in the middle ear A: Early extrusion A: Plugged tube

Four Most common causes of Malleus Head Fixation

A: Otosclerosis A: Tympanosclerosis A: Postinfectious A: Congenital fixation A: Trauma?

Histopathologic Stages of Otosclerosis

A: Otospongiosis - Histiocytes, Osteoclasts, Osteoblasts involved; Bone around Blood Vessels resorbed, vessels dilate, Osteocytes lay down rich amourphous Ground Substance that lacks collagen, rich Blue color under H&E (Blue Mantles of Manasse) A: Otosclerosis - Formation of Dense Bone in areas of previous bony resorption, Vascular channels narrowed due to bone deposition

List the sites of Ototoxic activity for Vancomycin

A: Ototoxicity disputed, No firm mechanism

Where do OAEs come from?

A: Outer Hair cells

What is the anatomic pathway of an Otoacoustic emission?

A: Outer hair cell A: Basilar membrane A: Cochlear fluids (ST) A: Oval window A: Ossicles A: Tympanic membrane

Define Peak Clipping for Hearing Aids

A: Output was limited at a predetermined level, in a linear amplification system

Most common Locations of a Spontaneous Perilymph Fistula

A: Oval window A: Round window A: Fissula Ante Fenestram A: Fissure of Round Window niche connected to PSCC ampulla A: Subluxed Stapes

Definition of and 6 treatment options for keloids (SCLERI)

A: Overgrowth of dense fibrous scar that extends beyond the borders of the original skin injury A: Silicone gel sheeting A: Cryotherapy A: Laser therapy A: Elastic compression A: Radiotherapy A: Intralesional steroids

Define Keystone area

A: Overlap of the Upper lateral cartilage by Nasal bone

Discuss Audiometric findings (PTA, SDS, Reflexes) in Acoustic Neuromas

A: PTA - 65% will have HF SNHL; 10% present with SSNHL (but only 5% of SSNHL is due to AN); 5% have Normal hearing A: SDS - Poor scores out of proportion to PTA suspicious; Rollover present A: Stapedial Reflex - 88% will have Absent Reflexes (afferent pattern) or Positive Reflex Decay

Management of Thyroid Storm (PPID)

A: PTU 800-1000mg NG qd A: Propranolol IV A: sodium Iodide 1g IV (inhibits hormone release) A: Dexamethasone 2mg IV q6h (suppresses the H-P-T axis & peripheral hormone conversion) 3: Admit to ICU, IVF, cooling,

List 5 absolute and 4 relative Contraindications to MRI

A: Pacemakers (most common) and Pacer-wires A: Swan-Gantz catheter A: Cochlear or Brainstem Implants A: Ocular (do xray) or Metallic foreign bodies A: Certain aneurysm clips A: Cardiac valve (relative, usually safe) A: Vascular clips (relative, usually safe) A: Orthopedic prosthesis (relative, usually safe) A: Claustrophobia (relative, premedicate)

Nine granulomatous diseases of the larynx that can simulate cancer

A: Pachyderma laryngis A: Coccidiomycosis A: Histoplasmosis A: Blastomycosis A: Rhinoscleroma A: Tuberculosis (most common) A: Leprosy A: Syphilis A: Sarcoidosis A: Wegener's granulomatosis 3: Remember: "Pachy Cocci Histo Blasto Rhino, TB Lepy, Syphi, Sarcoid, Wegener's"

Five Measures in the Management of Air Embolism during surgery

A: Pack surgical wound with wet sponges A: Trendelenburg prevents further leaks A: Left Lateral Decubitus position traps air in Right Heart and prevents lung embolism A: Oxygenate with 100% O2 until patient Stabilizes (air absorbed) or Aspirate air via venous catheter A: Fix source of air leak

Five Differences between Paget's Disease vs. Otosclerosis

A: Paget's involves all bones of the skull & often see enlarged Calvaria, vs. Temporal bone only involvement in Otosclerosis A: Paget's onset is 6th decade, Otosclerosis onset is usually 3rd A: Pagets initially Periosteal but eventually involves Endosteal and Endochondral layers, vs. only Endochondral involvement in Otosclerosis A: Paget's rarely involves Stapes Footplate or Ossicles A: Greater degree of SNHL seen in Pagets A: Elevated Alk Phos typically seen in Pagets

Ddx of Objective Non-pulsatile Tinnitus (PIP)

A: Palatal myoclonus A: Idiopathic stapedius muscle contraction A: Patulous eustachian tube 3: Management = Reassurance, Sedatives, Muscle relaxants, Surgery when indicated

Five muscles of soft palate (all innervated by pharyngeal branch of CN 10 except TVP is TV3)

A: Palatoglossus A: Palatopharyngeus A: Musculus uvulae A: Levator veli palatini A: Tensor veli palatini (innervated by V)

Indications for hemithyroidectomy or subtotal thyroidectomy in carcinoma of the larynx/hypopharynx

A: Palpable disease present A: Glottic or subglottic carcinoma with greater than 1cm subglottic extension A: T4 glottic carcinoma A: T4 pyriform sinus carcinoma

Eye analysis

A: Palpebral fissure ~10 mm A: Horizontal width ~30 mm A: Intercanthal distance (30-35 mm) = ½ interpupillary distance (60-70mm)

Six most common thyroid malignancies

A: Papillary 80% A: Follicular 10-15% A: Medullary 5% A: Anaplastic <5% A: Lymphoma A: Metastasis

Five indications for Type I Isshiki medialization thyroplasty

A: Paralysis - Glottic incompetence with dysphonia, poor cough, and aspiration A: Skull base surgery - High vagal paralysis A: Atrophy - Presyphonia A: Sulcus (difficult to treat) A: Scarring

Blood supply of Regional Skin Flaps

A: Paramedian forehead - Supratrochlear & Supraorbital arteries A: Forehead - Superficial Temporal artery (used for oropharynx recon) A: Temporoparietal - Superficial Temporal artery & vein A: Postauricular - Posterior Auricular artery A: Nape of neck - Occipital artery A: Nasolabial - Angular artery off the Facial A: Platysma - Submental branch of Facial (dominant), Transverse Cervical (secondary) A: SCM - Occipital (dominant); secondary branches from Posterior Auricular, Superior Thyroid, and Thyrocervical trunk; "SPOT" A: Trapezius - Transverse Cervical (dominant, Lateral island and Extended island); Occipital and Paraspinous perforators (secondary, Superior flap), Descending branch of Dorsal Scapular (secondary, Extended island); also "SPOT" A: Deltopectoral - 1st-4th perforators from Internal Mammary artery A: Pectoralis major - Thoracoacromial artery, Lateral Thoracic (supplies inferior 1/5th), and Internal Mammary artery perforators

Name 8 diseases or syndromes which can cause Cerebellar Ataxia (PET SCARF)

A: Paraneoplastic cerebellar degeneration - Most commonly associated with Small Cell Carcinoma of the lung, breast, ovary & female genital tract; Autoimmune mechanism A: Familial Episodic Ataxia - Recurrent vertigo and ataxia, triggered by Stress or Exercise; treatable with Acetazolamide A: Ataxia Telangiectasia - Autosomal recessive; slowly progressive cerebellar ataxia with telangiectasias of skin and conjunctiva, recurrent URTI A: Spinocerebellar atrophy - Cerebellar ataxia with extrapyramidal signs (bradykinesia, tremor, slow hesitant gait) A: Cerebellar atrophy (Dandy-Walker) - Highly localized atropy of the cerebellum A: Alcoholic cerebellar degeneration - Selective degeneration of the Anterior Vermis A: Refsum's disease - Retinitis pigmentosa, polyneuropathy, cerebellar ataxia, SNHL; due to Phytanic Acid storage disease, manage with diet restrictions A: Friedreich's ataxia - Instability of gait, progressive hearing loss (ANSD), cardiomyopathy, scoliosis; slowly progressive tri-nucleotide reapeat disorder

In order, what 10 sites do Meningiomas Commonly occur?

A: Parasagittal A: Falx A: Convexity A: Olfactory groove A: Tuberculum sellae A: Sphenoid ridge A: CPA (petrous face) A: Tentorium A: Lateral ventricle A: Clivus

Administrastion & Mechanism of Action of Immunotherapy

A: Parenteral administration of antigens identified on appropriate in vivo or in vitro tests A: Stimulate formation of allergen-specific IgG4 blocking antibodies which will compete with IgE for binding sites on Mast cells or Basophils A: Decreases IgE and reduces the seasonal rise of IgE A: Changes CD4+ cells from Th2 to Th1 phenotype A: Typically twice a week until a response is noted, then q1 week x 1 year, q2 weeks x 2 years, q3 weekly x 3 years

What levels should be addressed for salivary gland malignancy with a No neck

A: Parotid - I, II, III, IV (some debate about V) A: Submandibular - I, II, III

Pleomorphic adenoma locations

A: Parotid > Hard palate (minor salivary gland) > SMG > MSGs

Five parotid space contents

A: Parotid gland A: Facial nerve A: Lymphatics A: External carotid artery A: Retromandibular vein

Which area of TM is most susceptible to barotraumas

A: Pars Tensa

Two situations that may cause motion sickness

A: Passenger in the back seat of a car A: Passenger in ship cabin without windows

How is LPR different from classic GERD?

A: Patients have head & neck symptoms but heartburn is uncommon A: Predominantly upright (day-time) reflux A: Normal esophageal motility A: Most do not have esophagitis, as in GERD A: Laryngopharyngeal epithelium is more susceptible to reflux-related injury than esophageal epithelium

Total Glossectomy reconstruction

A: Pec major A: Rectus abdominus A: Lat dorsi

Differences between the pediatric & adult larynx

A: Pediatric larynx higher in neck (C3/4 vs C5/6) (about C2 at birth) A: Epiglottis curved/omega shaped, in contact with soft palate A: Thyroid cartilage oblique, no defining angle, overlapping with hyoid & cricoid A: Arytenoids are relatively large A: Infant vocal cords 4-4.5mm long at birth, adults 14-23mm A: Infant true vocal cord 50% composed of vocal process of arytenoid, in adults 25-33% A: Funnel shape, Infant subglottis narrowest portion of airway, 4.5-5 mm at full term A: Infant subglottis is loose, with a lot of submucosal glands

Name 3 autosomal recessive syndromes associated with SNHL (PUJ)

A: Pendred's syndrome A: Usher syndrome A: Jervell-Lange Nielsen syndrome

Classification of Temporal Bone Trauma (PB3)

A: Penetrating A: Blunt A: Blast A: Barotraumas

What are the frontal veins of Breschet in the frontal bone (diploic veins) and what is their significance?

A: Perforating veins connecting the intracranial and extracranial venous draining systems A: A potential pathway of hematologic spread of infection intracranially A: If not cleared in frontal sinus obliteration, can harbor mucosa and cause mucopyocele formation

Melolabial flap (superiorly based nasolabial) blood supply

A: Perforators of levator labii superioris muscle, off Angular artery

Neonatal tracheostomy safety factors intraoperatively and postoperatively

A: Perform the tracheostomy with the neck extended using a shoulder roll A: Stay sutures in tracheal incision A: Placement of ETT before performing tracheotomy A: Keep NG tube in situ to prevent mistaking esophagus for trachea A: Postoperative observation in PICU A: Tracheostomy set at the bedside A: Flexion of neck while applying ties A: Do not tack skin edges together to avoid subcutaneous emphysema A: CXR in recovery room to verify tube position and to R/O pneumothorax A: Always keep at bedside - Trach set with Hemostat, Suction, Same size and smaller trach tubes A: First tube change at 5-7 days

Cleft Palate repair

A: Performed at 8-12 months A: Restoration of soft palate sling incorporating tensor and levator palate muscles A: Schweckendiek - closure of soft palate only A: Von Langenbeck (bipedicle mucoperiosteal flaps of the hard and soft palate) A: Bardach two flap palatoplasty (for complete CP repair) A: Furlow double Z-plasty (for secondary CP repair) A: Wardill-Kilner-Peet V-Y pushback technique (for secondary CP repair)

Origin & ion content of Perilymph and Endolymph

A: Perilymph - Thought to be an ultrafiltrate of CSF & blood; Na = 139, K = 4, Cl = 0, Protein = 200-400, Glucose = Serum conc A: Endolymph - Stria Vascularis and the Dark cells of the Cristae and Maculae; Na = 13, K = 144, Cl = 120, Protein = 126, Glucose = low A: Protein content is higher in Endolymphatic sac

Subclassify Ameloblastomas

A: Peripheral vs Central A: Unicystic, Plexiform unicystic, or Multicystic 3: Plexiform unicystic more aggressive, requires excision with 3 cm margins

Describe Permanent Threshold Shift (PTS)

A: Permanent Cochlear damage, specifically to the OHCs A: Two main causes - A: Acoustic Trauma - Single, short lasting exposure to a very intense sound; causes Sudden, Painful loss of hearing A: Noise-Induced Hearing Loss - Chronic exposure to more Moderate sound levels

Preoperative intervention that may reduce CVA incidence over Simple Carotid Ligation

A: Permanent balloon occlusion A: Underlying principle is that high Embolization of the carotid eliminates the source of Stump Emboli A: Angiographic placement of permanent balloons or coils in the carotid siphon region just proximal to the ophthalmic artery, closely monitored for 72 hours, 2 weeks prior to carotid resection

Six bones of the Nasal Septum

A: Perpendicular plate of Ethmoid A: Vomer A: Maxilla - Anterior nasal spine and Maxillary crest A: Palatine bone - Maxillary crest A: Sphenoid A: Nasal bones (included in Schuenke)

Five early Signs of Impending Complication

A: Persistence of acute infection for 2 weeks A: Recurrence of symptoms within 2 weeks A: Fetid discharge during treatment A: Acute exacerbation of chronic infection, especially if fetid A: HiB or Anaerobes 3: "Two Week Fetid Exacerbation with Type B Anaerobes"

Six Indications for Open Exploration of CSF leak

A: Persistent leak despite Medical Management (10-14 days) A: Large Defect A: Brain/Meningeal Herniation A: Brain Penetration by bony Spicule A: Persistent Pneumocephalus A: Recurrent Meningitis

Four symptoms of Chronic Adenoiditis

A: Persistent nasal discharge A: Chronic congestion A: Postnasal drip A: Halitosis

Indications for PSG postsurgery for pediatric OSAS

A: Persistent snoring A: Preoperative FTT A: Preoperative cor pulmonale A: Age <1 year

Suture line that Longitudional temporal bone Fractures follow

A: Petrosquamous

Surgical Approaches to Petrous Apex for Drainage of infection

A: Petrous apex divided into Anterior/Posterior by plane running through IAC or Cochlea A: Posterior Petrous Apex pneumatized in 30%, Anterior Petrous Apex pneumatized in 10% A: Posterior approaches - SRS Subarcuate Retrofacial Sinodural A: Anterior approaches - IPI Infracochlear Peritubal Infralabyrinthine A: Endoscopic Trans-sphenoidal

Name the 5 structures that run between the external and internal carotid arteries

A: Pharyngeal branch of the glossopharyngeal nerve A: Pharyngeal branch of the vagus A: Stylopharyngeus A: Styloglossus A: Stylohyoid ligament

Adenoid blood supply (5)

A: Pharyngeal branch of the internal maxillary (major supply) A: Ascending pharyngeal artery A: Ascending palatine branch of the facial artery A: Ascending cervical branch of thyrocervical trunk A: Artery of the pterygoid canal

Describe Zenker's diverticulum

A: Pharyngoesophageal pseudodiverticulum pulsion type

Which of the parameters in rotary chair testing is the most reproducible?

A: Phase

What is the Axhausen two phase theory of Osteogenesis

A: Phase I - Initial bone formation comes from surviving transplanted cells, lays down osteoid randomly; Lasts 4 weeks, determines final size of graft A: Phase II - Replaces, reorganizes and remodeling of phase I bone; Last from 2 weeks to 6 months, peak at 6 weeks; Pluripotent host cells transformed into osteoblastic cells, bone morphogenic protein important for fibroblast ingrowth

Describe the phases of Clinical Trials

A: Phase I - Tolerance and Pharmacologic properties; determine spectrum of Toxicity A: Phase II - Efficacy of a drug in a specific Disease and Stage at a defined Dose; define Activity within tolerable levels of toxicity, endpoint is Response Rate A: Phase III - Comparison of 2 therapies in a Randomized manner; endpoints are Response Rate, Response Duration, Disease Free Survival, Overall Survival

Name 3 systemic decongestants

A: Phenylephrine A: Pseudoephedrine A: Phenylpropanolamine

Three disadvantages of Rigid endoscopes

A: Phonation is limited to sustained vowels A: Unnatural positioning with extended neck and protruded tongue A: The size of a glottic gap might appear exaggerated 3: A lateral approach to the larynx, or a 90-degree endoscope, to decrease neck extension during the examination, may help improve several of these points

Define Diplophonia

A: Phonation with two simultaneous fundamental frequencies A: Occurs with divergent travelling wave velocities due to asymmetric stiffness

Possible surgical modalities for RRP

A: Phono-microsurgery A: Microlaryngoscopy with Cup forceps A: Microlaryngoscopy with Microdebrider A: Microlaryngoscopy with Laser (KTP, Nd:YAG, CO2, flash scan) A: Cryosurgery

Describe two types of Glottography

A: Photoglottography - Change in light transillumination measures degree of glottic opening A: Electroglottography - Change in electrical conduction measures degree of glottic contact A: Speed quotient - Ratio of duration of glottic opening to duration of glottic closure A: Shift quotient - Ratio of time to peak glottic opening over the duration of glottic opening A: Open quotient - Ratio of duration of open folds to length of the glottic cycle (1/3) A: Closed quotient - Proportion of the glottic cycle during which the glottis is closed (2/3 split into 1/3 opening phase and 1/3 closing phase and when combine with 1/3 open phase it makes up complete glottic cycle)

Discuss Rhinophyma

A: Phymatous type of Rosacea is a sebaceous hyperplasia characterized by marked skin thickenings and irregular surface nodularities of the nose, chin, forehead, one or both ears, and/or the eyelids; affects ~50 yo males, may harbor occult BCC A: Four distinct histologic variants - Glandular, Fibrous, Fibroangiomatous, and Actinic A: Associated with Demodex folliculorum mite (inhabit hair follicles), and possibly H. pylori A: Treatment - Preventative = Deffating detergents, Anti-inflammatories, Antibiotics, Benzoyl peroxide, and Isotretinoin topical application; Surgical correction = Electrosurgery, Dermabrasion, Argon or CO2 Laser, +/- STSG

Three Advantages of ICW Mastoidectomy

A: Physiologic TM Position A: Deep Middle Ear A: No Mastoid Bowl

Clinical signs of post-obstructive pulmonary edema

A: Pink frothy secretions A: Hypoxemia A: Bilateral end expiratory wheezing with rales A: Radiographic findings (increased pulmonary markings & fluid overload)

Pediatric otologic development

A: Pinna - Near adult size at 4-5 years, full size by 9 years A: Tympanic membrane - Adult sized at birth, horizontal because of incomplete ossification of EAC, vertical position reached by 2 years A: External auditory canal - Incomplete ossification at birth leads to increased compliance on impedance audiometry until age? A: Eustachian tube - 50% adult length at birth, 10 degrees, enters nasopharynx @ hard palate; by 5-7 years lateral portion rises, tube lengthens & widens, 45 degrees, enters nasopharynx at inferior turbinate level A: Ossicles & petrous temporal bone - Adult sized at birth A: Mastoid antrum - Present at birth, increases in size during 1st year, pneumatization continues into childhood, fully developed mastoid & styloid by 3 years

What types of Alloplasts are used in mandibular reconstruction

A: Pins & wires (Kirschner wire, Steinmann pins) - No longer used anymore due to instability A: Alloplast appliances - Trays made of silicone, polyurethane-reinforced Dacron polyester mesh; used as spacer trays or carriers for bone grafts A: Bone plates & screws - Stainless steel, cobalt-chromium, titanium; used most commonly in mandibular reconstruction

Three subsites of the hypopharynx

A: Piriform sinus A: Posterior pharyngeal wall A: Postcricoid region

Four measures in a Standard Tinnitus Program

A: Pitch A: Loudness A: Minimum masking level A: Residual inhibition

Seven Ancillary tests for subglottic stenosis

A: Plain film A: Fluoroscopy A: Angiogram (R/O vascular ring) A: Flow-volume loops (Spirometry) A: pH probe A: cANCA A: Biopsy (R/O TB, WG, scleroma) HEAD & NECK ANSWERS

Airway imaging modalities

A: Plain soft tissue films of the neck, AP (croup) + lateral (epiglottittis & RPA) A: CXR AP + lateral (FB & tracheal stenosis) A: Inspiratory & expiratory chest films (FB) A: Airway Fluoroscopy (dynamic, awake & sleep, best for OSA) A: Barium swallow (vascular compression) A: Electron beam CT, or spiral CT scan with apnea A: MRI of the airway (intrathoracic vascular anomalies & masses) A: Bronchogram (after MRI, if difficult tracheobronchial stenosis) A: Laryngeal U/S (linear 7.0, 10 MHz, B-mode real-time)

Superior & inferior boundaries of the oropharynx

A: Plane through the Hard Palate A: Plane through the Hyoid

Ddx of Benign Salivary gland neoplasms in Adults

A: Pleomorphic adenoma A: Warthins tumor A: Oncocytoma A: Monomorphic adenoma A: Myoepithlioma

MRI findings of parapharyngeal space tumors

A: Pleomorphic adenoma - Low signal T1, high signal T2, displace carotid posteriorly A: Schwannoma - High signal T2, displace carotid anteriorly A: Paraganglioma - Salt and Pepper appearance on T1 due to hemmorhage and Flow Voids, displace carotid anteriorly

Etiologic agents in Chronic Sinusitis

A: Pneumococcus, H. influenzae, Moraxella still common, but Coag negative Staph, S. aureus & Pseudomonas play larger role A: Greatly increased role of anaerobes - Peptococcus, Bacteroides, Peptostreptococcus

Most common pollen, mold, and perennial allergens

A: Pollen - Short Ragweed A: Mold - Alternaria A: Perennial -Dust Mite (Dermatophagoides spp)

Ddx of benign vocal cord lesions (8)

A: Polyp (most common) - Mucoid or Angiomatous A: Nodules (bilateral) A: Cyst - Mucus retention or Epithelial inclusion A: Pseudocyst (no capsule)/Fibrous Mass? A: Polypoid degeneration/Reinke's edema A: Granuloma A: Sulcus vocalis A: Papillomas A: Lymphatic/vascular malformations A: Granular cell tumor A: Schwannoma A: Chondroma - Posterior cricoid A: Sarcoid A: Amyloid

Discuss cytokines, their mechanisms of action, and types

A: Polypeptide signal proteins that modulate cell function (proliferation, differentiation, regeneration, wound healing) A: Mechanisms of action (3) Autocrine - Secreted by cells which are modulated by it Paracrine - Secreted by cells which modulate neighboring cell populations Endocrine - Secreted by one population of cells with distant effects A: Categories - Angiogenic growth factors (TGF, EGF, PDGF, FGF), Bone morphogenetic protein, ILs, INF, TNF?

Two indications for surgery in Ankyloglossia

A: Poor feeding/sucking - First few months A: Impaired speech - 2-3 years FACIAL PLASTICS ANSWERS

Differential diagnosis of gingival hypertrophy

A: Poor oral hygiene A: Autoimmune - Diabetes, Crohn's A: Drug effects - Phenytoin most common, also cyclosporine A, calcium channel blockers A: Inherited - Cowden's disease A: Pregancy

Ten indications for Immunotherapy in allergic rhinitis include symptoms of allergy after natural exposure to aeroallergens and evidence of clinically relevant specific IgE, AND...

A: Poor response to Avoidance measures A: Allergens not easily avoided A: Poor response to Pharmacotherapy A: Unacceptable Adverse Effects of medications A: Severe symptomatology, persisting for >1 season A: Coexisting allergic rhinitis and asthma A: Possible Prevention of asthma in children A: Patient Wishes to reduce/avoid long-term pharmacotherapy and cost A: Motivated patients willing to undergo a program that may last up to 5 years

Three limitations of MRI for sinuses

A: Poor visualization of bony involvement A: Increased cost compared to CT scan A: Limitations due to metal

Two Disadvantages of Larger hearing aids

A: Poorer Cosmesis A: Occlusion effect

Merkels Cell carcinoma characteristics

A: Poorly Differentiated histology A: Neuroendocrine origin (think involved in fine touch) A: Aggressive tumours with High Recurrence rate A: High Lymphatic met rate, Need to Treat lymphatics A: Requires post op XRT after wide resection

Histologic subtypes of PTC with worse prognosis (PITCHS)

A: Poorly differentiated/Insular cell A: Tall cell A: Columnar cell A: Hurthle cell (variant of Follicular, worst of the WDTCs) A: Diffuse sclerosing 3: LN metastasis increases risk of local recurrence, not mortality; 5 year survival = 70-90%

Define Competence

A: Possession of the required knowledge, skill, and experience to perform a particular task reliably and produce an appropriate outcome A: It is a categorical variable; one is competent, or not, to provide a particular service or perform a particular operation

Two disadvantages of Cricotracheal Resections & Thyrotracheal Anastomosis

A: Possibility of injury to the Recurrent Laryngeal Nerve (lateral cricoid dissection is performed in subperichondrial plane & lateral resection is anterior to the Cricothyroid joint) A: Possible partial Dehiscence at anastomotic site resulting in Restenosis (laryngeal release only if >5 tracheal rings resected)

Six reasons for post-adenotonsillectomy desaturation

A: Post-obstructive pulmonary edema A: Loss of hypercapnic respiratory drive A: Airway swelling & obstruction A: Aspiration of blood clots A: Laryngospam A: Narcotic overmedication

Ten Steps in order of procedure for a Fisch A Approach

A: Postauricular Incision with Neck extension A: Neck Dissection - Expose CN IX-XII, ICA, IJV A: Superficial Parotidectomy and Identification of CN VII A: Wide field Mastoidectomy - Removal of Mastoid Tip, entire EAC, Middle Ear contents A: Identification of CN VII, Removal from canal and Anterior Translocation A: Anterior mobilization of Mandible A: Exposure of Posterior and Middle Cranial Fossa Dura for intracranial extension A: Removal of Disease A: Obliteration of cavity with Abdominal Fat, Rotated Temporalis Muscle, Tensor Fascia Lata A: Layered Closure and Compression dressing application

Most common locations of origin of Primary Acquired Cholesteatoma

A: Posterior Epitympanum - Originates in Prussak's space, breaks out by either penetrating Posterior into Superior Incudal space and into Aditus and Mastoid, Inferiorly into Posterior Pouch of von Troltsch, or Anteriorly into Anterior Pouch of von Troltsch A: Posterior Mesotympanum - Pars tensa retracts into Sinus Tympani and Facial Recess A: Anterior Epitympanum - Retraction pocket Anterior to Malleus Head, follows Anterior Pouch of von Troltsch into Supratubal area

Three indications for LTR with Division of the Posterior Cricoid lamina

A: Posterior Glottic/Subglottic stenosis A: Complete Glottic/Subglottic stenosis A: Significant Cricoid deformity

Four indications for Cartilage Grafting in the posterior glottis and subglottis

A: Posterior Glottic/Subglottic stenosis A: Isolated Subglottic shelves A: Circumferential Subglottic stenosis A: Total or near total obstruction at the glottic or subglottic level

Define the Iter Chordae Posterior

A: Posterior canal that the Chorda Tympani enters the Mesotympanum through

Three characteristics of Normal phonatory posture

A: Posterior commissure closure A: Symmetric thyroarytenoid stiffness A: Equal vertical position

Five indications for Long-term Stenting in pediatric airway reconstruction

A: Posterior cricoid split without cartilage grafting A: Lack of airway wall Rigidity A: Keloid formation A: Severely altered anatomy by stenosis or surgery A: Unstable cartilage grafts 3: "Posterior Rigid Keels are Severely Unstable"

Blood supply to the Posterior crus of the Stapes

A: Posterior crural artery - Terminal branch of Superficial Petrosal artery

Define tubercles of Zuckerkandl

A: Posterior extensions of each thyroid lobe where ultimobranchial bodies fused to thyroids, good landmard for RLN

Two indications for arytenoid adduction procedure

A: Posterior glottic gap not corrected with type I thyroplasty A: Vocal processes at different levels in the caudal/rostral axis 3: Usually necessary with posterior glottic gap ≥4 mm

Two routes of deep parotid lesions to enter the PPS

A: Posterior to stylomandibular ligament - round shaped lesion A: Anterior to stylomandibular ligament (ie via the tunnel) - dumbbell shaped mass 3: Stylomandibular tunnel - defined by posterior border of ramus of mandible, skull base and stylomandibular ligament

Describe the Canine space

A: Potential space between infraorbital & zygomatic heads of quadratus labii superioris, and this and the caninus muscle (Janfaza = between buccinators, orbicularis oris, and levator labii superioris) A: Important in anterior superior and bicuspid infection A: Treatment - Dental extraction/root canal, dependent drainage

Methods of determining Tip Projection

A: Powell & Humphreys - Height measured from Nasion to Subnasale, perpendicular drawn through Tip defining point, ratio of N-X/X-Tp ideally 2.8:1 A: Goode - Line drawn through Nasion (N) to Alar-Facial groove, perpendicular drawn through tip defining point (Tp), ratio of Tp-Y/N-Tp (Projection/Length) ideally 0.55-0.6:1 A: Crumley & Lancer - Defined relationship of tip projection, nasal length, vertical height as a right triangle whose sides follow a 3:4:5 ratio A: Simon - Tip projection (measured from Tip defining point to Subnasale) and length of the Upper Lip (measured from subnasale to upper vermilion border) should be equal

Variables that can be controlled with laser

A: Power A: Spot size A: Exposure time/frequency 3: Two other variables that determine tissue effect of laser are 1) laser type, and 2) tissue type

Name the 6 components of a laser

A: Power source A: Stimulation (excitation source - high voltage D/C vs flash lamp) A: Active medium (lasing medium) A: Rear mirror A: Output coupler A: Laser beam

Therapy for Autoimmune Inner Ear Disease

A: Prednisone 60 mg PO qd for 4 weeks A: Add Methotrexate 7.5-20 mg PO qweek with Folic Acid for people who Relapse during steroid Taper A: Add Cyclophosphamide 1-2 mg/kg/d for Nonresponders to steroids and methotrexate

Discuss Field Cancerization and 2 Theories of possible explanations

A: Premalignant changes and the propensity for second primary tumor development in the mucosal surface in the region adjacent to an invasive tumor A: Theory 1 - The areas of abnormality represent independent clones, with a unique pattern of genetic alterations A: Theory 2 - The areas of abnormality are genetically related and originate from a common cellular clone, with lateral spread of progenitor clones throughout squamous mucosal surfaces; predominant theory as per genetic studies 3: Subsequent alterations may demonstrate discordance, so the final pattern may be different. Therefore, the concept of field cancerization is likely a reflection that at least a proportion of lesions derived from lateral migration and subsequent expansion of clonally related cells develop histologic alterations surrounding the primary lesion, and ultimately lead to an increased incidence of subsequent primary tumors. OTOLOGY & NEUROTOLOGY ANSWERS

Complications of Stapedectomy

A: Preoperative - Anaesthesia A: Intraoperative - TM perforation/Flap tear (repair with gelfoam or paper patch for small perf, underlay graft for large) Facial nerve injury - Overhanging CN VII (gently retract superiorly and proceed) Chorda tympani injury (30%, metal taste, tongue soreness, dry mouth; better to cut than to stretch) Bleeding - High riding IJV, Persistent stapedial artery Perilymph gusher (<1%, tissue graft to OW and complete surgery, otherwise pack ear and stop, use CSF measures postop) Floating footplate/Depressed footplate (can drill inferior OW and use pick to elevate it, vs. placing shorter prosthesis over it, revise if unsuccessful) Incus fracture (complete surgery if not completely broken off, otherwise use TORP or IRP) Malleoincudal dislocation (remove incus and use TORP or IRP) Malleus fixation (remove incus and use TORP or IRP) A: Postoperative - Infection - Otitis media, Labyrinthitis, Meningitis Vertigo/Disequilibrium (5%, usually subsides after a few days; Persistent causes include Depressed footplate, Prosthesis too deep into oval window, Perilymph fistula, Oval window Granuloma) Perilymph fistula (3-10%, rare with small fenestra, causes mixed HL, revise) Oval window Granuloma (HL, Vertigo, and reddish mass in posterior superior TM 1-6 weeks postop, associated with use of gelatin sponges or fat in OW; treat with prompt removal of granuloma in OR) Fluctuating CHL - Due to Loose Prosthesis Incus necrosis Persistent TM perforation (freshen edges and place paper patch, myringoplasty for failures) Delayed facial palsy (onset up to 5 days postop, treat with prednisone, usually resolve) Dysgeusia (10%, most resolve by 4 months) Tinnitus Profound SNHL (<1%, treat with prednisone 60 mg x 5 days then taper) Delayed SNHL (1%) 3: IRP = Incus Replacement Prosthesis

Ddx of a Flat Hearing Loss (POOLS)

A: Presbycusis (Strial/Metabolic & Neural types) A: Ototoxicty (Macrolides) A: Otosyphilis A: Late Menieres A: Sudden SNHL A: Vascular loop

Calculation for Breakthrough Opioids doses

A: Prescribe 10% of the Total Daily Dose q1h A: When using Fentanyl, calculate 10% for the Breakthrough dose of Morphine SQ q1h, or multiply by 2 for Morphine PO q1h 3: For Fentanyl 25 mcg/h, Breakthrough = Morphine 2.5 mg SQ or 5 mg PO q1h

Four criteria for surgery in Aural Atresia

A: Presence of Cholesteatoma - Emergent indication, and reason every aural atresia requires CT at some point A: Normal Sensorineural function A: Ossicular mass present A: Middle ear space and mastoid at least 50% normal size

Four Indications for fluoride therapy

A: Presence of SNHL or Vertigo (cochlear otosclerosis) A: Non surgical candidates A: Positive Schwartze's sign pre-op (Rx for 6 months prior to surgery) A: Otospongiosis type bone seen intraop

Poor prognostic EMG findings

A: Presence of Spontaneous activity A: No Recruitment A: Fibrillations

Seven Poor prognostic indicators for Sudden SNHL (VASA RED)

A: Presence of Vertigo A: Age (advanced = poorer) A: Severity (total deafness worse) A: Audiogram shape (flat/downsloping worse) A: Other vascular Risk factors (HTN, DM) A: Elevated ESR A: Delay in initiation of treatment

Eight factors which may influence the ENG test/subtest outcomes (MC FA SLAB)

A: Presence of sedating Medications A: Congenital nystagmus A: Fatigue A: Level of Attentiveness A: Changes in skin resistance (Sweating) A: Ambient Light level can disturb recording A: Age A: Interference from eye Blinking 3: Test factors (3) & Patient factors (5); Recording based on corneoretinal potential; can only measure Horizontal eye movements, not vertical or torsional

Describe Jahrsdoerfer's grading system for congenital aural atresia

A: Presence of the stapes (2 points) A: Oval window status A: Round window status A: Facial nerve course through the middle ear A: Malleus/incus complex A: Incus-stapes connection A: Middle ear space A: Mastoid pneumatization A: External ear appearance 3: 10 point system based on high resolution CT findings of the temporal bone; ≤5 = not surgical candidates, ≥8 = 80% success

What is the most important factor in overall survival in melanoma

A: Presence or absence of Lymph Node Metastasis

Describe the spontaneous nystagmus of a acute unilateral vestibular loss

A: Present when head is still, dampened by visual fixation, increased or becomes apparent when fixation eliminated A: Horizontal component - Fast phase towards Intact ear A: Torsional component - Superior poles beat toward Intact ear

Discuss Mallory-Weiss tears

A: Present with UGI bleed secondary to vomiting/retching A: Develop nonpenetrating mucosal tear at lower esophagus/cardia, tears submucosal arteries A: Usually causes significant bleed and hypovolemic shock in 50%; bleeding usually stops, may require endoscopic cautery or vasoconstrictor application

Presentation, 4 investigations, and 2 interventions for TEF

A: Present with immediate feeding problems and aspiration A: Investigations - Inability to pass a feeding tube, X-rays (curled tube in the pharynx & air bubble in the stomach), Fluoroscopy, Endoscopy (mainstay) A: Management - Surgical correction, Dilation of strictures

Describe Chandler classification of orbital complications in sinusitis

A: Preseptal cellulitis A: Orbital cellulitis A: Subperiosteal abscess A: Orbital abscess A: Cavernous sinus thrombosis

Histologic features of necrotizing sialometaplasia

A: Preserved lobular architecture A: Lobular infarction with or without mucus extravasation A: Inflammation secondary to extravasated mucus A: Pseudoepitheliomatous hyperplasia at periphery of lesion A: Squamous metapasia of ducts & acini

3 functions of the Eustachian tube

A: Pressure regulation/ventilation A: Protection from nasopharyngeal reflux A: Drainage of middle ear secretion

Mechanism of action and 3 examples of Topoisomerase Inhibitor chemotherapeutic agents

A: Prevent unwinding of DNA for replication by Inhibiting Topoisomerases I or II 3: Irinotecan (I), Topotecan (I), Etoposide (II)

Two Absolute Contraindications to IV iodinated contrast for CT

A: Previous adverse reaction A: Severe renal insufficiency (esp. in Multiple Myeloma, DM, Nephrotoxic meds) A: Seven Relative Contraindications to IV iodinated contrast for CT A: Advanced age A: Asthma A: Atopy A: Beta-blockers A: Cardiac disease A: Dehydration A: Mild renal insufficiency A: Planned thyroid radio-ablation

Classification of Herpes Simplex infections (HSV 1 or 2)

A: Primary A: Secondary (recurrent)

Classification of hypothyroidism

A: Primary (Gland) A: Secondary (Pituitary) A: Tertiary (Hypothalamus) A: Peripheral resistence

Etiologies of Hyperparathyroidism

A: Primary - Adenoma or Hyperplasia A: Secondary - Gland hyperplasia due to malfunction of other organ system; CRF, MM, OI, PD, bone mets (PTBLK), pituitary basophilism A: Tertiary - Autonomous PTH production in patients with normal or low calcium levels; after cause of secondary HPT has been corrected

Two types of Acquired Cholesteatoma

A: Primary Acquired - AKA Retraction Pocket cholesteatoma; NOT secondary to infection, develop within a retraction pocket A: Secondary Acquired - Secondary to perforation from infection, surgery or trauma

Define Otosclerosis

A: Primary Metabolic Bone disease of the Otic Capsule and Ossicles, in which abnormal bone remodelling results in Fixation of the Ossicles and Conductive Hearing Loss; May have Sensorineural component if the cochlea is involved

Five Risk factors for lymphoma in Sjogren's

A: Primary Sjogren's A: Constant parotid enlargement A: Lymphadenopathy A: Splenomegaly A: Decreased IgM

Describe the TNM Staging of Esophageal cancer

A: Primary Tumor (T) TX - Primary tumor cannot be assessed T0 - No evidence of primary tumor Tis - Carcinoma in situ T1a - Tumor invades lamina propria T1b - Tumor invades submucosa T2 - Tumor invades muscularis mucosa T3 - Tumor invades adventitia T4 - Tumor invades adjacent structures A: Regional Lymph Nodes (N) NX - Lymph nodes cannot be assessed N0 - No lymph node metastasis N1 - Lymph node metastasis A: Distant Metastasis (M) MX - Distant metastasis cannot be assessed M0 - No distant metastasis M1 - Distant metastasis

Options for upper lid reconstruction

A: Primary closure A: Lateral cantholysis A: Split thickness skin graft A: Sliding tarso-conjunctival flap (posterior lamella from adjacent remaining lid, and advancement or FTSG) A: Posterior lamellar graft with myocutaneous flap (good for lid laxity, can supplement with ear cartilage) A: Pedicled flap from lower lid A: Tenzel flap A: Cutler-Beard bridge flap (>60%, 2 stage, borrows skin/muscle/conjunctiva from lower lid passing under the tarsus, with ear cartilage for support)

Options for lower lid reconstruction

A: Primary closure (30% loss in young and 45% loss in elderly) A: Lateral Cantholysis A: Full thickness skin graft A: Laterally based upper lid graft A: Tenzel rotational flap (up to 60%, laterally based semicircular musculocutaneous flap) A: Mustarde cheek rotational flap (for large defects, single stage, like exaggerated Tenzel, but lacks Orbicularis, so saggs; good for non seeing eye) A: Free tarsal grafts (for posterior lamella, from ipsi or contralateral lid) A: Hughes procedure (>50%, 2 stage, tarsoconjunctival flap from upper lid, and advancement or FTSG)

Discuss Complement

A: Primary humoral mediator of antigen-antibody reactions A: 2 Pathways of activation = Classical (activated by IgG or IgM, C1 -> C4b,2b) & Alternative (activated by PAMPs, C3 -> C3b,Bb) A: 4 Biologic activities - Opsonization, Chemotaxis and Cell Activation by fusion of fragments to neutrophils & macrophages, Cell Lysis through the membrane attack complex (MAC)

Indications for combined therapy for oropharyngeal cancer

A: Primary lesion - T3/T4, perineural/vascular invasion, positive/close margins A: Neck - N2, N3; or N0/N1 (clinical) with 2+ histologically positive nodes/nodes at multiple sites, Extracapsular invasion, Perineural or vascular invasion

Indications for radiation therapy for oropharyngeal cancer

A: Primary modality for T1 & T2 lesions with equal cure rate as surgery

Etiology of nasal tip Bosselation

A: Primary nasal tip asymmetry A: Iatrogenically created sharp edges A: Too narrow a rim strip

Six Roles for Chemotherapy in Head & Neck Oncology

A: Primary treatment of NPC... A: Organ preservation in Advanced Laryngeal cancer A: Unresectable disease A: Recurrent or metastatic disease A: Post-operative adjuvant therapy A: Investigative therapy

Contraindications to Botox

A: Prior allergic reaction A: Injection into areas of infection or inflammation A: Pregnancy (category C), or breastfeeding A: Myasthenia Gravis A: Medications that decrease NM transmission (can have botulism-like effects at high doses) - Aminoglycoside, Penicillamine, Quinine, Calcium channel blockers

Define Agger nasi cell

A: Product of 1st ethmoturbinal, found superior, lateral & anterior to attachment of the middle turbinate; can also refer to the anterior-most ethmoid cell, anterior to the frontal recess A: Boundaries are frontal process of maxilla anteriorly, nasal bones anterolaterally, frontal recess superiorly, lacrimal bone inferolaterally, uncinate process inferomedially

Define Dyskeratosis

A: Production of keratin at lower layers

Ddx of cervical disease in HIV

A: Progressive generalized lymphadenopathy (PGL, 12-45%) A: Mycobacterium tuberculosis A: Pneumocystiis carinii A: Lymphoma A: Kaposi sarcoma

Triad of symptoms for RRP

A: Progressive hoarseness (most common symptom) A: Stridor A: Respiratory distress

Indications for endoscopic examination in pediatric stridor (PAUSe)

A: Progressive stridor A: parental Anxiety A: Unusual features - Cyanotic attacks, apneic attacks, dysphagia, aspiration, recurrent pneumonias, failure to thrive, radiologic abnormality A: Severe stridor

Define Neoplasm

A: Proliferation of cells and formation of a mass

Stages of Hemangioma evolution

A: Proliferative - 6-12 months A: Involuting - 50% by 5 years, 70% by 7 years A: Involuted - Redundance, scarring, telangiectasias

Six General Disadvantages of MRI

A: Prolonged data collection times A: Higher sensitivity to patient motion A: Contraindications - Pacemakers, certain Implants, Metallic foreign bodies A: Inferior bony detail A: Claustrophobia may prohibit examination A: Higher equipment cost & exam cost

Passavant's ridge

A: Prominence on the posterior wall of the nasopharynx from contraction of the superior constrictor during swallowing

Difference between Proptosis and Exophthalmos

A: Proptosis = Pushing of the globe (some say when <18mm protrusion, others say when from non endocrinopathy) A: Exophthalmos = Uniform expansion of the orbital contents (some say > 18mm protrusion, others say when from an endocrinopathy ie. graves)

Three pros and 4 cons of radiotherapy for Paragangliomas

A: Pros - Lower/No risk to cranial nerves Lower/No risk of stroke Most tumours will stop growing A: Cons - Risk of microvascular and carotid artery disease Risk of radiation induced malignancies May have to operate in radiated bed Risk of Osteoradionecrosis

Most common sources of Temporal Bone Metastases

A: Prostate, Thyroid, Breast, Lung, Larynx, Kidney, GI tract 3: "P T Barnum Loves Little Kids & Girls"

Reasons for Stapedectomy Failure

A: Prosthesis Displacement (#1 cause) A: Incus tip Erosion A: Footplate Refixation A: Perilymph Fistula A: Otosclerotic Regrowth

Bacteriology of Chronic External Otitis

A: Proteus vulgaris A: Fungi (Aspergillus?)

Name 5 classes of drugs used to treat LPR, and give an example of each

A: Proton pump inhibitors - Pantoprazole A: H2-Blocker - Ranitidine A: Pro-kinetics - Metaclopramide, cisapride A: Cytoprotectants - Sucralfate A: Buffer - CaCO3

Two groups of Genes involved in cell Growth & Cancer

A: Protooncogene - Normal cell genes that influence cell growth in a positive way A: Tumor suppressor gene - Encode proteins that normally exert negative regulatory control in a cell

What constitutes an abnormal 24 hour pH probe result

A: Proximal/Hypopharyngeal probe pH <4 >1% of the time A: Distal probe pH <4 >4% of the time

Name the 6 clinical forms of oral candidiasis

A: Pseudomembranous (classic presentation) A: Acute atrophic A: Chronic atrophic (most common) A: Hyperplastic (increased epithelial atypia and malignant transformation) A: Median rhomboid glossitis A: Mucocutaneous (most serous, immunocompromise, 20% familial, AR)

Bacteriology of Acute External Otitis (4)

A: Pseudomonas aeruginosa A: Proteus mirabilis A: Staphylococci A: Streptococci A: Peptostreptococcus

What are the 4 bugs most commonly seen in COM? And 4 others seen in presence of Cholesteatoma?

A: Pseudomonas aeruginosa A: Staphylococcus aureus A: Corynebacterium diphtheria A: Proteus sp. A: Klebsiella pneumoniae 3: In presence of cholesteatoma - B. fragilis, Peptococcus, Peptostreptococcus, Proprionibacterium Anaerobes seen more commonly

Describe the 4 cell types making up the nasal epithelium

A: Pseudostratified Ciliated Columnar epithelium - ~50 cilia per cell beating, ~12/second at physiologic temperature, moves mucus 3-25 mm/minute A: Pseudostratified Nonciliated Columnar epithelium - Possess microvilli that expand the surface area of the epithelium A: Goblet cells A: Basal cells 3: Other cell types - Stratified squamous epithelium in nasal vestibule, and Olfactory epithelium found along roof of nose

Olfactory neuroepithelium histology

A: Pseudostratified columnar epithelium

Epithelium of the nasopharynx

A: Pseudostratified columnar majoritarily at birth, slowly replaced by stratified squamous in adolescence A: Transitional cell, found at junction of the two epithelial linings

Reflux Finding Score

A: Pseudosulcus - Subglottic edema (/2) A: Vocal fold edema (/4) A: Ventricular obliteration (/4) A: Diffuse laryngeal edema (/4) A: Posterior commissure hypertrophy (/4) A: Erythema/Hyperemia (/4) A: Granulation/Granuloma (/2) A: Thick endolaryngeal Mucus (/2) 3: Eight signs, Max score = 26; >7 is significant (95% chance of having a positive pH probe study)

Two conditions associated with Geographic tongue

A: Psoriasis A: Local inflammation RHINOLOGY ANSWERS

What are the 5 symptoms of Horner's syndrome?

A: Ptosis A: Meiosis A: Anhidrosis A: Enophthalmos A: Loss of the Ciliospinal reflex - dilation of the pupil ipsilateral to a pain stimulus applied to the neck

Four symptoms of Acute Adenoiditis

A: Purulent rhinorrhea A: Nasal obstruction A: Fever A: Otitis media

Ddx of Neonatal Nasal Obstruction

A: Pyriform aperture stenosis A: Midnasal stenosis A: Choanal atresia A: Nasolacrimal duct cyst A: Tumors - Encephalocele, Glioma, Dermoid, Teratoma, Hemangioma

Five concepts of Lee-Silverman voice training (MAISE)

A: Quantify with a Sound Meter A: Increase the Amplitude of vocal output A: Administer in Intensive fashion A: Increase Sensory perception A: Administer in High Effort style 3: Three problems in Parkinsons are 1) Decreased vocal drive, 2) Poor self-perception, and 3) Inability to regulate output

Describe Acoustic voice measures

A: Quantitate vocal harshness, measures perturbation in the peak to peak timing of each glottal pulse A: Jitter = Cycle to cycle Frequency fluctuations (normal ≤0.4%) A: Shimmer = Cycle to cycle Amplitude fluctuations (normal ≤0.5 dB) A: Harmonic-to-Noise Ratio (normal ≤11 dB) A: Time domain measures - Timing relationships in the acoustic signal (Jitter and shimmer)

Five complications of tracheal stenosis repair

A: RLN damage A: Granulation tissue A: Infection A: Dehiscence A: Fistulization

Paramedian position in vocal cord palsy suggests

A: RLN injury only, implies a lesion below the nodose ganglion A: Treated with type I thyroplasty

Screening tests for patients with Sudden SNHL

A: RPR/FTA-ABS A: MRI A: Total Cholesterol - if age 35-65 A: HIV - if high risk A: ESR, ANA, RF - if relapse after Steroid course

Virology of AOM (4 - RRIP!)

A: RSV (#1) A: Rhinovirus A: Influenza A: Parainfluenza

Name the 6 treatment options for early glottic cancer; which one has best voice function?

A: Radiation (best voice function, preserves normal anatomic relationships) A: Endoscopic cordectomy - cold steel, or laser A: Laryngofissure and cordectomy A: Vertical partial laryngectomy A: Hemilaryngectomy

What Spinal Cord Complication can occur Post-Radiotherapy? Sign/Dose of radiation is associated?

A: Radiation Myelopathy A: L'Hermitte's Sign - Electric shock sensations triggered by Flexing the Cervical Spine A: Doses of 50 Gy in 5-6 weeks 3: Also Transverse myelitis, Spinal cord Necrosis?

Classification of neck dissections

A: Radical A: Modified radical (types I, II, III/Functional) A: Selective - Lateral (II, III, IV), Posterolateral (II, III, IV, V), Anterolateral (II, III, IV, VI), Supraomohyoid (I, II, III) A: Extended

List 8 factors for treatment of glottic Ca with VPL instead of XRT

A: Radioresistant tumors (eg verrucous carcinoma) A: Salivary gland malignancies A: Benign laryngeal tumors A: Patients deemed unreliable for 6 weeks of XRT A: Young patients (due to the theoretical increased risk of late radiation-induced sarcoma) A: Neck nodes >2 cm in size favors primary surgery for both neck and primary A: Obese A: T1 extending to anterior commisure

Classification of local skin flaps based on blood supply (3)

A: Random cutaneous - No named vessels, rely on dermal and subdermal plexus vessels supplied by musculocutaneous arteries near the base of the flap A: Axial pattern/Arterial cutaneous - Incorporates a named subcutaneous artery running along length of the longitudinal axis of flap A: Myocutaneous/fasciocutaneous - Incorporates a segmental vessel that sends perforating vessels to overlying muscle & skin

Discuss Sinonasal Undifferentiated Carcinoma (SNUC)

A: Rare aggressive malignancy commonly affecting the Anterior Skull Base A: Incidence of Regional and Distant mets = 20 and 10% respectively A: Treatment - Surgical resection + Chemo-XRT A: Prognosis - Disease free survival up to 29% at 2 yrs, 9% long-term

Idiopathic subglottic stenosis (ISS)

A: Rare inflammatory process of unknown cause A: Limited to subglottis & upper 2 tracheal rings A: Young female >85% (estrogen altering wound healing response?) A: Surgery is the main treatment modality (endoscopic laser with mitomycin-c for < 1cm, open laryngotracheal surgery (CTR) for thicker complex scar)

Describe Chordoma

A: Rare slow growing skull base Malignancy derived from Notochord Remnant A: Histology - Physaliferous cells (Soap Bubble appearance) A: Treatment - Surgery, Radiotherapy for Incomplete excision

Five types of nasopharyngeal cysts

A: Rathke's pouch cyst - Remnant of invaginated ectoderm that forms the anterior pituitary gland, anterior to the pars intermedia; located high in nasopharynx near sphenovomeral junction; ciliated respiratory epithelium A: Tornwaldt's cyst - Remnant of notochord, inferior to Rathke's pouch; filled with jellylike material; ciliated respiratory epithelium A: Dermoid cyst - Benign developmental cyst derived from ectoderm and mesoderm; stratified squamous epithelium with adnexal structures A: Intraadenoidal - From median pharyngeal recess, opens onto adenoid bed A: Extraadenoidal - Deep within pharyngobasilar fascia, remnant of the pharyngeal bursa; usual findings are a cuff of granulation tissue rostral to the pharyngeal tubercle

Ten complications of Angio Embolization

A: Recurrence of bleeding (10-20%) A: CVA with permanent neurologic deficit (4%) A: Pulmonary embolus A: Blindness A: Temporofacial pain and paresthesia A: Trismus A: Skin necrosis A: Groin infection or hematoma A: Death

Eight Indications for PETs

A: Recurrent AOM (>3/6 mo, >4/1 yr, with failed medical management) A: OME (bilateral >3 mo or unilateral >6 mo; earlier when >25db CHL, speech/language delay, severe retraction pocket, disequilibrium/vertigo, or tinnitus present) A: Eustachian tube dysfunction with autophony, disequilibrium or vertigo, tinnitus, or Atelectatic TM/severe retraction pocket, unrelieved by medical management A: Patulous eustachian tube A: Barotitis media/Hyperbaric oxygen therapy A: Suspected unusual pathogen A: Suppurative complication, present or suspected A: Unsatisfactory response to antibiotics

Most common cause of oral ulcerative lesion

A: Recurrent Aphthous stomatitis

Definition of Sialosis/Sialoadenosis

A: Recurrent bilateral non-tender, non-inflammatory, non-neoplastic parotid swelling that occurs secondary to other underlying pathology (see below)

Anatomic associations between the inferior constrictor & esophagus

A: Recurrent laryngeal nerve A: Inferior laryngeal artery & vein

Most common benign neoplasm of larynx in children

A: Recurrent respiratory papillomatosis (RRP) 3: Second most common cause of hoarseness in children

What is Schwartze's sign?

A: Reddish/Vascular tinge to Promontory seen through TM in Otospongiosis phase of Otosclerosis

Pathognomic cell in Hodgkin's lymphoma

A: Reed Sternberg cell

Discuss VEMP

A: Reflex initiated by the Saccule A: Click stimulus producing a short-latency relaxation potential recorded by EMG of the ipsilateral SCM (an i-i response as per ART - ipsilateral inhibitory) A: Normally threshold is ~80 dB A: CHL abolishes VEMP A: SCDS depresses threshold

Describe general principles of Osseointegration

A: Rehabilitation of oral/maxillofacial or other head and neck defects with permanently implanted hardware, typically after resection & other therapy (chemo/RT) A: First surgery - Titanium implants placed into bone and allowed to integrate (form stable relationship) with bone (extracellular matrix etc forms stable relationship with titanium oxide on surface of titanium) A: 2nd surgery - Implants exposed and abutments placed which will connect the implant to the bone

Five historical open frontal sinus procedures

A: Reidel - Removal entire floor & anterior wall of frontal sinus; significant cosmetic deformity A: Lynch - Ethmoidectomy plus removal of entire frontal sinus floor, and part of the lamina papyracea A: Killian - Combination of Reidel and Lynch; removal of floor and anterior wall, retain frontal bar A: Lothrop/Chaput-Mayer - Superior nasal septum and inner sinus septum taken down A: Osteoplastic Flap - Anterior wall frontal sinus based inferiorly retracted and replaced with periosteum intact

Eight Pathologies causing Non-Organ Specific Autoimmune Inner Ear Disease

A: Relapsing Polychondritis A: Polyarteritis nodosa A: Behcet's syndrome A: Rheumatoid arthritis A: Systemic lupus erythematosis A: Wegener's granulomatosis A: Churg-Strauss syndrome A: Cogan's syndrome 3: "Peter PAN Be RA SLEing With Churg Cogan"

Surgical Strategies to increase support for dental prosthesis

A: Remove tooth and cut through a socket A: Preserve anatomy - Mandible (Alveolus, retromolar pad, buccal shelf), Maxilla (Tuberosity, alveolus, hard palate) A: Preservation of strip of mucosa - Make the mucosal cuts closer to the specimen than the bony cuts as to use the mucosa to fold into the defect A: Avoid true hemimaxillectomy - Save medial and lateral incisor for tripod effect A: Skin grafting of cheek flap, and areas of the obturator contact A: Skin grafting of the maxillary sinus A: Resection of inferior turbinate

Discuss the pathophysiology of Varices/Capillary Ectasias

A: Repeated microtrauma A: Capillary angiogenesis A: Dilated capillaries (proceeds from anterior to posterior, may look like capillary lake) making cord even more susceptible to trauma A: Vocal cord swelling A: Hemorrhagic polyp formation

Histopathologic origin of Vocal cord granuloma

A: Repetitive contact trauma leads to mucosal ulceration over the vocal processes of the arytenoids, cartilage exposure and inflammation leads to granuloma formation

Discuss Transverse temporal bone Fractures

A: Represent ~20% A: Cause - Due to Occipitomastoid trauma, usually of High Energy A: Clinical - Present with significant SNHL and Vestibular dysfunction; ~50% CN VII paralysis of Immediate onset A: TB course - Through Jugular Foramen & Foramen Magnum, courses through Otic Capsule through Vestibule or medial IAC, then through Foramen Spinosum & Lacerum

Discuss Longitudinal temporal bone Fractures

A: Represent ~80% A: Cause - Due to Lateral impacts; often with Tearing of TM and Hemotympanum A: Clinical - Present with Conductive HL; Vestibular involvement mild (Concussive); CSF leak ~20%; CN VII paralysis of Delayed onset (~20%), if fallopian canal damaged will usually be in tympanic segment A: TB course - Through EAC & tympanic ring, fractures Tegmen, avoids otic capsule, runs through Foramen Lacerum, Foramen Ovale, and/or Eustachian tube

Discuss Follicular thyroid carcinoma

A: Represents 10-15% (including Hurthle cell subtype) A: >4th decade, F:M 3:1 A: Varying degrees of Capsular and Vascular invasion (required for carcinoma diagnosis); other suggestive features are Necrosis, high Mitotic rate, and widespread Nuclear atypia; multicentricity uncommon A: IHC usually not necessary, but Tg, TTF-1, Cytokeratin, HBME-1, and Galectin-3 are positive A: Histologic variants - Oxyphilic (Hurthle), Clear cell, insular A: Lower rate of nodal metastasis (20%), higher rate of distant metastasis (65%) A: Size not a prognostic factor, angioinvasion a poor prognosis; 5 year survival 70%

Discuss Papillary thyroid cancer

A: Represents 80%, multicentric in 80%, A: 3rd & 4th decades, F:M = 3:1 A: Poorly encapsulated A: High locoregional metastasis to nodes (37-65%), lower rate of distant metastasis (2-17%) A: Age and Size are important prognostic factors; whereas vascular invasion and nodal metastasis are not significant A: Can display mucinous, ciliary, or squamous metaplasia

Discuss Medullary thyroid carcinoma

A: Represents ~5% A: 80% sporadic, 20% familial (better prognosis); RET proto-oncogene on chromosome 10q11.2, autosomal dominant A: Familial >90% bilateral, sporadic 30% bilateral A: Age of onset younger (20-30) in familial; mean age 50 in sporadic A: Early lymph node metastasis (50% at presentation) A: Histologic features - unencapsulated, Hyalinized bands of collagen, Amyloid deposits, Granular cytoplasm, Coarsely clumped chromatin, and prominent Nucleoli A: Markers - Calcitonin (most sensitive & specific), CEA, NSE, Chromogranin, Synaptophysin A: Total thyroidectomy for MEN IIA by 5-6 years, for MEN IIB by 6 months of age; 131I not indicated (not effective) A: 5 year survival 88%, 10 year survival 65%

Discuss Anaplastic thyroid carcinoma

A: Represents ~5% A: >65 years of age A: Risk factor is previous thyroid tumors = Papillary CA, Follicular CA, Follicular Adenoma A: Two genes associated = p53 and BRAF A: Histology - Bizarre, Giant & Spindle cells in various proportions, high mitotic rate, necrosis, infiltration A: 10% survival after 1 year

Discuss thyroid Lymphoma

A: Represents ~5%, more common than anaplastic carcinoma A: Hashimoto's disease increases patients risk 70x A: Almost always NHL, Immunoblastic most common subtype

Three Disadvantages of Preoperative Radiotherapy for SCC

A: Resection and Reconstruction more Difficult due to Fibrosis, Inflammation and decreased Blood Supply A: Obscured tumor Margins by tumor Shrinkage and Inflammatory response A: Wound Healing problems increase as dose >40 Gy A: Lower overall dose (Hart notes)

Five methods of Decreasing Tip Rotation in rhinoplasty

A: Resection of caudal septum near spine A: Shorten medial crura of alar cartilage A: Dorsal graft augmentation A: Infratip button A: Interrupted strip, especially if performed medial to domes?

Five Disadvantages of ICW Mastoidectomy

A: Residual Cholesteatoma may be Occult A: Recurrent Cholesteatoma may occur in Attic A: Delayed Canal Breakdown A: Incomplete Exteriorization of Facial Recess A: Second Stage often required

Causes for Continued Draining Ear Post-Mastoidectomy

A: Residual cells/mucosa (Mastoid Tip) A: Residual or Recurrent Cholesteatoma A: High Facial Ridge A: Inadequate Meatoplasty A: Poor Hygiene/Water Precautions

Describe the Techniques for Volume Reduction used in surgery of the nasal tip

A: Residual complete strip - Conservative cephalic trim; whenever possible, ideal to leave a complete strip of lateral crus of at least 5 mm in width A: Weakened complete strip - For further refining of tip, done by conservative cross hatching, morselization, or incomplete noncoalescent dome incisions A: Suture modification of complete strip A: Interrupted strip - For severe tip deformities, increased risk of asymmetric healing & scarring, causes increased cephalic tip rotation 3: In addition to volume reduction, other techniques include reconstruction, excision, augmentation, reorientation

Name the 3 Major nasal tip supports

A: Resiliency of medial & lateral crura A: Medial crural attachment to septum A: Alar attachment to upper lateral cartilages

Difference between Ossicular Stiffness and Mass

A: Resonance frequency of the middle ear ossicles is 800-1200 Hz A: Changes in ossicular Stiffness/Mobility affect Low frequency sound transmission A: Changes in ossicular Mass affect High frequency sound transmission 3: In otosclerosis, early lesions decrease the overall mobility of the ossicular chain, and subsequently will affect the ossicular mass; thus the hearing loss will spread from low into the higher frequencies

Name the 3 main functions, and 6 subfunctions, of the nasal airway

A: Respiration - Warming (37 C), Humidification (85%), and Nasal airflow (which can be altered through airway resistence by congestion & decongestion of the mucosa) A: Protection - Filtration, Mucocilliary clearance, and Immune protection A: Olfaction - To sense the environment through the specialized & general sensory nerves (olfactory & trigeminal)

What are the five component processes of speech?

A: Respiration/Generator - Lungs produce airflow through URT A: Phonation - Sound production by the vocal cords A: Resonance - Modulation of phonatory output by vibration in the chest, pharynx and head with selective amplification of certain component frequencies A: Articulation - Formation of consonants & vowels, controlled by lips, tongue, palate, pharynx A: Prosody - Intonation

What does primary hyperparathyroidism in infancy present with?

A: Respiratory distress A: Hypotonia A: Skeletal demineralization

Discuss Postmeningitis SNHL

A: Responsible for ~33% of all Hearing Deficits acquired after Birth A: 10% of patients experience Persistent SNHL A: Incidence of meningitis (and HL) varies with strain - Pneumococcus 31% (20%), N. meningitides 11% (5%), H. influenzae 6% (12%)

Five assessment tasks Using Continuous Light

A: Rest breathing A: Deep breathing A: Easy cough or throat clear A: Laryngeal diadochokinesis (DDK), which consists of rapid repetitions of "ee" or "hee" with glottal stops between productions A: Repetitions of a short "ee" followed by a quick sniff through the nose

Eight goals of primary mandibular reconstruction

A: Restore mandibular continuity A: Restore lower facial contour A: Reconstruct anatomic configuration of sulcus A: Provide rehabilitation with a functional lower denture A: Maintain mobility of residual tongue A: Improve mastication, deglutition, speech A: Restore sensation to denervated lower lip A: Restore sensation to resurfaced portions of oral cavity

Define Parakeratosis

A: Retention of nuclei in cells attaining the level of the stratum corneum

Name the 5 common subtypes of Lichen Planus

A: Reticular - Most common, fine lacy appearance on buccal mucosa (Wickham's striae), asymptomatic A: Atrophic - 1-5% malignancy rate A: Plaque - Leukoplakia A: Erosive - 1-5% malignancy rate, very painful A: Bullous - Painful 3: Remember "RAPE Bullous", other types include Ulcerative (Buccal, painful, greater risk of malignancy), and Annular (Lips, ringed edges); Lichenoid dysplasia? (Precancerous lesion)

Six alternative sites for parathyroid gland location

A: Retroesophageal (most common location for ectopic superior glands) A: Superior mediastinal (most common location for ectopic inferior glands) A: Intrathymic A: Intrathyroidal A: Carotid sheath A: Tracheoesophageal

Five craniofacial abnormalities associated with OSA

A: Retrognathia A: Shortened anterior cranial base A: Increased distance between mandibular plane and hyoid A: Narrow posterior pharyngeal space A: Enlarged soft palate 3: Other significant physical findings include - Macroglossia, collar size ≥17 in (40 cm), BMI, truncal obesity, HTN

Discuss 7 other Approaches used for Skull Base lesions requiring Drainage or Sectioning

A: Retrolabyrinthine - Main indication is Vestibular Nerve section, also for Resection of Selected Arachnoid Cysts, Meningiomas, Metastatic CPA lesions; Minimal Cerebellar Retraction; Hearing preservation A: Infralabyrinthine - After mastoidectomy, Sigmoid Sinus Decompressed & air cell tracts followed into Petrous Apex; Might be impossible with a high Jugular Bulb A: Transcochlear - Extension of Translabyrinthine Approach, Removal of Cochlea & Displacement of Facial nerve for access to Petrous Tip and Clivus A: Transotic - Similar to Transcochlear, but Facial nerve is Skeletonized and left in the Fallopian Canal A: Transcanal Infracochlear - Air cell tract between IJV and ICA is followed into Apex, permits Dependent Drainage, but requires EAC Transection which heals over prolonged period, also requires Exposure of Petrous Carotid artery A: Extended Middle Fossa - Extended by removal of Petrous Ridge and Posterior aspects of Temporal Bone up to Labyrinth A: Endoscopic Trans-sphenoidal

Venous drainage of the superficial head & neck

A: Retromandibular vein - formed from junction of superficial temporal and pterygoid plexus veins, splits to join the external jugular and the common facial vein A: Common facial vein - formed from junction of facial and branch from retromandibular vein, drains into internal jugular vein at the level of the hyoid

Posterior neck spaces

A: Retropharyngeal space - Found between visceral layer of deep fascia (buccopharyngeal) and alar layer of deep fascia; descends to mediastinum at level of carina A: Danger space - Found between alar layer and prevertebral layer of deep fascia; descends to diaphragm A: Prevertebral space - Found posterior to prevertebral fascia, descends to sacrum

Hismanal (H1-receptor blockers)

A: Reversible inhibition of histamine receptors

Seven Indications for Image Guided Surgery according to the AAO-HNS

A: Revision surgery A: Distorted anatomy A: Extensive NP/CRS A: Disease in Frontal, Posterior Ethmoid, or Sphenoid A: Disease abutting the Skull base, Orbit, or Optic nerve A: Skull base defect/CSF leak/Encephalocele A: Tumors

Most common cause of viral laryngitis

A: Rhinovirus

Viruses most commonly associated with acute rhinosinusitis

A: Rhinovirus A: Influenzae A: Parainfluenza A: Adenovirus 3: Others may include coronavirus, and RSV

Three patterns of calcification of thyroid masses

A: Rim (eggshell) - Benign A: Bilateral superolateral - Medullary Ca A: Extensive irregular - Multinodular goiter

Management of an exposed carotid

A: Risk factors - Prior radiation, Malnutrition, Diabetes, Fistula/Infection A: Inpatient considerations - Design incisions properly (avoid trifurcations), Perioperative antibiotics, Adequate tissue coverage (regional/free flaps)

Three indications for Z-plasty

A: Rotation of the dominant axis of a scar, especially those that cross RSTLs A: Reduction of scars distorting anatomic landmarks A: Avoidance of a web or band across a concavity

Two routes of access for Middle ear infection into the Inner ear

A: Round window - Most common A: Oval window 3: Although rare in comparision to the frequency of middle ear infections, this route is likely the most common for spread to the inner ear

Five options for Cleft Lip repair

A: Rule of 10's = 10 weeks, 10 pounds, 10 g of hemoglobin A: Straight line closure (rarely used anymore) A: Millard rotation advancement technique A: Millard bilateral cleft repair A: Tennison-Randall or Skoog techniques (single) triangular flap interdigitation A: Bardach (double) triangular flap interdigitation

Rule branchial arch anomaly relationships

A: Run deep to own arch structures A: Run superficial to next arch structures

Stridor history mnemonic "SPEC SPEAR"

A: S = Severity, parents' Subjective impression A: P = Progression of obstruction over time A: E = Eating/feeding difficulties A: C = Cyanotic spells A: S = Sleep disordered breathing A: P = history of Prematurity A: E = history of Endotracheal intubation A: A = possibility of foreign body Aspiration A: R = Radiographics that detect a specific abnormality

Identify the structures formed from the four embryologic sacs of the middle ear (Procter)

A: S. Anticus - Anterior pouch of von Troltsch A: S. Medius - Epitympanum and Petrous area A: S. Posterior - Sinus tympani, OW & RW niches A: S. Superior - Posterior pouch of von Troltsch, part of the mastoid, inferior incudal space 3: Main mucosal sacs of the middle ear, develop between 12th & 28th gestational weeks

Identify the 5 Histochemical Markers for Melanoma

A: S100 A: HMB-45 A: Melan-A/MART-1 A: Tyrosinase A: Vimentin

Five Muscles inserting on the Mastoid

A: SCM A: Posterior belly of digastric A: Longissimus capitis A: Splenius capitis A: Posterior auricular muscles

Two methods to help differentiate Spasmodic Dysphonia from Functional Dysphonia

A: SD will not respond to speech therapy, although functional will A: SD may be improved with whispering, singing or laughing

Lateral cord position in vocal cord palsy suggests

A: SLN & RLN injury, loss of cricothyroid with SLN injury causes increased abduction A: Treated with types I & IV thyroplasty

Symptoms of Noise-Induced Hearing Loss (STRADD)

A: SNHL (Usually bilateral) A: Tinnitus A: Recruitment A: Nonauditory (Anxiety, etc.) A: Diplacusis A: Distortion

Definition of Functional/Serviceable Hearing

A: SRT <50 dB and A: SDS >50% (Class 2) 3: Alternative uses BETTER hearing cutoffs of SRT <30 dB and SDS >70% (Class 1)

Causes of abnormal Oculomotor tests on ENG

A: Saccades - Central pathologic disorder, either Cerebellar lesion (dorsal vermis), Brainstem (PPRF and MLF), or Ocular muscles & Nerves A: Smooth pursuit system - Distributed throughout Brainstem and Cerebellum, anatomic localization not possible; Most common cause of bilateral impaired smooth pursuit = Medication side effects (anticonvulsants, sedatives), or Neurologic conditions (Parkinson's, Progressive SupraNuclear Palsy, Alzheimer's, schizophrenia) A: Optokinetic system - Responsible for optokinetic afternystagmus (OKAN), velocity storage mechanisms in the Cerebellum and Brainstem; Absence or asymmetry occurs with peripheral vestibular lesions 3: Cerebellum and Brainstem are common to all

Three findings for Chronic inflammation on sialography

A: Saccular dilatation of terminal ducts/acini A: Segmental strictures & dilatation A: Pseudocyst formation 3: Contrast to Sjogren's where there are punctuate, globular, cavitary lesions, or complete destruction

Four advantages of Cricotracheal Resections & Thyrotracheal Anastomosis

A: Safe effective treatment for Severe SGS A: Results are Superior to similar cases done by LTR techniques A: Voice quality results are better (preserves voice) A: No interference with normal growth of Larynx

Five Advantages of Postoperative Radiotherapy for SCC

A: Safer administration of Higher Total Doses of radiation A: Destruction of subclinical Residual tumor A: Surgical resection Easier, Healing is better in non-irradiated tissues A: Distinct tumor Margins which facilitates more accurate & complete surgical removal A: Ability to Direct Radiation specifically at areas Inaccessible by surgery A: Better staging possible

Management of hypercalcemic crisis

A: Saline hydration & electrolyte correction A: Furosemide diuresis A: Glucocorticoids A: Calcitonin 5 IU/Kg IM A: Bisphosphonates - Pamidronate 60 mg IV over 4 hours A: Organic phosphates: 8 mg IV, PO, PR A: Mithramycin 25 mcg/Kg IV over 4 hours A: Gallium nitrate A: Hemodialysis for life threatening situations 3: Symptoms - weakness, anorexia, N/V, drowsiness, confusion, stupor, coma 3: Causes - malignancy, hyperparathyroidism, increased ingestion of calcium or vitamins D&A (CHIMPANZEES)

Ddx of non-neoplastic, non-traumatic, and non-infectious subglottic stenosis in adults (SWAPIS)

A: Sarcoidosis A: Wegener's granulomatosis A: Amyloidosis A: Relapsing polychondritis A: Idiopathic A: SLE

Three indications for scar revision

A: Scars >2 cm in length or >2 mm in width A: Does not lie in RSTL A: Distortion of normal anatomy

Five things to consider when hiding facial incisions

A: Scars best hidden if within or parallel to RSTL's A: Junction of aesthetic subunits A: Place within skin creases or folds A: Locations that can be hidden by hair (requires incision parallel to follicles) A: Use of squamomucosal incisions (margins of orifices) 3: Other possible answers include Placing scars as far from midline as possible, and using a Geometric broken line

Three benign tumors arising from Schwann cells

A: Schwannoma - Benign, encapsulated, slow growing, rare malignant sarcomatous degeneration A: Neurofibroma - Proliferation of sheath cells & fibers, nonencapsulated, nodular & diffuse A: Granular cell tumor - Slow, painless growth, well circumscribed

Discuss primary neurogenic pathologies within parapharyngeal space

A: Schwannoma - Most common neurogenic tumor, most common poststyloid mass; Vagus > Sympathetic chain; anterior displacement of carotid A: Paragangliomas - 2nd most common neurogenic PPS tumor, seen in poststyloid space; arise from Nodose ganglion of vagus (most common in PPS)/carotid body/inferiorly from jugular bulb, less commonly glossopharyngeal/superior or inferior laryngeal; Familial in about 10% (PGL1 gene on 11q23, can be part of MENIIA or IIB), bilateral 10%, malignancy 5%, secretory 3%; Vagal - displace carotid anteriorly; Carotid body - Lyre sign A: Neurofibroma - 3rd most common neurogenic tumor of PPS, origin = Schwann cells and perineural fibroblasts, unencapsulated/involve nerve; higher malignancy rate associated with NF Type I

Differences between Vagal Schwannoma and Carotid Body tumor on Angio

A: Schwannoma will push vessels Anterior & Lateral A: Carotid body tumor will splay ECA and ICA Apart (Lyre sign) A: Filling effect from vessel ingrowth seen in Carotid Body tumors

Three BCC and 2 cSCC with worse prognosis

A: Sclerosing/morpheaform BCC - clinical classification A: Keratotic BCC - histiologic classification A: Recurrent BCC A: Spindle cell SCC A: De novo SCC

Discuss the three Belladonna derivatives

A: Scopolamine, Glycopyrrolate, Atropine A: Antimuscarinics A: Scopolamine & glycopyrrolate potent antisialogogues A: Atropine the most vagolytic, produces greatest increase in HR

Four components of Migraine aura

A: Scotomata - Dark spot in visual field A: Photopsia - Flashing lights A: Teichopsia/Fortification spectra - AKA Scintillating Scotoma; Bright, shimmering, jagged lines that can spread across the visual field A: Paresthesias

Epidemiology of Cleft Lip and Palate

A: Second most common malformation after club foot A: Cleft Lip and Palate occur together in 1/1000 births A: Isolated Cleft Lip and Cleft Palate occur in roughly 1/2000 births each, with CP slightly more common

Describe Dentigerous/Follicular cyst

A: Second most common maxillary/mandibular cyst, enlargement of follicle of Impacted tooth A: Treatment - Enucleation & curettage

Acinic cell carcinoma

A: Second most common salivary gland cancer in children A: Derived from serous acinar cells, thus for location Parotid >> SMG, rare in MSG A: Gross - well circumscribed, fibrous capsule A: Histologic - 2 types, one resembles Serous acinar cells, other has a Clear cytoplasm; Amyloid a hallmark pathologic finding A: Configurations - Solid, papillary cystic, follicular, microcystic, vacuolated A: Treatment - Surgery (no neck dissection), with post op XRT if residual disease

Discuss Sinonasal Adenocarcinoma

A: Second most common sinonasal malignancy A: Male predominace ~6:1, 5th-6th decade, usually arises in Middle Meatus A: Papillary, Sessile, and Alveolar mucoid

Define Metastasis

A: Secondary discontinuous cancerous growths

Options for reconstruction of Floor of Mouth

A: Secondary intention A: Primary closure A: Dermal graft (one pass with dermatome to remove skin then second pass to remove dermis and lay skin back down on donor site) A: STSG (small defect) A: Local flaps - Nasolabial flap (moderate defect) A: Regional flaps (moderate defects) Forehead flap (superficial temporal artery) Submental artery island flap (submental branch of facial art) Facial artery musculomucosal flap (FAMM; includes some buccinator, and buccal fat) Platysma flap Deltopectoral flap (historical) A: Free tissue transfer - Radial forearm Lateral arm Anterolateral thigh

Five Reconstructive options after Mohs surgery

A: Secondary intention - Concave areas (nasolabial), medial canthus, perioricular, temporal, and forehead A: Primary A: Skin Graft A: Local flap A: Regional flap

Four options for auricular reconstruction

A: Secondary intention - Lobule and Helix do the worst A: Primary closure - Helical defects <1.5cm A: FTSG - Post-auricular A: Composite graft from opposite ear - Half of the width of defect

What is the likely cause of superficial "snail-track" ulcers of the tonsils and soft palate?

A: Secondary syphilis

Major immunologic product of tonsils and adenoids

A: Secretory IgA

Define Brown's sign

A: Seen in Glomus Typmanicum tumors A: Reddish blush seen behind an intact TM which Blanches upon introducing Pneumatic Pressure that exceeds the Systolic BP

Define Aquino's sign

A: Seen in Paragangliomas A: Pulsatile Tinnitus of paraganglioma Decreases with Carotid artery Compression

Six contraindications to HBO

A: Seizure A: Claustrophobia A: Pregnancy A: Neoplasia A: Pneumothorax A: Reactive airways = URTI, Asthma/COPD

Eight Congenital immune deficiencies

A: Selective antibody deficiencies - IgA, IgG (IgG3 most common in RARS) A: Common Variable Immunodeficiency A: Severe Combined Immunodeficiency A: X-linked Agammaglobulinemia A: Complement deficiency A: Hyper IgM A: Hyper IgE/Job syndrome A: Ataxia-Telangiectasia

Three stages of sphenoid sinus pneumatization

A: Sellar (86%) A: Presellar (11%) A: Conchal (3%) 3: Pneumatization starts between 5 & 7 (actually 3) years of age, complete by 20-25 years (18 years according to Amin)

Ideal reconstruction of the lip

A: Sensate A: Intact sphincter function for control of watertight continent seal A: Sufficient opening for food, dentures, oral hygiene A: Aesthetically acceptable

ABR for hearing screens

A: Sensitivity 98%, specificity 96% A: Click stimuli from 30-40 dB nHL A: Estimates hearing in 1-4 kHz range A: Requires natural sleep or conscious sedation A: Newborn ABR composed of waves I, III, V A: ABR reaches adult levels by 18 months to 3 years

OAE's for hearing screens

A: Sensitivity ~95%, specificity ~85% A: TEOAE used most often, DPOAE being used more A: Estimates hearing in 1-6 kHz range, can go higher?

Name 5 otologic complications of temporal bone trauma

A: Sensorineural hearing loss A: Conductive hearing loss A: CSF leak A: Facial nerve injury A: Meningitis A: Transient or prolonged vertigo A: Hemotympanum

Five classical signs of Congenital Syphilis (may not present until after first 2 years of life)

A: Sensorineural hearing loss A: Frontal bossing A: Interstitial keratitis A: Snuffles - Nasal septal perforation & saddle deformity A: Hutchinson's teeth

Hutchinson's triad for Congenital Syphilis

A: Sensorineural hearing loss A: Interstitial keratitis A: Notched incisors

Describe the four types of Presbycusis

A: Sensory - Atrophy of Organ of Corti, initially at Basal turn; Middle age; High frequency loss, good SDS related to frequency range affected A: Neural - Atrophy of Spiral Ganglion cells and nerves of spiral lamina, affects All turns of cochlea; Later in life; Flat to down-sloping, severe Discrimination loss A: Strial/Metabolic - Atrophy of Stria Vascularis, initially at Apex of cochlea; Flat sensory loss affecting all frequencies, minimal discrimination loss A: Cochlear Conductive - Stiffness of Basilar Membrane, affects Basal-all turns of the cochlea; High tone gradual loss, discrimination related to steepness of slope

Describe the 2 Components of Dynamic Posturography

A: Sensory organization test - Tests integration of Vestibular, Visual and Proprioceptive balance cues A: Movement coordination test - Based on automatic muscle responses thought to be triggered by proprioceptive changes; measurements recorder are Latency to onset (principal), Amplitude, and Symmetry of neuromuscular responses

Four Complications of Nasal Folliculitis and Funruncle

A: Septal chondritis A: Septal abscess A: Saddle-nose deformity A: Cavernous sinus thrombosis 3: Infections of hair follicle caused by Staph aureus or Strep LARYNGOLOGY ANSWERS

Seven Anatomic variations influencing incidence of sinusitis

A: Septal deviation and spur A: Conchal bullosa A: Paradosical middle turbinate (Lateral convexity) A: Prominent ethmoidal bulla A: Pneumatization or deviation of the uncinate plate A: Prominent agger nasi cells A: Haller cells (Infraorbital ethmoids)

Steps of rhinoplasty

A: Septum A: Gross Tip Work A: Dorsum A: Fine tip work A: Osteotomies

Four stages of Suppurative Labyrinthitis (SAFO)

A: Serous or Irritative phase A: Acute or Purulent stage A: Fibrous or Latent stage A: Osseous stage

Why is the parotid gland more susceptible to infection?

A: Serous secretions less bacteriostatic than mucinous secretions, which contain glycoprotein & sialic acid that can aggregate bacteria; also contains lysozymes & secretory IgA

Five Indications for PSG in peds

A: Severe CNS disease A: Age <2 years A: Medical conditions that increase the risk of surgical management A: History is discordant with physical exam A: Nonsurgical therapy is more likely to be indicated by specific pathology or patient and family desires

5 indications for pediatric maxillary sinus aspirate

A: Severe Toxic child A: Immunocompromise A: Unresolving symptoms after 72 hours A: Suppurative complications A: Work up for fever of unknown origin

Indications for CT scanning for pediatric rhinosinusitis

A: Severe illness or Toxic condition A: Immunocompromise A: Acute RS that does not improve with medical therapy in 48-72 hours A: Suppurative Complication

Six Indications for Surgical intervention in acute bacterial sinusitis

A: Severe pain A: Toxic A: Impending complications of sinusitis A: Nonresponse to medical therapy A: Immunocompromised patient A: >4 infections per year

Four indications for 2-Step LTR

A: Severe stenosis A: History of reactive airway A: Poor pulmonary function A: Inadequate intensive care facilities

Six factors in the choice of procedure/treatment for unilateral vocal cord paralysis (Bailey p.856)

A: Severity of the symptoms (aspiration or maximum phonation time <10 sec) A: Needs of the patient (vocal demands) A: Duration of the paralysis and prognosis for recovery (based on nature of injury and/or LEMG findings) A: Medical comorbidities of the patient (disease prognosis) A: Position of the cord A: Likelihood of compensation

Discuss the Histopathology of Esthesioneuroblastoma

A: Sheets of Small Round Blue cells, Scant cytoplasm and Hyperchromatic nuclei A: Neurofibrils, Flexner-Wintersteiner Rosettes and Homer-Wright Pseudorosettes A: Neurosecretory Granules on electron microscopy A: Immunohistochemistry typically positive for neural markers - Neuron specific enolase (most consistent), S-100, Synaptophysin, +/- Chromogranin 3: Mets rare at presentation, but Regional and Distants mets may arise in 15-30%

Three options for Post-operative airway support for Glottic Edema following Cricotracheal Resections & Thyrotracheal Anastomosis

A: Short term ET intubation A: Montgomery T tube stenting for older child (4-6 weeks) A: Distal tracheotomy (4-6 weeks)

Factors influencing development of laryngomalacia

A: Shortened aryepiglottic folds A: Anterior collapse of cuneiform cartilage A: Immature neuromuscular control A: Reflux

Five implant options for Type I Isshiki medialization thyroplasty

A: Silastic A: Goretex A: Hydroxyapatite A: Titanium A: Prefabricated implants

Characteristics of the KTP laser

A: Similar frequency & efficacy A: Fiberoptic use

Define Choristoma

A: Similar to hamartoma except the component tissues are not normally present in that part of the body

Define Simultaneous, Synchronous, and Metachronous tumors (15% have second primary)

A: Simultaneous = Diagnosed at same time A: Synchronous tumor = found <6 months later A: Metachronous tumor = found >6 months later

Most Useful imaging for Tracking progress of Necrotizing OE

A: Single-Photon Emission Tomographs (SPECT) with two radionuclide tracers - Indium-111-labeled leukocytes and Technetium-99m (Cummings, Ch. 132)

Two Surgical options of BPPV

A: Singular neurectomy A: Posterior SCC occlusion

What are the 3 most common areas for Residual Cholesteatoma?

A: Sinus Tympani A: Facial Recess A: Anterior Epitympanum

Give 2 Diseases Associated with Keratitis Obturans

A: Sinusitis A: Bronchiectasis

Five clinical features of Kartagener's (immotile cilia) syndrome, abnormalities in cilia?

A: Sinusitis A: Bronchiectasis (chronic cough) A: Situs inversus & dextrocardia A: Otitis media A: Male Infertility (immotile sperm) 3: Autosomal recessive inherited lack of Dynein arms in A-tubules (9+2 microtubule structure)

Other causes of unilateral Proptosis in adults and children

A: Sinusitis complications (most common in children) A: Orbital fracture A: Lymphoma

Ten H&N manifestations of SLE

A: Skin - Malar rash (50%), Telangiectasias A: Nose - Septal ulceration/perforation (3-5%) A: Oral & Salivary - Acute parotid enlargement (10%), Chronic xerostomia, Painful oral mucosal ulcers A: Laryngeal - True vocal cord thickening/paralysis, cricoarytenoid arthritis, subglottic stenosis A: Neurologic - Cranial neuropathy (15%)

Nine Late Complications of Radiotherapy for NPC in 6 different sites

A: Skin necrosis A: Osteoradionecrosis A: Cataracts - 6 Gy A: Middle ear effusion secondary to ET dysfunction A: Xerostomia - 35 Gy A: Transverse myelitis (50 Gy), somnolence syndrome, brain necrosis (65 Gy)

Discuss the Methods of testing for Atopy

A: Skin tests - Detect presence of IgE-mediated allergy; takes little time to do (~1 hr), but uncomfortable, risk of anaphylaxis; generally classified as Epicutaneous (scratch tests and prick-puncture tests) or Intracutaneous (single-dilution and multiple-dilution intradermal tests); Negative control is GLYCEROL, and Positive control is Histamine Scratch tests - Neither sensitive nor reproducible Prick tests - Not sensitive for low levels of atopy Intradermal testing - More sensitive & reproducible Serial Dilution Endpoint testing/ Skin Endpoint Titration - Intradermal test, determines more specific levels of atopy, used to determine immunotherapy concentrations A: Direct measurement of Allergen-specific IgE in serum; serum incubated with known Antigens on a matrix; excess serum washed off, and residual complexes incubated with anti-IgE and a marker; More specific but less sensitive cf. skin tests, takes longer to do (days) RadioAllergoSorbent Test (RAST) - Radioactive marker Enzyme-Linked ImmunoAssay (ELISA) - Fluorescent marker

Name the 6 Minor nasal tip supports

A: Skin/Soft-tissue envelope - Alar cartilage attachments to overlying skin & soft tissue A: Soft tissue/sesamoid complex - Attachment to pyriform wall A: Interdomal soft tissue attachments A: Bony nasal spine A: Cartilaginous septal dorsum A: Membraneous septum

Six useful landmarks for Revision ESS

A: Skull base A: Superior attachment of Middle Turbinate A: Sphenoid sinus ostium A: Maxillary ostium A: Roof of maxillary sinus A: Lamina papyracea

Surgical approaches to the Lateral Temporal Bone

A: Sleeve Resection - Removes cartilaginous EAC & some or all of the Bony Canal Wall Skin circumferentially WITHOUT bone removal; for malignancies localized to the cartilaginous EAC A: Lateral Temporal Bone Resection - Removes En Bloc the entire Osseous & Cartilaginous EAC with the Tympanic Membrane; for malignancies localized to Osseous EAC WITHOUT encroachment on medial mesotympanum A: Subtotal Temporal Bone Resection - For En Bloc resection of the Medial surfaces of the Mesotympanum, WITHOUT air cells of Petrous Apex & portions of Bony Labyrinth; for tumors involving the Middle ear A: Total Temporal Bone Resection - For En Bloc resection of the Temporal Bone, including Petrous Apex & Sigmoid Sinus

Five histologic findings in Wegener's

A: Small & medium vessel vasculitis (angiocentric) A: Fibrinous "geographic" necrosis A: Necrotizing granulomas A: Mixed plasma cell, lymphocyte, histiocyte, and macrophage infiltration, with isolated multinucleated giant cells A: Angioinvasion, identified better on elastic stain 3: Hallmark = Necrotizing Granulomatous Vasculitis (found all together in <20% of single cases)

Options for reconstruction of Buccal mucosa

A: Small - Primary closure, STSG, or dermal graft A: Moderate - Temporoparietal fascia flap (with STSG) A: Larger/full thickness - Regional, Distant (Pectoralis major or Lat dorsi), or Free flap

Four Preventative measures for avoiding SGS

A: Smaller ETT without compromising safe ventilation (air leak at <25 cmH2O) A: Diagnosing (pH probe) & treating LPR A: Prophylactic Antibiotics when tracheotomy is performed following prolonged/traumatic intubation A: Prolonged intubation up to 6 months preferred over tracheostomy in neonates

Risk factors for esophageal carcinoma

A: Smoking A: EtOH A: Nutritional deficiencies A: Radiation exposure A: Chemical exposure (nitrates, nitrites, petroleum oils) A: Caustic ingestion A: Chronic esophagitis/strictures A: Barrett's esophagus (15% will develop adenocarcinoma) A: Achalasia A: Plummer-Vinson syndrome A: Tylosis - Autosomal dominant, hyperkeratosis of palms & soles

List 10 risk factors for cervical esophageal Ca

A: Smoking - 4.5x increased risk A: ETOH - 11x increased risk, synergistic with smoking A: Nitrosamines A: Prior H&N carcinoma A: Barrett's esophagus - distal ↑ 5-15% increased risk of adenocarcinoma A: Caustic burn/scar/stricture A: Achalasia A: Plummer-Vinson syndrome A: Oculopharyngeal dystrophy A: Pernicious anemia A: Tylosis (thickening palms and soles)

Seven predictors of poor FESS outcome

A: Smoking! A: Asthma A: Polyps A: ASA sensitivity A: Fungal disease A: Extensive disease (CT) A: Previous surgery

Sleep syndromes

A: Snoring A: Upper airway resistance syndrome (UARS) - Daytime hypersomnolence with normal PSG A: Obstructive sleep hypopnea syndrome (OSHS) - Daytime hypersomnolence with hypopneas; >15 hypopneas/hour on PSG diagnostic A: Obstructive sleep apnea syndrome (OSAS) - Daytime hypersomnolence with apneas & hypopneas

Ten Differences in adult vs pediatric OSAS

A: Snoring intermittent vs continuous A: Mouth breathing rare vs common A: Obesity common vs rare A: Failure to thrive & enuresis rare vs common A: Daytime somnolence common vs rare A: Hyperactivity, attention deficit, aggression rare vs common A: Nighttime arousals common vs rare A: Gender predilection male vs none A: CPAP mainstay vs selected (postop OSA, craniofacial anomaly, C/I to surgery) A: Surgery selected vs mainstay

Four sites of primary H&N cancer that always require addressing of both necks

A: Soft palate A: Base of tongue A: Supraglottis A: Hypopharynx A: Subglottis

Histopathologic classification of Congenital SGS

A: Soft tissue - Granulation tissue, submucosal gland hyperplasia, submucosal fibrosis A: Cartilaginous - Normal shape (cricoid small for infant's size) A: Cartilaginous - Abnormal shape (elliptical shape, large anterior lamina, large posterior lamina, generalized thickening, submucus/incomplete laryngeal cleft, other) A: Cartilaginous - Trapped first tracheal ring A: Combined stenosis

Two Mucous layers in upper resp tract, and function of each

A: Sol layer - Thin periciliary layer which allows cilia to be mobile; produced by microvilli A: Gel layer - Thick layer of mucoglycoproteins in which cilia embed themselves; produced by goblet cells and submucosal glands, propelled by cliliated epithelium; (antibacterial and clearance of foreign particles?)

Four histolopathologic classes of Basal Cell Carcinoma (SACK)

A: Solid - Undifferentiated A: Adenoid - Tubular/Glandular differentiation A: Cystic - Sebaceous gland differentiation A: Keratotic - More biologically aggressive

Glioma

A: Solid mass of Glial tissue with a fibrous stalk A: Dural connection in 15% A: 60% external, 30% internal, 10% combined A: Manage any intracranial portion first; surgical excision through vertical midline dorsal excision, external rhinoplasty, or bicoronal approach

Toxic CNS doses

A: Somnolence syndrome A: Myelopathy - 50 Gy in 25 fractions A: Transverse Myelitis - 50 Gy A: Brain Necrosis - 70 Gy

Describe Crossover

A: Sound energy that exceeds Interaural Attenuation and can stimulate the nontest ear

Define Distortion

A: Sound is heard sufficiently loud, but is of poor quality

Name 4 common dystonias in the head and neck

A: Spasmodic dysphonia A: Blepharospasm A: Torticollis A: Oromandibular dystonia

Four zones of antigen processing in adenotonsillar tissue

A: Specialized squamous epithelium (Dendritic cells) A: Extrafollicular area (T-cells) A: Mantle zone of lymphoid follicle (Mature B-lymphocytes) A: Germinal center of follicle (Active B-cells)

Speech Audiometry tests (3)

A: Speech Reception Threshold (SRT) - Softest intensity level a patient can repeat back a spoken word (Spondee) 50% of the time; Agreement between PTA and SRT should be within 10 dB A: Speech Discrimination Score (SDS) - Estimation of a person's ability to hear and understand speech; Percentage score of correctly repeated words off of a 50 phonetically balanced word list presented at 30-40 dB SL A: Word Recognition Score - Phonems presented at 30 dB SL

Treatment options for Drooling

A: Speech therapy A: Behavior therapy A: Dental appliance A: Drugs (glycopyorolate, trihexyphenidyl, scopolamine, Botox) A: Radiation A: Surgery - SMG excision or Duct re-routing Parotid duct ligation, Tympanic neurectomy, Intraductal laser photocoagulation

Management of Dysfluent speech

A: Speech therapy A: Dilate or repuncture A: Use correctly sized prosthesis with ideal airflow characteristics A: Pharyngeal constrictor myotomy, neurectomy, Botox injection A: Allow edema to subside, provide external pressure

Which anatomic sinus variation is more common in Asians

A: Sphenoethmoidal/Onodi cells (51% of asians)

What 2 vessels must be ligated in a transantral maxillary artery ligation to prevent recurrent epistaxis from collaterals?

A: Sphenopalatine artery A: Descending palatine artery

External carotid artery branches supplying the internal nose

A: Sphenopalatine artery - Lateral posterior nasal artery (supplies lateral nasal wall), and Septal posterior nasal artery (across anterior sphenoid and along septum up to Little's area) A: Descending Palatine artery - Splits into Lesser and Greater Palatine arteries (the latter supplies Little's area in the septum & floor of nose through the Incisive branch) A: Septal branch of Superior Labial artery - Supplies the Septum and Ala A: Pharyngeal branch (of IMAX)- Supplies posterior nose & nasopharynx 3: All are terminal branches of the internal maxillary artery except the superior labial artery, which is a branch off the facial artery

Define Sluder syndrome

A: Sphenopalatine neuralgia A: Localized facial pain A: Vasomotor abnormalities - lacrimation, rhinorrhea, salivation

Histiologic subtypes of Myoepithelioma

A: Spindle pattern A: Plasmacytoid A: Combination

Four characteristics of vertigo in Vestibular Neuronitis

A: Spontaneous A: Fast phase to contralateral side A: Unilateral decreased calorics

Four types of Peripheral Nystagmus

A: Spontaneous A: Positional A: Induced (calorics) A: VOR

What are the 4 subtypes of OAEs?

A: Spontaneous OAE - No external stimulus, do not occur where a Cochlear HL >30 dB exists; Occur in 35-60% of normal ears, usually narrow-band signals with 10-30 dB SPL A: Evoked OAE's - Not detectable in patients with a hearing loss >55 dB; 3 subtypes... A: Transient evoked OAE - Evoked by a click or pip ~4-15 ms after presentation of acoustic stimulus, contains multiple frequencies, do not occur where a Cochlear HL >40 dB exists; Most sensitive measure of hearing loss in children A: Distortion product OAE - Evoked by two simultaneously presented pure tones separated by a few hundred Hz; interaction produces a frequency that is lower than either of two frequencies (cubic distortion tone, 2f1-f2), do not occur where a Cochlear HL >50 dB exists; Sensitive test for cochlear function A: Stimulus frequency OAE - Evoked by a continuous pure tone stimulus, produce a continuous tonal emission of same frequency 3: Pasha also mentions Sustained frequency OAE, may be the same as Stimulus frequency?

Three elements of EMG analysis

A: Spontaneous activity - Present or absent A: Recruitment - Normal, reduced, or absent A: Motor unit morphology - Normal/bi/triphasic, low amplitude, polyphasic, fibillation

Name 9 tests done as part of a normal ENG battery

A: Spontaneous and Gaze nystagmus A: Positional testing A: Positioning testing (Dix-Hallpike) A: Caloric testing A: Fistula test A: Fixation suppression A: Saccade testing A: Smooth pursuit testing A: Optokinetic testing (moving target involving ≥90% of visual field) 3: 5 Vestibular subtests & 4 Oculomotor subtests; Calibration at beginning and throughout procedure

Boerhaave syndrome

A: Spontaneous esophageal rupture due to vomiting, coughing, heavy lifting or straining A: Typically left sided & transmural A: Symptoms typical for mediastinitis A: Treatment by surgical drainage of mediastinum/pleura, repair of perforation, nutritional support

Five clinical signs of acute unilateral vestibular loss

A: Spontaneous nystagmus - Fast phase towards contralateral ear A: Decreased VOR Gain - Corrective saccade on Head Thrust test A: Increased VOR Phase Lead at low frequency - Loss of velocity storage A: Ocular Tilt Reaction - Vertical skew deviation, head rotated towards weak labyrinth, ocular counter roll with superior polls rotated towards weak labyrinth A: Disturbance of gait - Fall towards hypoactive labyrinth

Six ENG test findings suggestive of a Central disorder

A: Spontaneous or positional nystagmus with normal caloric results A: Direction-changing nystagmus independent of stimulus changes A: Failure of fixation suppression A: Abnormal saccade, smooth pursuit, or optokinetic results, especially with normal caloric results A: Bilateral reduced or absent caloric responses without a history of labyrinthine or middle ear disease A: Hyperactive caloric responses (loss of cerebellum-generated inhibition) 3: Other characteristics of Central nystagmus are vertical beating (but can be any direction), no latency, non-fatiguing, and less associated N/V

Natural history of RRP

A: Spontaneous regression A: Not associated with puberty

Beckwith-Wiedemann syndrome

A: Sporadic occurrence A: "Overall growth syndrome", Macroglossia, Omphalocele, Visceromegaly, Cytomegaly of adrenal cortex A: Macroglossia may cause airway obstruction or chronic alveolar hypoventilation

Surgical options for Nasal Valve Obstruction

A: Spreader grafts A: Baten grafts A: Nasal-orbital suspension A: Valvuloplasty with composite graft A: Columellar strut graft (Bailey p. 331) A: Address other components of the valve - Septum, Turbinate 3: Obstruction may be fixed or dynamic

Histology of Thyroglossal duct epithelium

A: Squamous A: Respiratory A: Thyroid follicles and colloid

Mechanism of action and 2 examples of Taxane chemotherapeutic agents

A: Stabilize Tubulin Polymers & prevent progression of mitosis/cell division, cause cell cycle arrest in G2 3: Paclitaxel, Docitaxel

Kupferberg mucosal staging system for AFS

A: Stage 0 - No edema or Allergic Mucin A: Stage I - Edema A: Stage II - Polypoid edema A: Stage III - Polyps 3: Suffix A - Without Allergic Mucin; Suffix B - With Allergic Mucin

Three conditions for Mohs excision of lip cancer

A: Stage 1 or 2 disease A: Thickness ≤2.5 mm A: No muscle involvement 3: Beyond this, there will be regional mets, or there will be functional impairment

Current protocol for treatment of Mandibular ORN according to Marx

A: Stage I (Alveolar bone exposed) - Perform 30 HBO dives (1 dive per day, Monday-Friday) to 2.4 atmospheres for 90 Minutes; reassess patient to evaluate decreased bone exposure, granulation tissue covering, exposed bone, resorption of nonviable bone, and absence of inflammation; patients who respond favorably, continue treatment to a total of 40 dives A: Stage II (Does not respond to HBO) - Perform transoral sequestrectomy with primary wound closure followed by continued HBO to a total of 40 dives A: Stage III (Full-thickness ORN or pathologic fracture) - Perform transcutaneous mandibular resection, wound closure, and mandibular fixation with an external fixator or maxillomandibular fixation, followed by an additional 10 postoperative HBO dives A: Stage IIIR - Perform mandibular reconstruction 10 weeks after successful resolution of mandibular ORN; complete 10 additional postoperative HBO dives

McCaffrey system classifies laryngotracheal stenosis

A: Stage I - confined to the subglottis or trachea, <1 cm long A: Stage II - isolated to the subglottis, >1 cm long A: Stage III - subglottic/tracheal lesions not involving the glottis A: Stage IV - lesions involve the glottis

Most common infectious organism in pediatric bacterial cervical lymphadenitis

A: Staphylococcus aureus A: Streptococcus pyogenes

Bacteriology of pediatric Acute Sialadenitis

A: Staphylococcus aureus A: Streptococcus viridans A: Streptococcus pneumoniae A: Streptococcus micros A: Esherichia coli A: Bacteroides melaninogenicus

Seven Special tests for Pseudohypoacusis (Malingering)

A: Stenger test - Based on the Stenger effect, that when 2 tones of same frequency are presented simultaneously to both ears, the tone is heard by the better hearing ear A: Lombard test - Based on phenomenon that one increases the volume of one's voice in the presence of background noise, noise interferes with self-monitoring A: Lee's Delayed auditory feedback - Patient is asked to read aloud while listening to own recorded speech at a delay that would cause stuttering A: Doerffler-Stewart test A: OAE - Presence indicates that hearing loss can be no worse than 40 dB HL A: ABR - Objective test A: Bekesy audiometry - Type V tracing 3: Always check your hardware, reeducate patient

Definitive management for grade 3/4 Caustic Injuries

A: Stenting with NGT if risk of stricture (serial esophageal dilatation for the treatment of strictures) A: Thoracotomy for mural exam if difficult to determine transmural injury A: Early esophagectomy (blunt vs. thoracotomy) with reconstruction A: Esophagectomy/gastrectomy with exploratory laparotomy to remove necrotic tissue

Depolarization of the Vestibular Hair Cells occurs when

A: Stereocilia bend Toward the Kinocilium A: Results in increased vestibular neuronal firing rate

Name the four muscles controlling laryngeal depression

A: Sternohyoid A: Sternothyroid A: Thyrohyoid A: Omohyoid 3: All innervated by Ansa cervicalis (C1-3)

List five syndromes associated with Cleft Palate (STP ADD)

A: Stickler A: Treacher-Collins A: Pierre-Robin sequence A: Apert A: Deletion 22q11 - Velocardiofacial A: Down

JCIH Position Statement 1994 for infant (29d-2y) hearing screening (SPIT MORE)

A: Stigmata or other findings associated with a syndrome known to include SNHL A: Parent/caregiver concern regarding hearing, speech, language, or developmental delay A: Infections associated with SNHL (especially meningitis) A: Head Trauma with associated LOC or skull fracture A: Bacterial Meningitis A: Ototoxic medications A: Recurrent or persistent OME for at least 3 months

Discuss and name 3 expectorants

A: Stimulates secretions in respiratory tract via vagus A: Ammonium salts, Iodide salts, Guaifenesin

Mention 3 limitations in ENG excluding technical ones

A: Stimulus is limited to the Horizontal SCC (can miss Inferior vestibular nerve lesion) A: Low stimulus frequency on caloric stimulation (0.002-0.004 Hz) A: Results depend on patient's Alertness

Eight things to do with a tube fire during laser laryngoscopy?

A: Stop ALL gases including oxygen A: Remove damaged ET tube A: Douse the fire with saline A: Mask ventilate with 100% O2 A: Reintubate with patient paralyzed to facilitate reintubation A: Intraoperative bronchoscopy to remove any charred debris and assess extent of damage A: Intravenous steroids and antibiotics A: Delayed extubation and high humidity oxygen A: Reexamination of subglottis and trachea in 3-5 days to assess extent of further airway compromise 3: "Eliminate, extinguish, extract, and evaluate"

Management strategy for Subjective Tinnitus

A: Stop Caffeine, Chocolate, Smoking, Drugs (especially Aspirin) A: Patient Education - Tinnitus is Real, has a physical basis; may be Permanent, Reaction to tinnitus not tinnitus itself creates a problem; reaction is Manageable & can be Modified A: Patient Counselling - Emotional support, realistic understanding of tinnitus, attitude to pursue helpful activities, battery of tactics & strategies A: Masking - Hearing aids (25% success), Masker (33% success, for normal hearing patients), Tinnitus instrument (55% success) A: Stress Management - Habituation/Tinnitus retraining, Cognitive behavior therapy A: Medications - Alprazolam, Nortriptyline, Gabapentin, Melatonin A: Transcranial Magnetic Stimulation - 50% success A: Surgery - When associated with a condition (Otosclerosis, Meniere's, Acoustic neuroma, Glomus), tinnitus improves in ~50%; Auditory Nerve Section specifically for tinnitus will make it worse in 50%

Precautions for patients on Metformin receiving IV contrast

A: Stop Metformin for 48 hrs post-CT A: Check renal function prior to resumption 3: Causes ARF with lactic acidosis

Opera singer with Intracordal Hematoma and performing in 8 days; what are your recommendations?

A: Stop any anticoagulants A: Humidity & Hydration A: Voice rest A: Antireflux A: Reassess in a week prior to next performance

Most common cause of Bacterial Meningitis, and incidence of hearing loss

A: Strep pneumo (H.flu previous to Hib vaccine) A: 20%

Bacteriology of AOM

A: Streptococcus pneumonia A: Heamophilus influenza A: Moraxella catarrhalis A: Streptococcus pyogenes 3: Far less common are S. Aureus and GNBs

Bacteriology of Acute Mastoiditis not due to cholesteatoma

A: Streptococcus pneumonia A: Heamophilus influenza A: Streptococcus pyogenes A: Coag negative Staph A: Staphylococcus aureus (#3?) 3: Pseudomonas #2 in Bailey (in cholesteatoma?)

Etiologic agents in Acute Sinusitis

A: Streptococcus pneumonia A: Hemophylous influenzae A: Moraxella catarhallis A: Streptococcus pyogenes A: Peptostreptococcus A: Fusobacterium A: Bacteroides

Bacteriology of acute pediatric sinusitis

A: Streptococcus pneumonia A: Moraxella catarrhalis A: Haemphilus influenzae

Pathogen causing unilateral, bulging TM, T >100.6°

A: Streptococcus pneumoniae

Pathogen causing bullous myringitis

A: Streptococcus pneumoniae 3: Others include Mycoplasma, H. flu, Beta-hemolytic strep, M. catarrhalis, Parainfluenza & influenza virus???

Pathogen causing spontaneous perforation of TM

A: Streptococcus pyogenes

Bacteriology of intracranial Abscess

A: Streptococcus sp. (pneumonia, pyogenes) A: Staphylococcus sp. A: Proteus sp. A: Anaerobes (Peptococcus, Peptostreptococcus, B. fragilis)

What organisms are most commonly seen in odontogenic oral infections?

A: Streptococcus x 3 (Aerobic species, Microaerophilic, Peptostrep) A: Fusobacterium A: Eubacterium A: Veionella A: Eikenella A: Bacteroides 3: "Strep, **** You Vile Icky Bacteria!"

Fourteen precipitating factors in Migraine

A: Stress related - Stress, Letdown period after stress, Irregular sleep A: Hormone related - Menses, Pregnancy, OCP A: Head & neck related - Infection, Trauma, Surgery A: Diet related - Irregular diet, Dairy products, Red wine, Nuts, Shellfish, Caffeine withdrawal A: Irritant related - Perfumes, Strong odors, Bright sunlight, Flickering lights A: Vasodilating Medications

10 complications of caustic ingestion

A: Stricture formation (most common) A: Tracheoesophageal fistula A: Pneumonia A: Esophageal perforation A: Mediastinitis A: Gastric perforation A: Peritonitis A: Sepsis A: Death A: Schatzki's ring (late) A: Complete stenosis (late) A: Barrett's esophagus (late) A: Esophageal carcinoma (late)

10 Signs of airway obstruction

A: Stridor A: Dyspnea, tachypnea A: Tachycardia A: Diaphoresis, circumoral pallor, anxiety/restlessness A: Retractions - tracheal tug, suprasternal, intercostal, substernal A: Flaring of nasal alae, A: Use of accessory respiratory muscles A: Cyanosis, in extreme cases A: Respiratory arrest

Discuss Immunoglobulins

A: Structure - 2 Light chains (2 types = kappa & lambda), 2 Heavy chains (5 types, determine class of antibody = M, A, G, E, D) A: IgM - Predominates in early immune response; Pentameric, IgM & IgD main Ig's expressed on B-cell surface A: IgA - Predominates in bodily secretions; primary defense against local mucosal infection, dimeric with a secretory component bound to it A: IgG - 75% of total serum antibodies; 4 subtypes, crosses placenta to protect fetus, can fix complement A: IgE - Binds to mast cells & basophils, triggers inflammatory mediator release A: IgD - Found in serum; monomeric, main function unknown

Three muscles originating off the styloid and their innervations?

A: Stylohyoid (VII) A: Stylopharyngeus (IX) A: Styloglossus (XII) 3: Remember: "Glosso (pharyngeal n.) got Style (opharyngeus m.)!"

Contents of the parapharyngeal space

A: Styloid process, or the tensor-styloid fascia attaching the TVP to the styloid, divides space into two compartments A: Prestyloid - Fat, deep lobe of parotid, internal maxillary artery, & branches of V3 (lingual, inferior alveolar, auriculotemporal), LNs A: Poststyloid - CN's IX-XII, IJV, ICA, sympathetic chain

Inflammatory thyroiditides (SHARP)

A: Subacute/de Quervain's - Viral etiology, most common cause of painful thyroiditis A: Hashimoto's (autoimmune) - Most common; anti-TPO (90%) & anti-TG (50%) antibodies A: Acute suppurative A: Riedel's thyroiditis A: Painless/Silent thyrotoxic thyroiditis

Complication of laser of subglottic hemangioma

A: Subglottic stenosis

Six Methods of olfactory testing

A: Subjective odorant testing/Detection threshold - Sniffing sticks, 1 minute smell test A: Standardized psychophysical tests - OCM = Odorant Confusion Matrix; UPSIT = University of Pennsylvania Smell Identification Test, highly reliable A: Electrophysiologic tests - OERP = Odor event-related potentials, experimental, identifies brain EEG activity after odor presentation; far field potentials, cannot localize site of lesion; "EOG" = Electoolfactogram, electrode placed on olfactory epithelium identifies summated generator potentials of olfactory receptor neurons; no local anesthesia so uncomfortable; Flase positives in Kallman's and schizophrenia

Submandibular space, 7 contents

A: Submandibular gland A: Wharton's duct A: Facial artery A: Lingual nerves A: Hypoglossal nerves A: Fat A: Lymph nodes

Four ancillary procedures which assist in exposing the parapharyngeal space

A: Submandibular gland swing or excision (for transcervical) A: Separation of styloid structures (stylohyoid, styloglossus, stylopharyngeus, stylomandibular ligament) provides access to carotid at skull base A: Separation of sphenomandibular ligament A: Disarticulation of the TMJ

Typical presentation of mycobacterium lymphadenitis, most common organism, and treatment

A: Submandibular lymphadenopathy, most common in under 12, draining sinus if untreated, later calcifies A: M. avium intracellulare A: Surgical excision, can try Antibiotic triple therapy (Clarithromycin, Rifampin, & Ethambutol)

El Deeb's predisposing factors for Submandibular Gland calculi formation

A: Submandibular saliva (6) Alkaline pH High mucin content Higher percentage of organic matter High concentration of calcium & phosphate salts High phosphatase enzyme content Low carbon dioxide level A: Anatomy (4) Length & irregular course of Wharton's duct Dependent position of gland & duct system Position of ductal orifice Size of orifice smaller than duct lumen

Primary oral cavity spaces in infections

A: Submental A: Sublingual (anterior teeth, bicuspids, 1st molar) A: Submandibular (2nd, 3rd molars) 3: Site of infection determined by level of tooth roots in relation to mylohyoid line

Five indications for the professional voice user to cancel a concert

A: Submucosal Hemorrhage A: Enlarging vocal cord Varix A: Break in vocal fold mucosa A: Significant systemic Illness A: Severe laryngitis

Four Irreversible surgical procedures for Chronic Aspiration

A: Subperichondrial cricoidectomy A: Glottic closure A: Narrow field laryngectomy A: Total laryngectomy

Three surgical options for parathyroid hyperplasia

A: Subtotal parathyroidectomy - preferred if patient doesn't have MEN or renal failure A: Total parathyroidectomy with autotransplantation - CRF A: Total parathyroidectomy with cryopreservation of 1 gland - CRF or MEN

Ddx of a Sloping (High frequency) Hearing Loss (SOAP)

A: Sudden SNHL A: Ototoxicity (Cisplatinum, Aminoglycosides) A: Acoustic Neuroma A: Presbycusis (Sensory & Cochlear Conductive types) A: Diabetes A: Herpes Zoster Oticus A: MS A: Vascular loop

Four most common drugs associated with Toxic Epidermal Necrolysis

A: Sulfonamides A: Barbiturates A: Allopurinol A: Phenytoin 3: "Skin Bubbling And Peeling"

Four risk factors for lip cancer

A: Sun exposure A: Smoking A: Chronic alcoholism A: Poor dental hygiene

What are the Etiologic risk factors for BCC and cSCC? (Sun GRASSHOPEr)

A: Sun exposure (UVB 290-320 nm > UVA 320-400 nm) A: Genetic factors: Xeroderma pigmentosum, AD Nevoid BCC syndrome (Gorlin's Syndrome), B-K Mole syndrome, Atypical mole syndrome, p16 mutation A: Radiodermatitis from previous radiotherapy A: Actinic Keratosis A: Scar from former injury (Marjolin's ulcers) A: Suppressed immunity (iatrogenic or hemopoietic cancers) A: HPV 16 A: Occupation - Ranchers, farmers, sailors, fishermen (high sun exposure) A: Physical traits - Pale complexion, light hair, blue or green eyes, inability to tan, propensity for sunburn, history of multiple or severe sunburn, Celtic ancestry A: Environmental exposure - Arsenic induced BCC, soot, coal tar, paraffin oil, petroleum oil, asphalt

Proper surgical management of Type II First Branchial arch abnormality

A: Superficial parotidectomy A: CN VII identification A: Trace track back to origin (typically the bony/cartilaginous junction of EAC) A: Cartilage resection 3: Relation to CN VII variable

Six types of Melanoma, best prognosis, most common, etc.

A: Superficial spreading - 75%, Intermediate prognosis, late invasion A: Nodular - 15-20%, Worst prognosis due to vertical growth, early invasion A: Lentigo Maligna Melanoma - 4-10%, Best prognosis, only 30% ever metastasize A: Desmoplastic/Neurotropic - <1% of all cases, but 75% occur in the head & neck, amelanotic A: Mucosal - ~2% of all H&N melanoma, 50% arise in nasal cavity A: Metastatic with unknown primary 3: Head & neck melanoma ~20% of total incidence

Review the layers of the deep cervical fascia

A: Superficial/investing - from nuchal line & mandible to the clavicles; splits to form suprasternal space of Burns Muscles - envelops trapezius, SCM, and muscle of mastication (masseter, temporalis, and pterygoids) Bones - attaches to hyoid, mandible, and zygomatic arches Glands - Parotid & SMG A: Middle Muscular layer - surrounds strap muscles Visceral layer - Pretracheal fascia, surrounds larynx, trachea, pharynx, esophagus, thyroid and parathryroid; Buccopharyngeal fascia forms midline raphe adherent to prevertebral fascia, and pterygomandibular raphe in the lateral pharynx, superiorly ensheathes constrictors & buccinator muscle A: Deep Prevertebral - from skull base to coccyx, fuses with transverse processes Alar - lies between prevertebral & visceral layer of middle cervical fascia; extends from skull base to T2 level where it fuses with visceral layer (to end the retropharyngeal space at the carina) and continues down to level of the diaphragm where it fuses with the prevertebral fascia and ends the danger space

Classification of laryngeal (saccular) cysts

A: Superior (extending into ventricle) A: Posterior (extending into the AE folds and false cord)

Describe the borders of Level Va in the neck

A: Superior - Apex of convergence of the trapezius and SCM muscles A: Inferior - horizontal plane through inferior border of cricoid A: Medial - posterior border of SCM A: Lateral - anterior border of trapezius

Discuss the boundaries of the sella turcica/pituitary

A: Superior - Diaphragma sellae & optic chiasm A: Anterior - Tuberculum sellae A: Inferior - Sphenoid sinus A: Lateral - Cavernous sinuses A: Posterior - Dorsum sellae

Name the boundaries of the tympanic cavity

A: Superior - Tegmen A: Inferior - Jugular wall, Styloid prominence A: Medial - Labyrinth A: Lateral - TM, Scutum A: Anterior - Carotid wall, Eustachian tube, Tensor tympani S: Posterior - Mastoid, Stapedius, Pyramidal eminence (Sinus tympani?)

Describe the borders of Level Vb in the neck

A: Superior - horizontal plane through inferior border of cricoid A: Inferior - clavicle A: Medial - posterior border of SCM A: Lateral - anterior border of trapezius

Boundaries of the oropharynx

A: Superior - horizontal plane through the hard palate (and sulcus terminalis on tongue) A: Inferior - horizontal plane through the hyoid

Boundaries of the hypopharynx

A: Superior - horizontal plane through the hyoid A: Inferior - horizontal plane through inferior margin of cricoid

Boundaries of the Preepiglottic space

A: Superior - hyoepiglottic ligament and vallecular mucosa A: Anterior - thyrohyoid membrane and thyroid cartillage A: Posterior - epiglottis A: Inferior - thyroepiglottic ligament

Describe the borders of Level III in the neck

A: Superior - hyoid (clinical) or carotid bifurcation (anatomic) A: Inferior - lower border of the cricoid (clinical) or omohyoid over IJV (anatomic) A: Medial - lateral border of sternohyoid A: Lateral - posterior border of SCM

Describe the borders of Level VI (Prelaryngeal/Delphian, pretracheal, paratracheal) in the neck

A: Superior - hyoid bone A: Inferior - suprasternal notch A: Lateral - common carotids

Describe the borders of Level IV in the neck

A: Superior - lower border of the cricoid (clinical) or omohyoid crossing over IJV (anatomic) A: Inferior - clavicle A: Anterior - lateral border of sternohyoid A: Lateral - posterior border of SCM

Describe the borders of Level IIb (Submuscular recess) in the neck

A: Superior - skull base A: Inferior - hyoid (clinical) or carotid bifurcation (anat) A: Medial - lateral border of the sternohyoid muscle and the posterior belly of the digastric muscle and stylohyoid muscle A: Anterior/Lateral - plane defined by CN XI

Describe the borders of Level IIa in the neck

A: Superior - skull base A: Inferior - hyoid (clinical) or carotid bifurcation (anat) A: Posterior/Medial - plane defined by CN XI A: Lateral - posterior border of the SCM

Borders of parapharyngeal space and its connections

A: Superior - skull base along the sphenoid A: Inferior - lesser cornu of hyoid A: Lateral - fascia covering the medial pterygoid, ramus of the mandible, and deep surface of the parotid A: Medial - buccopharyngeal fascia covering the superior constrictor A: Anterior - pterygomandibular raphe A: Posterior - prevertebral fascia 3: Connects to paralingual, parotid, carotid, masticator, retropharyngeal, submandibular spaces

Describe the borders of Level Ia in the neck

A: Superior - symphysis A: Inferior - body of hyoid A: Lateral - anterior belly of ipsilateral digastric A: Medial - midline

Blood supply to superior & inferior parathyroid glands

A: Superior and inferior parathyroid arteries, originating from inferior thyroid artery (86% of time), some contribution from superior thyroid artery

Define Sinus/Recessus Terminalis

A: Superior boundary of the ethmoid infundibulum when the uncinate process terminates in the lamina papyracea (80%); the frontal recess drains medial to the uncinate process in this instance

Attachments of the pharyngeal constrictor muscles

A: Superior constrictor - Suspended from base of skull, medial pterygoid plate, Pterygomandibular raphe, Mylohyoid line, lateral tongue A: Middle constrictor - Stylohyoid ligament, Hyoid bone A: Inferior constrictor - Thyroid & Cricoid cartilages 3: All three muscles inserts with the corresponding opposite muscle in the midline at the pharyngeal raphe

Neurovascular supply of the Trapezius myocutaneous free flaps

A: Superior flap - Occipital artery, Paraspinous muscle perforators; only reliable choice following neck dissection A: Lateral island flap - Transverse cervical artery A: Lower island flap (Extended island) - Descending branches of the Transverse cervical artery and Dorsal Scapular artery

Anatomic associations between the middle & inferior constrictors (space closed by thyrohyoid membrane)

A: Superior laryngeal artery & vein A: Internal branch of the superior laryngeal nerve

Normal anatomic locations of the parathyroids

A: Superior pair - usually on deep surface of thyroid within 1cm of RLN entry at cricothyroid joint A: Inferior pair - usually within 1-2 cm of inferior thyroid artery entry into thyroid gland

Name the Seven voluntary muscles of the eye

A: Superior rectus A: Inferior rectus A: Lateral rectus A: Medial rectus A: Superior oblique A: Inferior oblique A: Levator palpebrae

Embryology of the Stapes

A: Superstructure & middle ear portion of footplate derived from Reichert's cartilage (2nd Arch) A: Otic portion of footplate and annular ligament derived from Otic Capsule A: Develops from mesenchyme in 6th week just before Malleus & Incus formation; Stapedial artery runs through it A: 8th week - IS joint forms A: 10th week - Stapedial artery degenerates, starts forming its adult shape A: 16th week - Ossification begins 3: The 2 Diarthrodial ossicular joints form in the 8th week, and Ossification begins from medial to lateral in the 16th week

Four techniques of voluntary control of swallowing

A: Supraglottic swallow - Close glottis, swallow followed by a cough & 2nd swallow before inspiration A: Super-supraglottic swallow - Take a breath and hold tightly while bearing down, swallow while still holding the breath, and then coughing immediately after the swallow; This technique can be used when there is reduced closure of the airway A: Mendelsohn maneuver - Voluntary elevation & anterior displacement of the larynx A: Effortful swallow - Muscle effort used during oral & pharyngeal phases

Interventions for severe laryngomalacia

A: Supraglottoplasty/epiglottoplasty - Unilateral or bilateral, division of aryepiglottic fold, partial epiglottis amputation, removal of redundant supra-arytenoid mucosa and lateral borders of epiglottis, removal of cuneiform & corniculate cartilages A: Epiglottopexy with glossoepiglottic adhesion A: Tracheostomy (rare)

Three surgical approaches to the Posterior pharyngeal wall

A: Suprahyoid pharyngotomy A: Lateral pharyngotomy A: Combined Suprahhoid and Lateral pharyngotomy

8 methods of reducing tension across the Tracheal Anastomosis suture line

A: Suprahyoid release A: Infrahyoid release A: Peritracheal mobilization A: Intercartilaginous incisions A: Perihilar release A: Dissection of the pulmonary vasculature A: Transplantation of the left mainstem bronchus A: Neck flexion with chin-to-chest suturing

One method of Nystagmus Suppression and 3 methods Enhancement

A: Supression - Fixation (peripheral) A: Enhancement - Gaze (Alexander's Law) A: Enhancement - Position (Otolithic) A: Enhancement - Fixation (Central)

Management of preoperative hyperthyroidism

A: Supressive therapy with thionamides (PTU, methimazole, carbimazole) used until patient normothyroid A: Iodides, lithium used 10-14 days prior to surgery once normothyroid to inhibit synthesis & release of hormone A: Β-blockers used 48-72 hours before surgery to suppress sympathetic manifestations of thyrotoxicosis

Classification of Obturators

A: Surgical A: Postsurgical A: Definitive

Alternative treatment to Botox for spasmodic dysphonia

A: Surgical Denervation of RLN +/- Reinnervation to ansa (crush, neurolysis)(SLAD-R) Also Type II thyroplasty and endoscopic laser thyroarytenoid myoneurectomy A: Speech therapy generally ineffective

Nine assessment tasks Using Stroboscopy

A: Sustained "ee" at normal pitch, normal loudness (NPNL) A: Quiet "ee" A: Loud "ee" A: Pitch glide from mid-range to high A: Sustaining the high note A: Pitch glide from mid-range to low A: Sustaining the low note A: Phonation on Inhalation A: "ee" at NPNL with locked phase A: Physical manipulation of the larynx or trial therapy (Unloading) also possible 3: Monitor the patient's voice so that all of the necessary information is recorded; Most vibratory parameters are rated from "ee" at NPNL.

Four Characteristics of Horizontal positional Nystagmus that suggest a central disturbance

A: Sustained, Large-Amplitude nystagmus that is present during visual Fixation A: Nystagmus in more than one head position A: Nystagmus associated with vertical (and especially downbeat) component A: Positional nystagmus associated with other neurologic signs or symptoms

Classification of Mandibular bony defects

A: Symphysis A: Lateral aspect of mandibular Body A: Ramus - Proximal segment includes condyle and coronoid process (coronoidectomy prevents superomedial pull by temporalis); Complete disarticulation of TMJ; or Oblique subcondylar osteotomy (Coronoid not part of proximal mandible)

Indications for surgery in euthyroid goiter

A: Symptoms - Compressive (airway, esophagus), recurrent painful hemorrhage A: Diagnosis - Inability to exclude cancer on FNA A: Cosmesis A: Substernal extension?

Ten Granulomatous diseases of the Sinonasal tract

A: Systemic (3) and Infectious (8) A: Sporotrichosis A: Blastomycosis A: Coccidiomycosis A: Rhinoscleroma A: Rhinosporidiosis A: TB A: Leprosy A: Syphilis A: Sarcoidosis A: Wegener's A: Histiocytosis X 3: "Sporo Blasto Cocci Rhino-Rhino, TB Lepy Syphi, Sarcoid, Wegener's & Histio"

Non-tuberculous vs tuberculous lymphadenopathy in Pediatrics

A: Systemic symptoms - Single large node with fevers and malaise in TB; Rare in NTB A: Lung disease - Often in TB; Rare in NTB A: Violaceous skin change in NTB A: PPD - Strongly positive in TB; weak or negative in NTB A: Treatment - Medical in TB; NTB often resistant to medical, therefore often require Surgical excision

Downs Peds patient with OSA and pulmonary hypertension: Two treatments

A: T and A A: Bronchoscopy (Rule out subglottic stenosis) 3: Use smaller ET tube in Down's patients

TNM pretreatment staging for rhabdomyosarcoma from IRS-IV

A: T staging T1 - Confined to anatomic site of origin T2 - Extension A ≤5cm in diameter B >5cm in diameter A: N staging N0 - Not clinically involved N1 - Clinically involved Nx - Clinical status unknown A: M staging M0 - No distant metastases M1 - Distant metastasis A: Stage groupings I - Head and Neck; T1 or T2, A or B; any N; M0 II - Parameningeal; T1 or T2, A; N0 or Nx; M0 III - Parameningeal; T1 or T2, B; any N; M0 IV - All sites; T1 or T2, A or B; any N; M1

Pathophysiology and Histopathology of Lichen Planus

A: T-cell mediated destruction of the basal cell layer A: Saw-tooth configuration of Rete Pegs A: Dense Lymphocytic Submucosal infiltrate A: Civatte bodies = degenerative eosinophilic basal keratinocytes

What is the 2001 AJCC TNM staging for melanoma?

A: T0 = Primary tumor cannot be assessed Tis = Melanoma within epidermis only (Clark's stage I) T1a = Invades papillary (II) or superficial reticular (III) dermis up to ≤1 mm, without ulceration T1b = Invades reticular (IV) dermis or subcutaneous tissue (V) up to ≤1 mm, with ulceration T2 = Invades 1.01-2 mm deep T3 = Invades 2.01-4 mm deep T4 = Invades >4.0 mm deep a = without ulceration b = with ulceration A: N0 = No regional LN involvement N1a = Single LN micrometastasis N1b = Single LN macrometastsis, +/- ECS N2a = 2-3 LN micrometastases N2b = 2-3 LN macrometastases N2c = 2-3 LNs with In-Transit Mets/Satellites N3 = 4+ LNs, Matted LNs, In-Transit Mets/Satellites with Metastatic LNs A: M0 = No distant metastasis M1a = Distant metastasis in Skin, Subcutaneous, or LNs beyond regional, with normal LDH M1b = Lung metastasis, with normal LDH M1c = All other Visceral mets, or Elevated LDH A: Staging is complicated

UCLA staging for Esthesioneuroblastoma

A: T1 - Involves Nasal/Paranasal sinuses (except sphenoid and most superior ethmoid cells) A: T2 - T1 plus Sphenoid, extends into Cribriform A: T3 - Extension to ACF/Orbit A: T4 - Extension to Brain

Biller staging for Esthesioneuroblastoma

A: T1 - Involves nasal/paranasal sinuses (except sphenoid) A: T2 - Extension into ACF or Orbit A: T3 - Extension into Brain, resectable with margin A: T4 - Unresectable

Tumor staging of Maxillary sinus cancer (UICC 2002)

A: T1 - Limited to antral Mucosa with no erosion or destruction of bone A: T2 - Bone erosion/destruction, includes extension to Hard Palate and/or Middle Meatus, except extension to posterior wall of maxillary sinus & pterygoid plates A: T3 - Invasion of any of following: Bone of Posterior maxillary sinus wall, Subcutaneous tissues, Floor or Medial wall of Orbit, Pterygoid fossa, Ethmoid sinuses A: T4a - Invasion of any of: Frontal sinus, Infratemporal fossa, Sphenoid sinus, Skin of cheek, Cribriform plate, Anterior Orbital contents, pterygoid pLates (FISSCAL) A: T4b - Invasion of Orbital Apex, Dura, Brain, MCF, CN's other than V2, Nasopharynx, Clivus

Krouse staging for Schneiderian papilloma

A: T1 - Limited to nasal cavity A: T2 - Extension ethmoids and/or to medial wall of maxillary sinus A: T3 - Extension to sup/lat/inf/ant/posterior wall of maxillary sinus and/or frontal or sphenoid sinus A: T4 - Extension outside the nasal cavity and sinuses and/or malignancy

Staging for Tumors of the Lateral Temporal Bone

A: T1 - Limited to the EAC without bony erosion or soft-tissue extension A: T2 - Limited EAC Bony erosion (not full-thickness) or limited Soft-tissue extension <0.5 cm A: T3 - Full thickness Bony erosion with limited Soft-tissue extension <0.5 cm, or involvement of Middle ear or Mastoid, or with Facial Paralysis A: T4 - Eroding the Medial wall of the Middle ear, Cochlea, Petrous apex, Carotid canal, Jugular bulb, Dura, or extensive Soft-tissue involvement >0.5 cm 3: Regional LN metastases are rare (10-15%), as are distant metastases, but both portend a poor prognosis

Tumor staging for Nasal cavity/Ethmoid sinus cancer (UICC 2002)

A: T1 - Restricted to 1 subsite of Nasal cavity or Ethmoid sinus, with or without bony invasion A: T2 - Involves 2 subsites in a single site, or extends to involve an adjacent site within the Nasoethmoidal complex, with or without bony invasion A: T3 - Extends to invade Floor of Medial wall of Orbit, Maxillary sinus, Palate, or Cribriform plate A: T4a - Invades any of following: Frontal sinuses, Infratemporal fossa, Sphenoid, Skin of nose or cheek, Anterior Orbital contents, Pterygoid plates (FISSAL) A: T4b - Invades of Orbital Apex, Dura, Brain, MCF, CN's other than V2, Nasopharynx, Clivus

TNM staging for lip & oral cavity cancer (UICC 6th edition)

A: T1 - Tumor 2cm or less in greatest dimension A: T2 - Tumor 2-4cm in greatest dimension A: T3 - Tumor >4cm in greatest dimension A: T4 (lip) - Tumor invades through Cortical bone, Inferior Alveolar nerve, Floor of mouth, or Skin A: T4a (oral cavity) - Tumor invades through Cortical bone, Deep/Extrinsic Muscles of tongue, Maxillary sinus, or Skin of face A: T4b (oral cavity) - Tumor invades Masticator space, Pterygoid plates, Skull base, or encases Carotid artery A: Seven poor prognostic indicators for lip cancer A: Cervical metastasis (below are RFs for occult regional mets, thus criteria for SND I-III) A: Size > 3 cm A: Thickness > 5 mm A: Mandibular invasion A: Perineural invasion A: Commissure lesion A: Recurrent tumors A: Poorly differentiated histology

TNM Staging for NPC (UICC 6th ed.)

A: T1 - Tumor confined to the nasopharynx T2 - Tumor extends to soft tissues of oropharynx or nasal fossa T2a - Without parapharyngeal extension T2b - With parapharyngeal extension T3 - Tumor invades bony structures and/or paranasal sinuses T4 - Tumor with intracranial extension and/or involvement of cranial nerves, infratemporal fossa, hypopharynx, orbit, masticator space A: NX - Regional nodes cannot be assessed N0 - No regional lymph node metastases N1 - Unilateral metastasis in lymph node(s), <6cm in greatest dimension, above supraclavicular fossa N2 - Bilateral metastasis in lymph node(s), <6cm in greatest dimension, above supraclavicular fossa N3a - Metastasis in lymph node(s) >6cm N3b - Metastasis in lymph node(s) extending to supraclavicular fossa

Staging for hypopharyngeal carcinoma (AJCC 1997)

A: T1 - Tumor limited to one subsite of hypopharynx AND <2 cm in greatest dimension A: T2 - Tumor involves more than one subsite of hypopharynx or an adjacent site, OR measures 2-4cm in greatest dimension without hemilarynx fixation A: T3 - Tumor >4cm in greatest dimension OR with hemilarynx fixation or esophageal involvement A: T4a - Tumor invades thyroid/cricoid cartilage, hyoid, central neck soft tissues, thyroid and/or esophagus A: T4b - Tumor invades carotid artery, prevertebral fascia/muscles, or mediastinum

T staging for Supraglottic carcinoma (UICC 2002)

A: T1 - Tumor limited to one subsite with normal vocal cord mobility T2 - Tumor invading more than one subsite of supraglottis or glottis or region outside supraglottis (BOT mucosa, vallecula, medial wall of pyriform sinus) without vocal cord fixation T3 - Tumor limited to larynx with vocal cord fixation, and/or invading postcricoid, paraglottic or preepiglottic spaces, and/or minor thyroid cartilage erosion T4a - Tumor invades beyond thyroid cartilage and/or tissues beyond larynx (soft tissues of neck, deep/extrinsic tongue muscles, strap muscles, thyroid, esophagus) T4b - Tumor invades prevertebral space, mediastinal structures, or encases carotid

Staging for Subglottic carcinoma (UICC 2002)

A: T1 - Tumor limited to the subglottis T2 - Tumor extending to the glottis with or without impaired vocal cord mobility T3 - Tumor limited to the larynx with vocal cord fixation T4a - Tumor invading cricoid or thyroid cartilages and/or invades tissues beyond the larynx (deep/extrinsic tongue muscles), trachea, strap muscles, thyroid, esophagus T4b - Tumor invades prevertebral space, mediastinal structures, or encases carotid

T Staging for Glottic carcinoma (UICC 2002)

A: T1 - Tumor limited to vocal cords (involving anterior or posterior commissure) with normal mobility T1a - Tumor limited to one cord T1b - Tumor involving both cords T2 - Tumor extending to supra- or subglottis with impaired vocal cord mobility T3 - Tumor limited to larynx with vocal cord fixation and/or invades preepiglottic/paraglottic/postcricoid space, and/or with minor thyroid cartilage invasion T4a - Tumor invading through thyroid cartilage, or invades tissues beyond larynx (trachea, soft tissues of neck, deep/extrinsic tongue muscles, strap muscles, thyroid, esophagus T4b - Tumor invades prevertebral space, mediastinal structures, encases carotid

T staging for oropharyngeal carcinoma (AJCC 1997)

A: T1 - Tumor measuring <2cm in greatest dimension A: T2 - Tumor measuring 2-4cm in greatest dimension A: T3 - Tumor measuring >4cm in greatest dimension A: T4 - Tumor invades adjacent structures (eg. pterygoids), mandible, hard palate, deep tongue muscle, larynx

Major salivary gland T staging (UICC 6th edition)

A: T1 = Tumor <2cm without extraparenchymal extension A: T2 = Tumor 2-4cm without extraparenchymal extension A: T3 = Tumor more than 4 cm and/or tumor with Extraparenchymal extension (clinical or macroscopic evidence of invasion of soft tissues or nerve, except T4) A: T4a = Tumor invades Skin, Mandible, Ear canal, Facial nerve A: T4b = Tumor invades Base of skull, Pterygoid plates, or encases Carotid artery

Surgical options for cancer of the Pyriform sinus

A: T1 lesions in high medial area - partial laryngopharyngectomy (could be endoscopic) A: T2+ lesions - usually require total laryngectomy, partial pharyngectomy if lateral extension present

Six Criteria for Supraglottic laryngectomy

A: T1, T2, some T3 lesions (preepiglottic space involvement or minimal medial piriform sinus involvement) A: Mobile vocal cords A: Anterior commissure uninvolved by cancer (>5 mm) A: Thyroid cartilage uninvolved by cancer A: Surmountable nodal metastasis A: General good health

Indications for SCPL-CHP (SCPL-CHEP for glottic Ca)

A: T2-T3 supraglottic carcinoma A: Floor of ventricle involvement A: Anterior commissure involvement A: Impaired vocal cord or arytenoids mobility A: Preepiglottic invasion A: Paraglottic space invasion below the glottic level A: Selected T4 supraglottic carcinoma A: Transglottic glottic carcinoma A: Radiation failure for glottic and supraglottic carcinoma

Three contraindications for laser microlaryngeal excision of early glottic carcinoma

A: T3 glottic tumor A: Posterior commissure involvement A: Subglottic extension

General Indications for Postoperative Radiotherapy

A: T3 or T4 lesion A: Perineural or vascular invasion A: Extracapsular spread A: Multiple histologicaly positive nodes (N2b, 2c) A: Positive resection margins (considered gross residual disease, >106 cells)

Seven indications for postop XRT for lip cancer

A: T3 or T4 tumor A: Recurrent tumor A: Positive margins A: Perineural invasion A: Multiple level cervical mets A: >2 LNs positive A: Extracapsular spread

Ddx of Pediatric Midline neck mass

A: TGDC A: Dermoid A: Teratoma A: Plunging ranula A: Thymic cyst A: Hemagioma A: Lymphatic malformation

Describe Austin's Classification of middle ear conductive apparatus lesions and associated hearing loss

A: TM perforation - Loss proportional to Size of perforation A: TM perforation with ossicular interruption - ~40 dB loss A: Total loss of TM and Ossicular chain - 50 dB loss A: Ossicular interruption with intact TM - 50-60 dB loss A: Ossicular interruption with intact TM and closure of oval window - 50-60 dB loss

Define Erythema multiforme

A: Target lesions caused by deposition of Ag-Ab complexes in small vessels of dermis & submucosa A: Severe form is Stevens-Johnson syndrome with systemic involvement of GI, genitals and eyes 3: Continuum = EM Minor, EM Major, SJS, Toxic Epidermal Necrolysis

An elderly diabetic is being treated for a malignant otitis externa; Which radionucleotide tests do you use to monitor the disease process?

A: Technetium99 scan - Used to identify presence of acute or chronic Osteomyelitis, identifies increased Osteogenic activity; stays positive for prolonged period & Lags behind gallium for Months A: Gallium67 scan - Incorporated into proteins & Leukocytes during Acute inflammatory process, used to Follow Resolution of the disease, will fade as disease process settles A: WBC Indium 111 - Best for following treatment progress, has largerly replaced Ga 67

Five pathways of spread to the orbit by dental infections

A: Teeth infect maxillary sinus, which can track posterior to infratemporal fossa/pterygomaxillary fossa to inferior orbital fissure A: Teeth infect maxillary sinus, which can track into ethmoids, then through lamina papyracea A: Venous spread along facial, angular & ophthalmic arteries A: Lymphatic seeding A: Traumatic/congenital dehiscences

Most common Sites of Middle Ear CSF Leak

A: Tegmen A: Fallopian canal A: Hyrtl's fissure

Name the 5 main branches of the extratemporal facial nerve

A: Temporal A: Zygomatic A: Buccal A: Marginal mandibular A: Cervical

Facial muscles supplied by CN VII

A: Temporal branch (4) - Occipitofrontalis, Orbicularis oculi, Corrugator supercilii, Anterior & superior Auriculares A: Zygomatic branch - Zygomaticus Major & Minor, Procerus, Levator labii superioris & Alaeque nasi, Nasalis, Depressor septi nasi, Orbicularis oris A: Buccal branch (3) - Buccinator, Orbicularis oris, Risorius A: Marginal mandibular branch (4) - Depressor anguli oris, Orbicularis oris, Depressor labii inferioris, Mentalis A: Cervical branch (1) - Platysma

Four Options for Dynamic Muscle Slings in Facial Paralysis

A: Temporalis (best vector forces) A: Masseter A: SCM A: Digastric

Brachytherapy time frames of use

A: Temporary - long lived isotopes used (RIC = Radium, Iridium, Cesium) A: Permanent - short lived isotopes used (PIG = Palladium, Iodine, Gold)

Five Pathologic conditions that Raise fundamental frequency

A: Tension-producing vocal fold masses A: Muscle tension phenomena A: Aging men A: Parkinson's disease A: Puberphonia

Five measures in the Prevention of tracheal stomal Stenosis

A: Tensionless anastomosis A: Cut the medial heads of the SCM A: Bevelled tracheal cuts, yet leave entire tracheal ring intact at anastamosis A: Use of a laryngectomy tracheostomal tube A: Bilaterally suture outer tracheal wall to clavicular periosteum 3: Infection & radiation predispose to stenosis

Five Anatomic structures running between the skull base & superior constrictor

A: Tensor veli palatini A: Levator veli palatini A: Eustachian tube A: Ascending pharyngeal artery A: Ascending palatine artery

Four muscles related to the eustachian tube

A: Tensor veli palatini/dilator tubae (CN V) A: Levator veli palatini (CN X) A: Salpingopharyngeus (originates off torus tubarius, interdigitates with Palatopharyngeus) (CNX) A: Tensor tympani (CN V)

Ddx of a Congenital Tonsillar mass (3)

A: Teratoma A: Hemangioma A: Lymphatic malformation

Blood supply to the Stapedius tendon

A: Terminal branches of anastomosis of Superficial Petrosal and Stylomastoid arteries supplying facial nerve

Seven Audiological findings suggestive of Pseudohypoacusis (Malingering)

A: Test retest reliability of pure tone thresholds should be within 5 dB A: Disagreement between SRT and PTA (should be normally within 10 dB) A: Bone conduction thresholds Worse than Air Conduction thresholds A: Good SDS at or near SRT A: Nonresponse by patient in Unmasked air conduction stimuli with suspected Unilateral hearing loss, indicating an Absence of crossover hearing (shadow curve, occurs at 0 dB BC, 40 dB AC) A: Presence of Acoustic Reflexes with audiometric Air-Bone Gaps

Six diagnostic criteria for Allergic Fungal Sinusitis (Bent & Kuhn)

A: Testing or history positive for fungal atopy (type I hypersensitivity) A: Nasal polyps A: Eosinophil mucin with Noninvasive fungal hyphae (Identification of fungus by stain, culture, ELISA, PCR) A: CT scan with hyperdense material in sinus cavity and possible sinus wall erosion or expansion of the sinus cavity A: Immunocompetence

Name the most important landmark for performing and Endoscopic Sphenopalatine artery ligation

A: The Crista Ethmoidalis, part of the perpendicular process of the Palatine bone

The new JCIH 2007 Position Statement was released in early October 2007.18 The following is a list of some of the significant changes from the previous statement:

A: The definition of targeted hearing loss was expanded to include neural hearing loss (eg. auditory neuropathy/dyssynchrony). A: Separate protocols are recommended for neonatal intensive care units (NICUs) and well-baby nurseries. Auditory brainstem response screenings are recommended for all NICU babies, as well as babies admitted for greater than 5 days, so that neural hearing loss will not be missed. A: Referrals should be made directly to an audiologist for comprehensive testing to include diagnostic auditory brainstem response (ABR) for all infants who do not pass ABR screening in the NICU. A: Rescreening of all infants should include re-evaluation of both ears, even if the infant only failed one ear in the initial screening. A: Audiologists with expertise in evaluating newborns should conduct diagnostic evaluations. A: Children identified with hearing loss should be fit with amplification within 1 month of diagnosis. A: A genetics consultation should be offered to families of infants diagnosed with hearing loss. A: All children identified with hearing loss should undergo an evaluation by an otolaryngologist and have at least one examination to assess visual acuity with a pediatric ophthalmologist. A: All children with any degree of bilateral or unilateral hearing loss should be considered eligible for early intervention services. A: Families should be made aware of all communication options and available hearing technologies (presented in an unbiased manner). A: Early intervention services should be provided by professionals with expertise in hearing loss.

Describe Ewald's First Law

A: The direction of a peripheral nystagmus is parallel to the semicircular canal being stimulated and opposite to the direction of endolymph flow A: Horizontal canal ampulopetal endolymph flow causes maximal stimulation A: Posterior/superior canal ampulofugal endolymph flow causes maximal stimulation

Discuss the Head Shake test

A: The examiner passively rotates the subject's head horizontally at 1-2 Hz for 10-20 cycles of rotation; Once the rotation stops, the eyes are observed under Frenzel lenses in order to prevent visual suppression of the nystagmus A: Normally, the velocity storage mechanism is charged equally on both sides, and there is no post-rotatory nystagmus as the stored velocities decay at the same rate on either side A: With unilateral (uncompensated) vestibular hypofunction, nystagmus does occur after head shaking; illusory continued rotation toward the intact side results in nystagmus with slow phases go toward the lesioned side, and the fast phases toward the intact side

Five Conditions that must be met for ethical research

A: The existence of clinical equipoise A: Good design A: Sound foundation of previous work must give good reason to believe that the proposed interventions will achieve the intended result A: Research interventions are not likely to cause serious harm to the research subjects A: Prior approval by a Research Ethics Board

What is the Masking Dilemma?

A: The fact that bilateral air-bone gaps of 50 dB cannot be masked

Define Genotype

A: The genetic constitution of a locus

Describe Alexander's Law

A: The intensity of pathologic vestibular nystagmus (slow phase component) increases during gaze in the direction of the fast phase and decreases during gaze in the direction of the slow phase

Define Phenotype

A: The observable characteristics of an organism controlled by a specific genetic locus

Describe Interaural Attenuation

A: The reduction in sound energy traveling through the skull A: 40 and 65 dB for air conduction using Head phones and Inserts, respectively A: 0-10 dB for bone conduction

Define Glottal puff

A: The release of air as the upper margins of the vocal cords separate

Define Phase delay

A: The time difference between closure of the upper margin of the vocal cords compared with the lower

Medical management of Graves disease (PIPI)

A: Thionamides - Propylthiouracil, Methamizole/Tapazole (C/I in pregnancy), provides thyroid peroxidase substrate, decreases peripheral T4 to T3 conversion; liver side effects A: I-131 (RAI - radioactive iodine ablation) - most common, C/I in pregnancy A: Β-blockers (Propranolol) - Decreases sympathetic overdrive, used as an adjunctive treatment A: Iodide (Lugol's but more likely sodium iodide) - Wolf-Chaikoff effect: inhibits organification & thyroid hormone release, decreases vascularity; most useful in thyroid storm, C/I in R/A

Define Clinical equipoise

A: This exists when the expert medical opinion remains divided over the best choice among treatment options; the medical community is unable to achieve consensus on the preferred treatment

List the sites of Ototoxic activity for Macrolides

A: Thought to affect Strial function, but no definite mechanism known A: Reversible, Flat SNHL A: Erythromycin, Azithromycin

Diagnostic criteria for Relapsing Polychondritis

A: Three of the clinical features in the absence of histologic confirmation A: Two of these features with response to steroids or dapsone A: Any one of these features with histologic confirmation 3: Features include recurrent chondritis of the auricular, nasal, laryngeal or tracheal cartilages; seronegative nonerosive inflammatory polyarthritis; ocular inflammation; cochlear (SNHL or tinnitus) or vestibular damage (vertigo)

What are the clinical applications of ECoG?

A: Threshold testing A: Meniere's disease A: Auditory neuropathy/Acoustic neuroma diagnosis (mainly ANSD) A: Intraoperative nerve integrity monitoring (near-field technique) A: Audiometric evaluation prior to Aural Atresia repair if EAC is Patent

What are 5 clinical applications of ABR?

A: Threshold testing in hard to test individuals (comatose, malingering) A: Neonatal/infant hearing screening A: Intraoperative monitoring in hearing preservation surgery A: Diagnosis of Retrocochlear/Brainstem pathology A: Diagnosis of Auditory Neuropathy 3: Suspicion of Vestibular schwannoma (Unilateral Hearing loss/Tinnitus/Stapedial reflex abnormality, Discrimiation asymmetry, Disequilibrium and vertigo and hearing difficulty)

Four Antibodies that may be present in Hashimoto's thyroiditis

A: Thryoid peroxidase (TPO, key enzyme in organification) A: Thyroglobulin (TBG) A: Colloid antigen A: Thyroid hormone

Two Lab markers of recurrent WDTC

A: Thyroglobulin A: Anti-Thyroglobulin antibody

Name 8 histologic barriers to laryngeal carcinoma spread

A: Thyroid and cricoid cartilages with overlying perichondrium A: Conus elasticus A: Quadrangular membrane A: Ventricle A: Hyoepiglottic ligament A: Thyrohyoid membrane A: Cricothyroid membrane

Test prior to PSG for suspected sleep apnea

A: Thyroid function

Bifid epiglottis - what other test required

A: Thyroid function 3: Also associated with Pallister-Hall syndrome

Hurthle cell carcinoma management

A: Thyroidectomy A: Post op thyroid suppression (TSH receptors on tumor surface) A: Unresponsive to XRT or I-131

Eight Surgical/Iatrogenic causes of vocal cord paralysis (Bailey p.849)

A: Thyroidectomy A: Skull base surgery A: ACDF (R>L) A: CEA (Carotid Endarterectomy) A: CABG (also mitral valve repairs, PDA ligations etc) A: Esophagectomy A: Pulmonary resection A: Mediastinoscopy A: Intubation (pressure-induced neuropraxia from cuff in proximal subglottis)

Describe the Residual Inhibition Phenomenon

A: Tinnitus will subside for periods after masking exposure

Resection margins based on melanoma thickness

A: Tis = 0.5cm A: <1 mm = 1 cm A: 1-2 mm = 1-2 cm A: >2 mm = 2 cm

Six considerations to minimize scar tissue formation when repairing facial laceration

A: Tissue eversion A: Patient's propensity for keloid/hypertrophic scar A: Patient's natural complexion, eg. light vs. dark skin A: Minimal tension on incision line A: Clean wound, avoidance of infection A: Absorbable vs. Nonabsorbable sutures

What are the three classes of dentures used for dental rehabilitation post-mandibulectomy?

A: Tissue-borne prosthesis - Simplest, complete or partial, stabilized by remaining teeth and a broad denture supporting area A: Implant-assisted prosthesis - Osseointegrated posts help stabilize the dentures against lateral displacement forces with use; mores stable than conventional dentures, cheaper than implant-borne prosthetics A: Fixed implant-borne prosthesis - Most stable form of restoration, supported by 5-6 fixtures attached to transepithelial connector implants, denture does not come into contact with the underlying mucosa

Conversion of Morphine to Fentanyl patch

A: Total Daily Dose of Morphine PO (mg) divided by 2 = Fentanyl patch dose (mcg/h) 3: Morphine 50 mg PO = Fentanyl 25 mcg/h

Surgical option for cancer of the Postcricoid area

A: Total laryngectomy, partial pharyngectomy, and cervical esophagectomy

Three etiologies of SNHL in Otosclerosis

A: Toxic metabolites in Endolymph causing Neuroepithelial Damage A: Vascular compromise from narrowing and sclerosis of Vascular channels A: Direct extension of lesion into Inner Ear, affecting Basilar Membrane Biomechanics and Electrolyte concentrations

Ddx of Central Vestibular disorders (TV Is CNN)

A: Toxic/Metabolic/Medications A: Vascular Headache - Migraine with/without aura, Basilar Migraine, BPV of childhood, Tension, Cluster Vertebrobasilar insufficiency - 4 D's = Dizziness, Diplopia, Dysphasia, Drop attack Brainstem - Lateral medullary infarction (Wallenberg syndrome, PICA), Lateral pontomedullary syndrome (AICA) Subclavian Steal - Compression of the subclavian artery proximal to the vertebral artery causing occasional reversal of flow with resultant Vertigo, Blurred vision, H/A, Arm pain, Bruits, and Differential brachial BPs Cerebellar - Infarction, Hemorrhage Orthostatic hypotension Labyrinthine apoplexy Paroxysmal torticollis A: Infectious Meningitis Encephalitis Epidural abscess Congenital Syphilis A: Craniovertebral junction disorders Basilar impression (CRROOP) Assimilation of the Atlas - C1 fused to Skull, Odontoid compression, Klippel-Feil (wildervanck) Atlantoaxial dislocation - Odontoid compression due to laxity of the transverse ligaments; can be Congenital (Down, Achondroplasia), or Acquired (R/A, RPA, Griesel) Chiari malformation Syringobulbia A: Neurologic Multiple sclerosis Huntington Parkinson Progressive Supranuclear Palsy Multiple Systems Atrophy Pseudotumor cerebri (BIH) - proper name is idiopathic intracranial hypertension Vestibular epilepsy Cerebellar ataxia syndromes (PET SCARF) Multisensory disturbances Waardenburg Cogan (Autoimmune) A: Neoplastic - Primary or Secondary Brainstem Gliomas Aqueductal/Ventricle compressing neoplasms Cerebellar neoplasms 3: Central causes of dizziness are estimated to account for 10-20% of cases

Eight tracheal causes of stridor

A: Tracheomalacia A: Hemangioma A: Vascular compression A: TE fistula A: Tracheal stenosis A: Tracheal cyst A: Tracheal agenesis (with a TEF) A: Tracheal bronchus

Six Therapeutic options for Bilateral vocal cord paralysis

A: Tracheostomy A: Laser cordotomy - Transverse cut between vocal process and vocal fold A: Laser cordectomy A: Arytenoidectomy - Endolaryngeal (unilat or bilat, laser partial or total), Extralaryngeal (Woodman lateral approach, or Anterior thyrotomy approach) A: Arytenoidopexy - Plicating vocal process to external laryngeal framework (Woodman lateral approach, or Anterior thyrotomy approach) A: Reinnervation with Neuromuscular pedicle transposition to PCA A: Type II Isshiki thyroplasty (lateralization thyroplasty)

Three methods of palliation for a high Tracheal Adenoid Cystic carcinoma

A: Tracheostomy A: Laser debulking A: Endoscopically inserted stent

Treatment modalities for RRP

A: Tracheostomy - Increased risk of distal spread, avoid unless absolutely necessary A: Surgical debulking - Standard of care, remove as much disease as possible, preserve normal structures A: Adjuvant therapy - 10% of patients will require some form

Five methods are used to identify the facial nerve during parotidectomy?

A: Tragal pointer A: Tympanomastoid suture A: Retrograde dissection from a distal branch A: Identification within mastoid bone A: Digastric muscle attachment to the digastric groove

Ten Surgical approaches to the sphenoid sinus

A: Transantral A: Transpalatal - Ideal for tumors involving the nasopharynx, posterior pharyngeal wall, and choanae A: Transseptal - Sublabial, Intranasal, External rhinoplasty A: Transethmoidal - Internal, External A: Endoscopic - Transnasal, Transethmoidal, Transantral

In Speech Aerodynamic (Airflow) studies, give the normal values for Transglottal airflow rate, Glottal resistance, and Subglottal pressure

A: Transglottal airflow rate = 100 cc/sec A: Glottal resistance = 30-50 cmH2O/lps A: Subglottal pressure = 6-8 cmH2O

Two approaches to Aural Atresia repair

A: Transmastoid approach A: Anterior approach

Characteristics of the Nd-YAG laser

A: Transmits through clear fluids (eg. can be used in the bladder) A: Deep zone of coagulation (~4mm) A: Fiberoptics

Characteristics of the Argon laser

A: Transmits through clear fluids (eg. vitreous) A: Absorbed by Hgb, thus good for pigmented lesions

Define Kallmann's syndrome

A: Transmitted X-linked (Cummings) A: Endocrine anomalies - Diabetes; Hypogonadotropic hypogonadism (deficiency of hypothalamic GnRH secretion) A: Genito-urinary anomalies - Micropenis, Cryptorchidism, Unilateral renal agenesis A: Anosmia - Agenesis of the olfactory bulbs A: Deafness A: Midline facial deformities

Surgical approaches for management of oropharyngeal carcinoma

A: Transoral - Oral, Visor lip & cheek flap, Lateral lip & cheek flap, Mandibular lingual release A: Transpharyngeal - Suprahyoid, Subhyoid, or Lateral (high or low) pharyngotomy A: Transmandibular - Midline labiomandibular glossotomy (good only for small midline BOT lesions), Mandibulotomy/Mandibular swing (midline or lateral), Mandibulectomy (composite resection)

Five surgical approaches to the parapharyngeal space

A: Transoral - seldom used, poor access A: Transfacial/transparotid - total parotidectomy performed A: Cervical parotid A: Transcervical - submandibular incision A: Transmandibular - midline mandibular osteotomy with lateral swing 3: Other possibilities may include Infratemporal fossa, or Transcervical-transmastoid approaches?

Surgical Approaches to the Central compartment of the Middle Cranial Fossa

A: Transseptal sphenoid A: Transethmoidal sphenoid A: Lateral rhinotomy A: Transantral/ LeFort I osteotomy? A: Midfacial degloving A: Transpalatal A: Transoral A: Mandibulotomy A: Extended maxillotomy A: Midfacial split/Facial translocation? A: Infratemporal fossa (Fisch B?) 3: Line through medial pterygoid plate and occipital condyle on both sides

Four Poor Prognostic indicators in Traumatic CN VII Paralysis?

A: Transverse temporal bone fracture A: Immediate onset of paralysis A: Complete paralysis cf. paresis A: Penetrating mechanism of injury

Diseases causing Oral Ulcerations (TANINS)

A: Trauma A: Autoimmune Behcet's disease - Japanese populations (uveitis, oral & genital ulcers, SNHL) Reiter's syndrome (conjunctivitis, urethritis, oral ulcers, arthritis) Inflammatory bowel disease Celiac disease Erythema multiforme Pemphigus vulgaris Lichen Planus Selective IgA deficiency A: Nutritional Vitamin B12 deficiency Vitamin C deficiency (Scurvy) A: Infectious/Idiopathic Recurrent aphthous ulcers HSV stomatitis VZV stomatitis Herpangina (Coxsackie) EBV HIV Bacterial (primary or secondary) Syphilis Borelia Vincenti (Trenchmouth) Deep mycotic infection Candida A: Neoplasia Epidermoid cancers MSG cancers A: necrotizing Sialometaplasia

Ten causes of Septal Perforation

A: Trauma A: Surgery A: Cocaine abuse A: Septal hematoma/abscess A: HHT A: Wegener's A: Lymphoma A: Syphilis A: TB A: Chrome workers A: Bilateral AgNO3 cauterization

Etiology of CSF Otorhinorrhea

A: Traumatic Accidental (80%) - 90% will close spontaneously Surgical/iatrogenic (16%) A: Nontraumatic (4%) High pressure - Tumors, Hydrocephalus Normal pressure - Congenital, Spontaneous, Osteitis/Osteomyelitis

The 3 Pollen seasons for seasonal allergens

A: Trees - Spring A: Grass - Summer A: Weeds - Fall

Discuss Melkersson-Rosenthal syndrome

A: Triad of symptoms - Swollen face (defining feature), Recurrent Facial paralysis (50-90%, may be Bilateral), Fissured tongue (50%) A: Diagnosis - Lip Biopsy = Noncaseating epithelioid cell Granulomas surrounded by Histiocytes, Lymphocytes, and Plasma cells; High serum ACE in some cases

Management of Auditory Neuropathy

A: Trial of hearing amplification A: Cochlear implantation increasingly viewed as an option

Name 4 benign skin appendage tumors

A: Trichilemmomas - seen in Cowden's disease A: Sebaceous hyperplasia/Rosacea/Rhinophyma A: Syringomas - Intraepidermal eccrine duct adenomas A: Pilomatricomas - Follicular metrical cornification

Height divided into thirds, where are the horizontal lines

A: Trichion-Glabella A: Glabella-Subnasale A: Subnasale-Menton

Six Topognostic tests/Localization studies for Facial nerve paralysis

A: Trigeminal (blink) reflex - Abnormal if lesion proximal to Orbicularis Oculi muscle A: Salivary flow testing - Unilateral reduction >25% abnormal; Abnormal if lesion proximal to Chorda Tympani in Mastoid segments A: Electrogustometry/Taste test? - Abnormal if lesion proximal to Chorda Tympani in Mastoid segments A: Stapedial reflex - Abnormal if lesion proximal to stapedius branch in Tympanomastoid segments A: Lacrimation (Schirmer test) - Paper strips placed in conjunctival fornix, unilateral difference of >50% or a bilateral total <5 mm after 5 minutes significant; lesion proximal to GG and GSPN A: Magnetic Evoked Neuromyography/Transcranial Magnetic Stimulation (TMS) - Noninvasive excitation of Intracranial facial nerve close to the Internal Acoustic Meatus (proximal to or within site of lesion in Bell's palsy); more informative than ENoG within first 4 days post-injury Rappaport also included hearing & balance testing 3: High resolution CT and MRI have largely replaced these tests for localization

Three ENT disorders associated with Vascular Loops

A: Trigeminal neuralgia A: Hemifacial spasm A: SNHL, tinnitus, vertigo, and Ménière's disease (controversial) 3: Loops in the cerebellopontine angle or internal auditory canal have been proposed to be a cause

Embryology of the Tympanic Membrane

A: Trilaminar structure comprising 1st Arch Ectoderm (lateral epidermis), Mesoderm (fibrous layer) & Endoderm (medial mucosa) A: Tympanic ring starts ossification in 3rd gestational month A: Horizontal position initially, assumes vertical position by 3 years

Define Pott's abscess

A: Tuberculous osteomyelitis of the spine

Top 4 presenting Signs & Symptoms of Superior Semicircular Canal Dehiscence Syndrome (SCDS)

A: Tullio phenomenon (Sound evoked vertigo/nystagmus) = 95% A: Chronic Disequilibrium = 75% A: Hennebert's sign (Pressure evoked vertigo/nystagmus) = 55% A: Gaze evoked Vertigo/Nystagmus = 20% 3: Other include Autophony, CHL mainly at lower frequencies, Weber lateralizes to the affected ear, dysfunction of the affected SSC (with dehiscences >5 mm)

Three tumor Factors affecting Chemotherapy success

A: Tumor burden - Number of tumor cells A: Percentage of cells in a chemo-Responsive phase of cell cycle A: Number of cells with inherent or acquired Resistance to chemotherapeutic agent

5 methods of Targeting Vectors to a tissue/cell type

A: Tumor-specific Promoter - Activate gene expression in cancer cells (COX-2, hTERT, CEA, AFP, SLPI) A: Non-specific Promoter - Consitutively high level expression (CMV, SV40) A: Indirect, Receptor targeting - Ligand for a specific Receptor A: Indirect, Inverse targeting - Selective Inhibition of infectivity with specific Receptor A: Indirect, Protease targeting - Selective infectivity of cells with specific Protease

Define Heterozygosity

A: Two different alleles at an autosomal gene locus

Define Homozygosity

A: Two identical alleles at an autosomal gene locus

Describe the histology of the Olfactory Bulb

A: Two layers - Internal and External Plexiform layers A: Synapses form aggregates called Glomeruli A: Three cell types - Mitral, Tufted, and Granular cells A: Synapses are mostly GABAergic and Dopaminergic 3: "My Tough Granny lives in the olfactory bulb, Dopi Grama!"

Hinderer's method for malar eminence position

A: Two lines drawn, one from Lateral Canthus to lateral Commissure, other from Tragion to lateral Ala A: Malar prominence should be in upper lateral angle formed from the lines' intersection

Nine characteristics of an ideal free flap

A: Two team approach for resection and harvest of flap A: Innervation with sensation/motor function feasible A: Adequate length and caliber of vessels A: Donor site previously unviolated A: Minimal donor site morbidity A: Tissue composition similar to that of defect A: Bulk and color matched to that of defect A: Excellent cosmetic potential A: Potential for osseointegration

TNM (2002) staging for thyroid carcinoma (non-anaplastic histology)

A: Tx - Primary cannot be assessed T0 - No evidence of primary tumor T1 - <2cm, limited to thyroid T2 - 2-4cm, limited to thyroid T3 - >4cm, limited to thyroid or any tumor with minimal extrathyroid extension T4a - Extension beyond thyroid capsule & invades Subcutaneous soft tissues, Larynx, Trachea, Esophagus, RLN T4b - Invades Prevertebral fascia, Mediastinal vessels, or encases Carotid artery A: N0 - no regional LN metastasis N1a - Level VI metastasis N1b - Metastasis in other unilateral, bilateral or contralateral cervical or upper/superior mediastinal nodes A: M0 - No distant mets M1 - Distant mets present

Most common area of Iatrogenic Facial nerve Injury

A: Tympanic segment

Four facial nerve findings in middle ear Atresia

A: Tympanic segment dehiscence A: Inferior/Lateral displacement of the tympanic segment A: Anterior/Lateral displacement of the mastoid segment A: More acute angle taken at 2nd genu (60 degrees vs. 90-120 normally)

Most common areas of Facial nerve dehiscence

A: Tympanic segment over oval window A: Geniculate ganglion A: Adjacent to retrofacial air cells in mastoid region 3: 30%

Surgical approaches for Glomus Jugulare & Tympanicum

A: Tympanicum - Type I = Endaural resection; Type II, III, IV = Extended Facial Recess approach A: Jugulare - Fisch type A Infratemporal Fossa approach 3: Efficacy of preoperative Embolization disputed

Keros classification of lateral cribriform plate lamella length/olfactory fossa depth

A: Type 1 - Cribriform plate 1-3 mm below fovea A: Type 2 - Cribriform plate 4-7 mm below fovea A: Type 3 - Cribriform plate 8-16 mm below fovea (greatest risk of intraoperative CSF leak) A: Type 4 - Asymmetrical

Types of Esophageal rings

A: Type A - Several cm proximal to the squamo-columnar junction A: Type B - Schatski's rings, occur at the squamo-columnar junction A: Type C - Rare, occur due to indentation by the diaphragma crura 3: Webs are thinner and do not contain muscularis

Discuss the types of Osteogenesis Imperfecta

A: Type I (van der Hoeve-de-Kleyn syndrome) - AD, Mildest form; Nondeforming pediatric fractures, Normal stature, Blue sclerae, CHL A: Type II - AR, Most severe; Lethal in utero or shortly thereafter; Shortened long bones, Beaded ribs, Platyspondylisis, Calvarial demineralization A: Type III - Variable Inheritance; Progressive growth failure, Frequent fractures, Severe deformity, Dentinogenesis imperfecta, Grey sclerae, CHL or SNHL A: Type IV - AD; Intermediate severity between types I and III, White sclerae 3: Mutations in COL1A1 and COL1A2 genes, encoding collagen type 1; CHL associated with Blue sclerae, SNHL associated with Grey sclerae; Order of Severity is Type I, IV, III, II

Classification of Glottic Webs

A: Type I - <35% obstruction, true cords visable, little or no subglottic extension, airway OK, mildly abnormal cry A: Type II - 35-50% obstruction, Anterior glottis involved, thickened web that extends into Subglottis, cricoid normal, airway a problem only with URTI or post intubation, weak voice A: Type III - 50-75% obstruction, thick web with vocal cords not visible, cricoid abnormalites, marked airway and voice problems A: Type IV - 75-90% obstruction, web thick and vocal cords not visible, immediate airway management needed at birth

Describe the Draf classification for endoscopic frontal sinus procedures

A: Type I - Ethmoidectomy including septa in the region of the frontal recess, with the inferior recess left untouched; indicated for minor pathology, in patient without adverse prognostic risk factors (ASA intolerance, asthma, NP, etc.) A: Type IIa - Extended drainage with ethmoidectomy and resection of the floor of the frontal sinus between the lamina papyracea and the middle turbinate A: Type IIb - Extended drainage with ethmoidectomy and resection of the floor of the frontal sinus between the lamina papyracea and the nasal septum, anterior to the ventral margin of the olfactory fossa A: Type III - Endonasal median drainage, the extended IIb opening is enlarged by resecting portions of the superior nasal septum in the neighborhood of the frontal sinus floor (diameter should be ~1.5 cm), followed by resection of the frontal sinus septum; this results in an opening from lamina papyracea to contralateral lamina papyracea 3: The AP diameter of the frontal sinus should be ≥8 mm to undergo a Draf III, otherwise an osteoplastic flap should be considered

Gell and Coombs Type Hypersensitivity Reactions (ACID)

A: Type I - IgE immediate (Allergic rhinitis, anaphylaxis, asthma) A: Type II - Cytotoxic IgG or IgM antibody mediated A: Type III - Immune complex mediated, Ag-Ab-Complement (serum sickness, Arthus reaction) A: Type IV - Delayed type hypersensitivity, T-cell mediated A: Type V - Receptor-stimulating Ig (Pasha)

Describe the Gell and Coombs Type Hypersensitivity Reactions (ACID)

A: Type I - Immediate IgE (Allergic rhinitis, anaphylaxis, asthma) A: Type II - Cytotoxic IgG or IgM antibody mediated A: Type III - Immune complex mediated, Ag-Ab-Complement (serum sickness, Arthus reaction) A: Type IV - Delayed type hypersensitivity, T-cell mediated

Classification of Tympanoplasty (Wullenstein)

A: Type I - Myringoplasty (no ossicular reconstruction) A: Type II - Malleus eroded, grafted on to Malleus remnant or Incus A: Type III - Myringostapedopexy or PORP A: Type IV - Graft laid on stapes Footplate A: Type Va - LSSC fenestration A: Type Vb - Stapes fenestration

What is the Shamblin classification of Carotid Body tumors?

A: Type I - Not encasing the carotid A: Type II - Partially encasing the carotid A: Type III - Completely encasing the carotid

Classification of Musculovascular pedicles (5)

A: Type I - Single pedicle A: Type II - Single dominant pedicle in mid belly of muscle, several minor distally (platysma, SCM, trapezius) A: Type III - Two dominant pedicles (orbucularis oris, rectus abdominis) A: Type IV - Multiple similar size pedicles along the belly A: Type V - Single dominant pedicle and multiple secondary segmental pedicles (pectoralis major, latissimus dorsi)

Describe the Glasscock-Jackson Classification for Glomus Tympanicum

A: Type I - Small mass limited to the Promontory A: Type II - Mass Completely Filling Middle Ear space A: Type III - Tumor filling Middle Ear and extending to Mastoid A: Type IV - Tumor filling Middle ear, extending to Mastoid, and through TM to EAC; may also extend Anterior to ICA

Describe the Glasscock-Jackson Classification for Glomus Jugulare

A: Type I - Tumor involves Jugular Bulb, Middle Ear, and Mastoid A: Type II - Tumor extends under IAC, may have Intracranial involvement A: Type III - Tumor extends into Petrous Apex; may have Intracranial extension A: Type IV - Tumor extending beyond Petrous Apex into Clivus or Infratemporal fossa; may have Intracranial involvement

Describe the Sisson classification of stomal recurrence

A: Type I - Tumor involves superior ½ of the stoma without esophageal involvement A: Type II - Tumor involves superior ½ of the stoma with esophageal involvement, or the inferior ½ of the stoma A: Type III - Tumor involves the inferior ½ of the stoma and extends to the mediastinum A: Type IV - Tumor extends beneath the clavicle out laterally

Classification of laryngeal framework (Isshiki) procedures

A: Type I - Vocal cord Medialization A: Type II - Vocal cord Lateralization A: Type III - Vocal cord Shortening, Relaxing = Decreasing pitch A: Type IV - Vocal cord Lengthening, Tensing = Increasing pitch

Compare two types of Hair Cells found in the Ampullae (containing neuroepithelium known as the Cristae Ampullaris)

A: Type I hair cells - Flask shaped nerve cells, Chalice shaped nerve ending; One nerve ending can synapse with 1-4 hair cells A: Type II hair cells - Cylinder shaped nerve cells; Multiple efferent and afferent nerve fibers synapsing on a single hair cell

Six steps to be done preoperatively for a secreting Carotid body tumor

A: Type and cross match (consider autologous blood) A: ICU bed A: Balloon occlusion test A: Consult vascular surgery A: Embolization (24-48 hours before) A: Alpha and beta antagonists

Definitions of diverticulae types, etiologies, and locations

A: Types True diverticula - Involvement of mucosa, submucosa, muscle Pseudodiverticula - Involvement of mucosa and submucosa A: Etiology Traction - External pull due to adjacent inflammation or neoplasia Pulsion - Due to increased intraluminal pressure combined with mural weakness A: Locations Pharyngoesophageal (Zenker's) - usually due to pulsion Midesophageal (mid-thoracic) - usually due to traction, and usually true diverticula Epiphrenic - usually due to pulsion

CT & MRI findings for Mucocele

A: Typically hypointense on CT A: Variable pattern, depends on hydration; Hyperintense on T1 (when dehydrated), Hyperintense secretions on T2 with hypointense central area

Five areas of extrinsic esophageal compression likely to be burned in Caustic ingestion

A: UES - Cricopharyngeus A: Aortic arch A: Left mainstem bronchus A: LES A: Gastroesophageal junction

Etiologies of deep space neck infections

A: URTI/pharyngitis/tonsillitis - most common for children A: Dental - most common for adults A: Salivary infection - second most common for adults? A: Trauma/Surgery/Instrumentation A: Foreign bodies A: Spread of localized/superficial infection A: IV drug abuse A: Congenital/Branchial anomaly

Four noninvasive localization studies for parathyroid adenoma

A: Ultrasonography - 22-82% sensitive A: CT - 47-76% sensitive A: MRI - 50-80% sensitive A: Sestamibi scan - 70-90% sensitive

Six Methods of identifying Occult Nodal disease in Melanoma

A: Ultrasound A: CT A: MRI A: SPECT or PET A: ELND = No benefit found Except in 1-2 mm thick lesions in patients <60 years of age A: Sentinal LNB - Should be offered to melanoma ≥1 mm (more complicated based on new NCCN guidelines)

CHAOS - how diagnosed and findings

A: Ultrasound findings in utero for diagnosis A: Dilated airways below the level of the upper airway obstruction, large echogenic lungs, flattened diaphrams, fetal ascites

Describe the 6 structures involved in the Osteomeatal Unit (OMU)

A: Uncinate process A: Ethmoid Bulla and anterior ethmoid ostia A: Ethmoid Infundibulum A: Maxillary sinus ostium A: Frontal recess A: Structures between Middle turbinate and lamina

Four anterior to posterior bony lamina encountered in ESS

A: Uncinate process A: Ethmoid bulla A: Vertical portion of the basal lamella of the middle tubinate A: Lamella of the Superior turbinate 3: Additional answers may include the Supreme turbinate, and Anterior wall of sphenoid sinus

Eight Criteria which should suggest an overnight stay post-tonsillectomy

A: Under 3 years of age A: OSA A: Craniofacial abnormalities A: Medical comorbidity (diabetes, seizures, Down syndrome, asthma, cardiac disease, etc) A: Peritonsillar abscess A: Emesis or hemorrhage A: Patient lives greater than 60 minutes away from hospital A: Poor socioeconomic class predisposing to neglect

Six complications of vocal cord medialization by injection

A: Underinjection - requires multiple procedures A: Overinjection - airway compromise A: Improper placement A: Migration of injection material A: Intrachordal injection - impairs vibratory ability A: Granuloma formation (polytef)

Three divisions and 10 branches of Mandibular division of trigeminal

A: Undivided Nerve Meningeal (Nervus spinosus) Medial pterygoid Tensor veli palitini Tensor typani A: Branches of the Anterior division Buccal N Lateral pterygoid Anterior and Posterior Deep Temporalis A: Branches of the Posterior division Auriculotemporal N Lingual N Inferior alveolar N (Mylohyoid, Incisive, Mental)

Conditions for partial thyroidectomy (hemithyroidectomy with isthmusectomy)

A: Unifocal, intrathyroidal, and nonmetastatic papillary carcinomas <1.0 cm in diameter A: No previous radiation A: Contralateral lobe is clinical normal

General management approach of Choanal Atresia

A: Unilateral - Nonurgent repair, ~1 year of age A: Bilateral - Establish airway & feeding pathway (McGovern nipple, Oropharyngeal airway; intubation not necessary unless mechanical ventilation required) A: Surgical repair approaches (SPAN = transSeptal, transPalatal, transAntral, transNasal) A: Postop care includes - ICU monitoring, frequent Suctioning, Antibiotics, PPI

Six diagnostic clues to the involved Labyrinth in uncompensated Peripheral Vestibular lesion

A: Unilateral fluctuating or progressive SNHL A: Consistently reproducible Unilateral Weakness on Calorics (ENG) A: Tinnitus A: Aural fullness A: Direction of observed Spontaneous or Positional nystagmus A: Rotatory chair asymmetries A: Head shake and Head thrust tests

Three Symptoms of SLN injury

A: Unstable pitch A: Difficult with high pitch A: Dysphagia/aspiration (loss of sensation)

Options for reconstruction of Anterior Tongue

A: Up to 50% resection, no reconstruction - Secondary intention, primary closure, STSG or dermal graft only A: Larger or composite resection - Free Radial forearm, or Lateral arm flap for increased bulk

Seven subsites of the oral cavity

A: Upper & lower lips A: Oral tongue A: Upper & lower alveolus A: Buccal mucosa A: Floor of mouth A: Retromolar trigone A: Hard palate

Frankfort plane passes through which structures?

A: Upper EAC margin A: Inferior orbital rim

Describe efferent lymphatics of the tonsils and adenoids

A: Upper deep cervical nodes (T+A) A: Retropharyngeal LN (A only) 3: No afferent lymphatics for T and A

Lip analysis

A: Upper lips - from Subnasale to Stomion = 1/3 lower facial third A: Lower lip - from Stomion to Menton = 2/3 lower facial third A: Upper lip fuller, projects slightly more than lower A: Lips should be anterior to line through subnasale & pogonion A: Oral commissure on same vertical line as Medial Limbus

Seven steps in thyroid metabolism

A: Uptake of Iodine by thyroid - ↑ by TSH A: Coupling of Iodine to Thryroglobulin (organification) - ↑ by TSH A: Storage of MIT/DIT in follicular space - ↑ by TSH A: Re-absorption of MIT/DIT A: Formation of T3 and T4 form MIT/DIT - ↑ by TSH A: Release of T3 and T4 into serum - ↑ by TSH A: Breakdown of T3 and T4 to release Iodine

Discuss the Head Thrust test

A: Uses unpredictable, high-acceleration head rotations (3000-4000 degrees/sec) through amplitudes of 10-20 degrees in order to demonstrate asymmetric VOR responses in unilateral labyrinthine weakness A: When the thrust excites the canal on the intact side, the VOR that results is nearly compensatory for the head movement; However, when the thrust excites the canal on the lesioned side, the VOR that results is markedly diminished, resulting in a corrective saccade

Four methods of evaluating for Choanal Atresia

A: Using cotton or mirror to detect airflow A: Inability to pass a small suction catheter A: Flexible scope A: CT scan

Grading Firm Mature SGS, Myer & Cotton (1994)

A: Using cuffless pediatric ET tube A: Assess air leak at 25 cm H2O pressure A: Comparing to expected size ET tube for patient age deduction % of lumen obstruction from the above 3: Usefulness is prognosis for decannulation, and number of operations required to decannulation

Bacterial flora of deep space neck infections

A: Usually due to mixed flora A: Aerobes - Streptococcus sp (viridans, beta-hemolytic, microaerophilic), Staphylococcus sp (aureus, epidermidis), Neisseria, Klebsiella, Haemophilus A: Anaerobes: Peptostreptococcus, Peptococcus; and Fusobacterium, Eubacterium, Veionella, Eikenella, Bacteroides 3: "**** You Vile Icky, Bacteria!"

Discuss Air Embolism during surgery

A: Usually through Diploic Veins in Skull into Jugular system & Sigmoid sinus A: 30cc of Air will cause signs & symptoms - Hypotension, Churning precordial sounds

Describe the components of VACTERL syndrome?

A: V - Vertebral/Vascular anomalies A: A - Anal atresia A: C - Cardiac anomalies (PDA, valve problems) A: TE - Tracheoesophageal fistula A: R - Renal/Radial bone anomalies A: L - Limb anomalies (extra digits, shortened limbs)

Describe the pathology for Aqueductal anomalies

A: VA normally 0.4-1 mm in diameter when measured halfway between common crus and external aperture A: Enlarged VA >1.5 mm, ~40% eventually develop Profound SNHL A: CA highly variable, usually 3-4 mm in diameter A: Enlarged CA ranges up to 10 mm

Travelling wave theory in Voice production

A: VC vibration occurs in lateral & vertical planes A: Sequence - Lower margin separates, elliptical volume of air formed in subglottal vault ("glottal puff"), which is released as upper margin separates; lower margin returns to midline, then upper margin does also A: Phase delay = Delay between closure of upper and lower vocal cord margins; phase closure (closing phase) should normally be ~1/3 of the vibratory cycle

Four contraindications against palatal procedures in mild OSA

A: VPI A: Bleeding disorder A: Voice/swallowing considerations A: Submucus cleft palate 3: For LAUP, add hyperactive gag reflex, and retrognathia with relative macroglossia

Eight complications of UPPP

A: VPI A: Voice change A: Bleeding A: Nasopharyngeal stenosis A: Foreign body sensation A: Oral candidiasis A: Perioperative upper airway obstruction A: Death from upper airway compromise

What are the 2 most common cranial nerves to be affected by a carotid body tumour?

A: Vagus (X) A: Hypoglossal (XII)

Therapy for Mild/Moderate Acute Vertigo (VG MD)

A: Valium 2 mg PO tid A: Glycopyrrolate 2mg PO tid A: Meclizine (Antivert/Dramamine) 25-50mg PO q6h A: Dimenhydrinate 50mg PO q4h

Define Sinus Lateralis/Retrobullar Recess

A: Variable air space found posterior and superior to the ethmoid bulla, in the anterior ethmoid region A: Boundaries are the ethmoid roof superiorly, lamina papyracea laterally, vertical portion of the basal lamella posteriorly

Clinical characteristics of Laryngomalacia

A: Variable inspiratory stridor - Begins in first few days/weeks after birth, worse with crying, feeding, or supine position with H&N flexed, better when prone or with H&N extended A: Signs of intermittent upper airway obstruction A: Normal cry A: Normal general health and development

General method of subclassifying Complications in Rhinology

A: Vascular A: Nerve damage A: Facial swelling or ecchymosis A: Orbital A: Intracranial A: Packing related 3: "Very Nice Face Or Brain Please!"

Neurovascular supply of the Jejunum visceral free flaps

A: Vascular - Based on single vascular arcade from SMA (usually 2nd arcade) A: Peristalsis maintained by action of autonomic plexuses

Neurovascular supply of the Rectus Abdominis myocutaneous free flaps

A: Vascular - Deep Inferior Epigastric arteries & veins; Inferior pedicle larger and provides the musculocutaneous perforators supplying the skin A: Nerve - any of the Intercostal nerves

Neurovascular supply of the Fibula the osseous composite free flaps

A: Vascular - Peroneal artery & vein; perforators run in posterior intermuscular septum A: Nerve - Lateral sural cutaneous A: Bone - Up to 25cm of bone available, contourable due to segmental blood supply; leave 10cm superiorly and inferiorly? (says 8 cm in ben's notes) 3: Blood supply to skin not completely reliable; loss in 5-10% of cases

Neurovascular supply of the Latissimus Dorsi myocutaneous free flaps

A: Vascular - Thoracodorsal vessels, off of Subscapular vessels A: Nerve - Thoracodorsal nerve 3: Can be elevated as a free or regional pedicled flap

Neurovascular supply of the Gastroomental visceral free flaps

A: Vascular - based on Right Gastroepiploic artery

Seven points in the assessment of the larynx using Continuous Light (VC SPASM)

A: Vascularity or risk for hemorrhage A: Vocal Cord Edges A: Laryngeal Structure A: Change in Laryngeal Position or Height A: Arytenoids Movement and Position A: Supraglottic Activity A: Mucous Color and Quantity

Describe the Sympathetics Autonomic supply to the nasal mucosa

A: Vasoconstrictor tone A: Preganglionics from sympathetic chain synapse in the Superior Cervical Ganglion A: Postganglionics travel with ICA, split off as deep petrosal nerve and joins with greater superficial petrosal nerve to form the vidian nerve A: Pass through the sphenopalatine ganglion without synapsing, into the sphenopalatine nerve, through the foramen and into the nasal cavity

Four cellular effects of histamine in atopy

A: Vasodilation A: Increases capillary permeability A: Bronchoconstriction A: Tissue edema 3: Main mediator of early allergic reaction

Describe the Parasympathetics Autonomic supply to the nasal mucosa

A: Vasodilation of capacitance & resistance vessels, also mediate nasal secretion A: Preganglionics arise in Superior Salivatory Nucleus, travel in nervus intermedius and branches off at the geniculate ganglion as the greater superficial petrosal nerve and becomes the vidian nerve, synapses at the Pterygopalatine Ganglion A: Postganglionics distributed with sympathetics through Maxillary trigeminal branches to the nose

Five factors controlling nasal airflow

A: Vasomotor control/Nasal cycle A: Exercise - Epinephrine A: Sex Hormones - Puberty, Menstruation, Pregnancy A: Nitric Oxide - Affects Nasal AND Pulmonary blood flow, Ciliary beat frequency A: Head & body Position

What 3 structures to evaluate using the Muller maneuver

A: Velopharynx A: Base of tongue A: Hypopharynx

Borders of vermilion

A: Vermilion border A: Innermost point of mucosa where lips contact 3: Made up of modified mucosa

Describe the 6 major types of Jugular Foramen Syndromes

A: Vernet syndrome - CN IX, X, XI palsies; due to jugular foramen neoplasm, most commonly Lymphadenopathy of Krause's nodes A: Collet-Sicard syndrome - CN IX, X, XI, XII palsies; most commonly due to Extradural Tumor of Posterior Fossa or Retroparotid space A: Villaret syndrome - CN IX, X, XI, XII palsies, Sympathetic chain involvement leads to Horner's; suggests lesion Distal to jugular foramen, usually Retrostyloid area A: Hughlings-Jackson syndrome - CN X, XI, XII palsies (Collet with no Glosso!) A: Tapia syndrome - CN X & XII lesions below level of inferior ganglion of X (vocal palsy seen, palate intact); due to Lesion in neck (usually Traumatic) A: Schmidt syndrome (vagal-accessory) - CN X & XI; Paralysis of Soft Palate, Pharynx & Larynx, flaccid weakness & atrophy of SCM/Trapezius; due to Lesion of Nucleus Ambiguous & Bulbar Spinal Nuclei of accessory A: Avellis' syndrome - CN X

Describe the Nystagmus in SCDS

A: Vertical and Torsional A: Loud sounds, Positive EAC Pressure or Insufflation against pinched nose = Upbeating and Superior pole rotating away from SCD on slow phase (fast phase is opposite - down and sup pole towards the SCD) A: Jugular venous compression, Negative EAC Pressure or Valsalva against closed gottis = Downbeating and Superior pole rotating towards SCD on slow phase (fast phase opposite - up and away from SCD)

Chin position analysis

A: Vertical line from Inferior Vermilion border (perpendicular to Frankfort plane) determines position; male Pognonion should be tangential to line, female 2-3 mm posterior to the line (Silver's method) A: Gonzales-Ulloa zero meridian (nasion) A: Legan angle - Tangents through Subnasale to Glabella, and Subnasale to Pogonion = 12 +/- 4 degrees ("Legan is Some Glutenous Pig!") A: Merrifield Z angle (Frankfort plane and Pogonion to Anterior Lips) = 80 +/- 5 degrees A: Length - Menton to suprasternal notch should be 50% of the head height (from vertex to menton)

Head & Neck manifestations of Multiple Sclerosis

A: Vertigo - presenting Sx in 7-10% A: Nstagmus - mainly horizontal A: Deafness - rare A: Diplopia - can be intermittent, paralyzed medial rectus (in INO) A: Bilateral internuclear ophthalmoplegia (INO) - diplopia on one side and unilateral nystagmus, lesion in the medial longitudinal fasciculus (MLF), suggestive of MS

Ddx of peripheral vertigo according to duration

A: Vertigo lasting seconds - BPPV A: Vertigo lasting minutes to hours - Idiopathic endolymphatic hydrops (Meniere's disease), or Secondary endolymphatic hydrops (Otic syphilis, Delayed endolymphatic hydrops, Cogan's disease, Recurrent vestibulopathy) A: Vertigo lasting days - Vestibular neuritis or Labrynthitis A: Vertigo of variable duration - Inner ear fistula, SCDS, Inner ear trauma (Penetrating, Non-penetrating, Barotrauma), Familial vestibulopathy, or Bilateral vestibular deficit (Ototoxicity, Bilateral temporal bone trauma, Autoimmune, Meningitis, Sepsis)

What are Orphan Annie eyes?

A: Vesicular, "ground-glass" nuclei

Neurovascular supply of the Lateral Arm osseous composite free flaps

A: Vessels - Profunda Brachii artery 3: Up to 10 cm and 1/6th of humeral circumference can be taken

Neurovascular supply of the Radial Forearm osseous composite free flaps

A: Vessels - Radial A: Up to 10 cm and 40% of radial circumference can be taken 3: Not suitable for osseointegration

Seven types of eye movements

A: Vestibular (VOR) - Reflex, Holds image of seen world steady on retina during brief head rotations A: Optokinetic - Reflex, Holds images of seen world steady on retina during sustained head rotations; Complements the vestibulo-ocular reflex by responding best to very slow, very long, or constant velocity (zero acceleration) head movements A: Nystagmus - Reflex, Resets eyes during prolonged rotation and direct gaze toward oncoming visual scene; Multiple types A: Fixation - Voluntary, Holds image of stationary object on fovea A: Smooth pursuit - Voluntary, Holds image of a moving target steady on fovea; like Fixation, it involves the Frontal and Occipital eye fields, Corticobulbar fibres, Brainstem nuclei, Cerebellum A: Saccades - Voluntary, Brings images of objects of interest onto fovea A: Vergence - Voluntary, Moves eyes in opposite directions to track target moving closer or farther away so that images of a single object are placed simultaneously on both foveas; Occipital cortex, Linked to the Accommodation reflex (activation of ciliary body and pupillary sphincter), Modifies VOR depending on target distance

In what six conditions may ENG be helpful?

A: Vestibular hypofunction - Unilateral or Bilateral A: Disorders of Oculomotor control A: Meniere's disease or Endolymphatic Hydrops A: Recurrent vestibulopathy A: Migraine-associated dizziness A: BPPV

Three points on pathophysiology in Vestibular Neuronitis

A: Vestibular nerve degeneration A: Sparing of peripheral receptor structure A: Viral etiology suspected

MRI characteristics of CPA and Petrous Apex Tumors

A: Vestibular schwannoma - T1 Iso/Hypo, T2 Iso/Hypo, Very Strong Gad enhancement A: Meningioma - T1 Iso/Hypo, T2 Iso/Hypo, Strong Gad enhancement A: Epidermoid - T1 Iso/Hypo, T2 Hyper (like CSF), Gad Nonenhancing, FLAIR Iso; FLAIR required to differentiate, will suppress CSF & Arachnoid cyst, DWI stays hyper (due to restricted diffusion) A: Arachnoid cyst - Well defined Rounded cyst, T1 Hypo, T2 Hyper (like CSF), Gad Nonenhancing, FLAIR Hypo, DWI is dark (high diffusion) A: Paraganglioma - Salt & Pepper on T1 & flow voids T2, Strong Gad enhancement A: Cholesterol granuloma - T1 Hyper, T2 Iso/Hyper, Gad Nonenhancing A: Dermoid - T1 Hyper, T2 Hypo, Gad Nonenhancing A: Lipoma - T1 Hyper, T2 Hypo, Gad Nonenhancing A: Chordoma & Chondrosarcoma - T1 Iso/Hypo, T2 Hyper, Strong Gad enhancement A: Petrous apicitis - T1 Hypo, T2 Hyper, Gad rim enhancement

What are 4 causes for the inability to adapt?

A: Vestibular suppressants A: Inactivity A: Advanced age A: Fluctuating vestibular deficit A: Other CNS or cerebellar pathology

Cochlea

A: Vestibulocochlear nerve A: Cochlear nucleus A: Trapezoid body (crossover to contralateral side here) A: Superior Olivary complex A: Facial motor nucleus A: Facial nerve A: Nerve to Stapedius

What three indirect immunofluorescence tests for antibodies to EBV play a role in NPC diagnosis and therapy?

A: Viral Capsid Antigen (IgA, 95% Sensitive) & Early Antigen (IgG, 95% Specific) - most specific tests for diagnosis, titers ~85% positive in cases of WHO II & III NPC A: ADCC titers - can be used to predict prognosis of WHO II & III NPC, significant relationship between lower titers & progression

Suspected Etiologies of Idiopathic Sudden SNHL (4)

A: Viral infection of the labyrinth A: Vascular A: Intracochlear Membrane rupture/Perilymph fistula A: Sudden expansion of CPA lesion A: Autoimmune inner ear disease (although AIED usually Bilateral by definition and slower in onset, may begin asymmetrically) 3: Ddx of SNHL much broader

False positive VDRL is associated with what diseases? (VDRL MMESHHH)

A: Viruses & infections - Measles, Malaria, EBV, Smallpox, HSV, HIV, Hepatitis A: Drugs A: Rheumatic fever, Rheumatoid arthritis A: Leprosy, Lupus

Three Indications for Treatment of Systemic Kaposi sarcoma

A: Visceral disease A: Pulmonary disease A: Extensive mucocutaneous involvement (>10 new in 1 month)

Name the physiologic basis for Motion Sickness

A: Visual-Vestibular Mismatch

Factors in nutrition that most influence wound healing

A: Vitamin A deficiency causes impaired collagen synthesis & epithelialization A: Vitamin C deficiency prevents hydroxylation of proline & lysine A: Zinc deficiency retards wound healing by preventing cell proliferation A: Magnesium A: Arginine

Three goals of voice therapy

A: Vocal Hygiene A: Decrease Phonotrauma A: Increase Coordinated voice production

Four risk factors for Vocal cord granuloma

A: Vocal abuse (chronic cough, throat clearing) A: Traumatic intubation A: GERD A: Male sex

Six Adjunctive procedures for Chronic Aspiration

A: Vocal cord Medialization - Injection, Thyroplasty, Arytenoid adduction A: Laryngeal Suspension A: Cricopharyngeal Myotomy or Botox injection A: Posterior cricoid resection A: Elevation of the Interarytenoid area A: Tracheostomy

Nine Contraindications to Supraglottic laryngectomy

A: Vocal cord fixation (absolute contraindication) A: Bilateral arytenoids cartilage involvement (absolute contraindication) A: Involvement at the glottic level A: Tumor within 5 mm of anterior commissure A: Ventrical or pyriform apex involvement A: Thyroid or cricoid cartilage involvement A: Involvement of the tongue base to within 1 cm of the circumvallate papillae A: Poor medical condition - extreme age, poor pulmonary function (FEV1/FVC < 50%) A: Prior irradiation (relative)

What is Ortner's syndrome?

A: Vocal cord paralysis caused by severe cardiomegaly, enlarged left atrium caused by mitral stenosis, thoracic aortic aneurysms, or dilated pulmonary arteries with pulmonary hypertension 3: AKA cardiovocal syndrome

Opera singer with good vocal hygiene and Acute Laryngitis; 4 Options

A: Vocal rest until day of performance A: Increase amplification A: Alter repertoire for the upcoming performance A: Offer steroids for short term

Five predisposing factors for Intracordal Hematoma

A: Voice abuse; extreme vocal effort (sporting event, forceful singing, sneezing) A: Usually Men A: Anticoagulants A: Laryngitis A: Capillary ectasia is a precursor, rupture of submucosal capillaries/vessels

Five syndromes with congenital neurofibromas or fibromas of tongue and jaw

A: Von Recklinghausen's disease (NF1) A: MEN IIB/III A: Tuberous sclerosis A: Cowden's disease A: Gardner?

Most common Bilateral lesions

A: Warthin's (10% of cases) A: Acinic cell (3% of cases) A: Oncocytoma 3: "Bilateral lesions are WACO!"

Parotid tumors arising from Striated duct cells

A: Warthin's tumor A: Oncocytoma

Hot lesions on Technetium scans

A: Warthin's tumor A: Oncocytoma 3: High mitochondrial content (striated duct)

Describe the normal Absolute Latencies on the ABR

A: Wave I - 2.0 ms A: Wave II - 3.0 ms A: Wave III - 4.1 ms A: Wave IV - 5.3 ms A: Wave V - 5.9 ms 3: Roughly 2, 3, 4, 5, and 6 ms

Describe the normal Interwave Intervals differences on the ABR

A: Wave I-III interval = <2.3 ms (Cummings & KJ) A: Wave III-V interval = <2.1 ms (Cummings & KJ) A: Wave I-V interval - <4.6 ms (KJ) A: Interaural Wave V latency - <0.2 ms (Bailey's), <0.3 ms (KJ), <0.4 ms (Cummings) 3: As in Bailey, roughly 2, 2, 4.4, and 0.2 ms

Five reasons for feedback in hearing aids

A: Wax A: Internal malfunction (eg: cracked tubing) A: Poor fit/leak A: High gain A: Small hearing aids

Four histologic and clinical differences of T-cell lymphoma and Wegener's granulomatosis of the nose

A: Wegener's has Diffuse nasal ulcerations; Lymphoma lesions are Focal, localized & explosive A: Wegener's has a Small & Medium vessel Vasculitis; Lymphoma has a polymorphic lymphoid infiltrate with Angiocentric & Angioinvasive features A: Wegener's has an Inflammatory cell infiltrate; Lymphoma has a primarily Lymphocytic infiltrate A: Otologic, Tracheal, Renal involvement rare in lymphoma

Seven selection Criteria for Anterior Cricoid Split

A: Weight > 1500 gm A: O2 requirement < 30% A: No ventilation support for at least 10 days A: No antihypertensive medications at least 10 days A: No CHF for at least 1 month A: No other airway abnormalities A: No acute respiratory tract infection

Treatment of Acute Rhinosinusitis

A: Whatchful waiting for 7 day IF Non-severe/mild pain, fever <38.3 C, followup ensured, able to re-evaluate if symptoms worsen; 5 adjunctive treatments: A: Double dose Mometasone spray A: Saline irrigations - No iodinated table salt as it inhibits ciliary action A: Mucolytics - Guaifensin 1200 mg bid, or N-Acetyl Cysteine 600 mg bid A: Decongestants A: Analgesics

Management of the Chylous fistula

A: When encountered intraoperatively, Ligate it off A: Postoperative Head elevation A: Closed suction drainage A: Pressure dressing A: Alter diet with MCT's - directly absorbed into portal system, avoids breakdown of LCT's into FFAs & glycerol which are transported in chylomicrons A: Monitor and correct electrolytes A: +/- Surgery vs tetracycline sclerosing therapy for failures

Warthin's tumor (papillary cystadenoma lymphomatosum, adenolymphoma) epidemiology

A: Whites A: Male/Female = 5/1 A: 5-6th decades A: Radiation & smoking influence A: 10% of parotid tumors, 10% bilateral, 10% multicentric

Name 4 X-linked Recessive syndromes associated with SNHL

A: Wildervaank syndrome A: Otopalatodigital syndrome A: Alport syndrome A: Norrie syndrome 3: "WOmAN"

Neufibromatosis type II subtypes (2)

A: Wishart - Early onset, rapid growth, other tumors than acoustic neuromas A: Gardner - Slower rate of growth & onset, usually only bilateral acoustic neuromas

What are the 3 standards for how the decision should be made, in decreasing order of priority?

A: Wishes - Prior expressions by the patient, while competent that seem to apply to the actual decision that needs to be made A: Values and beliefs A: Best interests - This is more important for children, where the decision maker is usually the parent; decisions should be respected unless they would cause direct and serious harm to the child

Six intrinsic muscles of larynx

A: With conus elasticus Thyroarytenoid (vocalis) Interartyenoid Lateral cricoarytenoid Posterior cricoarytenoid A: With quadrangular membrane Thyroepiglottic Aryepiglottic A: Cricothyroid (disputed)

Best Postoperative Time interval for Adjuvant XRT

A: Within 6 weeks

Eight Alaryngeal Communication Methods

A: Writing A: Gesturing A: Mouthing words A: AAC computer-based program, Alphabet boards A: Buccal-Pharyngeal speech A: Artificial Laryngeal devices A: Esophageal speech A: Tracheo-Esophageal speech

Hunter's syndrome (MPS II)

A: X-linked A: Deficiency of beta-galactosidase A: Macrocephaly, broad face, low nasal bridge, death usually occurs from infiltrative CMO & valvular disease leading to CHF

Name 4 conditions associated with Perilymph Gusher

A: X-linked Stapes Gusher Syndrome A: Congenital Fixed Footplate A: Mondini Deformity A: Enlarged Vestibular Aqueduct

Five Treatment modalities for small localized Kaposi sarcomas

A: XRT (for localized obstructing lesions) A: Laser excision A: Surgical excision A: Intralesional Chemotherapy - Vinblastine, or Interferon A: Cryotherapy

Nine Risk factors for Salivary gland malignancy

A: XRT (low dose) A: Full-mouth dental x-rays A: Skin CA? A: Rubber industry A: Nickel A: Silica dust A: Hair dye A: Kerosene cooking fuel A: Vegetables preserved in salt

Seven Complications of Radiotherapy for Neck disease

A: Xerostomia - 5-6 Gy in 5-6 weeks? A: Mucositis A: Dental caries A; Osteoradionecrosis - In up to 5% of patients, rare <60 Gy, increased if chemo-XRT A: Soft tissue fibrosis A: Hypothyroidism - 1% clinically overt, 10% occult after 50 Gy in 4 weeks A: Immunosuppresion A: Spinal cord Necrosis - Limit to 45-50 Gy in 5 weeks

What are Fordyce spots?

A: Yellow spots on the buccal mucosa lateral to the edges of the lips, usually bilateral, representing remnants of atrophied sebaceous cysts

Five clinical laryngeal findings in Presbylaryngeus

A: Yellow/dark gray discoloration of vocal fold (fibrofatty accumulation) A: Vocal fold atrophy, bowing and incomplete glottic closure A: Arrowhead configuration of glottis A: Visibility of ventricles A: Prominent vocal process A: Mucosal dryness

In comparison to Cholesteatoma, Patients with Keratitis Obturans are (4)

A: Younger A: Less or No Draining ear A: More generalized in position (compared to Choles. which is just localized to lateral to annulus) A: Widens EAC (but doesn't destroy bone)

Methods of scar irregularization

A: Z-plasty - Single or compound; best for scars across concavities (ie. webbing or banding) A: W-plasty - Best for long scars along curves (eg. angle of mandible); base <6mm, sides <6.5mm A: Geometric broken line - Best technique for long unbroken facial scars; shapes should be <5-6mm in any dimension

Name the four zones of the circulatory system applied to flap perfusion

A: Zone I - Macrocirculation = cardiopulmonary system, arteries & veins, lymphatic system; supplies flap through musculocutaneous, septocutaneous branches A: Zone II - Capillary circulation; flow controlled by Precapillary & Preshunt sphincters Precapillary sphincters under control by local hypoxia & metabolic byproducts Preshunt sphincters under autonomic control (norepinephrine) for thermoregulation and systemic BP maintenance A: Zone III - Interstitial space & mechanisms of nutrient delivery, including capillary wall; Diffusion (down concentration gradient) & Convection (bulk flow of plasma current) A: Zone IV - Cells and their membranes

Discuss the No-Reflow phenomenon

A: Zone II/III failure with intact zone I A: When the critical ischemia time for a flap is exceeded (12 hours), endothelial & parenchymal swelling, intravascular stasis, and thrombosis lead to loss of nutritive flow A: Severity of this effect is correlated with ischemia time A: A unique aspect of free flap physiology is microcirculatory failure related to showers of microemboli from the anastomosis

10 adjuvant therapies for RRP

A: a-Interferon therapy (most common, exact mechanism unknown) A: Indole-3-carbinol diet supplementation (found in cruciferous vegetables) A: Retinoic acid A: Methotrexate A: Antivirals (Ribavirin, Acyclovir, Cidofovir) A: HspE7 A: Photodynamic therapy A: Mumps vaccine - Gardisil (HPV) Vaccine is better answer

Four antagonistic relationships in cancer genes

A: bcl-2 and p53 A: p53 and Cyclin A: p16/p21 and Cyclin A: RB and TF E2F

Three borders of Trautmann's triangle (LSD)

A: bony Labyrinth A: Sigmoid sinus A: superior petrosal sinus or Dura

Define dB SPL, dB HL, dB SL

A: dB - 10 log (output intensity/reference intensity) A: SPL - Sound Pressure Level = Actual amount of sound pressure A: HL - Hearing level = Accounts for the human ear having different Sensitivities at different Frequencies (eg 6.5 dB SPL at 1000 Hz, 45 dB SPL for a 125 Hz, and both of these are 0 dB HL) A: SL - Sensation Level = Intensity of the stimulus in decibels above an individual's hearing threshold (a person has a hearing loss of 30 dB HL and if a test is to be given at 40 dB SL, then this would translate to 70 dB HL)

Seven Minor signs/symptoms of Rhinosinusitis (ABCDEFF)

A: headAche A: Bad breath (Halitosis) A: Cough A: Dental pain A: Ear pain/pressure/fullness A: Fatigue A: Fever

What are the 9 most Common Genes involved in HNSCC?

A: p53 - Tumor suppressor, involved in cell cycle regulation and Apoptosis; loss in >50% of HNSCC A: p16 & p21 - Tumor suppressor, suppresses Cyclin and Cyclin-dependent kinase pathways A: RB - Tumor suppressor, inhibits Transcription Factor E2F; loss in >60% of HNSCC, it is a target for HPV E7 gene A: HER-2/neu - Proto-oncogene, Receptor Tyrosine Kinase, ErbB/Epidermal Growth Factor Receptor (EGFR) family; overexpressed in 90% of HNSCC, chromosome 17q11.2 A: Cyclin D1 - Oncogene, accelerates cell cycle progression A: bcl-2 - Oncogene, inhibits apoptosis & counteracts p53 A: RET - Proto-oncogene (CC10), Receptor Tyrosine Kinase involved in cell growth, associated with MEN II A: C-myc - Proto-oncogene, Transcription regulator

Three most important questions on history about Caustic agents

A: pH A: Volume ingested A: Consistency

Most important prognostic factors for Papillary thyroid ca - MASH (MD Anderson)

A: presence of distant Metastatic diseases A: Age A: Sex A: Histology of the cancer

Contraindications of esophagoscopy (BASICS)

A: severe erosive Burns to esophagus A: Aortic aneurysm A: chronic Steroid use A: laryngeal Inflammation/edema A: Combative patient A: Spine abnormalities - severe cervical/thoracic (esp large osteophytes)

Percentage of Genetic hearing loss that is Syndromic

A: ~30%

Most common number of parathyroids

A: ~85% have 4 parathyroids A: ~10% have >4 A: ~5% have <4

Ratio of facial width/length

A: ¾

Nine thyroid hormone functions (CHROME NCL)

A: ↑ CO, HR A: ↑ Heat production A: ↑ RBC mass A: ↑ O2 consumption A: ↑ Metabolic rate (calorigenesis) A: Epinephrine potentiation A: Neural growth and development (*Neonatal brain) A: ↓ Cholesterol A: metabolism of Lipids, carbs, & protein

Four effects of TSH on thyroid cells

A: ↑ Iodine transport into cell A: ↑ formation of MIT & DIT (organification) A: Storage of T3 & T4 in the gland as colloid A: ↑ hydrolysis of the stored TG to release T3 and T4 into circulation A: ↑ TG synthesis and proteolysis 3: Maintains structure of thyroid cells, and stimulates gland size and vascularity

Effects of Parathyroid Hormone

A: ↑ serum calcium, ↓ serum phosphates A: Bone - ↑ osteoclasts A: Kidney- ↑ calcium reabsorption, ↓ phosphate reabsorption, ↑ 1,25-(OH)2-D3 synthesis A: Gut - ↑ calcium absorption via vitamin D stimulation

Effect of Calcitonin

A: ↓ serum calcium by inhibiting Osteoclast function

Repair options for Upper Lip defects

A: ≤50% - Perialar crescentic excisions and primary advancement closure for midline defects Primary closure for lateral A: 50-66% - Karapandzic flap for midline Abbe flap Estlander flap (if commissure involved) A: >66% - Webster modification of Bernard-Burow advancement flap for midline Gillies fan flap Temporal forehead flap, Regional flap, or Microvascular free flap if inadequate adjacent cheek tissue

Repair options for Lower Lip defects

A: ≤50% - Primary closure A: 50-66% - Karapandzic flap (orbicularis oris myocutaneous flap) Abbe flap Estlander flap (if commissure involved) A: >66% - Bernard-Burow advancement flap for midline defects Gillies fan flap Full thickness nasolabial transposition flap for lateral defect Regional, Distant, or Microvascular free flap if inadequate adjacent cheek tissue

Seven Adult Cochlear Implantation criteria

A: ≥18 years of age A: Severe to profound (>70 dB) SNHL; SDS ≤30% A: Little or no benefit from amplification trial (SRS <50% correct in best aided condition) of ≥6 months duration A: No middle ear disease A: No medical contraindications A: Psychologically and motivationally suitable

Six Indications to perform a detailed metastatic workup in Head and Neck Cancer

A: ≥4 lymph nodes metastases A: Level 4 lymph nodes metastases A: Bilateral lymph nodes metastases A: Lymph nodes metastases ≥6 cm in size A: Recurrent disease A: Second primary cancer

Seven Environmental Risk Factors for Sinonasal malignancy

Hardwood dust - Mahogany Nickel-refining processes Leather tanning Mineral oils Industrial fumes - Formaldehyde, Chlorophenol, Chromium, Isopropyl oils, Lacquer paints, Soldering and welding, Radium dial painting Asbestos Cigarette smoke


Kaugnay na mga set ng pag-aaral

Econ Ch. 14: Market Structure and Degrees of Market Power

View Set

Physics 1 - Chapter 7 - Linear Momentum Conceptual Questions

View Set

Science: Pure Substances and Mixtures

View Set

At least 100 of these will be on the OB test

View Set

Repro: Perimenopause - (Nelson) - JCN

View Set

Chapter 12- Gender, Sex, and Sexuality Test

View Set